Download as pdf or txt
Download as pdf or txt
You are on page 1of 577

FIRST YEAR

Outlines
FIRST YEAR
Outlines
Special thanks to: Lauren Yost, Esq., Adam Feren, Esq., Elizabeth Horowitz, Esq., Steven Marietti, Esq., Adam Maze, Esq., Nicole Pirog, Esq.,
Mike Power, Esq., Tammi Rice, Esq., Shalom Sands, Esq., Amit Schlesinger, Esq., Lisa Young, Esq.

This publication is designed to provide accurate information in regard to the subject matter covered as of its publication date, with
the understanding that knowledge and best practice constantly evolve. The publisher is not engaged in rendering medical, legal,
accounting, or other professional service. If medical or legal advice or other expert assistance is required, the services of a compe-
tent professional should be sought. This publication is not intended for use in clinical practice or the delivery of medical care. To
the fullest extent of the law, neither the Publisher nor the Editors assume any liability for any injury and/or damage to persons or
property arising out of or related to any use of the material contained in this book.

© 2020 by Kaplan, Inc.

Published by Kaplan Publishing, a division of Kaplan, Inc.


750 Third Avenue
New York, NY 10017

All rights reserved. The text of this publication, or any part thereof, may not be reproduced in any manner whatsoever without writ-
ten permission from the publisher.

10 9 8 7 6 5 4 3 2 1

ISBN: 978-1-5062-7232-0

Kaplan Publishing print books are available at special quantity discounts to use for sales promotions, employee premiums, or educa-
tional purposes. For more information or to purchase books, please call the Simon & Schuster special sales department at 866-506-1949.
Invest in
yourself.
Join the
ABA.
Join for free or go
Premium for just
$25 per year. ABA
law student members
can save on law
school necessities
and resources.

• After the Bar Digital Publication


• ABA Legal Ethics Bundle ($116 Value)
• ABA Student Leadership Opportunities
• National Law Student Competition Opportunities
• ABA Career Center
• ABA Member Groups (5 Free)
• The Student Lawyer magazine & Before the Bar blog
• ABA Journal ($75 Value)
• Member Discounts & Offers

ambar.org/FutureLawyer
KAPLAN BAR REVIEW
FIRST YEAR
SUCCESS PROGRAM

Access the Online Course Content


+ www.KaplanBarReview.com/1LSuccess, and click “log in.”
+ Once you’ve completed the free enrollment, you will receive a
confirmation email with your username and password.
Use that username and password to log in to your course at
www.KaplanBarReview.com/1LSuccess.

+ Get detailed lectures, complete outlines, and extensive practice


questions to help you navigate the 1L lifestyle.

Online Course Lecture and Study Materials Practice and


Manager Bar Notes Analysis

Log on to get started


MASTER TABLE OF CONTENTS

CIVIL PROCEDURE

I. Jurisdiction and Venue..........................................................................................................4


II. Law Applied by Federal Courts.............................................................................................25
III. Pretrial Procedures................................................................................................................29
IV. Jury Trials..............................................................................................................................63
V. Motions..................................................................................................................................66
VI. Verdicts and Judgments........................................................................................................73
VII. Appealability and Review......................................................................................................77

CONSTITUTIONAL LAW

I. The Nature of Judicial Review...............................................................................................82


II. Separation of Powers............................................................................................................92
III. The Relation of the Nation and the States in the Federal System.........................................105
IV. Type of Action Governed by the Constitution........................................................................112
V. Due Process and the Incorporation of Portions of the Bill of Rights.....................................115
VI. Equal Protection of the Laws.................................................................................................130
VII. Privileges and Immunities Clauses........................................................................................138
VIII. Retroactive Legislation..........................................................................................................140
IX. First Amendment Freedoms..................................................................................................142

CONTRACTS

I. Sources of Contract Law ......................................................................................................168


II. Contract Formation: Offer and Acceptance...........................................................................172
III. Consideration and Promissory Estoppel ..............................................................................192
IV. Statute of Frauds ..................................................................................................................201
V. Gap-Fillers, Interpretation, and the Parol Evidence Rule......................................................217
VI. Performance, Modification, and Excuse................................................................................231
VII. Defenses................................................................................................................................256
VIII. Remedies ..............................................................................................................................274
IX. Third-Party Beneficiaries.......................................................................................................293
X. Assignment of Rights and Delegation of Duties....................................................................298

V
CRIMINAL LAW

I. General Principles.................................................................................................................308
II. Crimes against the Person....................................................................................................316
III. Crimes against Property .......................................................................................................331
IV. Inchoate Crimes.....................................................................................................................344
V. Parties to Crime; Accomplice Liability...................................................................................349
VI. Defenses................................................................................................................................352

REAL PROPERTY

I. Ownership of Real Property..................................................................................................364


II. Rights in Land........................................................................................................................403
III. Contracts...............................................................................................................................421
IV. Mortgages/Security Devices.................................................................................................428
V. Titles.......................................................................................................................................438

TORTS

I. Intentional Torts.....................................................................................................................458
II. Negligence.............................................................................................................................475
III. Strict Liability.........................................................................................................................508
IV. Products Liability...................................................................................................................511
V. Nuisance................................................................................................................................522
VI. Defamation............................................................................................................................528
VII. Invasion of Privacy.................................................................................................................537
VIII. Wrongful Institution of Legal Proceedings.............................................................................540
IX. Economic Torts......................................................................................................................543
X. Miscellaneous Tort Concepts................................................................................................549

VI
Civil Procedure
2
TABLE OF CONTENTS

I. JURISDICTION AND VENUE

Federal Subject-Matter Jurisdiction..............................................................................................4


Territorial Jurisdiction...................................................................................................................12
Personal Jurisdiction....................................................................................................................12
Service of Process and Notice.....................................................................................................19
Venue, Transfer, and Forum non Conveniens..............................................................................22

II. LAW APPLIED BY FEDERAL COURTS

State Law in Federal Court..........................................................................................................25


Federal Common Law..................................................................................................................26

III. PRETRIAL PROCEDURES

Pleadings.....................................................................................................................................29
Rule 11.........................................................................................................................................33
Provisional Relief.........................................................................................................................34
Joinder of Claims and Parties......................................................................................................36
Pretrial Conferences and Orders.................................................................................................45
Discovery.....................................................................................................................................47
Adjudication without a Trial .........................................................................................................60

IV. JURY TRIALS

Right to Jury Trial.........................................................................................................................63


Demand for a Jury Trial ...............................................................................................................63
Jury Selection...............................................................................................................................64
Jury Instructions...........................................................................................................................64

V. MOTIONS

General Rules..............................................................................................................................66
Pre-Trial Motions..........................................................................................................................66
Post-Trial Motions........................................................................................................................69

VI. VERDICTS AND JUDGMENTS

Jury Verdicts.................................................................................................................................73
Judicial Findings and Conclusions...............................................................................................74
Effect of Verdicts—Claim and Issue Preclusion...........................................................................75

VII. APPEALABILITY AND REVIEW

Availability of Interlocutory Review...............................................................................................77


Final Judgment Rule....................................................................................................................78
Scope of Review for Judge and Jury...........................................................................................78

3
OUTLINE

I. JURISDICTION AND VENUE

A. Federal Subject-Matter Jurisdiction


1. Federal Question Jurisdiction
a. Federal district courts have original jurisdiction over all civil actions
“arising under the Constitution, laws, and treaties of the United States”
[28 U.S.C. § 1331]. A case arises under federal law if the federal ques-
tion appears on a fair reading of a well-pleaded complaint.
EXAMPLE: The Mottleys had free passes on the Louisville & Nashville
Railroad. They lost those passes when Congress passed a law that
banned free passes to railroads. The Mottleys brought suit in federal court,
but the Supreme Court ruled that the federal court did not have subject-
matter jurisdiction over the case because the case “merely arose under
federal law,” which was insufficient to establish federal question jurisdiction
[Louisville & Nashville Railroad Company v. Mottley, 211 U.S. 149 (1908)].
b. The Constitution or federal statutes may create a right of action. In that
case, federal courts have jurisdiction to hear those suits.
c. A defendant’s use of federal law as the basis for her defense will not,
by itself, create federal question jurisdiction. Under the well pleaded
complaint rule, to create federal question jurisdiction, the federal
question must arise within the plaintiff’s affirmative claim—regardless
of any defense the defendant might raise.
d. A violation of a federal statute does not create a federal cause of action
unless the statute also provides a remedy for the violation.
e. A state question that involves a question of federal law may be suffi-
cient to create federal question jurisdiction. The mere presence of
federal law in a claim is insufficient to create jurisdiction. The federal
law must have a substantial impact on the state law issue.
EXAMPLE: Federal question jurisdiction was found to arise in a land-
owner’s state law claim to quiet title against someone who had pur-
chased the land at auction, because embedded in that claim was an
argument that the Internal Revenue Service had given him inadequate
notice of the sale [Grable & Sons Metal Products, Inc. v. Darue Engi-
neering & Mfg., 545 U.S. 308 (2005)].
EXAMPLE: Federal question jurisdiction did not arise in a legal malprac-
tice claim in which the plaintiff alleged that his attorney did not raise a de-
fense under federal patent law. The disputed issue of federal law was not
substantial in any relevant sense [Gunn v. Minton, 568 U.S. 251 (2013)].
f. Federal courts have original jurisdiction over admiralty or maritime cases.
(1) Admiralty jurisdiction requires that a case have a “maritime nexus.” A
maritime nexus requires that the incident giving rise to the case had a
“potentially disruptive effect on maritime commerce,” and the general
character of the activity giving rise to the incident shows a “substantial

4
CIVIL PROCEDURE

relationship to traditional maritime activity” [Jerome B. Grubart, Inc.


v. Great Lakes Dredge & Dock Co., 513 U.S. 527 (1995)].
(2) Plaintiffs may pursue maritime claims as common-law claims
in state courts; however, remedies that are specific to admiralty
courts will not be available [28 U.S.C. § 1333].
g. Federal courts have subject-matter jurisdiction to review the state
court conviction of a prisoner through a writ of habeas corpus, chal-
lenging the confinement of the prisoner on federal constitutional
grounds [28 U.S.C. § 2254(a)].
(1) A petition for a writ of habeas corpus is a civil cause of action brought
against the jailor or custodian of a person currently under confinement.
(2) A writ of habeas corpus is not an appeal, and it does not continue
the criminal case against the prisoner in any manner. The writ does
not make a determination of the prisoner’s guilt or innocence.
(3) A petition for a writ of habeas corpus may be based only on the
following grounds:
(a) violation of the Due Process Clause of the Fifth and
Fourteenth Amendments;
(b) violation of the prohibition of compelled self-incrimination of
the Fifth and Fourteenth Amendments;
(c) violation of the protection against double jeopardy of the Fifth
and Fourteenth Amendments; or
(d) violation of the right not to be subject to cruel and unusual
punishment under the Eighth Amendment.
(4) Habeas corpus relief is only available when the state court’s
determination was contrary to, or involved an unreasonable appli-
cation of, clearly established federal law as determined by the
Supreme Court of the United States.
(5) In order to seek a writ of habeas corpus from a federal court, the
prisoner must have exhausted all remedies available at the state
level, including all state appellate review.
(6) A petition for a writ of habeas corpus must be brought within one
year of the final judgment of custody [28 U.S.C. § 2244(d)].
(a) A prisoner may not file successive petitions for habeas corpus
relief without the approval of the U.S. Court of Appeals.
2. Diversity Jurisdiction
a. Federal district courts have original jurisdiction over matters involving
litigants who are citizens of different states, U.S. citizens and citizens
of a foreign country, or a foreign state and a U.S. citizen, if the amount
in controversy is more than $75,000 [28 U.S.C. § 1332].
b. The amount in controversy between the litigants must exceed $75,000,
exclusive of the costs and expenses associated with the litigation.
(1) If the suit is based on a contract or statute that allows a winning
party to collect attorney’s fees, the amount in controversy
includes the likely amount of those fees.

5
OUTLINE

(2) A suit in which the amount in controversy is exactly $75,000 does


not meet the diversity jurisdiction requirement.
(3) An action based on diversity will be dismissed if it appears “to a
legal certainty” that the plaintiff’s claim does not exceed $75,000.
The burden falls on the defendant to prove that the amount in
controversy is not more than $75,000.
(4) There is a split of authority among the circuits regarding how the
amount in controversy is determined.
(a) Under the majority rule, which is the rule followed in most
circuits, the amount in controversy is determined according
to the amount sought by the plaintiff. This is referred to as
the “plaintiff’s viewpoint” rule.
(b) Under the minority rule, known as the “either viewpoint rule,”
the amount in controversy is not necessarily the amount
sought or recovered, but it is the value of the consequences
which may result from the litigation.
(5) A plaintiff may meet the amount in controversy requirement by
combining or aggregating the total claims against the defendant.
Claims against multiple defendants may not be aggregated to
meet the threshold unless the claims are common and undivided.
(a) Multiple plaintiffs cannot aggregate their claims for purposes
of 28 U.S.C. § 1332. The U.S. Supreme Court 5-4 deci-
sion in Exxon Mobil Corp. v. Allapattah Services, Inc., 545
U.S. 546 (2005), held that a federal court has supplemental
jurisdiction over claims of other plaintiffs who do not meet the
jurisdictional amount for a diversity action, when at least one
plaintiff in the action does satisfy the jurisdictional amount.
So we need at least one plaintiff with a claim over $75,000.
(6) If the suit seeks an injunction, the amount in controversy is either the
injunction’s worth to the plaintiff, or the cost of the injunction to the
defendant [JTH Tax, Inc. v. Frashier, 624 F.3d 635 (4th Cir. 2010)].
c. Diversity jurisdiction requires complete diversity, meaning a dispute that
involves citizens of different states. There will be no diversity jurisdiction
if any plaintiff and any defendant share citizenship of the same state.
(1) Diversity must be met only at the time the suit is filed. If a defen-
dant moves to the same state as the plaintiff after the suit has been
commenced, diversity will not be destroyed. However, if a complaint
is amended to add or dismiss a party, this can affect jurisdiction.
(2) A party is considered to be a citizen of her state of domicile. Domicile
is not the same as residence. Domicile requires a physical pres-
ence in the state and the intent to remain in that state indefinitely.
EXAMPLE: A and B, a married couple, live in Alabama. They are
registered to vote there, have a car registered there, and own a home
in that state. A and B are domiciled in Alabama. If A were to take a

6
CIVIL PROCEDURE

long-term work assignment in Texas, but she intended to move back


to her home in Alabama with B, she would not be domiciled in Texas.
(3) Alien refers to an individual who is a citizen of a foreign country.
For diversity purposes, if an alien is admitted to permanent resi-
dence in the United States (i.e., that person has a green card),
he is a citizen of the state in which he is domiciled. Therefore,
diversity is improper if a permanent resident alien is a resident of
the same state of the citizen whom he is suing or is being sued
by. In such a situation, there is no diversity jurisdiction.
(4) A corporation is a citizen of both the state or foreign country of its
incorporation and the state or foreign country where it has its principal
place of business. A corporation may only have one principal place
of business. The principal place of business is the state in which the
corporation has its “nerve center,” or the place from which the corpo-
ration’s high-level officers direct, control, and coordinate the corpora-
tion’s activities [Hertz Corp. v. Friend, 559 U.S. 77 (2010)].
EXAMPLE: ABC Corporation is incorporated in Delaware, but
has its principal place of business in New York. If ABC brought
suit against a competitor, XYZ, which has its principal place of
business in New Jersey, but which is also incorporated in Dela-
ware, there would be no diversity jurisdiction.
(5) The citizenship of an unincorporated association, such as a
partnership or limited liability company, is the citizenship of all of
the members of the association. For purposes of class actions,
however, an unincorporated association is a citizen of the state
under whose laws the association is organized, and the state in
which it has its principal place of business.
d. There is an exception to the complete diversity requirement for class
actions. In class actions where the class contains over 100 persons
and the amount in controversy exceeds $5,000,000, diversity need
only be “minimal,” meaning that federal jurisdiction exists if any single
member of the class is diverse from any single defendant.
e. There are two situations when a federal court will decline jurisdiction
even where the requirements for diversity jurisdiction are met, because
they are traditionally viewed as being matters for state courts to decide.
(1) Under the probate exception, federal courts will typically not exer-
cise jurisdiction over probate matters, such as the validity of a will
and the administration of an estate.
(2) Federal courts will also decline jurisdiction in family law matters,
such as divorce, child custody, and child support proceedings.
This is known as the family law exception.
3. Supplemental Jurisdiction
a. Supplemental jurisdiction allows a federal district court to hear
claims over which it would not ordinarily have jurisdiction (e.g., state

7
OUTLINE

law claims that do not involve diversity jurisdiction) [28 U.S.C. § 1367].
Supplemental jurisdiction permits a claim falling outside of federal
question jurisdiction or diversity jurisdiction to “piggy back” on a claim
that does fall within one of those jurisdictions.
b. To exercise supplemental jurisdiction over an additional claim, there
must be a claim over which a court has original jurisdiction [Id.]. In other
words, supplemental jurisdiction is only available over a claim if the claim
arises out of the same transaction or occurrence as a claim over which
the court would have federal question or diversity jurisdiction.
(1) Claims arise out of the same transaction or occurrence if they
arise out of a common nucleus of operative fact.
EXAMPLE: C was injured when he was wrongfully arrested by a
police officer. The federal courts have jurisdiction to hear his claim for
violation of his federal constitutional rights [42 U.S.C. §1983]. The fed-
eral courts have supplemental jurisdiction to hear his state law claim
against the arresting officer for battery arising out of the same incident.
c. Supplemental jurisdiction may include claims that involve the joinder or
intervention of additional parties, even if there would not otherwise be
federal jurisdiction over those parties.
(1) In a diversity case, supplemental jurisdiction may be exercised
over the claims of a party other than the plaintiff if the only reason
the other party’s claims do not qualify for diversity jurisdiction is
the failure to meet the amount in controversy requirement.
EXAMPLE: A nine-year-old child seriously injures her hand on
a can of fish sold by a corporation incorporated and having its
principal place of business in another state. The child’s injuries
meet the amount in controversy requirement. The child’s mother
also has a state-law claim against corporation, but the amount of
her claim does not meet the amount in controversy requirement.
The federal court may exercise supplemental jurisdiction to hear
mother’s claim along with child’s claim.
(2) In a diversity case, a court may not exercise supplemental jurisdic-
tion over a claim by a plaintiff proposed to be joined under Rule 19
(compulsory joinder), or a claim by a plaintiff seeking to intervene
under Rule 24, when exercising supplemental jurisdiction would
destroy complete diversity of the parties [28 U.S.C. § 1367(b)].
(3) Moreover, in a diversity case a court may not exercise supplemental
jurisdiction over claims by plaintiffs against persons made party to
the suit under Rule 14 (impleader) 19, 20 (permissive joinder), or 24.
(4) If non-diverse parties are joined to a case, and there is no supple-
mental jurisdiction to hear their claims, the court may dismiss
only the claims of the non-diverse parties.
d. Even if the additional claim arises out of the same transaction or occur-
rence and the plaintiff is not trying to circumvent diversity jurisdiction,

8
CIVIL PROCEDURE

the court may (but is not required to) decline to exercise supplemental
jurisdiction in the following situations:
(1) the state law claim raises a novel or complex issue of state law;
(2) the state law claim “substantially predominates” over the claim
over which the district court has jurisdiction;
(3) the court has dismissed all the claims over which it had jurisdiction; or
(4) in “exceptional circumstances,” there are other “compelling
reasons” to decline jurisdiction.
4. Concurrent Jurisdiction
a. A federal court has concurrent jurisdiction over a particular case if the
case could also have been brought in state court.
b. When a court has concurrent jurisdiction to hear a case, both state and
federal laws or policies may apply, resulting in a conflict of laws.
5. Removal
a. A case that was originally brought in state court may be removed to
federal court if the plaintiff could have brought the case in federal court
[28 U.S.C. § 1441]. The defendant in such a situation has a right to
remove or shift that case from state court to federal court.
(1) Note that removal is only an option for defendants; plaintiffs cannot
remove a suit to federal court after having chosen state court.
b. If removal is based on diversity, all of the defendants must be diverse
from the plaintiff. Removal on the basis of diversity will not be granted if
any of the defendants is a citizen of the forum state.
(1) Under the fraudulent joinder rule, the right to removal will not be
defeated if a defendant was fraudulently joined for the purpose of
defeating diversity jurisdiction and preventing removal to federal
court [Marshall v. Manville Sales Corp., 6 F.3d 229 (4th Cir. 1993)].
(a) A party who claims fraudulent joinder has the burden of proving:
1) that there is no possibility the plaintiff would be able to
establish a cause of action in state court against the
in-state defendant; or
2) that there was outright fraud in the plaintiff’s pleading of
jurisdictional facts.
(2) The fraudulent joinder rule applies to defendants named in the
original complaint, as well as to those joined as parties any
time prior to removal [Mayes v. Rapoport, 198 F.3d 457 (4th Cir.
1999)]. In other words, if the removal is based on diversity, diver-
sity is needed both at the time the original claim is filed and at the
time when the notice of removal is filed.
c. The defendant has the right to remove when the plaintiff is suing the
defendant in a jurisdiction that is not the defendant’s home state.
However, if the plaintiff is suing the defendant in state court located
in that defendant’s home state, there is no risk of being “homered.”
Therefore, that defendant cannot remove [28 U.S.C. § 1441(b)(2)].

9
OUTLINE

d. If a plaintiff’s suit is based on federal law, the defendant may have


the case removed without a showing of diversity.
e. Removal is allowed only when at least one of the plaintiff’s claims
would fall within the subject-matter jurisdiction of the federal courts.
EXAMPLE: A files suit in her home state court against B, a citizen of
another state. A’s suit claims several violations of her home state’s un-
fair trade practice laws. Her suit also alleges breach of contract, with a
claim of $80,000 in damages. B may have the suit removed to federal
court, and the federal court will hear her unfair trade practice claims.
EXAMPLE: C, a New York resident files suit against D, also a New
York resident in a New York state court. C’s complaint seeks $80,000
in damages. Two weeks later D gets a new job and moves to Florida,
establishing residency and an intent to permanently live there. D may
not have the suit removed to federal court because diversity must
exist both at the time of filing and the time of removal.
f. If the basis for removal is federal question jurisdiction, and appended
to the federal question claim is a state claim not within the original
or supplemental jurisdiction of the district court or a claim that has
been made non-removable by statute (i.e., a diversity claim against a
defendant from the forum state or a claim as to which the defendant
did not join in or agree to the removable notice), the entire case is
removed to federal court [28 U.S.C. § 1441(c)].
(1) Once in federal court, the federal district court judge must sever
those non-removable pieces and remand them back to state court.
g. Multiple Defendants
(1) In a case involving multiple defendants, all defendants who have
been properly joined and served in the case must consent to or
join in removal for removal to be proper [28 U.S.C. § 1446(b)].
(2) Absent that unanimity, removal is improper.
h. Procedure
(1) In order to remove a case, a defendant must file a notice in the federal
district court in the district in which the action is currently pending.
(a) The defendant must include with the notice a copy of all
pleadings served on the defendant.
(b) The defendant must notify all adverse parties.
(c) The defendant must file a copy of the notice with the state
court, after which the state court must proceed no further,
unless and until the case is remanded.
(2) The federal court will consider whether removal is proper. If
removal is not proper, the court will remand the action to state court.
i. Timing
(1) Notice must be filed within 30 days of service of the initial
pleading. If a case is not removable when the initial pleading is

10
CIVIL PROCEDURE

filed, and the complaint is amended in a way that now makes


the case removable, a notice of removal must be made within
30 days of service of the amended pleading.
(a) Each defendant receives a 30-day period to decide
whether to remove to federal court. This is important with
respect to later-joined defendants.
EXAMPLE: In the original lawsuit, the plaintiff is suing two
defendants. Those two defendants have 30 days to decide
whether or not to remove. If they do not remove and the
plaintiff later joins a third defendant, that new defendant has
30 days to decide whether or not to remove. If that defendant
decides to remove, the first two defendants who originally said
they did not want to remove can join in the new request for
notice of removal and the entire case will be deemed timely
removed to federal court [28 U.S.C. § 1446(b)(2)(B), (C)].
(2) A case removable on the basis of diversity jurisdiction may not be
removed more than one year after the commencement of the action
(i.e., when the action was filed not served) [28 U.S.C. § 1446].
(a) However, if the plaintiff acted in bad faith so as to prevent the defen-
dant from removing, that one-year time period can be relaxed.
j. The venue of a removed case is the district court where the state
court is located.
k. Removal is allowed only for state court cases. Agency or
administrative proceedings cannot be removed to federal court.
l. If a plaintiff believes that a case was improperly removed to federal
court, she may bring a motion to remand the case to state court.
EXAMPLE: A consumer from State A brings a $100,000 products
liability action in state court against the manufacturer of a product
that injured her and the retailer from whom she bought the product.
The manufacturer is a corporation that is a citizen of State B, while
the retailer is incorporated in State C but has its headquarters in State
A. The manufacturer and retailer have the case removed to federal
court, based on diversity of citizenship and the amount in controversy.
The consumer may bring a motion to remand to state court, on the
grounds that retailer is not a diverse defendant.
(1) A motion to remand on the basis of any defect other than lack of
subject-matter jurisdiction must be made within 30 days after
the filing of the removal notice [28 U.S.C. § 1447(c)].
(a) Objections to subject-matter jurisdiction can never be
waived and therefore a motion to remand based on lack of
subject-matter jurisdiction is not subject to any time limit. If
at any time before the final judgment a court lacks subject-
matter jurisdiction, the case must be remanded [Id.].

11
OUTLINE

(2) The defendant has the burden of showing that the removal was proper.
6. Dismissal
a. The court must dismiss an action if it determines that it does not have
subject-matter jurisdiction over the case [Fed. R. Civ. P. 12(h)(3)].
b. Lack of subject-matter jurisdiction may not be waived by the parties.
7. Lawsuit Based on Invalid Law
a. A lawsuit cannot be filed based on an invalid law. If an action has
been removed to federal court and the court determines that this is
the case, it must either be dismissed or remanded back to state
court for an amended complaint.
b. The federal court cannot consider any pleading subsequent to the removal
petition. If the action is remanded and an amended complaint is filed, the
defendants can file to remove the case to federal court a second time.

B. Territorial Jurisdiction
1. Territorial jurisdiction is the authority of a court to bind a party to the action.
There are three essential ways to establish territorial personal jurisdiction:
a. a federal court has territorial jurisdiction over the state in which that
federal district is located;
b. a federal statute that creates a cause of action may provide that
federal courts have nationwide jurisdiction; or
c. the “100 mile bulge” rule states that a federal court may exercise
jurisdiction over a defendant outside the state in which the court is
located if the defendant is joined under Rule 14 or 19 and is served
within a judicial district of the United States not more than 100 miles
from where the summons was issued [Fed. R. Civ. P. 4(k)].
(1) In order for jurisdiction to be proper under this rule, the defendant
must have the type of minimum contacts with the “bulge” that
would support personal jurisdiction for a court in the “bulge.”

C. Personal Jurisdiction
1. Personal jurisdiction refers to a court’s authority over a defendant.
Personal jurisdiction must be established separately for each defendant.
There are three types of personal jurisdiction:
a. in personam jurisdiction over the parties;
b. in rem jurisdiction over the property that is the subject of the action; and
c. quasi in rem jurisdiction over property that is attached to satisfy the
judgment in an action.
2. In personam jurisdiction is jurisdiction over the parties to a lawsuit. The
defendant typically is physically present in the forum state, or has some
significant contacts with the state. To determine whether a federal court has
personal jurisdiction over a particular party, the personal jurisdiction law of
the state in which the federal court sits must be satisfied and the exercise
of personal jurisdiction must comply with the United States Constitution.

12
CIVIL PROCEDURE

a. State Long-Arm Statute


(1) For a federal court to have personal jurisdiction over a
defendant, the state court in which the federal court sits must
be able to exercise personal jurisdiction over that defendant.
(a) If the state court could not assert personal jurisdic-
tion, then the federal court may not assert personal
jurisdiction either.
(2) A state court may exercise personal jurisdiction over an out-of-
state defendant if authorized by the state’s long-arm statute.
Long-arm statutes place limitations on the state’s ability to
establish personal jurisdiction by setting out the types of
contacts that will support jurisdiction. Typically, the types of
contacts that establish personal jurisdiction include the following:
(a) the defendant transacts business within the state;
(b) the defendant contracts to supply goods and services
anywhere within the state; or
(c) the defendant owns, uses, or possesses any real property
within the state.
b. Constitutional Considerations
(1) Just because state law permits personal jurisdiction over a
particular party does not mean that the federal court may
automatically assert it. The federal court must also determine
whether the state law is constitutional under the Due Process
Clause of the Fourteenth Amendment.

NOTE Many states authorize personal jurisdiction to the full extent of the Fourteenth
Amendment. If a state does that, all you need to do is determine whether
personal jurisdiction over a particular defendant is permissible under the
Constitution. If it is, then both state law and constitutional law are satisfied.

(2) A state law will be constitutional if it authorizes personal jurisdic-


tion in one of five following circumstances:
(a) residency;
(b) consent;
(c) service;
(d) minimum contacts; or
(e) substantial business.
(3) Residency
(a) If the defendant is domiciled in the forum state, personal
jurisdiction over the defendant is constitutional. A person is
domiciled in a jurisdiction if she:
1) resides in that jurisdiction; and
2) has expressed her intent to remain in that jurisdiction
indefinitely.

13
OUTLINE

(4) Consent
(a) If the defendant consents to the federal court’s personal
jurisdiction over him, personal jurisdiction is constitutional.
1) A person may expressly consent to personal jurisdiction
by agreement with the other party. For example, parties
to a contract often put a choice-of-forum clause in their
contracts that specifies the forum in which any lawsuit
arising from the contract must be litigated. If a defendant
signs a contract with a choice of forum clause, she has
consented to the power of the court specified in the clause.
EXAMPLE: A man obtains a line of credit from the
bank. The credit agreement provides that all disputes
relating to the agreement will be heard by the federal
district court in Delaware. The man has consented to
personal jurisdiction in Delaware.
2) A state may provide by law that a non-resident has consented
to personal jurisdiction in that state if the non-resident
engages in a particular activity that the state has a substan-
tial interest in regulating, such as driving on public roads.
EXAMPLE: A non-resident who drives a car in
another state has given his implied consent to
personal jurisdiction in that state, for suits arising out
of his operation of a car in the state.
3) A party can also consent to federal jurisdiction by waiving
her objection to personal jurisdiction. If a defendant
appears in court without making a motion to dismiss for
lack of personal jurisdiction, or does not include lack
of personal jurisdiction in a responsive pleading, the
defense is waived [Fed. R. Civ. P. 12(h)(1)]. The defendant
must object to personal jurisdiction in its initial filing or
appearance before the court. A defendant does not waive
objections to lack of personal jurisdiction by making an
appearance before the court to contest jurisdiction.
(5) Service
(a) If a defendant is physically present in the jurisdiction, and
is served with process while present, the court’s exercise of
personal jurisdiction over the defendant is constitutional.
1) It does not matter that the defendant’s presence in the juris-
diction is temporary, or that it is unrelated to the lawsuit.
2) The defendant’s presence must be voluntary, the
plaintiff cannot have coaxed the defendant into the
state under false pretenses. Moreover, personal
jurisdiction would not be constitutional if the defendant
is in the forum state to participate in a legal proceeding.

14
CIVIL PROCEDURE

EXAMPLE: A businessman is a resident of Missouri.


While driving to Louisiana on a business trip, he stops
for lunch at a restaurant in Arkansas. A process server
sees him in the restaurant, and serves him with a
summons and complaint for an action in federal district
court in Arkansas. The court has jurisdiction over the
businessman, based on service.
EXAMPLE: A singer, a resident of Maryland, receives a
telephone call telling her that she has won a prize in a raf-
fle. In order to pick up that prize, she is told that she must
go to the raffle sponsor’s headquarters in Virginia. When
the singer gets to the address she is given, she learns
that there is no raffle prize. Instead, she is served with a
summons and complaint. The court in Virginia does not
have jurisdiction over the singer based on service.
(6) Minimum Contacts
(a) For jurisdiction to be constitutional under a “minimum
contacts” analysis, three conditions must be satisfied:
1) the defendant must have established a minimum
contact with the forum state;
2) the claim against the defendant must be related to
that contact; and
a) A lawsuit is related to a defendant’s contacts if
the contacts played a role in causing the lawsuit.
3) the exercise of jurisdiction must not offend traditional
notions of fair play and substantial justice.
(b) A defendant has minimum contacts with a particular state
if he causes harm in the state, does business in the state
or has an interest in real property in the state. Importantly,
the defendant does not have to be in the state to cause the
harm, do the business or have a property interest, but his
contact with the state has to be purposefully established.
(c) Therefore, for a plaintiff to establish a minimum contact,
the defendant must “purposefully avail” itself to the privilege
of conducting activities within the forum state [World-Wide
Volkswagen Corp. v. Woodson, 444 U.S. 286 (1980)].
1) If a defendant purposefully availed itself of the protec-
tions and benefits of the forum state, sufficient minimum
contacts for personal jurisdiction will exist. Purposeful
availment involves actions that are directed towards a
particular state. A defendant who purposely involves
himself in transactions within the state receives the bene-
fits and privileges of that state’s laws, so it is fair that he
be exposed to the jurisdiction of that forum state as well.

15
OUTLINE

2) By purposefully availing itself, a party is deemed to


have reasonably foreseen the possibility of being haled
into court in that state to defend an action.
3) Merely placing a product into the stream of commerce
is not enough to establish purposeful availment; the
defendant must also have had additional contacts with
the forum state [Asahi Metal Industry Co. v. Superior
Court, 480 U.S. 102 (1987)].
(d) If a defendant has minimum contacts with the forum state,
the defendant bears the burden of proving that it would be
fundamentally unfair for him to litigate in that state in order
to avoid the court’s personal jurisdiction [Burger King v.
Rudzewicz, 471 U.S. 462 (1985)].
(e) Courts consider the following five factors when deter-
mining whether it is fair to require the defendant to liti-
gate in the forum state [World-Wide Volkswagen Corp. v.
Woodson, 444 U.S. 286 (1980)]:
1) the burden on the defendant;
2) the interests of the forum state;
3) the plaintiff’s interests in obtaining relief;
4) the interstate judicial system’s interest in efficiency; and
5) shared policy interests of the states.
(f) Generally, the burden on the defendant drives the analysis
and must be quite substantial for a court to reject personal
jurisdiction on the basis of fairness.
EXAMPLE: D, a resident of North Carolina, was driving
through Rhode Island on his way to Boston when he struck
P with his car. In a subsequent suit in Rhode Island, D is
subject to personal jurisdiction because he caused harm in
the state, the lawsuit arises from those contacts, and Rhode
Island has an interest in protecting its citizens from negligent
drivers and maintaining order on its roads.
EXAMPLE: P heard from a friend that D, a company in
Nebraska, makes great custom tailgates for pick-up trucks.
P, a resident of Kansas, called D and asked to order one.
D does all its business in Nebraska but agreed to make
P a tailgate and send it to him. P received the tailgate but
couldn’t install it because it was designed incorrectly. P can
sue D in Kansas because it did business in the state, the
lawsuit arose from that activity, and Kansas has an interest
in protecting its citizens from breaches of contract.
EXAMPLE: P, a citizen of Texas, was driving in Connecticut
when she got into a car accident with D. D is a Connecticut
citizen, but spends six weeks each summer in Texas with his

16
CIVIL PROCEDURE

grandmother (and has been doing so for the past 10 years). P


cannot sue D in Texas because his contacts with the state did
not give rise to the lawsuit. (P could, however, wait until D comes
back into the state next summer and have him served then.)
EXAMPLE: Motor MFG. manufactured an electric motor
in Michigan and sold it to Golf Cart Co. in Ohio who then
installed it in a golf cart. P, a resident of Georgia, purchased
the cart over the phone, and Golf Car Co. then shipped it from
Ohio to P at home. P was later injured when the cart’s electric
motor caught fire and he wants to bring suit in Georgia. Golf
Cart Co. will be subject to personal jurisdiction in Georgia
because it transacted business in the state or caused harm
in the state, the suit arises from those acts, and fair play
concerns are not implicated. In contrast, Motor MFG will not
be subject to personal jurisdiction in Georgia because it did
not purposefully establish any contacts with the state that
gave rise to P’s suit. Even if Motor MFG had sent other motors
to the state in the past, personal jurisdiction is not permissible
because those contacts did not give rise to P’s claim.
(7) Substantial Business
(a) A federal court can constitutionally assert personal
jurisdiction over a defendant if it is doing “substantial
business” in a particular state. The Supreme Court has
made clear that “substantial business” is a very high bar to
clear. There is no easy definition for “substantial business”
but recent opinions by the Supreme Court instruct that the
business activity in a particular state must be so significant
that the company is “essentially at home” in that state.
(b) A corporation would probably not be considered “essentially
at home” and thus subject to general jurisdiction in a state
in which it is not incorporated nor has its principal place of
business [Daimler AG v. Bauman, 571 U.S. 117 (2014)].
(c) As a corollary to this, the Court has suggested that a company
cannot be “essentially at home” in more than a couple of states.
EXAMPLE: P was vacationing in Utah when she slipped
and fell in a 7-11 convenience store. After returning home
to Arizona, P desired to bring suit against 7-11 in Arizona.
7-11 operates 131 stores in the state, roughly 8% of its total
operations. 7-11 is not subject to personal jurisdiction in
Arizona because its business, whether substantial or not, is
not the sort that renders it “essentially at home” because if
that was so, it would be essentially at home in all 50 states.
(d) The Supreme Court has recently limited the scope of
specific personal jurisdiction in this area as well. In order for

17
OUTLINE

a court to exercise specific jurisdiction over a claim, there


must be a connection between the forum and the underlying
controversy—in essence, an activity or an occurrence—
that takes place in the forum state [Goodyear Dunlop Tires
Operations, S.A. v. Brown, 564 U.S. 915 (2011)].
1) When there is no such connection, specific jurisdiction is
lacking regardless of the extent of a defendant’s uncon-
nected activities in the forum state [Bristol-Myers Squibb
Co. v. Superior Court of California, 137 S. Ct. 1773 (2017)].
2) Even regularly occurring sales of a product in a state
do not justify the exercise of jurisdiction over a claim
unrelated to those sales [Goodyear Dunlop Tires
Operations, S.A. v. Brown, 564 U.S. 915 (2011)].
3. In rem jurisdiction is based on the interests of a particular piece of
property or thing located within the forum state.
EXAMPLE: The courts of a state have in rem jurisdiction to hear an
action to quiet title to a parcel of real estate in the state.
a. A federal court may exercise in rem jurisdiction when:
(1) there is real or personal property of value located within the
state where the federal district is located;
(2) the court seizes the item; and
(a) A court’s in rem jurisdiction cannot begin until the court has
effective control of the asset.
(3) the owner of the property received proper notice of the proceeding.
b. A defendant who is no longer domiciled in the forum state may be
subject to in rem jurisdiction if:
(1) the cause of action is of a domestic nature; and
(2) the forum state was the place of matrimonial domicile, the defendant
abandoned the plaintiff in the forum state, or an agreement or
settlement for support was entered into in the forum state.
4. A federal court has quasi in rem jurisdiction in actions in which the
attachment of real or personal property is part of the relief requested.
The federal court located in the state in which that property is situated
has quasi in rem jurisdiction over the property being attached.
a. The following elements are required for quasi in rem jurisdiction:
(1) there is real or personal property of value located within the
state or territorial limits of the federal court;
(2) the defendant owns the property;
(3) the property is being attached or seized; and
(4) the defendant has been provided proper notice of the proceedings.
b. The court may not exercise quasi in rem jurisdiction unless there are
minimum contacts with the forum state or territory [Shaffer v. Heitner,
433 U.S. 186 (1977)].

18
CIVIL PROCEDURE

(1) If the dispute concerns the parties’ rights in the property itself,
the presence of the property in the state will provide sufficient
minimum contacts.
(2) The minimum contacts requirements for quasi in rem jurisdiction
are similar to those for in personam jurisdiction.
(3) The minimum contacts requirements make quasi in rem jurisdiction
a rarely used gap filler. It is used only when the constitutional require-
ments (minimum contacts, and a finding that the exercise of jurisdic-
tion does not offend traditional notions of fair play and substantial
justice) are satisfied, but in personam jurisdiction is unavailable
because the long-arm statute does not authorize jurisdiction.
5. Judgment Entered without Personal Jurisdiction
a. If the court entered a judgment against a defendant over whom it did
not have constitutional personal jurisdiction, that judgment is void and
not entitled to Full Faith and Credit by another court.
b. However, a judgment has preclusive effect on future litigation if the
defendant in the first case had the opportunity to challenge personal
jurisdiction and either:
(1) waived the personal jurisdiction challenge; or
(2) made the personal jurisdiction challenge and lost.

D. Service of Process and Notice


1. Commencement of an Action
a. A civil action in federal court is commenced by filing the complaint
with the court.
b. After the complaint is filed, the plaintiff presents a summons to the
clerk for signature and seal. If the summons meets the requirements
for format of a summons, the clerk certifies it and issues it to the
plaintiff for service on the defendant.
c. A summons is a court document that informs the defendant that the
complaint has actually been filed and that he must respond to it, or
else be held in default. The process served to compel the attendance
of witnesses is a subpoena [Fed. R. Civ. P. 45].
d. The formal requirements for a summons are set out in Rule 4, which
requires a summons:
(1) be signed by the clerk;
(2) identify the court and the parties;
(3) be directed to the defendant;
(4) state the name and address of either the plaintiff’s attorney or
the plaintiff itself, if unrepresented;
(5) notify the defendant of:
(a) the time period within which it must appear or file an
answer; and

19
OUTLINE

(b) the potential for a default judgment if the defendant fails to


appear within the time specified; and
(6) contain the seal of the court.
2. Service of Process
a. The plaintiff is charged with service of process.
b. A summons must be served with a copy of the complaint. Unless
both the complaint and summons are served on the defendant at
the same time, process has not been completed.
(1) The plaintiff is responsible for any insufficiency of either the
service of process or the process itself.
c. Service of process must occur within 90 days of filing of the
complaint [Fed. R. Civ. P. 3, 4(m)].
(1) If the plaintiff does not timely serve the defendant with process,
the court must dismiss the suit without prejudice unless the
plaintiff can show good cause why service was not timely.
d. Service may be effected by any person who is:
(1) at least 18 years of age; and
(2) not a party to the suit.
e. How Process is Served
(1) Process can be served on the defendant (whether a natural
person or corporation, partnership, or association) by following
the manner prescribed by the state court in the state where the
federal suit has been filed or the state court in the state where
the defendant will be served [Fed. R. Civ. P. 4(e)(1)].
(2) If the defendant is a natural person, process can also be
served by doing any of the following [Fed. R. Civ. P. 4(e)(2)]:
(a) personally delivering the process to the defendant himself,
wherever he maybe;
(b) leaving process at the defendant’s usual place of abode with
a person of suitable age and discretion who resides there; or
(c) serving the defendant’s registered agent.
(3) If the defendant is a corporation, partnership, or associa-
tion, process can also be served by delivering a copy of the
summons and the complaint to [Fed. R. Civ. P. 4(h)]:
(a) an officer;
(b) a managing agent or general agent; or
(c) any other agent authorized by appointment or by law to
receive service of process. Note that if the agent is one autho-
rized by law and the law requires service by mail, process
must be served by personal delivery to the agent and by mail.
f. Waiver of Service
(1) A defendant may also waive service of process, upon request of
the plaintiff. A waiver allows the plaintiff to save the costs of service.

20
CIVIL PROCEDURE

(2) Waiver of service is not a waiver of any objections the


defendant may have regarding the court’s subject-matter or
personal jurisdiction over the suit or as to the venue of the suit.
(3) A request for waiver of service of process must:
(a) be in writing and addressed to the individual defendant;
1) If the defendant is a corporation, partnership, or
association, the request is addressed to an officer,
manager, or authorized agent.
(b) be sent via first-class mail or other reliable means;
(c) be accompanied by a copy of the complaint;
(d) identify the court where the complaint was filed;
(e) inform the defendant of the consequences of waiving and
not waiving service;
(f) state the date on which the request was sent;
(g) allow the defendant a reasonable time to return the waiver;
1) at least 30 days after the request was sent; or
2) at least 60 days after the request was sent for defendants
located outside any judicial district of the United States; and
(h) provide the defendant with an extra copy of the request, as well
as prepaid postage or other means of compliance in writing.
(4) Upon request of the plaintiff, a defendant has a duty to waive
service unless the defendant is:
(a) a minor or incompetent person;
(b) the United States or an agency of the United States; or
(c) a state, local, or foreign government.
(5) A defendant who agrees to waive service need not answer
the complaint until 60 days after the waiver request. If the
complaint was sent to the defendant outside of any U.S.
judicial district, the defendant does not need to answer until
90 days after the waiver request.
(6) If a defendant located in the United States fails without good
cause to sign and return a waiver requested by a plaintiff located
within the United States, the court may impose on the defendant:
(a) expenses incurred in making service; and
(b) reasonable expenses, including attorney’s fees, of any
motion required to collect service expenses.
(7) Sanctions will not be imposed if the defendant had good cause
for failure to waive.
(a) “Good cause” does not include a belief that the complaint is
unfounded, or that the action was filed in the wrong court.
(b) Failure to receive the waiver request would constitute good cause.
g. If a plaintiff fails to effect service according to the rules, the case
should be dismissed for insufficiency of service of process.

21
OUTLINE

(1) A defendant may raise objections to service of process in a


motion to dismiss [Fed. R. Civ. P. 12(b)].
(2) Failure to raise the objection to insufficiency of service of
process in a pre-answer motion or in the answer waives the
defendant’s right to object.
(3) A court will dismiss an action for insufficient service of process
even if the defendant received actual notice of the action by
some means other than service of process.

E. Venue, Transfer, and Forum non Conveniens


1. Venue
a. While subject matter jurisdiction dictates whether a suit can be
brought in the federal court system, and personal jurisdiction
informs the plaintiff what state the suit can be brought in, venue
dictates the region within a particular state (known as a judicial
district) where a suit can occur.
b. The venue of an action is the federal court in which the action is filed.
c. In any action venue is proper in [28 U.S.C. § 1391]:
(1) the district in which any defendant resides, if all of the
defendants reside in the same state;
EXAMPLE: Plaintiff, a resident of Minnesota, brings a diversity
action against Defendant A, a resident of Madison, Wisconsin
(located in the Western District of Wisconsin), and Defendant B, a
resident of Milwaukee (located in the Eastern District of Wiscon-
sin). Venue is appropriate in either the Eastern or Western District.
(2) the district in which a substantial part of the events or
omissions giving rise to the claim occurred or the district in
which a substantial part of the property that is the subject of
the action is located; or
EXAMPLE: Plaintiff sues Defendant, a recording artist, for
copyright infringement for recording one of Plaintiff’s songs
without permission. The song was recorded at a studio in
Nashville. Venue is proper in the Middle District of Tennessee,
the district in which Nashville is located.
(3) the district in which any defendant would be subject to
personal jurisdiction, if there is no other district in which the
action could be brought.
d. Venue is also considered proper where a party either consents to the
venue, consents to personal jurisdiction in the venue, or waives his
objection by failing to raise it.
e. The court may raise the issue of improper venue sua sponte.
f. For purposes of venue, a defendant’s place of residence is
determined according to the following rules [28 U.S.C. § 1391(c)]:

22
CIVIL PROCEDURE

(1) Residence of Natural Persons


(a) Natural persons are deemed to reside where they
are domiciled.
(b) This applies to aliens who are admitted to permanent
residence in the United States. They are considered to
reside, for venue purposes, where they are domiciled.
(2) Residence of Entities
(a) If the entity is a plaintiff, venue is proper where that
plaintiff maintains its principal place of business.
(b) If the entity is a defendant, venue is proper where that
defendant is subject to personal jurisdiction.
(c) If a corporation resides in a state that has multiple districts,
venue is proper in the district that would have had personal juris-
diction over that corporation had that district been its own state.
(3) Non-Resident Aliens
(a) Venue over non-resident aliens is proper in any judicial district.
(b) In multi-defendant cases, we disregard non-resident aliens
when computing venue.
2. Transfer of Venue
a. If an action was brought in a particular venue, a federal court still
may transfer the action to any other district in which it might have
been brought, “for the convenience of parties and witnesses, in the
interest of justice” [28 U.S.C. § 1404(a)]. The court does not dismiss
the action so that the plaintiff may refile, but instead transfers the
action to the more appropriate judicial district.
(1) Courts usually give substantial deference to the plaintiff’s
choice of forum. The defendant bears the burden of proving that
an adequate alternative forum exists, and that the plaintiff’s
choice of forum is significantly inappropriate.
(2) The burden is on the party making a motion to transfer to show that:
(a) the convenience of the parties and witnesses, the ease of access
to proof, calendar congestion, and other factors favor transfer; and
(b) the court to which the action will be transferred has
personal and subject-matter jurisdiction over the suit.
(3) When an action filed in a proper venue is transferred to a
different court, the substantive law applied should be the same
law that would have been applied by the transferor court.
EXAMPLE: Plaintiff, a resident of Oregon, brings a diversity ac-
tion for wrongful termination of employment against Defendant, a
corporation incorporated in Delaware but located in Washington,
in U.S. District Court in Delaware. Defendant’s only physical
location is in Washington, and it has no assets or physical pres-
ence in Delaware. All of the records relating to Plaintiff’s employ-

23
OUTLINE

ment and termination are in Washington, and all of the witnesses


who might be called to testify are in either Washington or Oregon.
The court may transfer Plaintiff’s action under Rule 1404.
b. If a case is brought in an improper venue, a federal court “shall
dismiss, or if it be in the interest of justice, transfer such case to any
district…in which it could have been brought” [28 U.S.C. § 1406(a)].
(1) Transfers due to improper venue are appropriate only if the
action could originally have been brought in the transferee court.
(2) In an action transferred under § 1406, the action never
“belonged” in the transferor court. The transferee court will
therefore apply its own conflict of laws principles to determine
the applicable substantive law.
3. Forum non Conveniens
a. The doctrine of forum non conveniens allows a court to decline to
exercise its jurisdiction and dismiss an action if the court where the
action was brought would be a seriously inconvenient forum and an
adequate alternative forum exists in a foreign country.
(1) A federal court does not have the power to transfer a case to a
foreign country, so the case will be dismissed. The plaintiff may
then file his action in the alternative forum.
(2) Courts may give substantial weight to the possibility of any
change in substantive law as a result of bringing suit in
the foreign forum if the remedy is so clearly inadequate or
unsatisfactory as to be no remedy at all.
(3) Any deference given to the plaintiff’s forum carries less weight
when the plaintiff is from a foreign country.
b. Once the court has determined that an adequate alternative forum
exists, it will weigh the public and private interests to determine whether
to dismiss an action under the doctrine of forum non conveniens.
(1) Public interests include the interests in:
(a) resolving disputes locally and not burdening local courts
with distant disputes;
(b) avoiding application of foreign law, if the case will require
applying the law of another jurisdiction; and
(c) avoiding the imposition of jury duty for a case that would not
have a substantial impact on the potential jurors’ community.
(2) Private interests include:
(a) ease of access to evidence;
(b) cost of witness attendance at trial;
(c) availability of compulsory process; and
(d) any other factors that might lessen the time and expense of trial.

24
CIVIL PROCEDURE

II. LAW APPLIED BY FEDERAL COURTS

A. State Law in Federal Court


1. Rules of Decision Act
a. The Rules of Decision Act states that applicable provisions of the
federal Constitution, treaties, and constitutional statutes enacted by
Congress always take precedence over state law [28 U.S.C. § 1652].
b. The federal courts apply federal law when considering issues involving:
(1) the U.S. Constitution; and
(2) constitutional federal statutes.
2. The Erie Doctrine
a. The Erie doctrine states that a federal court that hears a state law
claim in a case based on diversity or supplemental jurisdiction must
apply the substantive law of the state in which the court sits [Erie
R.R. Co. v. Tompkins, 304 U.S. 64 (1938)].
(1) Under Erie, state law applies if it is a law that deals with the
substantive rights of state citizens. Federal courts use federal,
not state, procedural rules.
b. The Erie doctrine applies only where the following are both present:
(1) the court has subject-matter jurisdiction over a case, based on
diversity or supplemental jurisdiction; and
(2) the state law that would apply conflicts with the federal rule,
statute, doctrine, or procedure at issue.
c. If a valid federal statute or constitutional provision is on point, the
federal court must apply the relevant provision; it is irrelevant whether
the provision is substantive or procedural.
(1) The federal rule will not apply if the federal rule abridges, modi-
fies, or enlarges a substantive right.
(2) If there is a valid federal rule promulgated under the Rules
Enabling Act dealing with an issue, that rule is applied [28
U.S.C. § 2071, et seq.].
d. If there is no valid federal statutory or constitutional law on point, the
federal court will follow its ordinary practices unless doing so would
lead to either a preference for one court system over another (forum
shopping) or fundamental unfairness.

NOTE This part of the Erie analysis (when there is no valid federal statute or constitution
provision on point) is really the substantive versus procedure law distinction. If this
inquiry instructs the court to apply state law, the state law is called “substantive”
regardless of its true nature. If this inquiry instructs the court to stick with ordinary
federal practice, that practical is called “procedural,” regardless of its true nature.
e. Although the rules discussed above are the steps to apply in an Erie
analysis, there is general agreement that certain substantive state laws
should be applied in federal diversity cases.

25
OUTLINE

(1) Federal courts in a diversity action will apply the statute of limita-
tions of the state in which the court sits.
(2) Federal courts in a diversity action will apply the choice-of-law
rules of the state in which the court is located.
f. On the other hand, the Supreme Court has held that the right to a jury
trial in federal courts is to be determined as a matter of federal law.
The court held that “[i]n diversity cases, the substantive dimension of
the claim asserted finds its source in state law, but the characteriza-
tion of that state-created claim as legal or equitable for purposes of
whether a right to jury trial is indicated must be made by recourse to
federal law” [Simler v. Conner, 372 U.S. 221 (1963)]. The reason for
this was to ensure that the right to a jury trial, as guaranteed by the
Seventh Amendment, was exercised uniformly.
3. Uncertain State Law
a. If the federal court is uncertain about the state law to be followed, or if
the law is unclear or unsettled, the court may:
(1) abstain from hearing the state law claim;
(2) if state law allows, certify the question to the state courts and
obtain a ruling on the issue; or
(3) try to predict how the state courts would rule on the issue, using
all available and relevant resources.
b. Federal circuit courts of appeal do not give deference to district
courts’ interpretations of unsettled state law, but review those deci-
sions de novo.

B. Federal Common Law


1. In General
a. Under the Erie doctrine, there is no general federal common law.
Courts apply the laws of the states in which they are located.
b. Federal common law now exists to interpret Congressional intent, or
the meaning of statutes.
(1) Congress has made broad rules, with unclear or indefinite stan-
dards, for courts to interpret.
EXAMPLE: Federal law provides that the “fair use” of copyright-
protected material is not infringement; however, the statute does
not define “fair use” [17 U.S.C. § 107]. The courts have thus been
left to develop the parameters of the fair use defense.
(2) Congressional action will take precedence over common law rules.
EXAMPLE: The Clean Air Act gives the Environmental Protection
Agency authority to regulate greenhouse gases. The statutory
grant of authority precludes federal common law nuisance claims
against producers of those gases [American Elec. Power Co. v.
Connecticut, 564 U.S. 410 (2011)].

26
CIVIL PROCEDURE

2. Federal or Constitutional Interests


a. Federal courts may make federal common law when [Clearfield Trust
Co. v. U.S., 318 U.S. 360 (1943)]:
(1) federal or constitutional interests are at stake;
(2) Congress had inadequately addressed the situation concerned; and
(3) the application of individual state laws in various jurisdictions
would create unacceptable levels of diversity or uncertainty.
b. Federal common law rules are adopted when Congress has given
courts the power to make common law rules, or when Congress has
been silent on a question involving federal law.
EXAMPLE: Punitive damages are available in federal maritime cases
when the owner of a vessel fails to pay maintenance and cure to an
injured seaman. The damages are available as a matter of federal
common law, because Congress has not acted to prohibit such dam-
ages [Atlantic Sounding Co. v. Townsend, 557 U.S. 404 (2009)].
c. Generally, federal common law is confined to the following fields of law:
(1) maritime law;
(2) foreign relations;
(3) commercial rights and liabilities of the federal government; and
(4) property rights and liabilities of the federal government.

27
OUTLINE

ERIE DOCTRINE

A. Is the subject matter of the


YES Erie does not apply
case or claim based on
federal question jurisdiction

NO
Follow both
B. Is there a state-stipulated C. Can both procedures be procedures
procedure and a federally YES
followed simultaneously?
YES simultaneously;
stipulated procedure? Erie does not
apply

NO NO

Erie does not apply D. What is the source of


the federally stipulated
procedure?

The U.S. Federal Statute Federal Rules of Federal Practice Federal


Constitution that is Civil Procedure or Policy Common Law
constitutional

Use federal Use federal


Does the federal Is the federal common
procedure procedure Does the federal
rule abridge, law in one of the
practice conflict
modify, or following areas
with a state
enlarge a recognized as suitable
procedure?
substantive because a significant
right? national interest is
involved:
• The contractual powers
of the United States
• Admiralty and maritime
YES NO YES disputes
• International relations
Apply Apply Follow
state federal federal
rule rule practice
NO
NO
Use balancing
test: Is the state Apply
law at issue state
YES
integral to the procedure
substantive
rights and
obligations at E. If the state and federal
issue? procedures are still in
conflict:
• If the right is
primarily
YES substantive in
nature: apply the
Apply state procedure.
state • If the right is
law primarily procedural
in nature: apply the
federal procedure.

NO

Weigh the application of the state


law against the countervailing
federal considerations and consider
the likelihood of a different outcome
if the federal law applies.

28
CIVIL PROCEDURE

III. PRETRIAL PROCEDURES

A. Pleadings
1. Types of Pleadings
a. The following types of pleadings are allowed under the Federal
Rules of Civil Procedure:
(1) complaints;
(2) answers to complaints;
(3) answers to counterclaims;
(4) answers to cross-claims;
(5) third-party complaints;
(6) answers to third-party complaints; and
(7) replies to answers, if ordered by the court.
2. Rules of Pleading
a. To initiate a lawsuit, the first step is filing a complaint.
b. A pleading that states a claim for relief (including a complaint), must
contain [Fed. R. Civ. P. 8]:
(1) a short and plain statement of the grounds for the court’s jurisdiction;
(2) a short and plain statement of the claim showing that the pleader
is entitled to relief; and
(a) The “short and plain statement of the claim” must include
“sufficient factual matter” to state a plausible claim that is
more than a “sheer possibility that a defendant has acted
unlawfully” [Bell Atlantic Corp. v. Twombly, 550 U.S. 544
(2007); Ashcroft v. Iqbal, 556 U.S. 662 (2009)].
(3) a demand for the relief sought.
c. A pleading may make inconsistent claims or defenses. The court will
allow inconsistent pleadings to be determined by the trier of facts.
EXAMPLE: A sues B, claiming that B’s dog bit him. B’s answer denies
that A was bitten. The answer also claims that A was bit because he
provoked the dog.
d. A party may set out two or more statements of a claim or defense alternatively
or hypothetically, either in a single count or defense or in separate ones.
EXAMPLE: C is hit by a car driven by D. C sues D, alleging that D neg-
ligently hit C. C also alleges that D intentionally hit her with his car.
e. Generally, pleadings in federal court are to be liberally construed.
(1) Courts are directed to ignore technical defects in pleadings, if no
substantial right of the parties is prejudiced.
(2) Note that the Twombly and Iqbal cases referenced supra set out
heightened standards for the allegations in pleadings that may
affect how pleadings are construed.

29
OUTLINE

f. Parties must plead claims or defenses in a simple, direct, and concise manner.
(1) No technical pleading forms are necessary.
(2) Although parties should use general terms and omit evidentiary
material, a pleading must contain “sufficient factual matter” to set
out a plausible claim for relief.
g. Some cases involve special matters that must be pleaded with particu-
larity [Fed. R. Civ. P. 9]. Those special matters are:
(1) capacity or authority to sue, if required to show the court has jurisdiction;
(2) fraud, mistake, or condition of the mind;
(3) conditions precedent, but only when denying that the condition
has occurred;
(4) time and place, when testing the sufficiency of a pleading; and
(5) special damages.
EXAMPLE: A sues B for injuries sustained in a traffic accident. A al-
leges that B’s negligence caused her to incur medical bills, and to lose
wages because she was unable to work. A must plead with specificity
the amount of lost wages and the amount of her medical bills.
3. Responsive Pleading
a. An answer is a pleading in which the responding party admits or denies
the opposing party’s allegations and lists any defenses he might have.
(1) The responding party must admit those allegations, or parts of alle-
gations, that are true, and deny the others [Fed. R. Civ. P. 8(b)].
(2) A general denial is appropriate only when the responding party
intends in good faith to deny all the allegations in the pleading.
(3) If the defendant fails to deny an allegation, it is deemed admitted.
b. A party must plead certain affirmative defenses in its answer or reply to
a counterclaim.
(1) An affirmative defense is a defense that relies on factual issues
not presented in the complaint. It does not necessarily deny the
allegations of the complaint, but pleads additional facts.
(2) Affirmative defenses that must be pleaded in a responsive
pleading include:
(a) accord and satisfaction;
(b) arbitration and award;
(c) assumption of risk;
(d) contributory negligence;
(e) duress;
(f) estoppel;
(g) failure of consideration;
(h) fraud;
(i) illegality;
(j) injury by fellow servant;

30
CIVIL PROCEDURE

(k) laches;
(l) licenses;
(m) payment;
(n) release;
(o) res judicata;
(p) Statute of Frauds;
(q) statute of limitations; and
(r) waiver.
(3) Affirmative defenses that raise objections based on jurisdiction or
procedural matters may be raised either in a responsive pleading,
or in a pre-answer motion.
(a) If an objection will be raised by a motion, rather than in a
responsive pleading, the motion must be made before a
responsive pleading is made [Fed. R. Civ. P. 12(b)]. These
motions are therefore known as pre-answer motions.
1) A motion to dismiss, a motion for a more definite state-
ment, and a motion to strike are all pre-answer motions
that must be made before a responsive pleading is made.
(b) Objections that may be raised either by a pre-answer motion
to dismiss or responsive pleading are the following:
1) lack of subject-matter jurisdiction;
2) lack of personal jurisdiction;
3) improper venue;
4) insufficient process;
5) insufficient service of process;
6) failure to state a claim upon which relief can be granted; and
7) failure to join a party under Rule 19.
(4) A motion under this rule may be joined with any other motion allowed
by this rule. However, except as provided in Rule 12(h)(2) or (3), a
party that makes a motion under this rule must not make another
motion under this rule raising a defense or objection that was available
to the party but omitted from its earlier motion [Fed. R. Civ. P. 12(g)].
(5) Failure to raise an objection in an answer or pre-answer motion
to dismiss constitutes a waiver of these defenses with the excep-
tion of objections to subject-matter jurisdiction, failure to state a
claim upon which relief can be granted, and failure to join a party
[Fed. R. Civ. P. 12(h)(1)].
(6) A motion to dismiss for failure to state a claim upon which relief can
be granted, failure to join a necessary party, or failure to state a
legal defense to a claim may be raised [Fed. R. Civ. P. 12(g)(2)]:
(a) in any pleading allowed or ordered under Rule 7(a);
(b) by a motion under Rule 12(c); or
(c) at trial.

31
OUTLINE

(7) If the court determines at any time that it lacks subject-matter juris-
diction, the court must dismiss the action [Fed. R. Civ. P. 12(h)(3)].
c. Timing of Responsive Pleading
(1) An answer (or pre-answer motion) must be filed:
(a) within 21 days of service of process; or

NOTE The 21-day period does not include the date of service, but does include
weekends and holidays. If the 21st day is on a weekend or a holiday, the
answer must be filed on the next business day.

(b) if service was waived, within 60 days after the request for
a waiver was sent, or within 90 days after it was sent to the
defendant outside any judicial district of the United States.
(2) If the defendant responds to plaintiff’s complaint by filing a pre-
answer motion that is denied, the defendant must then file an
answer within 14 days of the denial.
4. Amended and Supplemental Pleadings
a. A party may amend a pleading once as a matter of course if the
amendment is filed [Fed. R. Civ. P. 15]:
(1) within 21 days of service of the original pleading; or
(2) if the pleading requires a response, within 21 days after service
of a responsive pleading or 21 days after service of a motion to
dismiss, a motion for a more definite statement, or a motion to
strike, whichever is earlier.
b. In situations in which a party is not entitled to amend as a matter of
course, a pleading may be amended with the written consent of the
opposing party, or with the leave of the court.
(1) Leave to amend must be freely given when justice requires. In
deciding whether to grant a request to amend, the court will consider:
(a) the reason for the delay in raising the matter to be raised by
the amendment; and
(b) the prejudice to the opposing party caused by the delay.
c. If a party objects at trial to evidence as not being within the scope of
the issues raised in the pleadings, the court may permit the pleadings
to be amended to conform to the evidence.
d. A response to an amended pleading is required if the original pleading
required a response.
e. Doctrine of Relation Back
(1) Under the doctrine of relation back, the court will treat an
amendment to a pleading as though it had been filed with the
original pleading.
(2) An amendment to a pleading that adds a new claim will be
considered filed on the date in which the original complaint was
filed as long as the amendment asserts a claim or defense that

32
CIVIL PROCEDURE

arose out of the same conduct, transaction, or occurrence as the


original claim [Fed. R. Civ. P. 15(c)].
(3) An amendment to a pleading that adds a new party will be
considered filed on the date in which the original complaint was
filed as long as [Id.]:
(a) the amendment asserts a claim or defense that arose out of the
same conduct, transaction, or occurrence as the original claim;
(b) the party to be added by the amendment received notice of
the original action within 90 days of service of the claim such
that it will not be prejudiced in defending on the merits; and
(c) the party to be added knew, or should have known, that the
original action would have been asserted against it, but for a
mistake concerning the proper party’s identity.

B. Rule 11
1. Every pleading, written motion, and other paper filed with the court must be
signed by at least one attorney of record in the attorney’s name, or by a party
personally, if the party is not represented by an attorney [Fed. R. Civ. P. 11].
a. The signed document must state the signer’s address, e-mail address,
and telephone number.
b. The court must strike an unsigned paper unless the omission is
promptly corrected after it is called to the attorney’s or party’s attention.
c. Rule 11 does not apply to discovery matter, whether it is a discovery
request, response, motion or any other matter, although other rules
may be applied to abusive discovery.
2. The attorney or unrepresented party who presents to the court a pleading,
written motion, or other document certifies that to the best of the person’s
knowledge, information, and belief, formed after an inquiry reasonable
under the circumstances that:
a. the document is not being presented for any improper purpose, such as to
harass, cause unnecessary delay, or needlessly increase the cost of litigation;
b. the claims, defenses, and other legal contentions are warranted by
existing law or by a nonfrivolous argument for extending, modifying, or
reversing existing law or for establishing new law;
c. the factual contentions have evidentiary support or will likely have
evidentiary support after a reasonable opportunity for further investiga-
tion or discovery; and
d. the denials of factual contentions are either warranted on the evidence
or are reasonably based on belief or a lack of information.
EXAMPLE: A brings a lawsuit against B for false light invasion of
privacy. A’s state has never recognized a cause of action for false light,
but A argues that, under existing precedent, the courts of the state
would likely recognize such an action. A has not violated Rule 11, even
if his action is ultimately unsuccessful.

33
OUTLINE

3. If the court finds that any of the above representations are untrue, it has
discretion to impose sanctions on the party or the party’s attorney. Sanctions
are ordered after notice to the offending party and an opportunity to be heard.
EXAMPLE: C brings a lawsuit against D for employment discrimination,
claiming that C’s application for employment was denied. C never applied
for a job with D. The court may order sanctions against C.
a. A party may not file a motion for sanctions without first serving the
motion upon the opposing party and providing the opposing party with
21 days to withdraw or correct the offending pleading, written motion,
or other paper (this is known as the safe harbor rule).
b. A court may sua sponte issue an order to show cause why conduct
described in the order does not violate Rule 11.
c. The nature of the sanction must be limited to “what suffices to deter
repetition of the conduct or comparable conduct by others.”
d. An order imposing sanctions must describe the sanctioned conduct
and explain the basis for the sanctions.

C. Provisional Relief
1. Purpose
a. Provisional relief is available to maintain the status quo and prevent irrepa-
rable damage or wasting of assets during litigation or pending arbitration.
b. Provisional relief consists of two remedies:
(1) temporary restraining orders (TROs); and
(2) preliminary injunctions.
c. Both are extraordinary remedies as they are issued prior to a decision on
the merits and provide immediate or quick but short-term injunctive relief.
(1) The temporary restraining order is used in an emergency situa-
tion, when the injunction must issue before any hearing, and lasts
only a few days, generally long enough for the parties to seek a
preliminary injunction.
(2) The preliminary injunction, if granted, will last until a decision on
the merits, however long that may take.
2. Standard for Granting
a. A party seeking either a temporary restraining order or a preliminary
injunction must first establish [Forsyth County v. United States Army
Corps of Eng’rs, 633 F.3d 1032]:
(1) a substantial likelihood of success on the merits;
(2) irreparable harm will be suffered unless the remedy sought is issued;
(3) the harm to the plaintiff if the temporary restraining order or
preliminary injunction is denied is greater than the harm to the
defendant if the remedy is granted; and
(4) the provisional remedy, if granted, will not be adverse to the
public interest.

34
CIVIL PROCEDURE

b. A court may issue a temporary restraining order or a preliminary injunc-


tion only if the moving party gives security sufficient to cover the costs
and damages sustained by any party who is wrongfully enjoined or
restrained [Fed. R. Civ. P. 65(c)].
3. Notice
a. Temporary Restraining Order
(1) A court may issue a temporary restraining order without written or
oral notice to the adverse party [Fed. R. Civ. P. 65(b)(1)].
(a) If a temporary restraining order is issued without notice, a
motion for a preliminary injunction must be set for hearing at
the earliest possible time, taking precedence over all other
matters [Fed. R. Civ. P. 65(b)(3)].
(b) If the party who obtained the temporary restraining order
does not proceed with the motion for a preliminary injunction,
the restraining order must be dissolved.
(2) Temporary restraining orders will issue without notice only if:
(a) specific facts in an affidavit or verified complaint clearly show
that immediate and irreparable injury, loss, or damage will
result to the movant before the adverse party can be heard
in opposition; and
(b) the movant’s attorney certifies in writing any efforts made to
give notice and the reasons why notice should not be required.
(3) Every temporary restraining order issued without notice must
[Fed. R. Civ. P. 65(b)(2)]:
(a) state the date and hour it was issued;
(b) describe the injury and state why it is irreparable;
(c) state why the order was issued without notice; and
(d) be promptly filed in the clerk’s office and entered in the record.
(4) A temporary restraining order issued without notice expires at the
time set by the court.
(a) The time for expiration must not exceed 14 days after entry.
(b) The court may, for good cause, extend the duration for a like
period or the adverse party may consent to a longer extension.
The reasons for an extension must be entered in the record.
b. Preliminary Injunction
(1) A preliminary injunction may only be issued by a court on notice
to the adverse party [Fed. R. Civ. P. 65(a)(1)].
(2) Before or after beginning a hearing on a motion for a preliminary
injunction, the court may advance the trial on the merits and
consolidate it with the hearing on the injunction.
(a) The court must preserve any party’s right to a jury trial.
(b) Whether or not consolidation is ordered, any evidence
entered on the motion for a preliminary injunction that would

35
OUTLINE

be admissible at trial becomes part of the trial record. This


evidence need not be repeated at trial.
4. Content and Scope of Relief
a. Every order granting a preliminary injunction or temporary restraining
order must [Fed. R. Civ. P. 65(d)(1)]:
(1) state the reasons why it was issued;
(2) state its terms specifically; and
(3) describe in reasonable detail the act or acts restrained or
required. The description may not be a reference to the complaint
or other document.
b. The order will only bind the following parties after actual notice is received,
whether by personal service or otherwise [Fed. R. Civ. P. 65(d)(2)]:
(1) the parties;
(2) the parties’ officers, servants, agents, employees, and attorneys; and
(3) any persons who are in active concert or participation with
anyone described above otherwise.

D. Joinder of Claims and Parties


1. Multiple Claims by Single Claimant against Single Defendant
a. A party may join as many claims in a single action as the party has
against an opposing party [Fed. R. Civ. P. 18].
b. There is no requirement that all of the claims in a complaint be related.
The goal of joining all claims in one action is to achieve a complete
resolution of all disputes between the parties.
(1) For convenience, to avoid prejudice, or to expedite and econo-
mize, the court may order a separate trial of one or more sepa-
rate issues, claims, cross-claims, counterclaims, or third-party
claims. When ordering a separate trial, the court must preserve
any federal right to a jury trial [Fed. R. Civ. P. 42(b)].
(2) Federal joinder of claims is usually permissive, but it is compelled
when the failure to join could result in splitting a cause of action.
The doctrine of claim preclusion prevents relitigation of a claim.
(3) Rule 18 addresses pleadings only. It cannot expand the jurisdic-
tion of a federal court, if there is not jurisdiction over the claim
otherwise. If supplemental jurisdiction is needed, then the parties
must still show that their additional claims “are so related to
claims in the action within such original jurisdiction that they form
part of the same case or controversy” [28 U.S. Code § 1367(a)].
2. Counterclaims
a. A counterclaim may be brought by any party against any other
opposing party and can be compulsory or permissive.
(1) A compulsory counterclaim is one that arises out of the same
transaction or occurrence as the original claim in the lawsuit.

36
CIVIL PROCEDURE

EXAMPLE: Plaintiff brings a breach of contract action against


Defendant, for failure to pay for construction work done by
Plaintiff. Defendant’s claim that the work was done negligently,
and caused Defendant to sustain personal injuries, is a compul-
sory counterclaim.
(a) A failure to raise a compulsory counterclaim waives the right
to assert the claim in any future action.
(b) The federal court has supplemental jurisdiction over a
compulsory counterclaim even in the absence of an indepen-
dent basis for federal jurisdiction [28 U.S.C. § 1367(a)].
(2) A permissive counterclaim is any claim that a party has against
an opposing party that does not arise out of the same transaction or
occurrence that is the subject matter of the opposing party’s claim.
EXAMPLE: Plaintiff sues Defendant for breach of contract for
failure to paint Plaintiff’s house. Defendant’s personal injury claim
against Plaintiff that arose out of a later traffic accident is a per-
missive counterclaim.
(a) A party is not required to assert a permissive counterclaim.
(b) Federal courts do not have supplemental jurisdiction over
permissive counterclaims, because they do not arise out of
the same transaction or occurrence as the original claim.
There must be an independent ground for federal jurisdiction
before the federal court will hear a permissive counterclaim.
3. Cross-Claims
a. A cross-claim is a claim filed by a party against a co-party (not
opposing party). Cross-claims may be brought by any party against
any co-party. In order for the court to have jurisdiction, the cross-claim
must either have an independent basis for jurisdiction (either federal-
question or diversity) or arise out of the same transaction or occur-
rence as the original claim or a counterclaim.
EXAMPLE: A sues B and C for failure to complete a construction proj-
ect as promised. B claims that C breached a contract with B to perform
certain work on the project. B may bring a cross-claim against C.
(1) A cross-claim may include a claim that the party against whom it
is asserted is or may be liable for all or part of the claim asserted
in the action against the cross-claimant.
EXAMPLE: A sues B, a surgeon, and C, an anesthesiologist, for
medical malpractice. C may bring a cross-claim against B, based on
C’s claim that the damages sustained by A were entirely B’s fault.
(2) The federal court has supplemental jurisdiction over a cross-claim
even in the absence of an independent basis for federal jurisdic-
tion, because the cross-claim arises out of the same transaction
or occurrence as the original claim [28 U.S.C. § 1367(a)].

37
OUTLINE

4. Impleader Claims
a. After a defendant to an action has served her answer, that defendant
may proceed against a non-party who may be liable for all or part of
the plaintiff’s claim against the defendant [Fed. R. Civ. P. 14].
b. Impleader thus allows a defending party to shift all or part of the liability
owed to the plaintiff to a new party who is or may be liable for it.
EXAMPLE: A sues B for injuries she sustained when B hit her while
driving her car. B claims that she hit A because the steering mecha-
nism on her car was damaged when C, a mechanic, did faulty repair
work. B may proceed against C.
c. The two most important impleader claims are contribution and
indemnity claims.
(1) A contribution claim is a claim against a joint tortfeasor that
shifts part of the defendant’s liability to a new party.
(2) An indemnity claim is a claim that shifts all of the defendant’s
liability to a new party (e.g., an insurance claim).
d. The defendant must file a third-party summons and complaint
with the court where the original action is pending. The defendant
becomes the third-party plaintiff.
(1) Service on the non-party makes that party the third-party defendant.
(2) A defendant may file a third-party complaint any time within 14
days of filing her answer. After that, third-party complaints may be
filed only with leave of court.
(3) When the defendant brings in a third-party, the plaintiff may amend his
original complaint to assert claims against the third-party defendant.
e. A plaintiff may file a third-party complaint to bring in a third-party defen-
dant if a counterclaim has been asserted against the plaintiff and there
is a basis for the plaintiff to argue that a third party is liable for all or
part of the liability pursuant to the counterclaim.
f. Third-party defendants have the same rights as any other party to the
action. These rights include the right to:
(1) service of process;
(2) file an answer to the third-party complaint (the answer may
include any defenses that the third-party plaintiff asserted in his
answer to the original complaint);
(3) assert counterclaims or cross-claims;
(4) implead additional non-parties by third-party complaint; and
(5) appeal orders or final judgment in the action.
g. The court may dismiss a third-party complaint without prejudice, or
order a separate trial of the third-party claim or any separate issue
within it, if it finds that the controversy between a third-party plaintiff
and third-party defendant would unduly delay the determination of the
original action or prejudice a substantial right of one of the parties.

38
CIVIL PROCEDURE

5. Permissive Joinder of Parties


a. Permissive joinder allows multiple plaintiffs to join together in a single
action and allows multiple defendants to be sued in a single action.
b. Multiple persons may be joined together as plaintiffs or defendants in
one action if [Fed. R. Civ. P. 20]:
(1) the joined parties claim relief (if plaintiffs) or face liability (if defendants)
that arises out of the same transaction or occurrence; and
(2) any question of law or fact is common to the joined parties.
EXAMPLE: A and B are injured while they are passengers
on an airplane that makes a bad landing. They may join their
claims in the same action.
6. Compulsory Joinder of Parties
a. A person who is subject to service of process and whose joinder will
not deprive the court of subject-matter jurisdiction must be joined as a
necessary party if [Fed. R. Civ. P. 19]:
(1) in that party’s absence, the court cannot grant complete relief
among existing parties; or
(2) the party claims an interest relating to the subject of the action,
and an adjudication without the party may:
(a) as a practical matter, impair or impede that party’s ability to
protect his interests; or
(b) leave an existing party subject to a substantial risk of incurring
double, multiple, or otherwise inconsistent obligations.
EXAMPLE: P leased a valuable painting to D. P believed
the lease term was 3 years, and D believed it was 4. After
possessing the painting for 2.5 years, D leased the painting
to X for 1 year. When the 3-year mark arrived, P brought suit
to reclaim the painting. X is a necessary party because it has
an interest in the subject matter of the litigation (the paint-
ing), because D could be subject to duplicative liability (D
could end up being liable to both P and X) and because the
court cannot order the painting returned unless the person
with the painting (X) is within the jurisdiction of the court.
EXAMPLE: A is injured while a passenger on a train owned and
operated by B. She was injured when another train, owned and
operated by C, collided with the train she was riding on. A sued
B. B argues that C is a necessary party and must be joined. This
argument will fail. As a rule, joint tortfeasors are not necessary
parties. There is no mandatory joinder in this situation.
b. If a person who is required to be joined cannot be joined for
jurisdictional reasons, the court must determine whether, in equity
and good conscience, the action should proceed among the existing
parties or be dismissed. The factors for the court to consider include:

39
OUTLINE

(1) the extent to which a judgment rendered in the person’s


absence might prejudice that person or the existing parties;
(2) the extent to which any prejudice could be lessened or avoided by:
(a) protective provisions in the judgment;
(b) shaping the relief; or
(c) other measures;
(3) whether a judgment rendered in the person’s absence would
be adequate; and
(4) whether the plaintiff would have an adequate remedy if the
action were dismissed for nonjoinder.
7. Interpleader
a. Interpleader is used where a plaintiff has some holding that would
expose the plaintiff to multiple liability from adverse claims. The
stakeholding party, or “stakeholder,” can commence an action for
interpleader to resolve liability where there are two or more adverse
claimants. Interpleader is only applicable where multiple claims
demand the same thing or obligation—usually, a piece of property,
prize, or, most commonly, the proceeds of an insurance policy.
b. Interpleader is either rule interpleader, authorized by the Federal
Rules of Civil Procedure, or statutory interpleader.
EXAMPLE: B dies leaving a life insurance policy, the proceeds
of which are claimed by his wife, ex-wife, and two children. The
insurance company is the stakeholder. As a neutral third party, it
may bring an action in court to determine the rights of each claimant.
c. Rule interpleader is authorized by Fed. R. Civ. P. 22.
(1) Rule interpleader may be initiated pursuant to Rule 22 by any
person who may be exposed to multiple liability.
(2) In an interpleader under Rule 22, the stakeholder may initiate
the claim or invoke the rule interpleader by his own initiative.
The stakeholder may also initiate interpleader by counter-
claiming or cross-claiming against a claimant in an action that
has already commenced against the stakeholder.
(3) Rule interpleader has no effect on jurisdictional or venue
requirements. Subject-matter jurisdiction must still be
established through diversity jurisdiction, federal question
jurisdiction, or supplemental jurisdiction.
d. Statutory interpleader is authorized by 28 U.S.C. § 1335.
(1) Statutory interpleader allows a person holding property
which may be claimed, or is claimed by two or more adverse
claimants, to interplead all possible claimants.
(2) Once a statutory interpleader suit is commenced, the court may
restrain all of the claimants from commencing any related action
or continuing any other action that has already begun.

40
CIVIL PROCEDURE

(3) Statutory interpleader has the following advantages over


rule interpleader:
(a) nationwide service of process is available for statutory
interpleader, but not for rule interpleader;
(b) the federal court has diversity jurisdiction as long as there
is minimal diversity between the adverse claimants and an
amount in controversy over $500; and
EXAMPLE: When A died, she left $1,000 in an account with Bank.
The money in the account is claimed by D, her surviving husband,
and E, her daughter. D is a citizen of the same state as Bank, but
E lives in a different state. The federal court has diversity juris-
diction to hear a statutory interpleader action brought by Bank.
(c) venue is established where any one of the adverse
claimants resides.
8. Intervention
a. Intervention allows a non-party to assert a right or interest in an
ongoing action. If the non-party can meet the test for intervention as a
matter of right, then the court must allow the party to intervene. A non-
party may also be allowed to intervene by permission of the court.
EXAMPLE: A conservation group may seek leave to intervene in an
action against a developer.
b. The purpose of intervention is to enable a person outside an action to
enter the suit to present an interest, a claim, or a defense relevant to
the issues presented by the existing parties.
(1) Intervention can contribute to judicial economy and protect
a non-party from having its interest adversely affected by the
litigation conducted without that person’s participation.
(2) Intervention can also delay or complicate the litigation by involving
parties that the plaintiff chose not to join in the first instance.
c. In order to intervene in an action, a non-party’s request for
intervention must be timely. The factors the court may consider in
deciding whether the application was timely are:
(1) the length of time the non-party knew or should have known
about its interest in the case before making the application;
(2) the extent of prejudice that the existing parties may suffer as a
result of the delay; and
(3) the likelihood and gravity of the prejudice the non-party may
suffer if the application is denied.
d. Upon a finding by the court that a non-party’s application was timely, the
non-party must be allowed to intervene in an action if [Fed. R. Civ. P. 24]:
(1) a federal statute confers an absolute right of the party to intervene; or
(2) a non-party is asserting a protectable interest relating to the
property or transaction involved in the lawsuit, and:

41
OUTLINE

(a) the non-party is so situated that disposing of the action


may as a practical matter impair or impede its ability to
protect its interest; and
(b) the non-party’s interests are not adequately represented by
existing parties.
EXAMPLE: The United States may seek to intervene as of right
in a discrimination action by students against a board of education
where plaintiffs request only monetary damages and do not repre-
sent the government’s interest in enforcing anti-discrimination laws.
e. If a non-party may not intervene in an action as a matter of right, the
court may permit intervention if:
(1) a federal statute allows a conditional right to intervention; or
(2) the non-party has a claim or defense that shares a common
question of law or fact with the primary action.
f. The trial court has discretion to grant permissive leave to intervene.
In making its determination, the court will consider:
(1) the complexity of the existing action and the complexity of the
applicant’s claim or defense;
(2) the length of time the primary action has been pending; and
(3) how much the existing parties will be delayed or prejudiced by
the addition of the applicant’s claim or defense.
9. Class Actions
a. A class action may be maintained by a party if the court finds that
each of the following requirements have been met [Fed. R. Civ. P. 23]:
(1) numerosity;
(a) The numerosity requirement is met if the number of
members of the class is so numerous that separate joinder
of each member is impracticable.
(b) No particular number is needed, nor is the number of
potential class members determinative of whether the court
will allow an action to proceed as a class action.
(2) commonality;
(a) The commonality requirement is met if there are questions
of law or fact common to the class.
(b) Rule 23 does not require that all questions of law and fact
be common, nor that the common questions predominate.
Instead, the party who seeks class certification must show
that the class members have suffered the same injury
[Wal-Mart Stores, Inc. v. Dukes, 564 U.S. 338 (2011)].
Specifically, there must be a common contention that is
capable of a class-wide resolution in one stroke [Id.]. Simply
showing common facts or a common accusational type will
not be enough to satisfy the commonality requirement.

42
CIVIL PROCEDURE

(3) typicality; and


(a) The typicality requirement is met if the claims or defenses
of the representative party are typical of those raised by
each member of the class.
(b) To meet this requirement, the representative’s interest in pros-
ecuting his own case must simultaneously tend to advance the
interests of the absent class members. This means that the
plaintiff’s claim cannot be so different from the claims of poten-
tial class members that those class members’ claims will not be
advanced by the plaintiff’s proof of his own individual claim.
(4) adequacy of representation.
(a) The adequacy of representation requirement is met if the
representative party can fairly and adequately protect and
represent the interests of each member of the class.
(b) The representative party must have standing to bring her claim.
b. In addition to the four requirements in Rule 23, courts will allow an
action to be maintained as a class action only if at least one of the
following is true:
(1) the prosecution of separate actions would create a risk of:
(a) inconsistent or varying judgments for individual class
members that would establish incompatible standards of
conduct for the party opposing the class; or
(2) judgments for individual members of the class that would
substantially impede or impair the ability of other members to
protect their interests; the party opposing the class has acted
or refused to act on grounds that are generally applicable to the
class as a whole; or
(3) the court finds that common questions predominate over
individual questions, and class representation is superior to
other methods for adjudicating the controversy. Factors used to
determine common questions include:
(a) pending individual litigation;
(b) the interests of class members in individually controlling
litigation of claims or defenses;
(c) the desirability of concentrating litigation in the forum; and
(d) the difficulties of managing the class action.
c. Federal rules for class certification apply to all class actions in
federal court, including those based on diversity. State laws on class
certification are superseded [Shady Grove Orthopedic Associates v.
Allstate Insurance Co., 559 U.S. 393 (2010)].
d. A motion to certify a class action must be made “at an early practi-
cable time” after the suit is commenced [Fed. R. Civ. P. 23(c)]. Local
rules may provide a specific deadline for a class certification motion.
(1) The order permitting a class action must describe the class.

43
OUTLINE

(2) When appropriate, the court may limit the class to those
members who do not request exclusion from the class within
the specified time after the notice.
EXAMPLE: A class certification order certifies a class of all
persons who bought a computer with a certain operating system
who do not inform the court that they do not wish to be a part of
the action within 90 days of the class certification order.
e. An action may be brought or maintained as a class action with
respect to any particular issue, or a class may be divided into
subclasses with each subclass treated as a class.
f. The court may direct appropriate notice to class members.
(1) For classes certified as a result of common questions of law or
fact, notice is required and must include:
(a) the nature of the action;
(b) the definition of the certified class;
(c) the class claims, issues, or defenses;
(d) the ability of a class member to make an appearance;
(e) the ability of a class member to request exclusion from the
class; and
(f) the binding effect of a judgment on all members of the class.
EXAMPLE: Notice to potential class members in an action
against the seller of computer operating systems must
inform members that if they do not opt out of the class, they
will be bound by the judgment in the class action, even if
they did not file an individual appearance.
(2) Where a class action is brought primarily for injunctive or
declaratory relief, notice need not be given to the class unless
the court finds it necessary in order to protect the class.
g. In every case, the court must have personal jurisdiction over every
defendant and each of the plaintiffs named in the action.
(1) The federal court is not required to have personal jurisdiction
over absent members of the plaintiff class as long as they
receive adequate notice of the pendency of the action and
are afforded the opportunity to opt out of the class [Phillips
Petroleum Co. v. Shutts, 472 U.S. 797 (1985)].
(2) It is unclear whether the court must have personal jurisdiction
over the absent members of the plaintiff classes where there is
no notice and/or opportunity to opt out of the class.
h. Class actions based on diversity jurisdiction are governed by
the Class Action Fairness Act (CAFA) passed in 2005, which has
transformed the doctrine in cases founded on diversity jurisdiction.
(1) Prior to CAFA, diversity jurisdiction required complete diversity between
all of the named representative plaintiffs and all of the defendants. In

44
CIVIL PROCEDURE

(2) CAFA expands federal subject-matter jurisdiction to include


classes where:
(a) the class has more than 100 persons;
(b) at least one member of the class is diverse from at least
one defendant; and
(c) the total amount in controversy exceeds $5 million.
(3) CAFA’s broad grant of subject-matter jurisdiction excludes local
controversies, certain civil rights cases, and other categories of cases.
i. Class actions may not be dismissed or compromised without court
approval. In evaluating a settlement proposal, a court must consider
whether it is fair, reasonable, and in the best interests of the
individuals affected by it [Fed. R. Civ. P. 23(e)].
j. A court that certifies a class action must appoint class counsel to
represent the class [Fed. R. Civ. P. 23(g)]. The court may deny
certification if suitable counsel is not found.
k. Counsel for the successful representative parties typically will be
awarded an appropriate fee. If a fee may not be awarded pursuant to
an applicable fee-shifting provision, the court may award fees out of
the common fund created by the recovery from the defendant.
l. A court of appeals may permit an appeal from an order granting or
denying class action certification if the petition for appeal is filed
within 14 days after the order is entered [Fed. R. Civ. P. 23(f)].

E. Pretrial Conferences and Orders


1. Discovery Planning Conference
a. Rule 26(f) requires the parties to meet and have a discovery
planning conference as soon as practicable, but at least 21 days
before a scheduling order is to be held or is due under Rule 16(b)
[Fed. R. Civ. P. 26(f)].
b. In the discovery conference, the parties must [Id.]:
(1) discuss the nature and basis of their claims and defenses and
the possibilities of timely settling or resolving the case;
(2) make or arrange for the mandatory disclosures required by
Rule 26(a)(1);
(3) discuss any issues about preserving discoverable information; and
(4) develop a proposed discovery plan.
c. A written report outlining the proposed discovery plan must then be
submitted to the court within 14 days after the discovery conference.
2. Pretrial Conferences
a. After a Rule 26(f) conference has been held, the court may order the
attorneys and any unrepresented parties to appear for one or more
pretrial conferences for such purposes as [Fed. R. Civ. P. 16(a)]:
(1) expediting disposition of the action;

45
OUTLINE

(2) establishing early and continuing control so that the case will
not be protracted because of lack of management;
(3) discouraging wasteful pretrial activities;
(4) improving the quality of the trial through more thorough preparation; and
(5) facilitating settlement.
b. Except in categories of actions exempted by local rule, the district judge—
or a magistrate judge when authorized by local rule—must issue a sched-
uling order that limits the time to join other parties, amend the pleadings,
complete discovery, and file motions. A schedule may be modified only
for good cause and with the judge’s consent [Fed. R. Civ. P. 16(b)].
c. The scheduling order must be issued within the earlier of [Id.]:
(1) 90 days after any defendant has been served with the complaint; or
(2) 60 days after any defendant has appeared.
d. At any pretrial conference, the court may consider and take
appropriate action on the following matters [Fed. R. Civ. P. 16(c)]:
(1) formulating and simplifying the issues, and eliminating frivolous
claims or defenses;
(2) amending the pleadings if necessary or desirable;
(3) obtaining admissions and stipulations about facts and
documents to avoid unnecessary proof, and ruling in advance
on the admissibility of evidence;
(4) avoiding unnecessary proof and cumulative evidence, and limiting
the use of testimony under the Federal Rules of Evidence;
(5) determining the appropriateness and timing of summary adjudication;
(6) controlling and scheduling discovery;
(7) identifying witnesses and documents, scheduling the filing and
exchange of any pretrial briefs, and setting dates for further
conferences and for trial;
(8) referring matters to a magistrate judge or a master;
(9) settling the case and using special procedures to assist in
resolving the dispute when authorized by statute or local rule;
(10) determining the form and content of the pretrial order;
(11) disposing of pending motions;
(12) adopting special procedures for managing potentially difficult
or protracted actions that may involve complex issues, multiple
parties, difficult legal questions, or unusual proof problems;
(13) ordering a separate trial of a claim, counterclaim, cross-claim,
third-party claim, or particular issue;
(14) ordering the presentation of evidence;
(15) establishing a reasonable limit on the time allowed to present
evidence; and
(16) facilitating in other ways the just, speedy, and inexpensive
disposition of the action.

46
CIVIL PROCEDURE

e. After any pretrial conference, the court should issue an order reciting
the action taken, which controls the course of the action unless the
court modifies it [Fed. R. Civ. P. 16(d)].
f. The court may hold a final pretrial conference to formulate a trial plan,
including a plan to facilitate the admission of evidence. The conference must
be held as close to the start of trial as is reasonable, and must be attended
by at least one attorney who will conduct the trial for each party and by any
unrepresented party. The court may modify the order issued after a final
pretrial conference only to prevent manifest injustice [Fed. R. Civ. P. 16(e)].
g. On motion or on its own, the court may issue sanctions if a party or
its attorney:
(1) fails to appear at a scheduling or other pretrial conference;
(2) is substantially unprepared to participate—or does not
participate in good faith—in the conference; or
(3) fails to obey a scheduling or other pretrial order.
(a) Instead of, or in addition to, any other sanction, the court
must order the party, its attorney, or both to pay the reason-
able expenses—including attorney’s fees—incurred because
of any noncompliance with this rule, unless the noncompli-
ance was substantially justified or other circumstances
make an award of expenses unjust [Fed. R. Civ. P. 16(f)].

F. Discovery
1. Overview
a. The primary purpose of the pleading stage is to provide notice to the
parties; thus, the facts may not be stated in detail in the pleadings.
b. The facts surrounding a cause of action come to light during discovery.
c. The primary function of the discovery process is to provide litigants with an
opportunity to obtain and review all of the pertinent evidence prior to trial.
d. There are mandatory disclosure obligations as well as specific
discovery devices that are designed to elicit information within the
permissible scope of discovery.
e. In general, discovery is initiated by the parties. A court will become
involved only when there is a discovery dispute that cannot be
resolved by the parties.
2. Mandatory Disclosures
a. Initial Disclosures
(1) With limited exceptions, parties are required to disclose some
information as a matter of course upon the commencement of
the litigation, without waiting for a discovery request. Mandatory
disclosure is required by both parties for the following
information [Fed. R. Civ. P. 26(a)(1)]:
(a) the name (and if known, address and telephone number)
of individuals likely to have discoverable information, along

47
OUTLINE

with the subject of that information, that the disclosing party


may use to support its claims or defenses (unless the use
would be solely for impeachment);
EXAMPLE: A plaintiff must disclose to the defendant the
names of all witnesses to an accident that is the subject of the
plaintiff’s action that the plaintiff may use to support her claim.
(b) a copy or description of all documents, electronically stored
information, and tangible things that the disclosing party has in its
possession, custody, or control and may use to support its claims
or defenses (unless the use would be solely for impeachment);
(c) a computation of damages claimed by the disclosing party,
together with supporting materials; and
(d) insurance agreements under which an insurance company
may be liable to satisfy all or part of a possible judgment.
(2) The initial mandatory disclosures must be made within 14 days
after the Rule 26 discovery conference. A party must make these
disclosures based upon the information then reasonably available.
b. Expert Disclosures
(1) A party must also disclose the identity of any witness that may
testify at trial [Fed. R. Civ. P. 26(a)(2)(A)].
(2) If the witness is retained or specially employed to provide
expert testimony in the case, or one whose duties as the
party’s employee regularly involve giving expert testimony, the
disclosure must be accompanied by the expert’s final written
report. The report must contain [Fed. R. Civ. P. 26(a)(2)(B]:
(a) all opinions, including the basis and reasons for them, that
the witness will testify to;
(b) all facts and data considered by the witness in forming
the opinions;
(c) any exhibits that will be used;
(d) the witness’s qualifications, including a list of all
publications authored in the previous 10 years;
(e) a list of all cases during the previous 4 years in which
witness testified as an expert; and
(f) a statement of the compensation to be paid for the study
and testimony in the case.
(3) If the witness does not need to provide a written report, the
disclosure must state [Fed. R. Civ. P. 26(a)(2)(C]:
(a) the subject matter on which the witness will testify; and
(b) a summary of the facts and opinions that the witness will testify to.
(4) Unless provided otherwise by court order, these disclosures
must be made [Fed. R. Civ. P. 26(a)(2)(D]:
(a) at least 90 days before trial; or

48
CIVIL PROCEDURE

(b) if the evidence is intended solely to contradict or rebut


evidence on the same subject matter identified by another
party through the mandatory witness disclosure, within 30
days after the party’s disclosure.
c. Pretrial Disclosures
(1) In addition to the mandatory disclosures discussed above,
at least 30 days before trial each party must provide to the
other parties and promptly file the following information about
evidence that it may present at trial [Fed. R. Civ. P. 26(a)(3)(A)]:
(a) the name (and if not previously provided the contact
information) for each witness the party expects to present
and those it may call if needed;
(b) a list of witnesses whose testimony will be presented
through a deposition; and
(c) a list of documents or physical evidence the party expects
to present and may present if need be.
d. Supplementation
(1) A party must supplement a discovery response with any
information that would have been subject to the mandatory
disclosure requirement [Fed. R. Civ. P. 26(e)].
(2) Failure to comply may lead to the exclusion of that evidence at
trial [Fed. R. Civ. P. 37(c)(1)].
EXAMPLE: Using the above example, the plaintiff must
supplement its witness list if the plaintiff learns of any additional
witnesses after it first discloses the list to the defendant.
3. Scope of Discovery
a. In general, a party is entitled to demand the discovery of any matter
that is [Fed. R. Civ. P. 26(b)]:
(1) nonprivileged;
(2) relevant to any party’s claim or defense; and
(3) proportional to the needs of the case.
b. The factors to consider when determining whether a matter is propor-
tional to the needs of the case include the importance of the issues
at stake in the action, the amount in controversy, the parties’ relative
access to relevant information, the parties’ resources, the importance
of the matter sought in resolving the issues, and whether the burden or
expense of the proposed discovery outweighs its likely benefit [Id.].
c. Information is relevant if it is likely to make any fact in dispute
more or less likely to be true, regardless of whether the information
would be admissible at trial. A party is entitled to discovery of not only
material that is relevant and admissible at trial, but also information
“that appears reasonably calculated to lead to the discovery of
admissible evidence” [Fed. R. Civ. P. 26(b)(1)].

49
OUTLINE

EXAMPLE: Plaintiff may question Defendant about a document


even if that document would not be admissible into evidence, as long
as the question is relevant.
d. If the requested information can be obtained from another source
that is more convenient, less burdensome, or less expensive, a party
may be required to obtain it from the other source. A request for
electronically stored information could be particularly vulnerable to
this criticism.
e. Privilege
(1) Privileged matter is not discoverable.
(2) A privilege may arise under the laws of evidence or constitu-
tional principles. The most frequently invoked privilege is the
attorney-client privilege, which precludes the discovery of confi-
dential communications between an attorney and her client for
the purposes of obtaining or rendering legal advice.
EXAMPLE: Defendant may object to Plaintiff’s request for
attorney invoices as being privileged.
(3) Many states also recognize some combination of the
following privileges:
(a) priest-penitent;
(b) doctor-patient;
(c) psychotherapist-patient; and
(d) spousal.
f. Work-Product
(1) A party may not discover documents and tangible objects
prepared in anticipation of litigation or for trial by or for another
party or its representative, including an attorney, consultant,
surety, indemnitor, insurer, or agent [Fed. R. Civ. P. 26(b)(3)(A)].
This is known as an attorney’s work-product.
(2) However, such materials are still discoverable if [Id.]:
(a) they are otherwise discoverable under Rule 26(b)(1); and
(b) the party shows both substantial need for the material to
prepare its case and it cannot, without undue hardship,
obtain their substantial equivalent elsewhere.
(3) If the court orders the materials discoverable, the court must
protect from disclosure the attorney’s mental impressions,
conclusions, opinions, or legal theories, which always remain
privileged [Fed. R. Civ. P. 26(b)(3)(B)].
g. Experts
(1) A party may depose any person who has been identified as an
expert who is expected to testify at trial, but only to a limited
extent because communications between an attorney and his
expert are considered work-product.

50
CIVIL PROCEDURE

(2) Besides the information contained in the mandatory


expert report (discussed above) a party can also obtain
communications relating to [Fed. R. Civ. P. 26(b)(4)(C)]:
(a) compensation for the expert’s study or testimony;
(b) any facts or data provided by the attorney to the expert
that the expert considered in forming his opinion; and
(c) any assumption the attorney provided and the expert relied
on in forming his opinion.
(3) Expert reports that are prepared in draft (i.e., non-finalized) are
also considered work-product.
(4) A party may not, by interrogatories or deposition, obtain facts
known or opinions held by an expert retained in anticipation
of litigation or to prepare for trial, but who is not expected to
be called to testify at trial, unless the party shows exceptional
circumstances under which it is impracticable to obtain the facts
or opinions by any other means, or in the case of an expert who
conducts a Rule 35 examination [Fed. R. Civ. P. 26(b)(4)(D)].
4. Objections
a. Parties who receive discovery requests that are beyond the scope of
discovery (or are otherwise allegedly defective) can either object to
the requests or request a protective order.
b. In the event of an objection, the party requesting discovery has at
least two options:
(1) abandon or reframe the discovery request; or
(2) bring the dispute to the attention of the court by filing a motion to compel.
c. If the responding party requests a protective order, the party thereby
brings the discovery dispute to the attention of the court.
5. Discovery Devices
a. Depositions
(1) Depositions permit the direct questioning of a party or witness
under oath. They are typically conducted orally, and every word
that is spoken is recorded verbatim and transcribed.
(2) A party may, by oral questions, depose any person, including a
party, without leave of court except as provided in Rule 30(a)(2).
The deponent’s attendance may be compelled by subpoena
under Rule 45 [Fed. R. Civ. P. 30(a)(1)].
(3) A party must obtain leave of court, and the court must grant
leave to the extent consistent with Rule 26(b)(1) and (2)
[Fed. R. Civ. P. 30(a)(2)]:
(a) if the parties have not stipulated to the deposition and:
1) the deposition would result in more than 10 depositions
being taken under this rule or Rule 31 by the plaintiffs,
or by the defendants, or by the third-party defendants;

51
OUTLINE

2) the deponent has already been deposed in the case; or


3) the party seeks to take the deposition before the time
specified in Rule 26(d), unless the party certifies in
the notice, with supporting facts, that the deponent is
expected to leave the United States and be unavailable
for examination in this country after that time; or
(b) if the deponent is confined in prison.
(4) The witness (called the deponent) is given an opportunity to review
the transcript and make technical corrections [Fed. R. Civ. P. 30(e)].
(5) Scheduling
(a) A deposition may be scheduled on reasonable notice
in writing. If the deponent is a party, the deposition is
scheduled by a notice of deposition served on all counsel.
(b) The party requesting the deposition may ask the witness to
bring documents to the deposition.
(c) If the deponent is a non-party, the party requesting the depo-
sition must also provide for the attendance of the witness,
usually by serving a subpoena.
(d) A deposition may not be used against a party at trial if the
party received less than 14 days’ notice of the deposition and
promptly moved for a protective order that it not be taken, and
the motion was still pending when the deposition was taken.
(6) Depositions of corporations are permitted. The corporation
must designate one or more persons whose answers bind
the corporation and who must have completed a reasonably
diligent investigation prior to testifying.
EXAMPLE: A corporation could designate its financial officer to tes-
tify at a deposition concerning the corporation’s financial statements.
(7) A party may take only 10 depositions in an action as a matter of right.
The primary advantages of depositions vis-à-vis other discovery
techniques include engaging in a spontaneous dialogue with the
witness and assessing the demeanor and credibility of potential
trial witnesses. The most significant disadvantage is the expense.
(8) Use of Depositions in Court Proceedings
(a) In General
1) In general, at a hearing or trial, all or part of a deposition
may be used against a party on these conditions [Fed.
R. Civ. P. 32(a)(1)]:
a) the party was present or represented at the taking
of the deposition or had reasonable notice of it;
b) it is used to the extent it would be admissible under
the Federal Rules of Evidence if the deponent were
present and testifying; and
c) the use is allowed by Rule 32(a)(2) through (8).

52
CIVIL PROCEDURE

2) Any party may use a deposition to contradict or impeach


the testimony given by the deponent as a witness, or
for any other purpose allowed by the Federal Rules of
Evidence [Fed. R. Civ. P. 32(a)(2)].
3) An adverse party may use for any purpose the deposi-
tion of a party or anyone who, when deposed, was the
party’s officer, director, managing agent, or designee
[Fed. R. Civ. P. 32(a)(3)].
4) A party may use for any purpose the deposition of a
witness, whether or not a party, if the court finds [Fed.
R. Civ. P. 32(a)(4)]:
a) that the witness is dead;
b) that the witness is more than 100 miles from the
place of hearing or trial or is outside the United
States, unless it appears that the witness’s absence
was procured by the party offering the deposition;
c) that the witness cannot attend or testify because of
age, illness, infirmity, or imprisonment;
d) that the party offering the deposition could not
procure the witness’s attendance by subpoena; or
e) on motion and notice, that exceptional circumstances
make it desirable—in the interest of justice and with
due regard to the importance of live testimony in open
court—to permit the deposition to be used.
5) A deposition must not be used against a party who, having
received less than 14 days’ notice of the deposition,
promptly moved for a protective order under Rule 26(c)(1)
(B) requesting that it not be taken or be taken at a different
time or place—and this motion was still pending when the
deposition was taken [Fed. R. Civ. P. 32(a)(5)(A)].
6) A deposition taken without leave of court under the
unavailability provision of Rule 30(a)(2)(A)(iii) must not
be used against a party who shows that, when served
with the notice, it could not, despite diligent efforts,
obtain an attorney to represent it at the deposition
[Fed. R. Civ. P. 32(a)(5)(B)].
7) If a party offers in evidence only part of a deposition, an
adverse party may require the offeror to introduce other
parts that in fairness should be considered with the part
introduced, and any party may itself introduce any other
parts [Fed. R. Civ. P. 32(a)(6)].
8) Substituting a party under Rule 25 does not affect the
right to use a deposition previously taken [Fed. R.
Civ. P. 32(a)(7)].

53
OUTLINE

9) A deposition lawfully taken and, if required, filed in any


federal- or state-court action may be used in a later
action involving the same subject matter between the
same parties, or their representatives or successors
in interest, to the same extent as if taken in the later
action, and may also be used as allowed by the Federal
Rules of Evidence [Fed. R. Civ. P. 32(a)(8)].
(b) Form of Presentation
1) Unless the court orders otherwise, a party must provide
a transcript of any deposition testimony the party offers,
but may provide the court with the testimony in nontran-
script form as well. On any party’s request, deposition
testimony offered in a jury trial for any purpose other
than impeachment must be presented in nontranscript
form, if available, unless the court for good cause orders
otherwise [Fed. R. Civ. P. 32(c)].
(c) Objections to Admissibility
1) Subject to Rules 28(b) and 32(d)(3), an objection may be
made at a hearing or trial to the admission of any deposi-
tion testimony that would be inadmissible if the witness
were present and testifying [Fed. R. Civ. P. 32(b)].
(d) Waivers of Objections
1) An objection to an error or irregularity in a deposition
notice is waived unless promptly served in writing on the
party giving the notice [Fed. R. Civ. P. 32(d)(1)].
2) An objection based on disqualification of the officer
before whom a deposition is to be taken is waived if not
made [Fed. R. Civ. P. 32(d)(2)]:
a) before the deposition begins; or
b) promptly after the basis for disqualification
becomes known or, with reasonable diligence,
could have been known.
3) An objection to a deponent’s competence—or to the
competence, relevance, or materiality of testimony—is
not waived by a failure to make the objection before or
during the deposition, unless the ground for it might have
been corrected at that time [Fed. R. Civ. P. 32(d)(3)(A)].
4) An objection to an error or irregularity at an oral exami-
nation is waived if [Fed. R. Civ. P. 32(d)(3)(B)]:
a) it relates to the manner of taking the deposition, the
form of a question or answer, the oath or affirma-
tion, a party’s conduct, or other matters that might
have been corrected at that time; and
b) it is not timely made during the deposition.

54
CIVIL PROCEDURE

5) An objection to the form of a written question under Rule


31 is waived if not served in writing on the party submitting
the question within the time for serving responsive ques-
tions or, if the question is a recross-question, within seven
days after being served with it [Fed. R. Civ. P. 32(d)(3)(C)].
6) An objection to how the officer transcribed the testimony—
or prepared, signed, certified, sealed, endorsed, sent, or
otherwise dealt with the deposition—is waived unless a
motion to suppress is made promptly after the error or
irregularity becomes known or, with reasonable diligence,
could have been known [Fed. R. Civ. P. 32(d)(4)].
b. Interrogatories
(1) Interrogatories are written questions that must be answered by
another party in writing under oath [Fed. R. Civ. P. 33].
EXAMPLE: The defendant serves interrogatories upon the
plaintiff regarding information related to the time and place of a
particular accident.
(2) Interrogatories may only be served on parties to an action.
(3) Each party may submit up to 25 questions on any other party.
(4) Interrogatories must be answered in writing, under oath, within
30 days after service of the interrogatories.
c. Document Requests
(1) A party may serve on any other party a request within the
scope of Rule 26(b) [Fed. R. Civ. P. 34(a)]:
(a) to produce and permit the requesting party or its representa-
tive to inspect, copy, test, or sample the following items in
the responding party’s possession, custody, or control:
1) any designated documents or electronically stored informa-
tion—including writings, drawings, graphs, charts, photo-
graphs, sound recordings, images, and other data or data
compilations—stored in any medium from which information
can be obtained either directly or, if necessary, after transla-
tion by the responding party into a reasonably usable form; or
2) any designated tangible things; or
(b) to permit entry onto designated land or other property
possessed or controlled by the responding party, so
that the requesting party may inspect, measure, survey,
photograph, test, or sample the property or any
designated object or operation on it.
(2) More than 21 days after the summons and complaint are
served on a party, a request under Rule 34 may be delivered
[Fed. R. Civ. P. 26(d)(2)]:
(a) to that party by any other party; and

55
OUTLINE

(b) by that party to any plaintiff or to any other party that has
been served.
(3) The request [Fed. R. Civ. P. 34(b)(1)]:
(a) must describe with reasonable particularity each item or
category of items to be inspected;
(b) must specify a reasonable time, place, and manner for the
inspection and for performing the related acts; and
(c) may specify the form or forms in which electronically stored
information is to be produced.
(4) The party to whom the request is directed must respond in writing
within 30 days after being served or—if the request was delivered
under Rule 26(d)(2)—within 30 days after the parties’ first Rule 26(f)
conference. A shorter or longer time may be stipulated to under
Rule 29 or be ordered by the court [Fed. R. Civ. P. 34(b)(2)(A)].
(5) For each item or category, the response must either state that
inspection and related activities will be permitted as requested
or state with specificity the grounds for objecting to the request,
including the reasons [Fed. R. Civ. P. 34(b)(2)(B)].
(a) The responding party may state that it will produce copies
of documents or of electronically stored information instead
of permitting inspection. The production must then be
completed no later than the time for inspection specified in the
request or another reasonable time specified in the response.
(6) An objection must state whether any responsive materials are
being withheld on the basis of that objection. An objection to
part of a request must specify the part and permit inspection of
the rest [Fed. R. Civ. P. 34(B)(2)(C)].
(7) The response may state an objection to a requested form for
producing electronically stored information. If the responding
party objects to a requested form—or if no form was specified
in the request—the party must state the form or forms it intends
to use [Fed. R. Civ. P. 34(B)(2)(D)].
(8) Unless otherwise stipulated or ordered by the court, these
procedures apply to producing documents or electronically
stored information [Fed. R. Civ. P. 34(b)(2)(E)]:
(a) a party must produce documents as they are kept in the
usual course of business or must organize and label them
to correspond to the categories in the request;
(b) if a request does not specify a form for producing
electronically stored information, a party must produce it in
a form or forms in which it is ordinarily maintained or in a
reasonably usable form or forms; and
(c) a party need not produce the same electronically stored
information in more than one form.

56
CIVIL PROCEDURE

(9) As provided in Rule 45, a nonparty may be compelled to


produce documents and tangible things or to permit an inspec-
tion [Fed. R. Civ. P. 34(c)].
EXAMPLE: A plaintiff may request minutes of the defendant’s
corporate board meetings for the previous five years.
d. Requests for Admissions
(1) A party may submit to any other party a request for admission of
any matter within the scope of discovery [Fed. R. Civ. P. 36].
(2) Requests for admissions are typically “question and answer”
statements that are used by either party to further explore specific
contentions. Any request that is admitted is deemed established
for all purposes in the litigation.
EXAMPLE: During a wrongful death lawsuit resulting from a
car crash, the plaintiff may ask the defendant to admit that he
was driving the Toyota involved in the crash. If the defendant
makes the admission, the plaintiff will not have to prove that
issue at trial.
(3) A party served with a request for admission has 30 days to respond.
e. Physical and Mental Examinations
(1) When a party’s condition is in controversy, a physical or mental
examination of the person may be requested [Fed. R. Civ. P. 35].
(2) Physical and mental examinations are the only discovery tools for
which advance court approval is required. The court requires a
showing of “good cause” for the examination.
EXAMPLE: If the plaintiff is claiming emotional distress, the
defendant may request that the plaintiff submit to a psychologi-
cal exam.
6. E-discovery
a. Electronic discovery (or e-discovery) refers to discovery regarding the
exchange of electronically stored information, or ESI. The e-discovery
process generally involves the following steps.
(1) Electronically stored data (and their custodians) are identified by
counsel as potentially relevant for further analysis and review.
(2) ESI identified as potentially relevant is placed on a legal hold to
ensure that the data cannot be destroyed.
(3) The data is transferred from a company to its legal counsel, who
will determine relevance.
(4) Files are prepared to be loaded into a document review platform.
(5) Documents are reviewed for responsiveness to discovery
requests and for privilege.
(6) Documents are turned over to opposing counsel, based on
agreed-upon specifications.

57
OUTLINE

b. Procedure
(1) A response in writing is required within 30 days after being served or,
if the request was delivered under Rule 26(d)(2), within 30 days after
the parties’ first Rule 26(f) conference [Fed. R. Civ. P. 37(b)(2)].
(2) Instead of permitting inspection of documents or ESI, the
responding party can produce copies of the documents or the
ESI. The production of the copies of ESI must take place within
the same timeframe as the requested inspection or another
reasonable time specified in the response [Fed. R. Civ. P.
37(b)(2)(B)]. ESI does not have to be reproduced in the form
requested. If a party objects to the requested form of producing
ESI, or if the form is specified in the request, the response must
state the form the party intends to use [Fed. R. Civ. P. 37(b)(C)].
(3) Unless otherwise stipulated or ordered by the court, these proce-
dures apply to producing documents or ESI [Fed. R. Civ. P. 37(b)(E)]:
(a) a party must produce documents as they are kept in the
usual course of business or must organize and label them
to correspond to the categories in the request;
(b) if a request does not specify a form for producing ESI,
a party must produce it in a form in which it is ordinarily
maintained or in a reasonably usable form; and
(c) a party need not produce the same ESI in more than one form.
c. Sanctions
(1) If ESI that should have been preserved in the anticipation or
conduct of litigation is lost because a party failed to take reason-
able steps to preserve it, and it cannot be restored or replaced
through additional discovery, the court [Fed. R. Civ. P. 37(e)]:
(a) upon finding prejudice to another party from loss of the infor-
mation, may order measures no greater than necessary to
cure the prejudice; or
(b) if the courts finds that the party acted with the intent to
deprive another party of the information’s use in the litigation,
the court can:
1) presume that the lost information was unfavorable to
the party;
2) instruct the jury that it may or must presume the
information was unfavorable to the party; or
3) dismiss the action or enter a default judgment.
7. Discovery Sanctions
a. Motion to Compel
(1) On notice to other parties and all affected persons, a party may
move for an order compelling disclosure or discovery. The
motion must include a certification that the movant has in good
faith conferred or attempted to confer with the person or party

58
CIVIL PROCEDURE

failing to make disclosure or discovery in an effort to obtain it


without court action [Fed. R. Civ. P. 37(a)].
(2) A motion for an order to a party must be made in the court where
the action is pending. A motion for an order to a non-party must
be made in the court where the discovery is or will be taken.
b. Failure to Comply with Court Order
(1) If the court where the discovery is taken orders a deponent to be
sworn or to answer a question and the deponent fails to obey, the
failure may be treated as contempt of court [Fed. R. Civ. P. 37(b)].
(2) If a party or a party’s officer, director, or managing agent, or a
designated witness, fails to obey an order to provide or permit
discovery, the court where the action is pending may issue further
just orders, including the following:
(a) declaring that the facts sought are established in favor of the
requesting party;
(b) prohibiting the disobedient party from supporting or opposing
designated claims or defenses, or from introducing desig-
nated matters in evidence;
(c) striking pleadings in whole or in part;
(d) staying further proceedings until the order is obeyed;
(e) dismissing the action or proceeding in whole or in part;
(f) rendering a default judgment against the disobedient party; or
(g) treating as contempt of court the failure to obey any order
except an order to submit to a physical or mental examination.
(3) If a party fails to provide required information or identify a witness,
the party is not allowed to use that information or witness to supply
evidence on a motion, at a hearing, or at a trial, unless the failure
was substantially justified or is harmless [Fed. R. Civ. P. 37(c)].
(4) If a party fails to make an admission, and if the requesting party later
proves a document to be genuine or the matter true, the requesting
party may move that the party who failed to admit, pay the reasonable
expenses, including attorney’s fees, incurred in making that proof [Id.].
c. Additional Sanctions
(1) The court where the action is pending may, on motion, order
sanctions if [Fed. R. Civ. P. 37(d)]:
(a) a party or a party’s officer, director, or managing agent fails,
after being served with proper notice, to appear for that
person’s deposition; or
(b) a party, after being properly served with interrogatories or a
request for inspection, fails to serve its answers, objections,
or written response.
(2) Instead of, or in addition to, the sanctions listed above, the court
must require the party failing to act, the attorney advising that party,

59
OUTLINE

or both, to pay the reasonable expenses, including attorney’s fees,


caused by the failure, unless the failure was substantially justified
or other circumstances make an award of expenses unjust [Id.].
(3) If a party or its attorney fails to participate in good faith in devel-
oping and submitting a proposed discovery plan, the court may,
after giving an opportunity to be heard, require that party or attorney
to pay to any other party the reasonable expenses, including
attorney’s fees, caused by the failure [Fed. R. Civ. P. 37(f)].

G. Adjudication without a Trial


1. Most cases do not result in a full trial. Possible methods of resolving
disputes without a trial include:
a. voluntary dismissal;
b. involuntary dismissal;
c. default judgments;
d. settlements; and
e. pretrial motions (discussed infra).
2. Voluntary Dismissal
a. A plaintiff may voluntarily dismiss an action by filing a notice of
dismissal at any time prior to service of defendant’s answer or
motion for summary judgment [Fed. R. Civ. P. 41(a)].
b. Before or after a responsive pleading, a plaintiff may voluntarily
dismiss an action by filing a stipulation signed by all parties.
c. If the plaintiff cannot obtain another party’s agreement to the dismissal,
the plaintiff may make a motion for voluntary dismissal. Such a
motion rests within the broad discretion of the trial court. The court
will likely grant the motion unless the opponent will suffer prejudice.
d. The filing of the notice of dismissal automatically terminates the case without
prejudice. If the plaintiff has already voluntarily dismissed the action once
before, the notice of dismissal operates as an adjudication upon the merits.
3. Involuntary Dismissals
a. Dismissal for Failure to State a Claim
(1) Pleadings must contain “a short and plain statement of the
claim showing that the pleader is entitled to relief” [Fed. R. Civ. P.
8(a)(2)]. If this requirement is not met, the party against whom
the claim is alleged may move to dismiss for failure to state a
claim for which relief can be granted [Fed. R. Civ. P. 12(b)(6)].
EXAMPLE: Defendant may move to dismiss for failure to state
a claim on the ground that the complaint fails to allege that
Defendant is responsible for Plaintiff’s injuries.
(2) When an action is dismissed based on failure to state a claim,
the dismissal is with prejudice unless the court states otherwise
in its order [Fed. R. Civ. P. 41(b)].

60
CIVIL PROCEDURE

(3) A court would dismiss the claim without prejudice (or with leave
to amend) in circumstances where the pleading defect can and
should be rectified.
b. Dismissal for Failure to Prosecute
(1) If a plaintiff fails to prosecute or to comply with the Federal Rules
or a court order, a defendant may move to dismiss. A dismissal
for failure to prosecute is a dismissal with prejudice, unless the
dismissal order states otherwise [Fed. R. Civ. P. 41(b)].
EXAMPLE: Plaintiff brings a lawsuit against defendant. Eighteen
months later, plaintiff has not engaged in any discovery, and has
not made any efforts to move the case forward. Defendant may
bring a motion to dismiss for failure to prosecute.
4. Default Judgment
a. Default judgments involve two steps:
(1) the entry of default; and
(2) the entry of the default judgment.
b. An entry of default must be entered on behalf of a party against whom
a judgment for affirmative relief is sought when the party has failed to
plead or otherwise defend the claim and that failure is shown by affi-
davit or otherwise [Fed. R. Civ. P. 55(a)].
c. Only after an entry of default has been entered may a default judgment
be entered.
d. A default judgment can be entered by the clerk or the court.
(1) If the plaintiff’s claim is for a sum certain, upon the plaintiff’s
request and with an affidavit showing the amount due, the
clerk must enter judgment for that amount and costs against
a defendant who has been defaulted for not appearing [Fed.
R. Civ. P. 55(b)].

NOTE A clerk cannot enter default judgments against minors or incompetent persons.

(2) In all other cases, the party must apply to the court for a default
judgment to be entered.
(3) If the party against whom a default judgment is sought has
appeared personally or by a representative, that party or repre-
sentative must be served with written notice of the application for
default judgment at least seven days before the hearing [Id.].
(4) The court may conduct hearings or make referrals when, to enter
or effectuate judgment, it needs to [Id.]:
(a) conduct an accounting;
(b) determine the amount of damages;
(c) establish the truth of any allegation by evidence; or
(d) investigate any other matter.

61
OUTLINE

e. The court may set aside an entry of default for good cause. If the
clerk or court entered a default judgment, the court may set it aside
in accordance with a Rule 60(b) post-trial motion.
5. Settlements
a. Settlements may be achieved through counsel and the parties
themselves, or through alternative dispute resolution (ADR) methods
such as arbitration or mediation.
b. A settlement conference is appropriate at any time. It may be held
with a pretrial or discovery conference, or separately.
c. Settlement conferences are not mandatory under the Federal Rules.
d. A judge may, on his own or at a party’s request, refer a settlement
conference to another judge or magistrate.

62
CIVIL PROCEDURE

IV. JURY TRIALS

A. Right to Jury Trial


1. The right to a jury trial in a civil action can be conferred by statute and is
guaranteed by the Seventh Amendment to the U.S. Constitution.
2. Equitable actions are not triable by a jury as a matter of right. In
determining whether a right to a jury trial exists, the court will consider:
a. whether the claim more closely resembles actions in law or in equity; and
b. whether the remedy sought is legal or equitable in nature (i.e.
monetary relief or injunctive relief).

NOTE If both legal and equitable relief are sought in an action, the right to a
jury exists for any issue of fact underlying a damages claim, even if the
resolution of that issue may also support injunctive relief.

B. Demand for a Jury Trial


1. If the right to a jury trial exists in an action, any party can exercise the
right by filing with the court and serving on the other parties a written
demand for a jury trial.
2. The demand for a jury trial must be made within 14 days after the service
of the last pleading directed to the triable issue.
3. If the demand for a jury trial is not timely filed and served, the right to a
trial by jury is waived [Fed. R. Civ. P. 38].
a. Issues on which a jury trial is not properly demanded are to be tried
by the court. However, the court may, on motion, order a jury trial
on any issue for which a jury might have been demanded [Fed. R.
Civ. P. 39(b)].
b. In an action not triable of right by a jury, the court, on motion or on its
own [Fed. R. Civ. P. 39(c)]:
(1) may try any issue with an advisory jury; or
(2) may, with the parties’ consent, try any issue by a jury whose
verdict has the same effect as if a jury trial had been a matter
of right, unless the action is against the United States and a
federal statute provides for a nonjury trial.
4. Removal
a. If a party has expressly demanded a jury trial in accordance with
state law prior to the case being removed to federal court, the
demand need not be renewed after removal.
b. If the state law did not require express demand, a party need not
make such demand unless the court orders the parties to do so. The
court may so order on its own, or must so order at a party’s request
[Fed. R. Civ. P. 81(c)(3)(A)].
c. Failure to make a demand when ordered will constitute waiver [Id.].

63
OUTLINE

d. If all necessary pleadings have been served at the time of removal, a


party entitled to a jury trial under Rule 38 must be given one if the party
serves a demand within 14 days after [Fed. R. Civ. P. 81(c)(3)(B)]:
(1) filing a notice of removal; or
(2) being served with a notice of removal filed by another party.

C. Jury Selection
1. A federal jury can be composed of six to twelve members. Each juror
must participate in the verdict unless excused [Fed. R. Civ. P. 48(a)].
2. The federal jury is selected by the voir dire process [Fed. R. Civ. P.
47]. The court may conduct the examination or permit the attorneys to
question directly the prospective jurors.
3. The purpose of the voir dire is to explore germane factors that might
expose a basis for challenge, whether for cause or peremptory.
4. Attorneys may ask the court to excuse any juror for cause. There are
three categories of challenges for cause:
a. general disqualification—a person may lose the right to sit on
any jury because of a felony conviction or some other inherent
disqualification;
b. implied bias—potential jurors may be removed if the attorney
believes a bias may exist; and
EXAMPLE: A juror who has a relationship with the parties or their
attorneys could be removed due to an implied bias.
c. actual bias—jurors may be removed for cause if they indicate during
voir dire that they would use predetermined beliefs or principles to
decide the case instead of deciding the case based upon the facts.
5. Peremptory Challenges
a. In federal court, each party may exercise three peremptory challenges
to excuse jurors without having to state the reason before the court.
b. Peremptory challenges may not be used to exclude jurors on the
basis of race or gender.
c. Peremptory challenges may be objected to by the opposing party
only where the exclusion gives rise to an inference of racial or
gender discrimination. If objected to, the excluding party must
provide a nondiscriminatory explanation for the strikes or else
rescind its strike of each juror involved.

D. Jury Instructions
1. Requests
a. At the close of the evidence or at any earlier reasonable time that
the court orders, a party may file and furnish to every other party
written requests for the jury instructions it wants the court to give
[Fed. R. Civ. P. 51(a)].

64
CIVIL PROCEDURE

b. After the close of the evidence, a party may [Id.]:


(1) file requests for instructions on issues that could not reasonably
have been anticipated by an earlier time that the court set for
requests; and
(2) with the court’s permission, file untimely requests for
instructions on any issue.
2. Instructions
a. The court [Fed. R. Civ. P. 51(b)]:
(1) must inform the parties of its proposed instructions and
proposed action on the requests before instructing the jury and
before final jury arguments;
(2) must give the parties an opportunity to object on the record and
out of the jury’s hearing before the instructions and arguments
are delivered; and
(3) may instruct the jury at any time before the jury is discharged.
3. Objections
a. A party who objects to an instruction or the failure to give an
instruction must do so on the record, stating distinctly the matter
objected to and the grounds for the objection.
b. An objection is timely if [Fed. R. Civ. P. 51(c)]:
(1) a party objects on the record and out of the jury’s hearing before
the instructions and arguments are delivered; or
(2) a party was not informed of an instruction or action on a request
before that opportunity to object, and the party objects promptly
after learning that the instruction or request will be, or has been,
given or refused.
c. The timing requirements for objecting to jury deliberations are strict,
and the key thing is that the objection has to be made before, and not
after, the jury begins its deliberations. Once the jury begins deliber-
ating, it is too late to object.
4. Assigning Error
a. A party may assign as error [Fed. R. Civ. P. 51(d)]:
(1) an error in an instruction actually given, if that party properly
objected; or
(2) a failure to give an instruction, if that party properly requested it
and—unless the court rejected the request in a definitive ruling
on the record—also properly objected.
5. A court may consider a plain error in the instructions that has not been preserved
as required by the Federal Rules if the error affects substantial rights [Id.].

65
OUTLINE

V. MOTIONS

A. General Rules
1. Requirements Regarding Form
a. Any applications to the court for an order shall be made by motion.
b. A motion must:
(1) be in writing, unless made during a hearing or trial; and
(2) state with particularity the grounds for the request and a
request for relief.
c. All rules governing matters of form in pleadings also apply to motions.
2. Other Requirements
a. Local court rules may set out specific requirements for motion practice,
such as the papers to be filed, and the timing for making a motion.
b. The court may issue standing orders in a particular case regarding
the timing of motions.

B. Pre-Trial Motions
1. Motion to Dismiss
a. A motion to dismiss is a motion filed by a defending party seeking the
dismissal of a claim filed against him.
b. A motion to dismiss may be made on any of the following grounds
[Fed. R. Civ. P. 12(b)]:
(1) lack of subject-matter jurisdiction;
(2) lack of personal jurisdiction;
(3) improper venue;
(4) insufficient process;
(5) insufficient service of process;
(6) failure to state a claim upon which relief can be granted; and
(7) failure to join a party under Rule 19.
c. A motion to dismiss based on lack of personal jurisdiction, improper
venue, insufficient process, or insufficient service of process must
be made in the defending party’s first response to the court—either a
pre-answer motion to dismiss or an answer.
(1) If the defending party makes a motion to dismiss and one of
the aforementioned defenses is not included, that defense is
waived and cannot later be included in the answer.
(2) If no pre-answer motion to dismiss is made, the defense must
be in the defending party’s answer, or else it is waived.
d. A motion to dismiss based on failure to state a claim upon which relief
can be granted or failure to join a necessary party can be raised at
any time before the trial ends, or else it is waived.
e. Lack of subject-matter jurisdiction is never waived, it can be raised at
any time, even on appeal.

66
CIVIL PROCEDURE

f. Rule 12(b)(6) Dismissal


(1) A court will dismiss a complaint for failure to state a claim upon
which relief can be granted if the complaint:
(a) fails to state a cognizable claim;
(b) provides insufficient facts; or
(c) contains an allegation that negates one or more elements
of the cause of action.
(2) Dismissal of a complaint under Rule 12(b)(6) is with prejudice
unless the court states otherwise.
(3) If there is a defect in the pleading that the court deems
rectifiable, the court may:
(a) dismiss the complaint without prejudice; or
(b) grant the plaintiff leave to amend the complaint.
2. Motion for Judgment on the Pleadings
a. After the pleadings are closed—but early enough not to delay trial—a
party may move for judgment on the pleadings [Fed. R. Civ. P. 12(c)].
b. If, on a motion under Rule 12(b)(6) or 12(c), matters outside the
pleadings are presented to and not excluded by the court, the motion
must be treated as one for summary judgment under Rule 56. All
parties must be given a reasonable opportunity to present all the
material that is pertinent to the motion [Fed. R. Civ. P. 12(d)].
3. Motion for a More Definite Statement
a. A party may move for a more definite statement of a pleading to which a
responsive pleading is allowed but which is so vague or ambiguous that
the party cannot reasonably prepare a response [Fed. R. Civ. P. 12(e)].
b. The motion must be made before filing a responsive pleading and
must point out the defects complained of and the details desired.
c. If the court orders a more definite statement and the order is not obeyed
within 14 days after notice of the order or within the time the court sets,
the court may strike the pleading or issue any other appropriate order.
4. Motion to Strike
a. The court may strike from a pleading an insufficient defense or any
redundant, immaterial, impertinent, or scandalous matter. The court
may act [Fed. R. Civ. P. 12(f)]:
(1) on its own; or
(2) on motion made by a party either before responding to the
pleading or, if a response is not allowed, within 21 days after
being served with the pleading.
5. Summary Judgment Motions
a. A motion for summary judgment is a motion challenging a claim or
defense on the merits. Summary judgments are intended to pierce
the pleadings to determine if there is credible evidence to factually
support a party’s claim.

67
OUTLINE

b. Summary judgment will be granted if the moving party “shows that


there is no genuine dispute as to any material fact and the movant is
entitled to judgment as a matter of law” [Fed. R. Civ. P. 56].
c. Any party may move for summary judgment as to any individual
issue that has been joined, or as to the cause of action as a whole.
d. The parties may move with or without supporting affidavits for
summary judgment.
e. Unless a local rule or court order provides otherwise:
(1) a party may move for summary judgment at any time before 30
days after the close of discovery;
(2) a party opposing a motion for summary judgment must file
a response within 21 days after the motion is served or a
responsive pleading is due, whichever is later; and
(3) a reply may be filed within 14 days after the response is served.
f. When a party moves for summary judgment, the court must first
consider whether the moving party has shown—based on their
arguments alone—that the non-moving party lacks sufficient facts
and/or law to prevail on the claim or defense in question. The moving
party can make this argument by either:
(1) pointing out holes in the opposing party’s claims or defenses; or
(2) adducing new evidence demonstrating that the claim or defense
cannot be true.
g. If the moving party can establish that the non-moving party lacks
sufficient facts or law to prevail on the claim or defense in question,
the court then turns to the non-moving party to defeat the motion.
(1) To survive summary judgment, the non-moving party must
adduce evidence sufficient for a reasonable jury to find in her
favor on a claim or defense.
(2) The non-moving party may not rely on allegations in the
complaint to defeat a motion for summary judgment.
h. Although both plaintiffs and defendants can move for summary
judgment, defendants do so far more often because it is harder
for a plaintiff to prevail. For a plaintiff to prevail on her own motion
for summary judgment, she must produce evidence in support of
every element of her claim and the defendant must not respond with
evidence of any element of the claim.
i. The evidence presented at a summary judgment motion will be
viewed in the light most favorable to the non-moving party to deter-
mine whether the movant is entitled to prevail as a matter of law.
(1) A party may offer affidavits and/or evidence obtained during discovery.
(2) A party may not rely on evidence that will be inadmissible at trial
to satisfy the production burden. If the evidence is in an inad-
missible form at the summary judgment stage, but will be in an
admissible form at trial, then the production burden is satisfied.

68
CIVIL PROCEDURE

EXAMPLE: The inadmissibility of an affidavit that contains


hearsay would be remedied by a hearsay exception of
presentation of the witness who made the statement.
(3) Plaintiffs may not rely on evidence that no rational fact finder
would believe or would find sufficient to establish the element.
EXAMPLE: If the only expert testimony offered by the plaintiff
is “implausible,” a court will enter summary judgment on behalf
of the defendant [Matsushita v. Zenith, 475 U.S. 574 (1986)].
j. Powers of the District Judge
(1) The judge may grant or deny summary judgment in favor of or
against the person asking for it.
(2) After giving notice to all parties and a reasonable time to
respond, the judge can also [Fed. R. Civ. P. 56(f)]:
(a) grant summary judgment even when nobody asks for it (a
sua sponte grant of summary judgment is proper);
(b) grant summary judgment on grounds that neither party has
asked for—i.e., for completely different reasons than the
moving party was seeking for summary judgment; or
(c) grant summary judgment not just for the moving party, but
against the moving party and in favor of the non-moving party.
(3) Judges may grant partial summary judgment.
(4) Judges are required to state on the record the grounds for
granting or denying summary judgment.
(5) Judges do not have to canvas the entire record hunting for a
genuine issue of material fact. Instead, the trial judge can limit
his review on a motion for summary judgment to only those
portions of the record cited by the parties.

C. Post-Trial Motions
1. Judgment as a Matter of Law
a. A motion for judgment as a matter of law (JMOL) was formerly
known as a motion for a directed verdict.
b. The defendant or the plaintiff may move for JMOL after the other
party closes her case. If the motion is granted, the motion results in
judgment for the moving party [Fed. R. Civ. P. 50(a)].
c. A JMOL will be granted if the court finds that a reasonable jury would
not have a legally sufficient evidentiary basis to find for the non-
moving party on the issue at hand [Id.].
EXAMPLE: In an action for copyright infringement, the plaintiff
fails to show that she is the owner of the copyright in question. The
defendant’s motion for JMOL should be granted.
d. The analysis for JMOL motions is identical to that on a summary
judgment motion.

69
OUTLINE

(1) A summary judgment motion is based on evidence that a party


may introduce at trial, while the JMOL motion is based on
evidence actually introduced at trial.
(2) The evidence will be viewed in the light most favorable to the
non-moving party to determine whether the movant is entitled
to prevail as a matter of law.
(3) A JMOL must specify the judgment sought and the law and
facts that entitle the moving party to the judgment [Id.].
e. Motions for JMOL are typically made at the close of the non-moving
party’s case, but may be made at any time after all of the non-moving
party’s evidence has been submitted but before the case is submitted
to the jury.
f. Most attorneys will make a motion for JMOL as a matter of course, in order
to preserve the right to renew the motion at the conclusion of the trial.
2. Renewed Motion for Judgment as a Matter of Law
a. A renewed motion for judgment as a matter of law was formerly
known as a motion for judgment notwithstanding the verdict (JNOV,
or judgment non obstante veredicto).
b. Within 28 days after the entry of judgment (or discharge of the jury
if the motion addresses a jury issue not decided by verdict), a party
who has timely moved for JMOL may serve a motion to set aside the
verdict and any judgment entered on the verdict [Fed. R. Civ. P. 50(b)].

NOTE A party that did not timely move for a JMOL during trial cannot move for a
renewed JMOL.

c. A renewed motion for JMOL requires analysis identical to the


JMOL motion.
(1) Even after the jury has rendered a verdict, if a judge finds that
no reasonable jury could interpret the evidence presented as
supporting the verdict, a judge may grant to the party against
whom the judgment was rendered JMOL.
d. In ruling on a renewed motion for JMOL, the court may [Id.]:
(1) allow judgment on the verdict to stand, if the jury returned a verdict;
(2) order a new trial; or
(3) direct the entry of judgment as a matter of law.
e. When making a renewed motion for JMOL, the moving party may
also move jointly or in the alternative for a new trial under Rule 59.
(1) If the court grants the renewed motion for JMOL, it must also
issue a conditional ruling on any motion for a new trial [Id.].
3. Motion for a New Trial
a. A new trial may be granted to all or any of the parties on all or some
of the issues litigated at trial [Fed. R. Civ. P. 59].
b. A motion for a new trial must be filed within 28 days of the entry of judgment.

70
CIVIL PROCEDURE

c. A motion for a new trial is generally made as a form of alternative


relief to a renewed JMOL motion.
d. A motion for a new trial is generally appropriate in the following instances:
(1) in order to avoid an inevitable appeal and reversal if the trial
judge has committed reversible error;
(2) when a jury verdict is so excessive as to demonstrate that the jury
has misunderstood its duty or has acted with extreme prejudice;
(3) if evidence of jury misconduct exists, and Fed. R. Evid. 606(b) limits
judicial inquiry to external influences on the deliberation process; or
(4) when the verdict is against the clear weight of the evidence.
e. On a motion for a new trial, the court is not required to view the evidence
in a light most favorable to the verdict and may grant the new trial, even
though there was enough evidence to prevent JMOL or renewed JMOL.
f. The court may also consider a new trial conditioned on
certain circumstances.
EXAMPLE: If the jury verdict awarded an improper or excessive
amount of damages, the court may grant a new trial on the issue of
damages alone. Liability would not be relitigated.
4. Remittitur and Additur
a. A motion for remittitur asks the judge to reduce the award of damages that
are excessive. Federal courts will routinely grant motions for remittitur.
b. The converse of a remittitur is an additur, or an increase in the award
of damages. Additur is unconstitutional in federal court.
5. Motion for Relief from a Judgment or Order
a. The court may correct a clerical mistake or a mistake arising from
oversight or omission whenever one is found in a judgment, order, or
other part of the record.
(1) The court may do so on motion of a party or on its own, with or
without notice [Fed. R. Civ. P. 60(a)].
(2) If an appeal has been docketed with the appellate court and is
pending, however, the mistake may be corrected only with the
appellate court’s leave [Id.].
b. On motion and on just terms, the court may grant a motion for relief
form judgment for the following reasons [Fed. R. Civ. P. 60(b)(1)–(6)]:
(1) mistake, inadvertence, surprise, or excusable neglect;
(2) newly discovered evidence that, with reasonable diligence,
could not have been discovered in time to move for a new trial
under Rule 59(b);
(3) fraud, misrepresentation, or misconduct by an opposing party;
(4) the judgment is void;
(a) A judgment is void where it is “so affected by a fundamental
infirmity that the infirmity may be raised even after the

71
OUTLINE

judgment becomes final” [United States Aid Funds, Inc. v.


Espinosa, 559 U.S. 260 (2010)].
(b) Rule 60(b)(4) applies only in the rare instance when a judgment
is premised either on a certain type of jurisdictional error or on
a violation of due process that deprived a party of notice or the
opportunity to be heard. This standard does not include erro-
neous judgments or situations where a legal error has occurred.
(5) the judgment has been satisfied, released, or discharged, it
is based on an earlier judgment that has been reversed or
vacated, or applying it prospectively is no longer equitable; or
(6) any other reason that justifies relief.
c. A motion under Rule 60(b) must be made within a reasonable time. If
the motion is being made for any of the first three reasons listed above
(mistake, new evidence or fraud), the motion must be made no more than
a year after entry of the judgment, order, or the date of the proceeding.
d. A motion for relief from judgment does not affect the finality of the
judgment, nor does it suspend its operation [Fed. R. Civ. P. 60(c)].
e. Rule 60 does not limit a court’s power to entertain an independent
action to relieve a party from a judgment, order, or proceeding, grant
relief to a defendant who was not personally notified of the action, or
set aside a judgment due to fraud on the court [Fed. R. Civ. P. 60(d)].

72
CIVIL PROCEDURE

VI. VERDICTS AND JUDGMENTS

A. Jury Verdicts
1. In General
a. Unless the parties stipulate otherwise, a jury verdict must be
unanimous and must be returned by a jury of at least six members
[Fed. R. Civ. P. 48(b)].
b. After a verdict is returned but before the jury is discharged, the court
must on a party’s request, or may on its own, poll the jurors individu-
ally. If the poll reveals a lack of unanimity or lack of assent by the
number of jurors that the parties stipulated to, the court may direct the
jury to deliberate further or may order a new trial [Fed. R. Civ. P. 48(c)].
2. General Verdicts
a. The court may direct the jury to return a general verdict, which
simply states which party should win, the plaintiff or defendant,
without addressing specific findings on each issue of fact.
3. Special Verdicts
a. The court may require a jury to return only a special verdict in the
form of a special written finding on each issue of fact. The court may
do so by [Fed. R. Civ. P. 49(a)(1)]:
(1) submitting written questions susceptible of a categorical or
other brief answer;
(2) submitting written forms of the special findings that might
properly be made under the pleadings and evidence; or
(3) using any other method that the court considers appropriate.
b. The court must give the instructions and explanations necessary to
enable the jury to make its findings on each submitted issue [Fed.
R. Civ. P. 49(a)(2)].
c. A party waives the right to a jury trial on any issue of fact raised by the
pleadings or evidence but not submitted to the jury unless, before the jury
retires, the party demands its submission to the jury. If the party does
not demand submission, the court may make a finding on the issue. If the
court makes no finding, it is considered to have made a finding consis-
tent with its judgment on the special verdict [Fed. R. Civ. P. 49(a)(3)].
4. General Verdict with Answers to Written Questions
a. The court may submit to the jury forms for a general verdict, together
with written questions on one or more issues of fact that the jury must
decide. The court must give the instructions and explanations necessary
to enable the jury to render a general verdict and answer the questions
in writing, and must direct the jury to do both [Fed. R. Civ. P. 49(b)(1)].
b. When the general verdict and the answers are consistent, the court
must approve, for entry under Rule 58, an appropriate judgment on
the verdict and answers [Fed. R. Civ. P. 49(b)(2)].

73
OUTLINE

c. When the answers are consistent with each other but one or more is incon-
sistent with the general verdict, the court may [Fed. R. Civ. P. 49(b)(3)]:
(1) approve for entry an appropriate judgment according to the
answers, notwithstanding the general verdict;
(2) direct the jury to further consider its answers and verdict; or
(3) order a new trial.
d. When the answers are inconsistent with each other and one or more
is also inconsistent with the general verdict, judgment must not be
entered; instead, the court must direct the jury to further consider its
answers and verdict, or must order a new trial [Fed. R. Civ. P. 49(b)(4)].

B. Judicial Findings and Conclusions


1. In an action tried on the facts without a jury or with an advisory jury,
the court must find the facts specially, and state its conclusions of
law separately. The findings and conclusions may be stated on the
record after the close of the evidence or may appear in an opinion or a
memorandum of decision filed by the court. Judgment must be entered
under Rule 58 [Fed. R. Civ. P. 52(a)(1)].
2. In granting or refusing an interlocutory injunction, the court must
similarly state the findings and conclusions that support its action [Fed.
R. Civ. P. 52(a)(2)].
3. The court is not required to state findings or conclusions when ruling on a
motion to dismiss or for summary judgment or, unless these rules provide
otherwise, on any other motion [Fed. R. Civ. P. 52(a)(3)].
4. A master’s findings, to the extent adopted by the court, must be
considered the court’s findings [Fed. R. Civ. P. 52(a)(4)].
5. A party may later question the sufficiency of the evidence supporting the
findings, whether or not the party requested findings, objected to them,
moved to amend them, or moved for partial findings [Fed. R. Civ. P. 52(a)(5)].
6. Findings of fact, whether based on oral or other evidence, must not be
set aside unless clearly erroneous, and the reviewing court must give due
regard to the trial court’s opportunity to judge the witnesses’ credibility
[Fed. R. Civ. P. 52(a)(6)].
7. On a party’s motion filed no later than 28 days after the entry of judgment,
the court may amend its findings—or make additional findings—and may
amend the judgment accordingly. The motion may accompany a motion
for a new trial [Fed. R. Civ. P. 52(b)(1)].
8. If a party has been fully heard on an issue during a nonjury trial and the
court finds against the party on that issue, the court may enter judgment
against the party on a claim or defense that, under the controlling law, can
be maintained or defeated only with a favorable finding on that issue. The
court may, however, decline to render any judgment until the close of the
evidence. A judgment on partial findings must be supported by findings of
fact and conclusions of law [Fed. R. Civ. P. 52(c)].

74
CIVIL PROCEDURE

C. Effect of Verdicts—Claim and Issue Preclusion


1. Claim Preclusion
a. Claim preclusion (also known as res judicata) and issue preclusion
(also known as collateral estoppel) are judicial doctrines intended to
promote the policy of judicial economy and the finality of litigation. The
general rule created by these doctrines is that once a case has reached
a final judgment, then that case, and related claims and issues, have
been decided or settled permanently and are not eligible for relitigation.
b. The doctrine of claim preclusion prevents relitigation of a claim:
(1) between the same parties and those who are in privity with them;
(2) arising out of the same transaction or occurrence underlying the
prior suit; and
(3) that was determined on the merits by a court with proper
subject-matter and personal jurisdiction.
EXAMPLE: During trial, the jury determines that the defendant was
not liable for the plaintiff’s injuries arising out of a car accident. The
plaintiff may not file a subsequent action against the same defendant
seeking property damages resulting from the same car accident.
c. A judgment is not on the merits if it is a dismissal for lack of personal
jurisdiction, subject-matter jurisdiction, or venue. A dismissal with
prejudice is considered a dismissal on the merits.
d. Claim preclusion operates as an affirmative defense that is waived
if not properly asserted. It can also be asserted as a basis for a
summary judgment motion.
e. Claim preclusion also prevents relitigation of claims that could
have been brought in the earlier action, if they arise from the same
transaction or occurrence.
2. Issue Preclusion
a. Issue preclusion prevents relitigation of issues that were fully and
fairly litigated, and were necessarily decided in a proceeding that
reached a final judgment on the merits.
b. An issue is narrower than a claim. The litigation of a single claim may
involve the determination of several different issues.
c. For litigation of a particular factual issue to be precluded, four
elements must be met:
(1) the issue must have been litigated and determined in the prior suit;
(2) the issue must have been essential to the judgment;
(3) the prior suit must have ended in a valid final judgment on the
merits; and
(4) the party against whom preclusion is asserted must have had a full
and fair opportunity and incentive to litigate the issue in the first suit.
EXAMPLE: During a trial against John, a jury determines that
the light was red when Adam crossed the street. In a subsequent

75
OUTLINE

action against Mark arising from the same occurrence, Adam


cannot relitigate the issue of whether the light was red or green.
d. Parties
(1) Issue preclusion may not be used against someone who was
not a party to the previous action.
(2) Although the party invoking the doctrine need not have been
a party to the previous action, it would be a violation of due
process to bind a non-party (or someone not in privity with a
party) since that person had not yet had her day in court.
e. Offensive Use
(1) Issue preclusion may be used offensively by one who was not a party
to the first action against one who was a party in the earlier suit.
(2) Courts are a bit reluctant to permit offensive use of issue
preclusion and will base their determination on the following:
(a) whether the plaintiff in the second suit could have easily
joined in the first action;
(b) whether there are procedural opportunities available to the
defendant in the second suit that were unavailable in the
earlier action; and
(c) whether the defendant had incentive to litigate the issue in
the first action.
EXAMPLE: If the defendant was forced to defend in an
inconvenient forum or was unable to engage in meaningful
discovery, it may be unfair to deny the defendant the
opportunity to relitigate the issue.
(3) Use of Criminal Convictions in Civil Proceedings
(a) If a defendant has been convicted of a crime that has
an element common to an issue in a subsequent civil
proceeding, the conviction may have issue preclusive
effect in the civil case, provided that the issue was fully and
fairly litigated and was necessarily decided.
(b) An acquittal is not likely to have issue preclusive effect
in a subsequent action, as the burden of proof is more
modest in a civil case.

76
CIVIL PROCEDURE

VII. APPEALABILITY AND REVIEW

A. Availability of Interlocutory Review


1. Interlocutory orders are orders either asserting provisional relief or
made upon motion or application during the trial, but which are not final
adjudications on the merits. Interlocutory orders may not be reviewed
before final judgment with limited exceptions.
EXAMPLE: An order requiring a party to produce documents claimed to
be protected by privilege is an interlocutory order.
2. Collateral Order Doctrine
a. Appellate review is authorized for an interlocutory order rendered by
a trial judge that:
(1) conclusively determines the disputed question;
(2) resolves an important issue completely separate from the
merits of the action; and
(3) is effectively unreviewable on appeal from a final judgment.
EXAMPLE: An order determining that a party is immune from cer-
tain claims would be reviewable under the collateral order doctrine.
3. Injunctive Relief
a. Congress has given to the federal courts subject-matter jurisdiction over
appeals from an interlocutory order of the trial court that grants, denies,
continues, modifies, or dissolves an injunction [28 U.S.C. § 1292(a)(1)].
b. Interlocutory orders for injunctive relief are not appealable if a similar
suit is pending in state court.
4. Certification of Interlocutory Orders
a. Federal trial courts may certify an order for appeal when
[28 U.S.C. § 1292(b)]:
(1) the order involves a controlling question of law as to which a
substantial ground for difference of opinion exists; and
(2) an immediate appeal from the order may materially advance
the termination of the litigation.
b. Following the written certification, the party seeking appeal must
apply to the federal court of appeals, which has discretion to grant or
deny the application.
5. Federal courts of appeals have the power to issue extraordinary writs
[28 U.S.C. § 1651].
6. Multi-Claim or Multi-Party Judgments
a. When a final decision has been made on at least one of the claims in
a multi-claim or multi-party case, the trial judge may direct the entry of
a final judgment as to that claim by a finding on the record that there
is no reason to delay judgment on that claim until the entire case is
resolved [Fed. R. Civ. P. 54(b)].

77
OUTLINE

b. Any party affected by the judgment may appeal the judgment while
the rest of the action remains pending.

B. Final Judgment Rule


1. Congress has given to the federal courts subject-matter jurisdiction over appeals
from all final decisions of the federal district trial courts [28 U.S.C. § 1291].
2. The basic principle in the federal system is that only final judgments
may be taken to the appellate courts. This rule is modified by a few other
principles, which when available, provide the appellant with an avenue
for interlocutory appeal even though the underlying action has not gone
to final judgment. Appellate courts will apply different standards of review
depending on the subject matter of the appeal.
3. A final judgment is generally defined as one that disposes of all issues as
to all of the parties, it disposes of the entire case.
EXAMPLE: A judgment on behalf of the plaintiff in an action for medical
malpractice is a final judgment.

C. Scope of Review for Judge and Jury


1. Although an appellant may want an appeals court to review all errors
alleged to have occurred at the trial court level, there are restrictions on
what is reviewable:
a. with few exceptions, courts will not review errors that are not on the
record of the trial court proceeding; and
EXAMPLE: If the appellant did not raise or object to an error at a time
when the trial court had the opportunity to correct it, the issue is not
preserved and is, thus, unreviewable.
b. errors that do not affect substantial rights will be labeled harmless
and may be unreviewable.
2. Errors will receive different levels of scrutiny upon appellate review
depending on the category of error.
a. Conclusions of law will typically be reviewed de novo. De novo review
means the appellate court will conduct a non-deferential review and
address the legal issue as if it has never been addressed in the case.
b. Factual findings may be disturbed only if clearly erroneous.
c. Other discretionary determinations by the judge will be affirmed
absent some indication that the judge abused his discretion.

78
Constitutional Law
TABLE OF CONTENTS

I. THE NATURE OF JUDICIAL REVIEW

Organization of the Courts in the Federal System.......................................................................82


The United States Supreme Court...............................................................................................89

II. SEPARATION OF POWERS

The Powers of Congress..............................................................................................................92


Executive Power..........................................................................................................................99
Interbranch Checks upon the Exercise of Federal Power............................................................103

III. THE RELATION OF THE NATION AND THE STATES IN THE FEDERAL SYSTEM

Nature and Scope of Federal and State Powers..........................................................................105


Intergovernmental Immunities......................................................................................................105
Authority Reserved for the States................................................................................................107
Reserved State Power in Taxation...............................................................................................109

IV. TYPE OF ACTION GOVERNED BY THE CONSTITUTION

State Action versus Private Action...............................................................................................112


National Power to Override State Authority..................................................................................114

V. DUE PROCESS AND THE INCORPORATION OF PORTIONS OF THE BILL OF RIGHTS

Incorporation of the Bill of Rights.................................................................................................115


Procedural Due Process..............................................................................................................117
Substantive Due Process.............................................................................................................121
Takings Clause.............................................................................................................................127

VI. EQUAL PROTECTION OF THE LAWS

Constitutional Basis......................................................................................................................130
Standards of Review....................................................................................................................130
Proving Discrimination.................................................................................................................131
Suspect Classifications................................................................................................................132

VII. PRIVILEGES AND IMMUNITIES CLAUSES

Privileges or Immunities under the Fourteenth Amendment........................................................138


Privileges and Immunities under Article IV, Section 2..................................................................138

80
VIII. RETROACTIVE LEGISLATION

The Contract Clause....................................................................................................................140


Ex Post Facto Laws.....................................................................................................................140
Bills of Attainder...........................................................................................................................141

IX. FIRST AMENDMENT FREEDOMS

Freedom of Religion and Separation of Church and State..........................................................142


Freedom of Expression................................................................................................................148

81
OUTLINE

I. THE NATURE OF JUDICIAL REVIEW

A. Organization of the Courts in the Federal System


1. Federal Court System
a. Source of Federal Judicial Power
(1) Article III, Section 1 provides that the “judicial power of the
United States shall be vested in one Supreme Court and in
such inferior Courts as the Congress may from time to time
ordain and establish.”
b. Scope of Federal Judicial Power
(1) Article III, Section 2 limits the jurisdiction of the federal courts to:
(a) cases, in law and equity, arising under the U.S. Constitution,
federal laws, and treaties;
(b) cases affecting ambassadors, public ministers, and consuls;
(c) cases of admiralty and maritime jurisdiction;
(d) controversies to which the United States shall be a party;
(e) controversies between two or more states;
(f) cases between a state and citizens of another state; and
(g) cases between citizens of different states (diversity of
citizenship cases).
c. The Eleventh Amendment prohibits the citizens of one state from
suing their own state or another state in federal court on federal
claims for money damages, without the state’s consent.
(1) The states also have sovereign immunity from suits by citizens
for damages in their own state courts, unless sovereign immunity
has been waived [Alden v. Maine, 527 U.S. 706 (1999)].
(2) The Eleventh Amendment recognizes the states and their
governmental immunity.
(3) It applies not only to diversity suits but to federal question
cases as well.
(4) The concept of governmental immunity, or sovereign immu-
nity, means that the government may not be sued without its
consent. However, subdivisions of a state (e.g., cities, towns,
and counties) do not have immunity from suit under the Eleventh
Amendment [Lincoln County v. Luning, 133 U.S. 529 (1890)].
(5) Despite Congress’s enumerated powers under Article I, Section
8, the Eleventh Amendment nonetheless prohibits federal court
adjudication of claims by private parties against a state.
EXAMPLE: Congress could not use Article I to circumvent the
limitations placed on federal jurisdiction and, therefore, could
not allow a Native American tribe to sue a state in federal court
[Seminole Tribe of Florida v. Florida, 517 U.S. 44 (1996)].

82
CONSTITUTIONAL LAW

(a) However, pursuant to its enforcement powers under the


post-Civil War amendments (Thirteenth, Fourteenth,
and Fifteenth), Congress can authorize private suits by
individuals to compensate for state violations of those
amendments [Fitzpatrick v. Bitzer, 427 U.S. 445 (1979)].
(6) Exceptions to the application of the Eleventh Amendment include:
(a) suits against state officials for abusing their power in
enforcing an unconstitutional state statute [Ex parte Young,
209 U.S. 123 (1908)];
(b) federal suits brought by one state against another state, or
suits brought by the federal government against a state; and
(c) most suits for injunctions—e.g., a private citizen may sue to
enjoin a state official from acting in violation of the plaintiff’s
federal constitutional rights.
(7) A state may consent to suit in federal court if it clearly waives
its Eleventh Amendment immunity and does so expressly
and unequivocally (or by voluntarily invoking a federal court’s
jurisdiction). A state will not be held to have impliedly or
constructively waived its immunity simply because Congress
provides that a state will be subject to private suit if it engages
in certain federally regulated conduct (such as infringing a
federally granted patent) and the state voluntarily elects to
engage in that conduct [College Savings Bank v. Florida Prepaid
Postsecondary Education Expense Board, 527 U.S. 666 (1999)].
(8) Generally, Congress may abrogate a state’s immunity where:
(a) the act asserts that it is abrogating the state’s immunity; and
(b) Congress enacts the act under a grant of power that may
abrogate the state’s immunity.
EXAMPLE: State workers alleging age discrimination
under the federal Age Discrimination in Employment
Act may not sue their employers (i.e., the state) using
the federal statute as a cause of action, because
Congress lacks the power to override the state’s Eleventh
Amendment immunity from federal lawsuits in the absence
of a pattern of unconstitutional action by the states
violative of the Fourteenth Amendment [Kimel v. Board of
Regents, 528 U.S. 62 (2000)].
(9) Complete preclusion of all jurisdiction may occur in certain instances.
EXAMPLE: Probation officers owed money under federal law
cannot sue for it in either federal or state court.
EXAMPLE: States can infringe patents or copyrights and thus
violate federal law, but cannot be sued in either federal or state
court by private parties.

83
OUTLINE

(a) Remember, however, that state officials can be sued for


injunctive relief, but injured private plaintiffs have no
damage remedy against state governments.
2. Limitations on Jurisdiction of Federal Courts
a. Case or Controversy
(1) Article III, Section 2 limits the jurisdiction of federal courts to
“cases” and “controversies.” A case or controversy is a real
and substantial dispute that touches the legal relations of
parties having adverse interests and that can be resolved by a
judicial decree of a conclusive character [Aetna Life Insurance
Co. v. Haworth, 300 U.S. 227 (1937)].
(a) The Supreme Court will not give advisory opinions to either
the president or Congress concerning the constitutionality
of proposed action or legislation.

NOTE State courts may be allowed to render advisory opinions.

(b) The prohibition against advisory opinions does not


preclude federal courts from granting declaratory judgments.
A declaratory judgment is a decision in which the court
is requested to determine the legality of proposed conduct
without awarding damages or injunctive relief. However, the
plaintiff must meet the case or controversy tests, as well as
the “RAMPS” requirements explained below.

Federal Court Jurisdiction RAMPS Up

Ripeness, Abstention & Adequate State Grounds, Mootness, Political Questions, Standing

b. Mootness
(1) If a controversy or matter has already been resolved, then the
case will be dismissed as moot. An actual case or controversy
must exist at all stages of the litigation [Liner v. Jafco, Inc., 375
U.S. 301 (1964)].
(a) Although the principal issue in a lawsuit has been resolved,
if a party still has an interest in resolving collateral (or lesser)
matters, the case will not be dismissed.
EXAMPLE: A case about wrongful termination is not moot even
though the plaintiff had his employment restored if issues of
back pay remain [Powell v. McCormick, 395 U.S. 486 (1969)].
(b) The case will not be dismissed for mootness if the injury is
“capable of repetition, yet evading review,” meaning that it is
a practical impossibility for there to be adjudication or appel-
late review before the claims of the plaintiff, or other individ-
uals who are members of the class, become moot.

84
CONSTITUTIONAL LAW

EXAMPLE: A pregnant woman’s suit challenging the consti-


tutionality of a state abortion statute was held not to be moot
even though she was no longer pregnant at the time the case
reached the Supreme Court. Pregnancy is “capable of repeti-
tion, yet evading review,” for the plaintiff and for members of
the class she represents [Roe v. Wade, 410 U.S. 113 (1973)].
c. Ripeness
(1) Whereas mootness bars consideration of claims after they have
been resolved, ripeness bars consideration of claims before
they have fully developed.
(a) Generally, a court may not review or grant a declaratory
judgment of a state law before it is enforced or when there
is no real threat the statute will ever be enforced [Poe v.
Ullman, 367 U.S. 497 (1961)].
EXAMPLE: The Supreme Court dismissed a claim against
the Army’s data-gathering activities because there was no
showing that the Army’s surveillance system resulted in
any specific present harm or threat of future harm to the
complainants [Laird v. Tatum, 408 U.S. 1 (1972)].
EXAMPLE: The Supreme Court refused to grant equitable
relief because there was no evidence that the pattern of al-
legedly unconstitutional racial discrimination in the bail and
sentencing practices in the Cairo, Illinois court system threat-
ened the plaintiffs, who were not in imminent danger of being
prosecuted [O’Shea v. Littleton, 414 U.S. 488 (1974)].
EXAMPLE: The Supreme Court did rule on the
constitutionality of a statute prohibiting the teaching of
evolution in public schools even though the statute had not
been enforced [Epperson v. Arkansas, 393 U.S. 97 (1968)].
(b) However, if the plaintiff can show, before enforcement of
the law, that the law presents a specific or present harm
or a threat of specific future and imminent harm, the court
may grant a declaratory judgment [Abbott Laboratories v.
Gardner, 387 U.S. 136 (1967)].
d. Abstention
(1) The federal court may abstain, or refuse to hear a particular
case, when there are undecided issues of state law presented.
The abstention doctrine permits the state court to resolve issues
of state law, thereby making a decision of the constitutional issue
unnecessary. Thus, proper deference is paid to the state court
system and harmonious federal-state relations are furthered
[Railroad Commission v. Pullman, 312 U.S. 496 (1941)].
(a) The federal court may abstain if the meaning of a state
law or regulation is unclear. In this situation, the state

85
OUTLINE

court might interpret the statute so as to avoid the consti-


tutional issue [Harris County Commissioners Court v.
Moore, 420 U.S. 77 (1975)].
(b) Where state criminal proceedings are pending, the federal
court will abstain in a suit seeking an injunction against the
state prosecution, absent a showing of bad-faith harassment
on the part of the state prosecutors [Younger v. Harris, 401
U.S. 37 (1971)]. This principle has been extended to cases:
1) where state civil proceedings had commenced (seeking
an injunction against operation of a state public nuisance
statute used to close a pornography movie house)
[Huffman v. Pursue, Ltd., 420 U.S. 592 (1975)]; and
2) where civil contempt hearings had begun [Judice v.
Vail, 430 U.S. 327 (1977)].
(2) There are two areas where federal courts traditionally decline to
hear cases.
(a) Under the probate exception to federal court jurisdiction,
federal courts always decline to probate estates, viewing
these as traditionally belonging to the state courts.
(b) Federal courts generally do not hear family law claims (i.e.,
divorce, child custody, or child support matters), as these
belong to states. This is known as the family law exception.
e. Standing
(1) Article III requires a person litigating a constitutional question to show:
(a) injury-in-fact;
1) The plaintiff must show a direct and personal injury,
actual or imminent, caused by the action that he is
challenging. Where the plaintiff has not suffered any
personal injury or harm, he does not have standing
[Sierra Club v. Morton, 405 U.S. 727 (1972)].
(b) causation (the injury was caused by the challenged action); and
1) The injury must be caused by the violation of a duty
affecting the plaintiff’s rights arising under the constitu-
tion or federal law [Simon v. Eastern Kentucky Welfare
Rights Organization, 426 U.S. 26 (1976)].
(c) redressability.
1) The plaintiff must show that he will benefit from the
remedy sought in the litigation.
EXAMPLE: Environmentalists sued the government
because they did not receive notice when the
government sold burnt timber after a particular forest
fire. After settling the suit, the environmentalists
brought a second suit, asking the court to require the
government to give notice before conducting any future

86
CONSTITUTIONAL LAW

timber-salvage activities. Held: The environmentalists


had standing in the first action because they suffered
the concrete harm of being excluded from the sale of
particular burnt timber. However, they lacked standing in
the second action, because there was no new imminent
harm caused by the government’s general policy of not
giving notice before timber-salvage activities [Summers
v. Earth Island Institute, 555 U.S. 488 (2009)].
(2) Specialized Problems of Standing
(a) As a general rule, federal taxpayers do not have standing
to challenge allegedly unconstitutional federal expen-
ditures on the grounds that their injury is comparatively
minute and indeterminative, and their interest is too remote
[Massachusetts v. Mellon, 262 U.S. 447 (1923)].
EXCEPTION: In 1968, the Supreme Court held that a federal
taxpayer had standing to challenge federal expenditures to
aid parochial schools where the taxpayer was challenging
the expenditure of money by Congress under its taxing and
spending power and the expenditure allegedly violated the
Establishment Clause [Flast v. Cohen, 392 U.S. 83 (1968)].

NOTE The Flast v. Cohen principle has not been extended to other areas of
government activity.

EXAMPLE: Taxpayers did not have standing to challenge a


transfer of valuable real estate, under the Property Clause,
to a Christian college from a local government [Valley Forge
Christian College v. Americans United for Separation of
Church and State, 454 U.S. 464 (1982)].
EXAMPLE: Citizens did not have standing to challenge a sena-
tor’s or representative’s service in the Armed Forces Reserve as
violating Article I, Section 6, prohibiting a member of Congress
from “holding any office under the United States” [Schlesinger
v. Reservists Committee to Stop the War, 418 U.S. 208 (1974)].
(b) Under the traditional view, a litigant lacks standing to assert
the rights of third parties not before the court [Tileston v.
Ullman, 318 U.S. 44 (1943)].
1) The Supreme Court has permitted a party to raise the
constitutional rights of a third party where he himself
has suffered injury and:
a) a special relationship exists between the claimant
and third party because of the connection between
the interests of the claimant and the constitutional
rights of the third person; and

87
OUTLINE

EXAMPLE: The Supreme Court acknowledged the


right of a physician to raise the rights of his patients in
challenging an abortion ruling because of the close re-
lationship between the doctor and his patient, and also
because the patient was unable to bring suit on her
own behalf [Singleton v. Wulff, 429 U.S. 106 (1976)].
EXAMPLE: A vendor of beer had standing to
assert the rights of males under the age of 21 in
a challenge to a law prohibiting the sale of beer to
them [Craig v. Boren, 429 U.S. 190 (1976)].
EXAMPLE: A seller of contraceptives had standing to
assert the rights of potential purchasers against a law
prohibiting the sale of such devices [Carey v. Popu-
lation Services International, 431 U.S. 678 (1977)].
b) the third party is unable or finds it difficult to bring
suit on his own behalf.
2) An association has standing to assert the claims of its
members, even if the association has not suffered any
injury itself, if [Hunt v. Washington Apple Advertising
Commission, 432 U.S. 333 (1977)]:
a) the members would otherwise have standing to
sue in their own right;
b) the interest asserted is germane to the
association’s purpose; and
c) neither the claim asserted nor the relief requested
would require participation by the individual
members in the lawsuit.
f. Political Questions
(1) Federal courts cannot hear cases involving political questions. A
political question is a matter assigned to another branch by the
constitution or incapable of a judicial answer. the Supreme Court set
forth the following relevant factors to consider to determine if the polit-
ical question doctrine applies [Baker v. Carr, 369 U.S. 186 (1962)]:
(a) whether there is a textually demonstrable constitutional
commitment of the issue to a coordinate political department;
(b) a lack of judicially discoverable and manageable standards
for resolving it;
(c) the impossibility of deciding without an initial policy
determination of a kind clearly for nonjudicial discretion;
(d) the impossibility of a court’s undertaking independent
resolution without expressing lack of respect due
coordinate branches of government;
(e) an unusual need for unquestioning adherence to a
political decision already made; and

88
CONSTITUTIONAL LAW

(f) the potential for embarrassment from multifarious


pronouncements by various departments on one question.
(2) There are two principal factors related to the political question
doctrine. First, considerations (a), (d), (e), and (f) enumerated
above are rooted in the separation of powers. On the other hand,
considerations (b) and (c) recognize the limitations of the judiciary
in resolving certain types of controversies.
EXAMPLE: the ability of a grand jury to subpoena documents in
the possession of the president against a claim of executive privi-
lege does not present a political question apart from a claim based
on national security [United States v. Nixon, 418 U.S. 683 (1974)].
(3) Other areas of political questions include decisions in regard to:
(a) the impeachment process;
(b) the amendment ratification process;
(c) the president’s power to unilaterally terminate a treaty;
(d) foreign affairs; and
(e) Guaranty Clause issues under Article IV.

NOTE A claim that a state has redrawn its electoral districts in a racially discrimi-
natory manner is not a political question. However, a claim that a state has
redrawn electoral districts to benefit one political party is a political question
and, therefore, is nonjusticiable. The Supreme Court has ruled that there
are manageable judicial standards for deciding claims of race discrimination,
but that there are no such standards for deciding when a political party has
been unfairly advantaged when electoral districts are redrawn.

B. The United States Supreme Court


1. Jurisdiction of the Supreme Court
a. Under Article III, Section 2, the Supreme Court has original jurisdic-
tion “in all cases affecting ambassadors, other public ministers and
consuls and those in which a state shall be a party.” Congress may
neither enlarge nor restrict the Supreme Court’s original jurisdiction
[Marbury v. Madison, 5 U.S. 137 (1803)].
b. Article III, Section 2 further provides that “in all other cases before
mentioned, the Supreme Court shall have appellate jurisdiction, both
as to law and fact, with such exceptions, and under such regulations
as the Congress shall make.” The Supreme Court has the power to:
(1) hold acts of the other branches of the federal government (the
executive branch and Congress) unconstitutional [Marbury v.
Madison, 5 U.S. 137 (1803)];
(2) hold state statutes unconstitutional [Fletcher v. Peck, 10 U.S. 87 (1810)];
(3) review state court decisions to ensure that the states act in
conformity with the U.S. Constitution and federal statutes [Martin
v. Hunter’s Lessee, 14 U.S. 304 (1816)]; and

89
OUTLINE

(4) decide other state law questions.


(a) In cases of diversity jurisdiction:
1) the Supreme Court will defer to an existing state court
interpretation of state law;
2) the Supreme Court will interpret a state law which has
not already been interpreted by the state court based
on the Supreme Court’s prediction of how the state
court will interpret the statute; and
3) the Supreme Court will abstain from a decision if the
state court’s interpretation of unsettled state law could
end the dispute and the Supreme Court cannot predict
how the state court will rule.
c. Under federal law, there are two methods for invoking Supreme
Court appellate jurisdiction [28 U.S.C. § 1257]:
(1) by appeal (where jurisdiction is mandatory); and
(a) In 1988, an act of Congress reduced obligatory review on
appeal to decisions of three-judge federal district courts.
(2) by writ of certiorari (discretionary review where four or more
justices vote to hear the case).
(a) Since the two categories of decisions that were subject
to obligatory review on appeal have been eliminated,
practically all decisions from state supreme courts and
federal courts are now reviewable by a writ of certiorari,
except decisions made purely on state law.
(b) Grounds for certiorari include:
1) cases involving conflicts between different federal
courts of appeal;
2) cases involving conflicts between the highest courts of
two states;
3) cases involving conflicts between the highest state
court and a federal court of appeals; or
4) cases from state courts or U.S. courts of appeal
involving important, yet unresolved, issues.
d. Adequate and Independent State Grounds
(1) Although a state court decision may involve a federal question,
if the state court judgment can be supported on an adequate
and independent state ground, the Supreme Court will not
take jurisdiction. To do so would be tantamount to rendering an
“advisory opinion” [Herb v. Pitcairn, 324 U.S. 117 (1945)].
(a) Unlike other doctrines of judicial review, which apply to the
entire federal judiciary, adequate state grounds apply only
to the Supreme Court.
(2) Where a state court clearly states that a state law violates other state
law or a provision of the state constitution, that decision will be an

90
CONSTITUTIONAL LAW

adequate and independent state ground (regardless of whether the


opinion also decides that the state law violates a federal law as well).
(a) Where a state court holds that a state law violates both the
state and federal constitutions, the doctrine of adequate
state grounds will apply.
(3) Where a state court’s decision is based upon a federal interpretation
of a similar federal law, adequate and independent state grounds
will not apply. The Supreme Court may review in this situation.
(4) Where it is unclear whether the state court made its decision based
upon state or federal interpretations of statutes, the Supreme Court
may take the case, although it also has the power in that situation
to dismiss or remand the case for clarification from the state court.
(a) The independence prong focuses on whether it is “apparent
from the four corners” of the opinion that the state court judg-
ment was settled based on a state court interpretation of state
law precedent or the state’s constitution. If not, the Supreme
Court may [Michigan v. Long, 463 U.S. 1032 (1983)]:
1) obtain clarification from the state court; or
2) presume that the state court decision was rooted—at
least in part—in federal law, and review the case.

91
OUTLINE

II. SEPARATION OF POWERS

A. The Powers of Congress


1. Legislative Power
a. Legislative power is primarily the power to make laws, but incidental to that
power is the right to conduct investigations and hearings, consider matters
upon which legislation may be enacted, and do all other things “necessary
and proper” to the enactment of legislation [U.S. Const. art. I, § 1].
(1) Congress has enumerated powers to collect taxes and spend
money for the general welfare, to borrow money on the credit
of the United States, to regulate commerce with foreign nations
and among the several states, to declare war, and to raise and
support the army, navy, and militia [U.S. Const. art. I, § 8].
(2) The Necessary and Proper Clause gives Congress the implied
power “to make all Laws which shall be necessary and proper for
carrying into Execution the foregoing Powers, and all other Powers
vested by this Constitution in the Government of the United States,
or any Department or Officer thereof” [U.S. Const. art. I, § 8].
(3) The Enabling Clauses of the Thirteenth, Fourteenth, and
Fifteenth Amendments give Congress the power to enforce those
amendments by “appropriate legislation.”
2. Commerce Power
a. Despite the broad language used by the Supreme Court in Gibbons v.
Ogden [22 U.S. 1 (1824)], subsequent cases used various criteria to
restrict the power of Congress to regulate “commerce which concerns
more states than one.” Today, Congress can regulate:
(1) channels of interstate commerce (i.e., highways, waterways,
and air traffic);
(2) instrumentalities of interstate commerce (i.e., cars, trucks, ships,
and airplanes); and
(3) activities that “substantially affect” interstate commerce.
(a) Under the affectation doctrine, Congress now has the
power to regulate any economic activity, whether carried on
in one state or many, that has a substantial effect (whether
directly or indirectly) upon interstate commerce.
1) This doctrine was first formulated in National Labor
Relations Board v. Jones and Laughlin Steel Co. [301
U.S. 1 (1937)], which upheld the constitutionality of
the Wagner Act in requiring collective bargaining in all
industries “affecting” interstate commerce.
(b) The affectation doctrine was expanded upon by the cumula-
tive effect doctrine, in which the Supreme Court held that
the federal commerce power permitted regulation of the
amount of wheat a farmer could grow on his own land, for

92
CONSTITUTIONAL LAW

his own consumption, because his activity, together with that


of other growers of wheat for their own consumption, had
a substantial cumulative effect upon interstate commerce
[Wickard v. Filburn, 317 U.S. 111 (1942)].
(c) Congress’s plenary commerce power is not without limits. In
United States v. Lopez [514 U.S. 549 (1995)], for the first time
in more than half a century, the Supreme Court struck down a
federal law that made it a crime for any individual knowingly to
possess a firearm in a school zone. Since gun possession near
schools is neither itself an “economic” activity nor an activity
that “substantially” affects interstate commerce, and since no
“jurisdictional element” connecting particular gun possession
to interstate commerce was expressed in the language of the
statute, the Supreme Court held that Congress was acting
beyond the limits of the Commerce Clause.

NOTE To validly exercise its Commerce Clause power under the “substantial effects”
test, Congress must now show: (1) that the regulated activity is “economic” in
nature; and (2) that the regulated activity (when taken cumulatively through-
out the nation) has a substantial effect on interstate commerce.

(d) The Commerce Clause has been used as the vehicle to


uphold laws aimed at barring racial discrimination in activities
connected with interstate commerce.
EXAMPLE: Congress may prohibit racial discrimination in
private restaurants if a substantial portion of the food con-
sumed traveled in interstate commerce [Katzenbach v. Mc-
Clung, 379 U.S. 294 (1964)].
EXAMPLE: The Court has upheld provisions of the Civil Rights
Act of 1964 barring discrimination in places of public accommoda-
tion (e.g., as applied to a motel) as “affecting” interstate commerce
[Heart of Atlanta Motel v. United States, 379 U.S. 241 (1964)].
(e) The Supreme Court has upheld, under the commerce power,
a federal law prohibiting extortionate credit transactions (i.e.,
loan sharking) on the grounds that such transactions provide
a major source of revenue for organized crime, and that
organized crime, in turn, has an adverse effect upon interstate
commerce [Perez v. United States, 402 U.S. 146 (1971)].
(f) The Tenth Amendment, which provides that powers not dele-
gated to the federal government are reserved to the states, or
to the people, serves as a very weak limitation on the federal
commerce power today. Generally applicable federal laws
that regulate the states (state businesses and/or employees)
do not violate the Tenth Amendment [Garcia v. San Antonio
Metropolitan Transit Authority, 469 U.S. 528 (1985)].

93
OUTLINE

NOTE The Tenth Amendment does prevent Congress from interfering with a state’s
lawmaking processes. Congress may not commandeer the legislative pro-
cesses of the states by directly compelling them to enact and enforce a fed-
eral regulatory program.

EXAMPLE: The Supreme Court has held invalid as violating


the Tenth Amendment a federal law that required the state
of New York to pass legislation to arrange for the disposal of
radioactive waste generated within its borders [New York v.
United States, 505 U.S. 144 (1992)].

NOTE The Tenth Amendment also prevents Congress from commandeering state
executive officials. That is, Congress may not order state officials to act as
administrators of federal programs.

EXAMPLE: The Supreme Court held unconstitutional the


federal Brady Handgun Violence Protection Act, because it
commandeered state and local law enforcement officers for
performance of various acts required by federal legislation
[Printz v. United States, 521 U.S. 898 (1997)].
3. Taxing Power
a. Congress has the power to lay and collect taxes, duties, imposts, and
excises to pay the debts and provide for the common defense and
general welfare [U.S. Const. art. I, § 8].

EXAM TIP A congressional act purporting to be a “tax” should be upheld as a valid


exercise of the taxing power provided that it does, in fact, raise revenue (the
objective test) or that it was intended to raise revenue (the subjective test).

NOTE The Sixteenth Amendment gives Congress the power to collect taxes on
incomes derived from any source.

b. Congress has used its taxing power as a necessary and proper means
of achieving a regulatory effect. Generally, as long as Congress has
the power to regulate the activity taxed, the tax can then be used as a
regulating device rather than for revenue-raising purposes.
EXAMPLE: The Supreme Court sustained a tax on coal producers who
violated a fair competition law, even though it was clearly designed to
be a penalty rather than revenue-raising [Sunshine Anthracite Coal Co.
v. Adkins, 310 U.S. 381 (1940)].
(1) Even when Congress does not have power to regulate the
activity taxed, the tax will nevertheless be upheld if its “domi-
nant intent” is revenue-raising. Thus, even though the tax may
have a substantial regulatory effect, if the tax in fact raises
revenue, it will be valid.

94
CONSTITUTIONAL LAW

EXAMPLE: The Supreme Court upheld as revenue-raising a dis-


criminatory tax on colored oleomargarine, even though Congress
did not have the power to regulate its production directly [McCray
v. United States, 195 U.S. 27 (1904)].
(2) As a general rule, the modern judicial trend is to uphold any tax
as valid if it is, in fact, a revenue-raising measure.
EXAMPLE: Although a tax on bookmaking activities had a regu-
latory effect, it was upheld as producing revenue (albeit negligible
in amount) [United States v. Kahriger, 345 U.S. 22 (1953)].
EXAMPLE: The taxing power was held by the Supreme Court
to justify a penalty on individuals who opt not to purchase health
insurance [National Federation of Independent Business v.
Sebelius, 567 U.S. 519 (2012)].
4. Spending Power
a. Congress has the power to lay and collect taxes “to pay the Debts and
provide for the common Defense and General Welfare of the United
States” [U.S. Const. art. I, § 8, cl. 1].
(1) The Supreme Court has construed the General Welfare Clause
as a limitation on Congress’s taxing and spending powers and
not as an independent source of congressional power. In other
words, Congress’s power to tax and spend must be exercised
for the “general welfare” of the United States [United States v.
Butler, 297 U.S. 1 (1936)].
EXAMPLE: The Federal Election Campaign Act, which limited the
amount a candidate could spend if he accepted money from the
federal government, is a valid exercise of the power to spend for
the “general welfare.” The Supreme Court sustained the power
of Congress to finance elections because this would reduce the
harmful effect of candidates’ reliance on large private contributions
[Buckley v. Valeo, 424 U.S. 1 (1976)]. However, the power is not
unlimited. Threatening the states with complete loss of Federal
Medicaid funding if the states refused to comply with the expansion
of the program was held unconstitutional [National Federation of
Independent Business v. Sebelius, 567 U.S. 519 (2012)].
(2) By exercising its spending power, Congress can require states to
comply with specified conditions in order to qualify for federal funds.
EXAMPLE: Congress conditioned further grants of highway
funds upon the resignation of a state highway administrator who
was also the state Democratic Party chairman [Oklahoma v. Civil
Service Commission, 330 U.S. 127 (1947)].
(3) The Supreme Court held that Congress may place a condition on
the receipt of federal funds by a state if [South Dakota v. Dole,
483 U.S. 203 (1987)]:

95
OUTLINE

(a) the spending serves the general welfare;


(b) the condition is unambiguous;
(c) the condition relates to the federal program;
(d) the state is not required to undertake unconstitutional
action; and
(e) the amount in question is not so great as to be considered
coercive to the state’s acceptance.
EXAMPLE: The Supreme Court rejected South Dakota’s
Tenth and Twenty-First Amendment arguments and upheld
Congress’s withholding of federal highway funds from states
permitting the purchasing of alcoholic beverages by individu-
als under age 21.
5. War and Defense Powers
a. Congress may [U.S. Const. art. I, § 8]:
(1) declare war;
(2) raise and support armies;
(3) provide and maintain a navy; and
(4) organize, arm, discipline, and call forth a militia.
b. The war power confers upon Congress a very broad authority to initiate
whatever measures it deems necessary to provide for the national
defense, in peacetime as well as in wartime.
(1) Thus, the military draft and selective service systems have repeat-
edly been upheld [United States v. O’Brien, 391 U.S. 367 (1968)].
(2) Similarly, the power of Congress to initiate wage, price, and
rent control of the civilian economy during wartime has been
sustained [Yakus v. United States, 321 U.S. 414 (1944)].
c. The exclusion of civilians from certain restricted areas during wartime
has been approved [Korematsu v. United States, 323 U.S. 214
(1944)]. Congress has the power to establish military courts and tribu-
nals. Since military courts are not Article III courts, the accused in court-
martial proceedings is not entitled to the same procedural safeguards
set forth in the Bill of Rights (e.g., the right to jury trial or grand jury indict-
ment). Rather, an accused is safeguarded by the procedures provided in
the Uniform Code of Military Justice [U.S. Const. art. I, § 8, cl. 14].
(1) Military courts have jurisdiction over offenses committed by
servicemen on a military post or in an area under military control.
Jurisdiction is determined by the status of the individual as an
armed service member, not the connection between service and
the offense [Solorio v. United States, 483 U.S. 435 (1987)].
(2) Military courts also have jurisdiction over current members of the armed
forces, even while on a pass or on leave, for nonservice matters.

96
CONSTITUTIONAL LAW

(3) As long as civilian courts are available, military courts are denied
jurisdiction over civilians and their dependents unless Congress
has affirmatively granted such jurisdiction.
EXAMPLE: The Supreme Court held that, absent military exi-
gency, the president may not try alleged terrorists in special military
courts if Congress has enacted a statute denying the president that
authority. Congress, however, may affirmatively grant that authority
to the president [Hamdan v. Rumsfeld, 548 U.S. 557 (2006)].
(4) Although civilian courts have no general power of review over
court-martial proceedings, there can be a limited examination of
the regularity of such proceedings by a civilian court [Burns v.
Wilson, 339 U.S. 103 (1950)].
6. Investigatory Power
a. Although Congress does not have any express constitutional power
to investigate, the Necessary and Proper Clause permits Congress
to conduct investigations incident to its legislative power [McGrain v.
Daugherty, 273 U.S. 135 (1927)].
(1) Congress’s investigatory power is broad, and it may extend to
any matter within a “legitimate legislative sphere.”
(2) If a witness fails to appear after being summoned before a
congressional committee or fails to answer a question posed by
such a committee, Congress may either:
(a) cite the witness for contempt; or
(b) refer the matter to the U.S. attorney general for prosecution.
(3) Generally, a witness can raise as a defense:
(a) the privilege against self-incrimination;
(b) lack of due process safeguards; or
(c) interference with the First Amendment rights to privacy and
freedom of association.
EXAMPLE: The Supreme Court upheld the refusal of a
member of the NAACP to release membership lists to a leg-
islative committee because there was not a sufficient nexus
between the investigation and the records sought [Gibson v.
Florida Legislative Committee, 372 U.S. 539 (1963)].
7. Property Power
a. Congress has the power “to dispose of and make all needful Rules and
Regulations respecting the Territory or other Property belonging to the
United States” [U.S. Const. art. IV, § 3].
8. Power of Eminent Domain
a. Although the Constitution does not expressly give Congress the power
of eminent domain, the power to take property is “implied in aid of
the other powers granted to the federal government” [Kohl v. United
States, 91 U.S. 367 (1876)].

97
OUTLINE

b. This power is limited, however. The Fifth Amendment provides, in


part, that private property shall not be taken for public use without
just compensation.

EXAM TIP Be sure to consider and include an analysis under the Takings Clause.

9. Admiralty and Maritime Power


a. The Supreme Court has determined that the Necessary and Proper
Clause gives Congress complete and plenary power to fix and deter-
mine the maritime laws throughout the country [Southern Pacific Co. v.
Jensen, 244 U.S. 205 (1917)].
10. Bankruptcy Power
a. Congress has the power “to establish uniform Laws on the subject of
Bankruptcies throughout the United States” [U.S. Const. art. I, § 8, cl. 4].
11. Postal Power
a. Congress may establish post offices and post roads [U.S. Const.
art. I, § 8, cl. 7].

NOTE The post office has now been moved out of the Cabinet and is a self-operat-
ing entity within the government.

12. Copyright and Patent Power


a. Congress may “promote the Progress of Science and useful Arts, by
securing for limited Times to Authors and Inventors the exclusive Right to
their respective Writings and Discoveries” [U.S. Const. art. I, § 8, cl. 8].
13. Speech and Debate Clause
a. “The Senators and Representatives…for any Speech or Debate in
either House…shall not be questioned in any other Place” [U.S.
Const. art. I, § 6, cl. 1].
b. The Speech and Debate Clause protects legislators and their aides
against criminal or civil proceedings for “legislative acts” [United States
v. Johnson, 383 U.S. 169 (1966); Gravel v. United States, 408 U.S. 606
(1972)]. Members of Congress, however, can be liable based on defa-
mation for issuing press releases or newsletters that injure reputation
[U.S. Const. art. I, § 6, cl. 1].
(1) The clause insulates members of Congress for “acts that occur
in the regular course of the legislative process,” but does not
bar prosecution for taking a bribe to influence legislation [United
States v. Brewster, 408 U.S. 501 (1972)].
(2) The Speech and Debate Clause forbids the courts from enjoining
members of Congress or their aides from issuing subpoenas or
conducting their inquiry [Eastland v. United States Servicemen’s
Fund, 421 U.S. 491 (1975)].
14. Civil War Amendments
a. Congress has the power to enforce:

98
CONSTITUTIONAL LAW

(1) the Thirteenth Amendment, banning slavery;


(2) the Fourteenth Amendment, prohibiting the states from violating
protections regarding due process, equal protection, and privi-
leges and immunities; and
(3) the Fifteenth Amendment, prohibiting the states from discrimi-
nating in voting rights.
b. To validly enforce the Fourteenth or Fifteenth Amendment, Congress
must show that:
(1) state governments have engaged in widespread violations of the
amendment; and
(2) the legislative remedy is “congruent with and proportional to”
the violations.
c. Congress may enforce the Thirteenth Amendment ban on slavery even
when the state governments have not violated the amendment. However,
Congress may enforce the Fourteenth and Fifteenth Amendments only
when state governments have widely violated those Amendments.

B. Executive Power
1. Chief Executive
a. “The Executive Power shall be vested in a President of the United
States.” This provision confers broad authority in the president to
execute the laws of the United States. There are few enumerated
powers expressly granted to the president under Article II. Many of the
president’s domestic and foreign powers are implied. The Supreme
Court has held that the president has no power to make laws, but has
the power to execute them [U.S. Const. art. II, § 1].
(1) Appointment Power
(a) Article II gives the president the power, “with the Advice
and Consent of the Senate,” to nominate and appoint all
“Ambassadors, other public Ministers and Consuls, Judges
of the Supreme Court, and all other Officers of the United
States, whose appointments are not herein otherwise
provided for” [U.S. Const. art. II, § 2].

NOTE The presidential appointment power should be construed as a limitation on


the congressional appointment power. Although Congress can appoint of-
ficials to its legislative committees, it cannot appoint members to any agency
or commission with administrative powers.

EXAMPLE: An attempt to vest the power to appoint mem-


bers of the Federal Elections Commission in the speaker
of the House of Representatives was held unconstitutional
[Buckley v. Valeo, 424 U.S. 1 (1976)].
(b) The president, therefore, has the exclusive power to
nominate high-level officials such as cabinet members,

99
OUTLINE

ambassadors, and heads of agencies. The Senate, however,


has the power to confirm or reject the president’s nominees
for such high-level appointments.
(c) Congress can delegate the appointment of “inferior” officers
(including special prosecutors) to:
1) the president;
2) the judiciary; or
3) heads of departments.
(2) Removal Power
(a) Although the U.S. Constitution is silent with respect to
removal power, it is generally agreed that the president may
remove any executive appointee (e.g., an ambassador or
cabinet member) without cause.
(b) However, the president must have cause (i.e., good reasons)
to remove executive officers having fixed terms and officers
performing judicial or quasi-judicial functions (e.g., a member
of the Federal Trade Commission).
(c) Congress has no power to summarily remove an executive
officer [Bowsher v. Synar, 478 U.S. 714 (1986)].
(d) Federal judges cannot be removed by either Congress or
the president during “good behavior”; formal impeachment
proceedings are required for removal.
(3) Veto Power
(a) Once Congress has passed legislation and presented it to
the president, the president has 10 days in which to act upon
it. Unless he vetoes it within the 10-day period, the proposed
legislation will become law [U.S. Const. art. I, § 7].
(b) The president can pocket veto a bill passed within 10 days
of the end of the congressional term by not signing it.
(c) Congress has the power to override a veto by a two-thirds
vote of both the Senate and House of Representatives [U.S.
Const. art. I, § 7].
(d) A line-item veto, which gives the President the power to
cancel particular provisions of new federal legislation, has
been held unconstitutional as a grant of power to the presi-
dent to amend a statute. Amendments are valid only when
Congress enacts a new statute followed by a presidential
signature [Clinton v. City of New York, 524 U.S. 417 (1998)].
(4) Congressional Legislation and Presidential Power
(a) The president may exercise only those powers expressly
or impliedly granted by the Constitution or an act of
Congress. Moreover, if Congress validly exercises one
of its powers and overrides the president, then Congress
prevails over the president.

100
CONSTITUTIONAL LAW

EXAMPLE: The president may not validly use his power as


commander-in-chief to seize a factory in order to prevent a strike
during wartime if Congress opposes such action [Youngstown
Sheet and Tube Co. v. Sawyer, 343 U.S. 579 (1952)].
(b) Congress can enact legislation that delegates rulemaking
power to an executive or administrative agency in some
designated subject area (such as securities law, labor law,
environmental law, and so on), creating a huge exception to
the general rule that the executive branch may not make law.
(c) The Supreme Court has found that Congress has the power
to delegate the authority to make rules having the power of
law to the executive branch as long as Congress provides
an intelligible principle to guide the executive branch [J.W.
Hampton, Jr., & Co. v. United States, 276 U.S. 394 (1928)].
(5) Pardon Power
(a) The president may “grant reprieves and pardons for
offenses against the United States, except in cases of
impeachment” [U.S. Const. art. II, § 2].
(6) Executive Privilege
(a) The president has an absolute privilege to refuse to
disclose information relating to military, diplomatic, or
sensitive national security secrets.
(b) Other confidential communications between the president
and his advisors are presumptively privileged [United
States v. Nixon, 418 U.S. 683 (1974)]. The presumption
is overcome when confidential communications are
subpoenaed as evidence in a criminal trial.
(7) Obligation to Report
(a) The president “shall from time to time give to the Congress
Information of the State of the Union, and recommend
to their Consideration such Measures as he shall judge
necessary and expedient.” This obligation has evolved
into the annual “State of the Union” address given by the
president [U.S. Const. art. II, § 3].
(8) Faithful Execution
(a) The Constitution requires that the president “take Care that
the Laws be faithfully executed” [U.S. Const. art. II, § 3].
This has been referred to as the Take Care Clause, Faithful
Execution Clause, and Faithfully Executed Clause.
(b) This clause has been interpreted as placing a duty on the
president and executive branch to enforce the laws of the
United States, even where the president may disagree with
the purpose of that law. It does not imply a power to forbid a
law’s execution [Kendall v. United States, 37 U.S. 524 (1838)].

101
OUTLINE

(c) However, while the exact scope of this has never been
defined, courts have construed the provision as vesting the
executive branch “with substantial discretion in choosing
when and how to prosecute cases” [United States v. Bolden,
353 F.3d 870 (10th Cir. 2003)].
2. Commander-in-Chief
a. “The President shall be the Commander in Chief of the Army and
Navy of the United States, and of the militia of the several States
when called into the actual service of the United States by Congress”
[U.S. Const. art. II, § 2].
(1) Military Powers
(a) The president has the power to deploy military forces
without a formal declaration of war in response to an attack
upon the United States (as determined by the Prize Cases)
[Brig Amy Warwick, 67 U.S. 635 (1863)].
(b) He additionally has the power to seize private property
during wartime unless Congress denies him that power
[United States v. Pewee Coal Co., 341 U.S. 114 (1951)].
(c) However, the president does not have the power to declare war.
(2) Unsettled Areas
(a) Clearly, the president may commit troops to repel a sudden
attack on the United States. However, certain issues remain
unclear, such as:
1) whether the president may commit forces without
congressional approval to aid a U.S. ally under attack
(although provisions in certain defense treaties would
authorize such intervention); and
2) whether the president may order a preemptive strike in
anticipation of an enemy attack.
3. International Affairs
a. Treaty Power
(1) The president has the power to make treaties with foreign
nations “by and with the Advice and Consent of the Senate”
[U.S. Const. art. II, § 2, cl. 2]. Treaties require consent of two-
thirds of the Senate before they can be enacted.
(a) A treaty is self-executing when it takes effect without the
necessity of any further action by Congress beyond consent
or ratification [Missouri v. Holland, 252 U.S. 416 (1920)].
(b) A treaty is not self-executing when it requires Congress
(or state legislatures) to pass legislation to implement its
provisions—i.e., requiring a change in either federal or state
law which would enable fulfillment of treaty obligations.
(2) In accordance with the Supremacy Clause, treaties are “the
supreme Law of the land; and the Judges of every State shall

102
CONSTITUTIONAL LAW

be bound thereby, any Thing in the Constitution of Laws of any


State to the Contrary notwithstanding” [U.S. Const. art. VI, cl. 2].
(a) Self-executing treaties are considered the supreme law
of the land, whereas treaties which are not self-executing
become part of the supreme law of the land, and thus
supersede existing federal and state statutes, only when
further legislative action is taken by Congress.

NOTE Treaties take precedence over any conflicting state law regardless of whether
a treaty precedes or follows the enactment of the state law [Nielsen v. John-
son, 279 U.S. 47 (1929)]. However, when a treaty and federal statute conflict
on the issue in question, time determines the controlling authority—the last-
in-time will prevail.

b. Executive Agreements
(1) The president has the power to enter into executive agreements
and compacts with foreign nations. Such agreements are valid
and prevail over inconsistent state law [United States v. Belmont,
301 U.S. 324 (1937)].
(2) Executive agreements are the sole responsibility of the president,
and need not be ratified by Congress.
(3) Executive agreements do not prevail over federal statutes.

TREATIES AND EXECUTIVE AGREEMENTS: PRECEDENCE CHART


U.S. Constitution Federal Law State Law

Treaties Constitution prevails. Last-in-time prevails. Treaty prevails.

Executive Agreements Constitution prevails. Conflicting federal Executive agreement


statute prevails. prevails.

C. Interbranch Checks upon the Exercise of Federal Power


1. Congressional Limits on the Executive
a. Foreign Affairs
(1) If Congress is acting within its constitutional powers, Congress
may block the president from acting.
b. Impeachment Power
(1) “The President, Vice President, and all civil Officers of the
United States shall be removed from Office on Impeachment for,
and Conviction of, Treason, Bribery or other high Crimes and
Misdemeanors” [U.S. Const. art. II, § 4].
(a) The House of Representatives has the sole power to impeach.
(b) The Senate has the sole power to try impeachments. A two-
thirds vote in the Senate is required for conviction.

103
OUTLINE

c. Legislative Veto
(1) A legislative veto occurs when Congress enacts a law
containing a provision that Congress can change the law
without a new Congressional vote or presidential signature.
(2) The Supreme Court has held that a legislative veto violated the
constitutional requirements of bicameralism and presentment to
the president [Immigration & Naturalization Service v. Chadha,
462 U.S. 919 (1983)].
d. Investigative Power
(1) As indicated earlier, under the Necessary and Proper Clause,
Congress has the implied power to conduct investigations
concerning all matters over which Congress has jurisdiction.
e. Delegation to Executive
(1) On numerous occasions Congress has delegated to the
executive branch the authority to make rules having the power
of law (through rulemaking by such agencies, such as the
Department of Health, Education, and Welfare; the Environmental
Protection Agency; and the Nuclear Regulatory Commission).
f. Appropriations Power
(1) Where Congress by legislative act explicitly directs the president to
spend appropriated money, the president has no power to impound
(i.e., refuse to spend or delay spending) the authorized funds.
2. Presidential Limits on Congress
a. Every act of Congress must be approved and signed by the president
before it can become law, or, being disapproved, must be passed by a
two-thirds vote of each House [U.S. Const. art. I, § 7].
3. Judicial Limits on Congress and the President
a. The federal judiciary is the ultimate arbiter of cases whose disposition
depends upon construction of the Constitution, an act of Congress,
or a federal treaty [U.S. Const. art. III, § 2].

PYRAMID OF POWER IN U.S. LAW

U.S.
Constitution

Treaties and
Federal Statutes

Executive Orders
and Agreements

State Constitution

State Law and Ordinances

104
CONSTITUTIONAL LAW

III. THE RELATION OF THE NATION AND THE STATES


IN THE FEDERAL SYSTEM

A. Nature and Scope of Federal and State Powers


1. The fundamental principle of federalism is the co-existence of the national
government and the state governments.
2. The Tenth Amendment provides that “[t]he powers not delegated to the
United States by the Constitution, nor prohibited by it to the States, are
reserved to the states respectively, or to the people.”
3. While each state has a general police power, there is no federal police
power. The federal government must legislate through one of its enumer-
ated powers, whereas the states may regulate any health, safety, welfare,
moral, or aesthetic interest through their respective police powers.
4. The U.S. Constitution limits the state’s police powers by:
a. reserving certain enumerated powers exclusively for the federal government;
b. restricting both the federal and state governments from acting in viola-
tion of constitutional provisions; and
c. providing under the Supremacy Clause that if Congress enacts legis-
lation with the intention of preempting state law, the congressional
regulation will control.

B. Intergovernmental Immunities
1. Immunity of the Federal Government
a. The federal government and its agencies are immune from suits by
private individuals except where they allow themselves to be sued
(e.g., the Federal Tort Claims Act).
EXAMPLE: The U.S. Postal Service is part of the federal government
rather than a separate antitrust “person” under the Sherman Act, and,
therefore is not subject to antitrust liability [U.S. Postal Service v. Fla-
mingo Industries (USA) Ltd., 540 U.S. 736 (2004)].
b. Taxation of the Federal Government by a State
(1) In accordance with McCulloch v. Maryland [17 U.S. 316 (1819)],
the federal government and its agencies are immune from state
taxation and state regulation. States may nevertheless collect a
nondiscriminatory tax on persons who deal or contract with the
federal government.
(a) A nondiscriminatory gross-receipts tax applied to a contractor
performing work for the federal government has been upheld
[James v. Dravo Contracting Co., 302 U.S. 134 (1937)].
(b) A state’s sales tax on goods purchased by a contractor, even if
the goods will eventually be sold to the United States, is valid. In
a “cost-plus” contract with the federal government, the tax will be
part of the cost [Alabama v. King & Boozer, 314 U.S. 1 (1941)].

105
OUTLINE

1) However, such a sales tax is invalid if it falls upon the


federal government itself.
(c) A state property tax is valid when applied upon a building
owned by the United States but used by a contractor [Detroit
v. Murray Corp., 355 U.S. 489 (1959)]. Likewise, a building
owned by a contractor doing work for the United States will
be subject to a property tax.
(d) A state income tax applied to employees of the United States
domiciled within that state is valid [Graves v. O’Keefe, 306
U.S. 466 (1939)].
c. As a general rule, the Supremacy Clause impliedly prevents the states
from regulating the activities of agents or instrumentalities of the federal
government if the regulation will interfere with the government’s ability to
carry out its federal functions [Johnson v. Maryland, 254 U.S. 51 (1920)].
(1) Congress has the power “to dispose of and make all needful rules
and regulations respecting the territory or other property belonging
to the United States.” Thus, federal lands (e.g., military bases,
Indian reservations, FBI offices, etc.) are subject to the authority
of the federal government, except to the extent that Congress has
ceded jurisdiction to the state [U.S. Const. art. IV, § 3, cl. 2].
EXAMPLE: States cannot require a federal contractor to obtain
a state license to construct facilities at an Air Force base located
within the state [Miller Inc. v. Arkansas, 352 U.S. 187 (1956)].
2. Immunity of State Governments
a. The federal government, or one of its agencies or instrumentalities,
may sue a state without its consent. In this case, the Supreme Court
has original, but not exclusive, jurisdiction.
b. A state may be sued by a sister state without its consent. In this case,
the Supreme Court has original and exclusive jurisdiction.
c. As a general rule, the Eleventh Amendment prohibits citizens of one
state from suing another state in federal court. This rule has been
extended to prohibit suits by a citizen of a state against his own state.
However, some exceptions do exist to this rule. In particular [Ex parte
Young, 209 U.S. 123 (1908)]:
(1) a state may be sued if it consents to the suit; and
(2) a state officer may be sued for injunctive relief on the theory that his
allegedly unlawful conduct was beyond the scope of his authority.
d. Congress is generally limited to the powers granted to it by the
Constitution and, as explained above, is prohibited by the Tenth
Amendment from “commandeering” state governments. The Supreme
Court has attempted to limit Congress’ use of the Commerce Clause to
regulate integral operations in areas of traditional government functions
[National League of Cities v. Usery, 426 U.S. 833 (1976)].

106
CONSTITUTIONAL LAW

(1) However, Usery was overruled in 1985 by a decision in which the


Supreme Court found that drawing immunity boundaries around
“traditional government functions” was unworkable and “incon-
sistent with established principles of federalism” [Garcia v. San
Antonio Metro. Transit Auth., 469 U.S. 528 (1985)].
e. A state now enjoys immunity from federal taxation if the tax is
applied to either:
(1) unique state activities; or
(2) essential governmental functions.

NOTE Where a state engages in a proprietary business (i.e., one similar in nature to
a business operated by a private individual), then the state may be taxed to
the same extent as the private citizen.

C. Authority Reserved for the States


1. The Tenth Amendment
a. The Tenth Amendment provides that the “powers not delegated to the
United States by the Constitution, nor prohibited by it to the states, are
reserved to the states respectively, or to the people.”

EXAM TIP The Tenth Amendment is frequently a “red herring” wrong answer choice ex-
cept when the facts show the federal government “commanding” the states.

b. The Constitution specifically prohibits any state from [U.S. Const. art. I, § 10]:
(1) making treaties with other nations;
(2) coining money;
(3) passing a bill of attainder;
(4) enacting an ex post facto law;
(5) impairing the obligation of contracts;
(6) laying any duty on imports or exports, except where necessary
for executing its inspection laws;
(7) engaging in war; or
(8) maintaining a peacetime army.
c. The Tenth Amendment prohibits the federal government from using an
enumerated power to force a state legislature to pass a law or a state
executive official to administer a federal program. This is known as the
anti-commandeering doctrine.
2. The Dormant Commerce Clause
a. The Commerce Clause gives Congress the power to regulate interstate
commerce. Where Congress has not enacted legislation, the states are
free to regulate local transactions affecting interstate commerce, subject
to certain limitations. These limitations are generally known as the
dormant Commerce Clause, or the negative implications doctrine.

107
OUTLINE

b. If a state law discriminates on its face between in-state and out-of-state


economic actors, the state must show that:
(1) the regulation serves a compelling state interest; and
(2) the regulation is narrowly tailored to serve that interest.
c. If a state law merely incidentally burdens interstate commerce, the
court will apply a balancing test and the law will be upheld unless the
burden imposed on interstate commerce clearly outweighs the local
benefits [Pike v. Bruce Church, Inc., 397 U.S. 137, 142 (1970)].
d. There are some exceptions to the dormant Commerce Clause which
should be noted:
(1) Congress may affirmatively authorize states to legislate in areas
that would violate the dormant Commerce clause; and
(2) when states act as market participants, they may discriminate
between in-state and out-of-state businesses.
e. Traditionally, it is within the state’s police power to enact legislation for
the protection of the health, safety, and welfare of its citizens.
EXAMPLE: A Detroit smoke-abatement ordinance that affected ships
traveling in interstate commerce was upheld as a valid health measure
[Huron Portland Cement Co. v. City of Detroit, 352 U.S. 440 (1960)].
(1) The Privileges and Immunities Clause under Article IV, Section
2 prohibits states from enacting legislation that discriminates
against out-of-state residents. No state or municipality may deny
citizens of other states any rights that it affords its own citizens
without substantial justification. This applies only to individuals,
not aliens or corporations. The dormant Commerce Clause is
applicable to aliens, businesses and corporations, as well as indi-
viduals, as it deals with laws affecting interstate commerce.
f. If a state regulation furthers no ostensible benefit and imposes a substan-
tial burden on interstate commerce, it will likely be held unconstitutional.
EXAMPLE: An Illinois statute requiring all trucks to be equipped with a
new type of contour mudguards (instead of the flat mudguards permit-
ted in all other states) was declared unconstitutional because it placed
an undue burden on interstate carriers [Bibb v. Navajo Freight Lines,
Inc., 359 U.S. 520 (1959)].
EXAMPLE: A state law limiting the length of trucks traveling on state
highways or trains rolling through the state is unconstitutional because
the regulation unduly burdened interstate commerce, and the state
failed to demonstrate any significant safety benefit [Raymond Motor
Transportation, Inc. v. Rice, 434 U.S. 429 (1978)].
g. Generally, public health measures (e.g., quarantine and inspection
laws) are upheld so long as they do not discriminate against or unduly
burden interstate commerce [Hannibal & St. Joseph Railroad v. Husen,
95 U.S. 465 (1877)].

108
CONSTITUTIONAL LAW

D. Reserved State Power in Taxation


1. As a general rule, state taxation of interstate commerce is permissible as
long as the tax does not discriminate against or unduly burden interstate
commerce. In determining the validity of a state tax affecting interstate
commerce, the court will generally consider the following four factors
[Complete Auto Transit, Inc. v. Brady, 430 U.S. 274 (1977)]:
a. there must be a substantial nexus between the activity taxed and
the taxing state;
(1) In other words, the taxpayer must have “sufficient contacts” or
presence within the taxing state.
(2) “Substantial nexus” is more than “minimum contacts” under the
Due Process Clause.
EXAMPLE: The solicitation by mail and shipping of goods by
common carrier or postal service, although enough to show “mini-
mum contacts,” does not satisfy the “substantial nexus” require-
ment [Quill Corp. v. North Dakota, 504 U.S. 298 (1983)].

NOTE A regular operation within a state, despite owning no property, is enough to


show “substantial nexus”—for example, an airline operating regularly sched-
uled flights from a rented station [Braniff Airways, Inc. v. Neb. Bd. of Equaliza-
tion, 347 U.S. 590 (1954)].

b. the tax must be fairly apportioned;


(1) The taxpayer has the burden of proving unfair apportionment.
EXAMPLE: The Supreme Court invalidated an apportionment based
upon track mileage within the state, as compared with total track
mileage, which resulted in a disproportionate tax because the railroad
had an unusually large amount of track in the taxing state [Norfolk &
Western Railway v. Missouri Tax Commission, 390 U.S. 317 (1968)].
c. the tax must not discriminate against interstate commerce; and
(1) A tax favoring local commerce over interstate commerce will be invali-
dated under the Commerce Clause unless it is authorized by Congress
[Prudential Insurance Co. v. Benjamin, 328 U.S. 408 (1946)].
(2) Article IV, Section 2 (the Privileges and Immunities Clause)
prohibits state taxes that discriminate against nonresidents
[Austin v. New Hampshire, 420 U.S. 656 (1975)].
d. the tax must be fairly related to the services provided by the taxing state.

THE STATE TAX ON INTERSTATE COMMERCE IS VALID IF…

• It is not preempted by federal law.


• It does not discriminate against interstate commerce.
• It has a substantial nexus.
• There is a fair apportionment and relationship.

109
OUTLINE

2. Various Types of State Taxes


a. States are not permitted to levy an ad valorem tax on “goods” that happen
to be in the taxing state on the tax day if those goods are still in the course
of transit [Standard Oil Co. v. Peck, 342 U.S. 382 (1952)]. However, the
validity of such ad valorem property taxes as applied to “instrumentalities
of commerce” (e.g., airplanes or railroad cars) depends upon:
(1) whether there is a taxable situs (or nexus) within the state; and
(2) whether the tax is fairly apportioned to the amount of time the
equipment is in the state.
b. A sales tax is a tax upon the transfer of title of goods consummated
within the state.
(1) If the sale is consummated within the state (even though the buyer
takes the goods outside the state), a sales tax is valid. If, however,
the sale is made to a buyer outside the state (i.e., if the seller delivers
the goods to an out-of-state buyer), then the sales tax is invalid.
c. On the other hand, a use tax is a tax upon the use of goods within the
state that were purchased outside the state.
(1) The ability of a state to collect a use tax usually depends upon
whether the interstate seller, who receives goods from outside
the state, has a sufficient nexus within the taxing state.
(a) Where the seller maintains offices within the taxing state
(or even sends salespeople into the state as employees or
independent contractors), there is a sufficient nexus even
though the use tax was imposed on interstate mail-order
sales [National Geographic Society v. California Board of
Equalization, 430 U.S. 551 (1977)].
(b) In contrast, if no sales office is maintained in the state and all
selling is done by mail, there is not a sufficient nexus with the
taxing state to justify the collection of a use tax [National Bellas
Hess, Inc. v. Department of Revenue, 386 U.S. 753 (1967)].
d. Doing-business taxes (variously referred to as “privilege,” “occupa-
tion,” “franchise,” “license,” “gross receipts” or “net income” taxes) can
be measured by either a flat annual fee or a graduated rate based in
proportion on the amount of revenue derived from the taxing state. As
a general rule, such taxes must “relate to the benefits” conferred by the
taxing state upon the interstate business.
EXAMPLE: A fee applied to airport users was held constitutional
because it bore a reasonable relationship to use of the facilities by the
passengers [Evansville-Vanderburgh Airport Authority v. Delta Airlines,
Inc., 405 U.S. 707 (1972)].
e. Net income taxes may be applied by the state upon a company
engaging in interstate commerce or on a nonresident engaged
in business in the taxing state. A net income tax is valid only as
long as the tax is fairly apportioned, has a significant nexus, and

110
CONSTITUTIONAL LAW

is nondiscriminatory [Northwestern States Portland Cement Co. v.


Minnesota, 358 U.S. 450 (1959)].
f. A flat license fee is generally unconstitutional. This is a fee levied
by the state upon drummers, or solicitors who solicit local orders and
then fill them out-of-state and ship them through interstate commerce
[Nippert v. Richmond, 327 U.S. 416 (1946)].
g. A license tax on a peddler, or itinerant salesperson, is valid when a state
levies it upon a seller who actually sells and delivers the product within the
state. A license is valid as nondiscriminatory only as long as the tax is fairly
apportioned with an equal application to local salespeople.

111
OUTLINE

IV. TYPE OF ACTION GOVERNED BY THE CONSTITUTION

A. State Action versus Private Action


1. In order to show a constitutional violation either by the state or federal govern-
ments, the plaintiff must first show that there is governmental action. Constitutional
rights can be violated only by governmental actors, not by private actors. This is
called the state action requirement. In this context, state action is a term of
art referring to governmental action either by the federal or state governments.
2. Fourteenth and Fifteenth Amendments
a. The Fourteenth Amendment says, “No State shall make or enforce any
law which shall abridge the privileges or immunities of citizens of the
United States; nor shall any State deprive any person of life, liberty or
property without due process of the law; nor deny to any person within
its jurisdiction the equal protection of the laws.”
b. The Fifteenth Amendment prohibits the state and federal governments
from denying the right to vote based on race.
c. The Fourteenth and Fifteenth Amendments therefore prohibit govern-
mental conduct, and not private conduct which infringes upon
protected individual rights.
3. Federal Violation of Constitutional Rights
a. Private actors cannot violate the Bill of Rights and other constitutional
amendments that constrain the federal government. Those constitu-
tional amendments only bind the federal government; private actors
are not bound by them.
4. Exceptions
a. There are two exceptions to the rule that constitutional rights can be
violated only by governmental action. State action can be found in the
actions of private actors under:
(1) the public function theory, where a private entity is carrying on
activities traditionally and exclusively performed by the government; or
(2) the significant state involvement, endorsement, or encour-
agement theory, where the government and private entity are
so closely related that the action by the private party fairly can be
treated as action by the government.
5. Examples of State Action
a. Where the Private Activity Performs an Exclusive Public Function
(1) In Marsh v. Alabama [326 U.S. 501 (1946)], a private company
owned an entire town. A member of the Jehovah’s Witnesses
was arrested and convicted of violating a state trespass law that
made it a crime “to enter or remain on the premises of another.”
The Supreme Court reversed the trespass conviction because
the town’s streets, although privately owned, were the func-
tional equivalent of city streets. The Supreme Court held that the
company’s actions were in violation of the First and Fourteenth

112
CONSTITUTIONAL LAW

Amendments because neither a state nor a private owner can


totally ban the expression of free speech.

NOTE The Supreme Court has ruled that the “company town” rationale did not apply
to a privately owned shopping center [Hudgens v. National Labor Relations
Board, 424 U.S. 507 (1976)].

(2) In Smith v. Allwright [321 U.S. 649 (1944)], the Supreme Court
held that because holding an election is a public function, a
political party could not racially discriminate against blacks by
excluding them from voting in a primary election.
b. Where there is “significant state involvement” in private discrimination,
the constitutional right of equal protection may be applicable.
EXAMPLE: A restaurant owner, whose business was located in a
building owned by the city, was prohibited from discriminating against
racial minorities. The Court found that there was a “symbiotic relation-
ship” between the city and the restaurant [Burton v. Wilmington Parking
Authority, 365 U.S. 715 (1961)].
EXAMPLE: State court enforcement of restrictive covenants prohibiting the
sale of property to blacks was held to involve sufficient state involvement
so as to constitute state action [Shelley v. Kraemer, 334 U.S. 1 (1948)].
EXAMPLE: However, state action was not found in granting a liquor
license to a private club that racially discriminated against blacks
[Moose Lodge No. 107 v. Irvis, 407 U.S. 163 (1972)].
EXAMPLE: More recently, the Supreme Court found state action be-
cause of the “entwinement” between a state and a private organization
that regulated interscholastic athletic competitions among public and
private secondary schools [Brentwood Academy v. Tennessee Second-
ary School Athletic Association, 531 U.S. 288 (2001)].
(1) 42 § U.S.C. 1983 provides citizens with the opportunity to seek
redress against those who are acting under the authority of a
state or local government, and denying citizens their federal
constitutional and statutory rights.
6. Thirteenth Amendment
a. The Thirteenth Amendment prohibits slavery and involuntary servi-
tude. The amendment has been interpreted as providing Congress
with the power to pass legislation prohibiting badges and incidents of
slavery, and thus can be used to proscribe purely private acts of forced
labor without the requirement of state action. The Peonage Abolition
Act of 1867 was designed to help enforce the Thirteenth Amendment
and abolished “the voluntary or involuntary service or labor of any
persons... in liquidation of any debt or obligation.”
b. The Thirteenth Amendment to the United States Constitution applies to
both state action and private action. It prohibits slavery and involuntary

113
OUTLINE

servitude, except as punishment for crime.


(1) The phrase “involuntary servitude” can mean being compelled to
work by the use of force, the threat of force, or the threat of legal
coercion.
(2) The United States Supreme Court has held that the nation’s
military draft does not violate the Thirteenth Amendment, but that
laws regarding peonage (involuntary service to work off a debt)
do violate this provision.
(3) The Second Circuit has ruled that community service required
to graduate from high school does not violate the Thirteenth
Amendment.

B. National Power to Override State Authority


1. Supremacy Clause
a. The Supremacy Clause provides that the Constitution, treaties, and laws
of the United States are “the supreme Law of the Land.” In general, a
federal law will supersede any state law with which it is in direct conflict.
Furthermore, Congress can preempt any state law in an area in which
the exercise of federal power is constitutional [U.S. Const. art. VI, cl. 2].
(1) Where Congress does not intend to occupy a field completely,
and state laws are not otherwise preempted, the states may
enact similar legislation.
EXAMPLE: A state statute prohibiting racial discrimination was
held valid even though there was a substantially identical federal
law [Colorado Anti-Discrimination Commission v. Continental Air
Lines, Inc., 372 U.S. 714 (1963)].
(2) Greater state protection is also permitted. Where Congress has
legislated to establish minimum standards (such as in the areas of
health and safety requirements pertaining to food and drugs or the
regulation of roads and highways), then the states are free to enact
more stringent standards than those mandated by federal law.
EXAMPLE: A municipal smoke-abatement statute was held valid
even though it imposed stricter standards than the federal regulation
[Huron Portland Cement Co. v. City of Detroit, 362 U.S. 440 (1960)].

114
CONSTITUTIONAL LAW

V. DUE PROCESS AND THE INCORPORATION OF PORTIONS


OF THE BILL OF RIGHTS

A. Incorporation of the Bill of Rights


1. As originally enacted, the Bill of Rights (the first 10 amendments to the
U.S. Constitution) was applicable only to the federal government, not to the
states. In 1868, the Fourteenth Amendment was adopted, which provided:
“No State shall make or enforce any law which shall abridge the privileges
or immunities of citizens of the United States; nor shall any State deprive
any person of life, liberty, or property, without due process of law; nor deny
to any person within its jurisdiction the equal protection of the laws.”
2. Fourteenth Amendment Privileges or Immunities Clause
a. A few years after the Fourteenth Amendment was adopted, the
Supreme Court held that the fundamental rights set forth in the Bill
of Rights were not privileges and immunities of national citizenship
[Slaughter-House Cases, 83 U.S. 36 (1873)].
b. In 1999, the Supreme Court held that under the Fourteenth Amendment
Privileges or Immunities Clause (along with the fundamental right to
travel), new residents could challenge a state’s denial of full unemploy-
ment benefits until a specified waiting period had been satisfied.
c. The Court struck down a California law that limited the payment of
welfare benefits for first-year residents to the amount they would have
received from their former state. The law violated the right to travel
freely from state to state, which the Court said is protected by the
Fourteenth Amendment Privileges or Immunities Clause as a right of
national citizenship [Saenz v. Roe, 526 U.S. 489 (1999)].
3. Fourteenth Amendment Due Process Clause
a. Although the Supreme Court rejected the argument that the Due
Process Clause incorporated all of the Bill of Rights, under the doctrine
of selective incorporation, the following specific provisions are now
applicable to the states [Adamson v. California [332 U.S. 46 (1947)]:
(1) the First Amendment freedom of speech and freedom of press,
the right to assemble and petition the government for a redress of
grievances, the right to free exercise of religion, and the prohibi-
tion against the establishment of religion;
(2) the Fourth Amendment provisions guaranteeing the right to be
free from unreasonable searches and seizures [Ker v. California,
374 U.S. 23 (1963)];
(3) the Fifth Amendment protection against double jeopardy [Benton
v. Maryland, 395 U.S. 784 (1969)], privilege against self-incrimi-
nation [Malloy v. Hogan, 378 U.S. 1 (1964)], and requirement of
just compensation when private property is taken for public use;
(4) the Sixth Amendment rights guaranteeing the accused in criminal
prosecutions a speedy and public trial [Klopfer v. North Carolina,

115
OUTLINE

386 U.S. 213 (1967)], the right to confront and cross-examine


witnesses [Pointer v. Texas, 380 U.S. 400 (1965)], the right to
counsel [Gideon v. Wainwright, 372 U.S. 335 (1963)], and the
right to a jury trial in criminal cases [Duncan v. Louisiana, 391
U.S. 145 (1968)];
(5) the Eighth Amendment prohibition against cruel and unusual
punishments;
(6) the Supreme Court recently held that, in addition to applying to
the District of Columbia, the Second Amendment right to keep
and bear arms applies to state and local governments; and
EXAMPLE: A Chicago law banning possession of handguns by al-
most all private citizens was found to be invalid. In dictum, however,
the Court said that many traditional state gun regulations will still be
upheld [McDonald v. City of Chicago, 561 U.S. 742 (2010)].
(7) the Supreme Court has also held that the Due Process Clause
does not permit a State to infringe on the fundamental right of
parents to make childrearing decisions, and that parents can
make decisions concerning the care, custody, and control of their
children [Troxel v. Granville, 530 U.S. 57 (2000)].
b. Major rights not incorporated include:
(1) the Fifth Amendment right to a grand jury in criminal cases; and
(2) the Seventh Amendment right to a jury trial in civil cases.

THE BILL OF RIGHTS STOPS HERE!


The Fourteenth Amendment
extends the Bill of Rights EXCEPT:

• Prevent quartering soldiers in


your home (Third).
NO RIGHT TO • Grand jury indictment (Fifth).
• Civil jury trial (Seventh).
• Prevent excessive fines (Eighth).

c. Scope of the Due Process Clause


(1) The Due Process and Equal Protection Clauses of the Fourteenth
Amendment protect the rights of “persons,” and not merely “citizens.”
(2) A corporation is considered a “person” for purposes of due
process and equal protection. Note, however, that a corporation
is not entitled to the privilege against self-incrimination.
(3) Aliens are considered “persons” for purposes of due process and
equal protection.

116
CONSTITUTIONAL LAW

B. Procedural Due Process


1. Both the Fifth and Fourteenth Amendments protect against the deprivation
of life, liberty, or property without the due process of the law. Where there
is a deprivation of one’s “life,” “liberty,” or “property” interests, the individual
is entitled to fundamentally fair procedural safeguards (e.g., some form of
notice and a meaningful hearing within a reasonable time).
2. Deprivation of Liberty
a. Acts that invade a liberty interest include:
(1) freedom from bodily restraints;
(a) A state must grant a parolee an evidentiary hearing before
it revokes parole or probation [Morrissey v. Brewer, 408
U.S. 471 (1972)].
(2) physical punishment; and
(a) Corporal punishment of pupils in a public school is valid,
although a liberty interest exists [Ingraham v. Wright, 430
U.S. 651 (1977)].
(3) commitment to a mental institution.
(a) An adversary hearing must be provided to adults before they
are committed to a mental institution against their will. Minor
children have a liberty interest in not being confined unnec-
essarily for medical treatment. In the case of minor children,
a screening by a neutral fact-finder is required [Parham v.
J.R., 442 U.S. 584 (1979)].
b. Acts that do not invade a liberty interest include:
(1) injury to reputation; and
(a) Injury to one’s reputation in the absence of a related harm
to a more tangible interest (such as an employment oppor-
tunity) is not a deprivation of a liberty interest [Paul v. Davis,
424 U.S. 693 (1976)].
(2) forced administration of medicine.
(a) The federal government may administer antipsychotic drugs
against a defendant’s will in order to render him competent to
stand trial, as long as the treatment is medically appropriate,
does not cause substantial side effects, and is necessary to
significantly further important governmental trial-related inter-
ests [Sell v. United States, 539 U.S. 166 (2003)].
3. Property Interests
a. Public Education
(1) There is a constitutionally protected property interest in the statu-
tory entitlement to continued attendance at a public school [Goss
v. Lopez, 419 U.S. 565 (1975)].
(2) On the other hand, no prior evidentiary hearing is required when
a student is dismissed for academic reasons [Board of Curators
of University of Missouri v. Horowitz, 435 U.S. 78 (1978)].

117
OUTLINE

b. Continued Welfare Benefits


(1) A property interest is created by a statutory entitlement to continued
welfare benefits [Goldberg v. Kelly, 397 U.S. 254 (1970)].
c. Retention of Driver’s License
(1) A state may not revoke a driver’s license without a hearing [Bell v.
Burson, 402 U.S. 535 (1971)].
d. Public Employment
(1) There is a property interest in a person keeping his job if the employ-
ment is under a tenure system or there is a clear understanding,
either express or implied, that the employee can be terminated only
for “cause” [Perry v. Sindermann, 408 U.S. 593 (1972)].
(2) However, the Supreme Court has held that there was no property
interest where a police officer held his position “at the will of” the
public employer [Bishop v. Wood, 426 U.S. 341 (1976)].
(3) Furthermore, there is no property interest when a state refuses
to renew a fixed-term contract [Board of Regents v. Roth, 408
U.S. 564 (1972)].
e. Prejudgment Garnishment
(1) Prejudgment attachment or garnishment of wages, without notice
or hearing, violates procedural due process [Sniadach v. Family
Finance Corp., 395 U.S. 337 (1969)].
(2) A due process property interest does not arise when an indi-
vidual is first applying for employment. However, if the person
has already been getting employment benefits, he has a property
interest in continuing the receipt of those benefits.
f. Forfeiture of Property
(1) Due process is satisfied where the government sends a certi-
fied letter to prison to notify an inmate that property seized will
be forfeited, because such an action is “reasonably calculated,
under the circumstances, to apprise interested parties of the
pendency of the action and afford them an opportunity to present
their objections.” Actual notice is not required; constructive notice
will suffice [Dusenbery v. United States, 534 U.S. 161 (2002)].
g. Business Licensing
(1) The First Amendment and due process require that a licensing
scheme for adult businesses provide applicants with “prompt judicial
determination” of their claim that the government unconstitution-
ally denied a license, as opposed to mere prompt access to judicial
review [City of Littleton v. Z.J. Gifts D-4, L.L.C., 541 U.S. 774 (2004)].

NOTE As with any other constitutional right, procedural due process is not required
where there is no “state action” [Flagg Brothers v. Brooks, 436 U.S. 149 (1978)].

4. Life Interests
a. Capital punishment is clearly a deprivation of a life interest, and rigorous

118
CONSTITUTIONAL LAW

due process protections are applicable. When life interests have been
claimed in other contexts, such as in abortion or right-to-die situations, the
Supreme Court has addressed it under other provisions of the Constitution.
5. Type of Process Required
a. Once it is determined that there is a sufficient deprivation of life, liberty, or prop-
erty, the next step is to decide what process is required. In order to determine
what procedural safeguards are necessary, the Supreme Court set forth the
following factors to look at [Mathews v. Eldridge, 424 U.S. 319 (1976)]:
(1) the private interest that will be affected by the official action;
(2) the risk of an erroneous deprivation of this interest through the
procedures used, and the probable value of additional or substi-
tute procedures; and
(3) the government’s interest in streamlined procedures, including the
function involved and the fiscal and administrative burdens that the
additional or substitute procedural requirements would entail.
EXAMPLE: The Supreme Court held that a full hearing was not
required for the dismissal of a medical student from a state medical
school for academic deficiency because the decision was an eval-
uative one, made by faculty officers and outside practitioners [Bd.
of Curators of Univ. of Missouri v. Horowitz, 435 U.S. 78 (1979)].
EXAMPLE: The Supreme Court held that it was not necessary
for a public school to give a student a hearing before imposing
corporal punishment [Ingraham v. Wright, 430 U.S. 651 (1997)].
EXAMPLE: The Supreme Court ruled that a temporary suspen-
sion of a driver’s license without a hearing, where the driver
refused to take a breathalyzer test, was valid, because he had a
right to an immediate hearing following the suspension [Mackey
v. Montrym, 443 U.S. 1 (1979)].
b. Due process requires a judge to recuse himself when the judge has
a pecuniary interest in the case, such that an average judge would
“possibly be tempted” to render an “imbalanced or untrue” judgment.
EXAMPLE: If one party to a case had a “significant and disproportion-
ate influence” in getting the judge elected, then due process requires
that the judge recuse himself. Such a disproportionate influence exists
when, for example, an appellant contributed more than 50% of all cam-
paign contributions to the appellate judge’s election campaign [Caper-
ton v. A. T. Massey Coal Co., 556 U.S. 868 (2009)].
6. Irrebuttable Presumptions
a. In recent cases, the Supreme Court has held that irrebuttable
presumptions violate procedural due process.
b. Civil Proceedings
(1) The Supreme Court held that a statutory presumption that a
pregnant schoolteacher was physically incapable of performing

119
OUTLINE

her duties was unconstitutional [Cleveland Board of Education v.


LaFleur, 414 U.S. 632 (1974)].
(2) A state law requiring children of unwed fathers to become wards
of the court was invalidated because its statutory effect was to
create a conclusive presumption that unwed fathers were unfit
parents [Stanley v. Illinois, 405 U.S. 645 (1972)].
c. Criminal Cases
(1) A statutory presumption that possession of marijuana was conclu-
sive proof that the drug had been imported was invalidated,
because marijuana is frequently grown in the United States as
well as abroad [Leary v. United States, 395 U.S. 6 (1969)].

CONSTITUTIONAL LAW ANALYSIS TIPS


when you see this… …remember this

Thirteenth Amendment • Best answer when private individuals, who do not qualify as state
actors, discriminate.

Fourteenth Amendment • All the elements of the Constitution incorporated into the amendment
are, hence, protected by it.
• Frequently best answer when First Amendment right of free speech
or freedom of religion is not offered as a choice.

Commerce Clause • Grants Congress the power to enact legislation that affects
interstate commerce.
• Important source of congressional power over civil rights.
• Does not empower Congress to push states into acting.

Contract Clause • Can be an issue where the state revokes a contract to which it is a party.

Dormant Commerce • Legal doctrine inferred from Commerce Clause.


Clause • Prohibits a state from passing legislation that improperly burdens or
discriminates against interstate commerce.

Due Process Clause • Best choice for an unconstitutional argument when:


1. state acts; or
2. state does not grant a hearing or invades privacy rights.

Equal Protection Clause • Best option when state action interferes with a fundamental interest
(e.g., right to vote or travel, or classifications by race, alienage, or sex).

General Welfare Power • Congress’ power to spend treasury funds, but not to regulate activity.
or Spending Power • Best answer when voluntary cooperation by party with federal
government is the ONLY way to reach result.

Police Power • Can be the answer only when state action versus federal action.

120
CONSTITUTIONAL LAW

CONSTITUTIONAL LAW ANALYSIS TIPS


when you see this… …remember this

Privileges and • Article IV’s Privileges and Immunities Clause bars states from
Immunities Clauses discriminating against nonresident citizens on fundamental matters,
unless the regulation in question specifically targets a problem
arising from such nonresident’s behavior.
• The Privileges or Immunities Clause of the Fourteenth Amendment
is almost never a correct answer.

Supremacy Clause • Not a congressional power.


• Not a good choice unless there is a congressional statute or
provision conflicting with a state activity.

Taxing Power • Allowed for regulatory purposes if there is a revenue-raising function.


• Best answer if Commerce Clause isn’t available.

EXAM TIP When answering a question involving the various arguments that can be used
to attack the constitutionality of a statute regulating economic activity, you
should still consider the argument that it violates substantive due process, es-
pecially if the statute operates in an arbitrary and unreasonable manner and
there is no legitimate reason for the legislature to pass the statute.

WHEN SHOULD THE SUPREME COURT AVOID A CONSTITUTIONAL DECISION?

1. When there is no issue in controversy—no adversarial conflict between the parties.


2. When the complainant cannot show injury by the statute in question.
3. When the issue isn’t ripe and the Supreme Court doesn’t feel the issue has arisen yet.
4. When the decision requires a ruling broader than the facts of the case.
5. When the complaint can be resolved in a nonconstitutional manner (e.g., by a statute).
6. When the Court can determine a statutory construction that avoids constitutional analysis.
7. When the complainant benefited from the statute it now argues is unconstitutional.

C. Substantive Due Process


1. Economic Regulation
a. In the past, the doctrine of substantive due process was frequently used
to protect rights of property and contract and to invalidate legislation that
regulated economic activity. However, this approach to substantive due
process was abandoned by the Supreme Court in the 1930s.
b. The Supreme Court held that challenges to economic regulation are
to be subjected to deferential rational basis scrutiny, with a challenger
having the burden of proving that a regulation lacks a rational connec-
tion to a legitimate government interest. As a result, since the New

121
OUTLINE

Deal, the Supreme Court has not struck down any economic regulation
on substantive due process grounds.
c. Such an economic regulation will be upheld if it is rationally related to a
legitimate government interest.
2. Fundamental Rights
a. The substantive due process doctrine is now used to evaluate govern-
mental regulations that affect fundamental rights of personhood, rather
than rights of property. Strict scrutiny review, or other forms of height-
ened scrutiny, apply to laws that burden the exercise of fundamental
rights, including the right to vote, the right to travel, the right to privacy,
First Amendment rights, family rights, and other rights referred to as
fundamental rights.
b. The following are some of the categories that fall under the penumbra
of privacy rights.
c. Contraceptives
(1) The Supreme Court invalidated a state law prohibiting the use of
contraceptive devices, thus recognizing a right of marital privacy
[Griswold v. Connecticut, 381 U.S. 479 (1965)].
(2) The Supreme Court later expanded the Griswold decision and held
that the right to use contraceptives belonged to single as well as
married persons [Eisenstadt v. Baird, 405 U.S. 438 (1972)].
(3) In Carey v. Population Services International [431 U.S. 678 (1977)],
the Supreme Court invalidated a state law that prohibited the sale of
contraceptives to minors except through a licensed pharmacist.
(4) But in 2014, the Court held that corporations cannot be required to
pay for contraception coverage for their female workers (the first
time that the court has recognized a for-profit corporation’s claim of
religious belief) [Burwell v. Hobby Lobby, 134 S. Ct. 2751 (2014)].
d. Marriage
(1) The right to marry is deemed fundamental. Any substantial inter-
ference with that right must be necessary to further a compelling
interest [Zablocki v. Redhail, 434 U.S. 374 (1978)].
(2) In June 2015, the United States Supreme Court announced that
governmental bans on same-sex marriage are unconstitutional
under the U.S. Constitution. Specifically, the Court held that such
bans violate the substantive due process doctrine and the funda-
mental right to marriage. This ruling impacts every state with a statu-
tory or constitutional ban on same-sex marriage. All states must now
recognize the same-sex marriages legally performed in other states,
assuming that such marriages are otherwise in accord with the
laws of the state where the marriage occurred. Most significantly, all
states must now allow same-sex couples to get married and accord
such couples the same rights and obligations otherwise accorded
people who marry [Obergefell v. Hodges, 135 S. Ct. 2584 (2015)].

122
CONSTITUTIONAL LAW

e. Abortion
(1) Planned Parenthood of Southeastern Pennsylvania v. Casey [505
U.S. 833 (1992)] modified the Supreme Court’s approach to repro-
ductive freedom that was established in Roe v. Wade [410 U.S.
113 (1973)]. The Casey holding rejected the trimester approach of
Roe and instead adopted an “undue burden” standard.
(a) A woman has a protected privacy interest in choosing to
have an abortion before the fetus is viable.
(b) During the first two trimesters (that is, before viability), the
government may regulate (but not ban) abortion in the
interest either of the mother’s health or of the potential life of
the fetus. The regulation may not impose an “undue burden”
on the woman’s right to choose an abortion.
(c) For the period of time subsequent to viability, the Supreme
Court reaffirmed Roe in concluding that a state may “regu-
late, and even proscribe, abortion except where it is neces-
sary…for the preservation of the life or health of the mother.”
(2) Consent Requirements
(a) Neither spousal notification nor spousal consent may be
required before a woman may obtain an abortion [Planned
Parenthood of Missouri v. Danforth, 428 U.S. 52 (1976)].
(b) However, parental consent may be required before an uneman-
cipated woman under the age of 18 obtains an abortion if the
state establishes a “judicial bypass” procedure through which
a minor may obtain an abortion with the consent of a judge
[Hodgson v. Minnesota, 497 U.S. 417 (1990)].
(3) Public Funding
(a) There is no constitutional right for indigent women to obtain
government funding for abortions [Maher v. Roe, 432 U.S. 464
(1977)]. Furthermore, a state may prohibit the use of public
facilities and publicly employed staff in performing abortions
[Webster v. Reproductive Health Services, 492 U.S. 490 (1989)].
(4) Late-Term Abortion
(a) Where a Nebraska statute prohibited “deliberately and
intentionally delivering into the vagina a living unborn
child, or a substantial portion thereof, for the purpose of
performing a procedure that the person performing such
procedure knows will kill the unborn child and does kill the
unborn child,” such a statute was unconstitutional because
it did not provide an exception for the health of the mother
and because it prohibited the dilation and evacuation
procedure used in most second-trimester abortions, as
well as the less frequently used “partial-birth” procedure
[Stenberg v. Carhart, 530 U.S. 914 (2000)].

123
OUTLINE

(b) Subsequently, the Supreme Court upheld a federal ban on partial-


birth abortions [Gonzales v. Carhart, 550 U.S. 124 (2007)]. The
Court rejected a facial challenge to a prohibition against use of
this particular type of abortion procedure, but left open the possi-
bility of challenges to the act as applied to specific individuals.
(5) Protests
(a) The Supreme Court reversed a Massachusetts law which made it
a crime to stand on a public road or sidewalk within thirty-five feet
of a reproductive health care facility, allowing protests, counseling,
and other speech and rejecting the concept of “buffer zones”
around these clinics [McCullen v. Coakley, 134 S. Ct. 2518 (2014)].
f. Family Relations
(1) A fundamental right exists for related persons to live together.
A zoning ordinance prohibiting members of an extended family
from living in a single household has been subjected to height-
ened scrutiny and held unconstitutional [Moore v. City of East
Cleveland, 431 U.S. 494 (1977)].

NOTE This fundamental right does not apply to unrelated persons [Belle Terre v.
Boraas, 416 U.S. 1 (1974)].

(2) Parents were held to have a protected liberty interest concerning


the control of visitation with their children by others, including
grandparents [Troxel v. Granville, 530 U.S. 57 (2000)].
(3) Parents have the substantive due process right to make funda-
mental decisions affecting the lives of their children.
g. Sexual Orientation
(1) In Lawrence v. Texas [539 U.S. 558 (2003)], the Supreme Court
applied a stringent form of rational basis scrutiny and held that
a statute making it a crime for a person to engage “in deviate
sexual intercourse with another individual of the same sex”
furthered no legitimate state interest.
(2) In 2013, it was held unconstitutional to restrict U.S. federal inter-
pretation of “marriage” and “spouse” to apply only to heterosexual
unions, giving same-sex couples the rights of married couples in
terms of tax benefits, health care, retirement, and other areas of
federal regulation [United States v. Windsor, 570 U.S. 744 (2013)].
(3) As of 2015, the fundamental right to marry is guaranteed to same-
sex couples [Obergefell v. Hodges, 135 S. Ct. 2584 (2015)].
h. Private Education
(1) Parents have a right to privately educate their children outside the
public school system [Pierce v. Society of Sisters, 268 U.S. 510 (1925)].
i. Obscene Material
(1) The right to possess obscene material in the privacy of one’s
home is protected [Stanley v. Georgia, 394 U.S. 557 (1969)].

124
CONSTITUTIONAL LAW

(2) However, the government can severely restrict the sale,


purchase, receipt, transport, and distribution of obscene mate-
rials, both in stores and through the mail. Furthermore, the state
can criminalize even the private possession of child pornography
[Osborne v. Ohio, 495 U.S. 103 (1990)].
j. Right to Die
(1) The right of a terminally ill or comatose person to choose to die is
not presently a fundamental right.
(2) Nevertheless, a person has a well-established due process
“liberty” interest in not being forced to undergo unwanted medical
procedures, such as life-support [Cruzan v. Missouri Department
of Health, 497 U.S. 261 (1990)].
(3) However, the state has a countervailing interest in preserving
life. A terminally ill patient has no right to assisted suicide
[Washington v. Glucksberg, 521 U.S. 702 (1997)].
k. Right to Travel
(1) The Privileges and Immunities Clause of Article IV, Section 2
and the Commerce Clause mutually enforce the right of every
citizen to travel freely from state to state [Griffin v. Breckenridge,
403 U.S. 88 (1971)].
(2) Durational residency requirements for dispensing government
benefits are subject to strict scrutiny.
EXAMPLE: A one-year waiting period before receiving welfare
benefits or state-subsidized medical care is invalid.
EXAMPLE: Reasonable residency requirements are valid for
obtaining a divorce as well as for obtaining reduced tuition at
state universities.
(3) The right to international travel is not absolute and may be
subject to reasonable restrictions, and Congress may authorize
the president to restrict travel to certain countries or danger areas
[Zemel v. Rusk, 381 U.S. 1 (1965)].

EXAM TIP The Privileges or Immunities Clause of the Fourteenth Amendment has fre-
quently been a wrong answer on the MBE. Almost all the privileges and immunities
are protected by the Commerce Clause, Due Process Clause, or Equal Protection
Clause. However, the Privileges or Immunities Clause is not entirely dead: In
Saenz v. Roe [526 U.S. 489 (1999)], the Supreme Court ruled that it was an al-
ternative to an Equal Protection Clause protection with regard to the right to travel.
l. Right to Vote
(1) The fundamental right of U.S. citizens over age 18 to vote
extends to all federal, state, and local elections, as well as to
primaries. Strict scrutiny review is used to adjudicate restrictions
on the right to vote.

125
OUTLINE

(2) However, government regulations of ballot-access by candi-


dates–based upon age, duration of residency, or payment of filing
fees–require only minimum rational basis scrutiny.
(a) In addition, voter registration requirements and regulation of the
time, place, and manner of casting ballots are valid so long as
they do not impose an “undue burden” on the right to vote.
(3) A rule of “one-person, one-vote” is generally followed. Whenever
the government attempts to establish a new apportionment or
redistricting scheme, fairly exact mathematical equality between
districts is required so as not to dilute the fundamental right to vote.
(4) There must be equality in the distribution of the right to vote.
EXAMPLE: Imposition of a poll tax as a prerequisite to vote vio-
lates equal protection and is unconstitutional [Harper v. Virginia
Board of Elections, 383 U.S. 663 (1966)].
EXAMPLE: Limiting voting for school board elections to parents
of schoolchildren and/or property owners is unconstitutional
[Kramer v. Union Free School District, 345 U.S. 662 (1969)].
EXCEPTION: Voting can be limited to landowners for a special
purpose “water-storage district” (which cannot enact laws or per-
form government functions) [Ball v. James, 451 U.S. 355 (1981)].
(5)In 2013, the Court effectively struck down the portions of the Voting
Rights Act of 1965 that required certain states and local governments
to obtain federal pre-clearance before changing their voting laws
or practices, enabling states to implement new voter identification
laws, eliminate same-day registration, and other measures that
affect voting rights [Shelby County v. Holder, 570 U.S. 529 (2013)].
m. Medical Records
(1) There is some right to privacy recognized for medical records,
but “informational privacy is not a fundamental right” and the
privacy interest must be balanced against the public interest
in disclosure. A law requiring physicians to report certain
prescriptions to the state for storage in a database has been
held constitutional [Whalen v. Roe, 429 U.S. 589 (1977)].
(2) In 1977, the Supreme Court seemed to hold:
(a) that there is an implied zone of privacy into which medical
records would fall; but
(b) a patient ID requirement was a reasonable exercise of the
state’s broad police powers, and no Fourteenth Amendment
Due Process violation occurred.
(3) The Court recognized two privacy interests: “the individual
interest in avoiding disclosure of personal matters” and “the
interest in independence in making certain kinds of important
decisions” [Whalen v. Roe, 429 U.S. 589 (1977)].

126
CONSTITUTIONAL LAW

(4) However, Whalen v. Roe provided little guidance for lower courts
dealing with medical privacy issues. Regarding the first of the
Court’s dual privacy interests, “the individual interest in avoiding
disclosure of personal matters,” Whalen left unclear what consti-
tuted a violation of the right, and failed to establish what type of
constitutional treatment the courts were to use when assessing it.
The Court’s second privacy interest in “independence in making
certain kinds of important decisions,” had been fleshed out by other
Supreme Court decisions, but there was no clear legal precedent
for a privacy interest in nondisclosure of personal matters [Id.].
(5) The courts of appeals have split over whether Whalen v. Roe
created a constitutional right to confidentiality. Most courts of
appeals have affirmed the existence of the right; two courts of
appeals have practically denied it.
(6) In addition, the constitutional treatments used to assess the
right contrast greatly among all the courts of appeals. In some
circuits, the federal courts hold that the needs of the government
always supersede the patient’s implied right of privacy. In
other circuits, the federal court apply a balancing test. While
the factors of the balancing tests differ between circuits, the
main factors used to determine whether an intrusion into an
individual’s privacy is justified center around [United States v.
Westinghouse Electric Corp., 638 F.2d 570 (3rd Cir. 1980)]:
(a) the type of record requested;
(b) the information it does or might contain;
(c) the potential for harm in any subsequent nonconsensual
disclosure;
(d) the injury from disclosure to the relationship in which the
record was generated;
(e) the adequacy of safeguards to prevent unauthorized disclosure;
(f) the degree of need for access; and
(g) whether there is an express statutory mandate, articulated
public policy, or other recognizable public interest militating
toward access.

D. Takings Clause
1. The Fifth Amendment provides that private property shall not be taken for
public use without just compensation. This prohibition also applies to the
states through the Due Process Clause of the Fourteenth Amendment.
2. In Agins v. City of Tiburon [447 U.S. 255 (1980)], the Supreme Court held
that a taking of property occurs if a regulation of private property “does not
substantially advance legitimate state interests.”
a. Agins was overruled in Lingle v. Chevron U.S.A., Inc. [544 U.S. 528
(2005)], in which the Supreme Court held that the extent to which a

127
OUTLINE

regulation advances a legitimate state interest is not a relevant factor


in determining if a taking has occurred.
3. Property can be taken by a variety of means, including:
a. direct government appropriation;
(1) Where the state validly regulates for health, safety, or welfare
purposes under its police power, the government action merely
amounts to a regulation, and payment of just compensation is
not required. However, a taking does occur where there is an
actual appropriation of one’s property [Loretto v. Teleprompter
Manhattan CATV Corp., 455 U.S. 904 (1982)].
b. a regulatory taking;
(1) Any permanent, physical invasion, no matter how minor, will
constitute a taking.
(2) A land-use regulation is a taking if it denies an owner all
reasonable, economically beneficial uses of his land. This is
otherwise known as an inverse condemnation.
(3) To analyze regulations that merely decrease economic value,
the court may use a balancing test to determine if there is a
taking. The court must balance (1) “[t]he economic impact
of the regulation on the claimant,” (2) “the extent to which
the regulation has interfered with distinct investment-backed
expectations,” and (3) “the character of the governmental action”
[Pennsylvania Coal Co. v. Mahon, 260 U.S. 393 (1922); Penn
Central Transportation Co. v. New York, 438 U.S. 104 (1978)].
EXAMPLE: A state zoning law was passed after the owner
purchased the property, and was held to constitute a taking
because the law precluded the owner from erecting any
permanent structure on his land [Lucas v. South Carolina
Coastal Council, 505 U.S. 1003 (1992)].
c. temporary restrictions; or
(1) When considering whether a taking has occurred, a court will
look at both the dimensions of a property interest (metes and
bounds) and the term of years.
EXAMPLE: A temporary restriction causing a diminution in value
(such as a 32-month moratorium on development in an area) is
not a taking of the parcel as a whole because the property will
recover value when the prohibition is lifted, and thus the regula-
tion will not “permanently deprive” the owner of “all economically
beneficial uses” of his land [Tahoe-Sierra Preservation Council,
Inc. v. Tahoe Regional Planning Agency, 535 U.S. 302 (2002)].
d. conditional permits.
(1) The government can place a condition on the grant of a permit for
land development if [Nollan v. California Coastal Commission, 483
U.S. 825 (1987); Dolan v. City of Tigard, 512 U.S. 374 (1994)]:

128
CONSTITUTIONAL LAW

(a) there is a logical nexus between the condition and the


governmental purposes; and
(b) there is rough proportionality between the impact on the
proposed development and the governmental objectives
served by the condition.
4. The Supreme Court has interpreted the “public use” language of the
Takings Clause to mean “public purpose,” and held that a city has the
power to take private property and transfer it to a private developer
because of the public benefits of the proposed development [Kelo v. City
of New London, 545 U.S. 469 (2005)].

129
OUTLINE

VI. EQUAL PROTECTION OF THE LAWS

A. Constitutional Basis
1. The Fourteenth Amendment provides that no state shall “deny to any
person within its jurisdiction the equal protection of the laws.” The Supreme
Court has ruled that equal protection also applies to the federal government
under the Due Process Clause of the Fifth Amendment.
2. The guarantee of substantive due process assures that a law will be fair
and reasonable, not arbitrary; equal protection review is triggered where
persons similarly situated are treated differently.
a. Substantive due process review applies where a law affects the rights
of all persons with respect to a specific activity (e.g., a state law
prohibits the sale of birth control devices except by prescription).
b. Equal protection review applies where a law affects the rights of
some persons with respect to a specific activity (e.g., a state law
prohibits the sale of birth control devices to unmarried persons
except by prescription).

B. Standards of Review
1. Under the strict scrutiny standard, the burden of persuasion is on the
government to prove that the measure being challenged is necessary to
further a compelling interest.
a. The word necessary means that there is no less restrictive alternative
means available. There must be a very close “fit” between the means
and the end.
b. The government usually fails to prove its burden under strict scrutiny,
so an equal protection challenge to a law is generally successful (i.e.,
the law is presumptively invalid).
c. Strict scrutiny review applies to government action that uses suspect
classifications—race, alienage, and national origin.
2. Under the intermediate scrutiny standard, the burden of persuasion is
placed on the government to prove that the measure being challenged is
substantially related to the achievement of an important governmental interest.
a. The key term, substantially related, means that an exceedingly
persuasive justification must be shown. Intermediate scrutiny is much
closer to strict scrutiny than it is to rational basis.
b. Intermediate scrutiny applies to government action using quasi-suspect
classifications—gender and illegitimacy.
3. Under the rational basis standard of review, the burden of persuasion is on
the plaintiff to show that the measure being challenged serves no legitimate
government interest or is not rationally related to any legitimate interest.
a. Rational relationship is a minimal requirement which means that the
law cannot be arbitrary or unreasonable.

130
CONSTITUTIONAL LAW

b. Practically any police power regulation which furthers a health, safety,


or welfare purpose will be found “legitimate.” For this reason, laws
scrutinized under rational basis are almost always upheld. From the
plaintiff’s standpoint, an equal protection challenge under rational basis
is generally unsuccessful.
c. Rational basis review applies to all classifications not falling under
strict or intermediate scrutiny, such as classifications based on age,
poverty, wealth, disability, and need for necessities of life (food, shelter,
clothing, medical care).

THE THREE STANDARDS OF SCRUTINY


Rational Basis Intermediate Scrutiny Strict Scrutiny

The plaintiff must prove no The government must prove The government must prove
legitimate governmental interest. its classification is substantially its classification is necessary
related to an important to achieve a compelling
government interest. government interest.

C. Proving Discrimination
1. Discriminatory intent—purposeful discrimination—must be shown to
trigger strict or intermediate scrutiny. Mere discriminatory effect is insuf-
ficient. Discriminatory intent may be shown facially, as applied, or where a
discriminatory motive exists.
a. Facial discrimination arises where a law, by its very language,
creates distinctions between classes of persons (e.g., “only white, male
U.S. citizens may apply for positions with the state police department”).
b. Where a law that appears neutral on its face but in its application has a
disproportionate effect on a particular class of persons, strict or intermediate
scrutiny will apply only if the court finds a discriminatory purpose exists.
EXAMPLE: The fact that black applicants scored lower than white ap-
plicants on a police qualifying test did not per se prove a discriminatory
purpose in hiring practices, so strict scrutiny review was not triggered
and no equal protection violation was found [Washington v. Davis, 426
U.S. 229 (1976)].
c. A facially neutral law can be applied in a discriminatory manner. Where the
challenger can show a discriminatory purpose, the law will be invalidated.
EXAMPLE: Where a law prohibited the operation of laundries in wood-
en buildings, most of which were owned by Chinese individuals, and
government officials granted discretionary exemptions to white-owned
laundries, the Supreme Court found that purposeful discrimination in
the application of the law violated equal protection [Yick Wo v. Hopkins,
118 U.S. 356 (1886)].

131
OUTLINE

D. Suspect Classifications
1. Strict Scrutiny
a. Strict scrutiny applies to classifications based on race, alienage,
and national origin. Such laws will be presumptively invalid absent
a showing by the state that the measure is necessary to achieve a
compelling state interest.
(1) A state law prohibiting interracial marriages was held unconstitu-
tional [Loving v. Virginia, 388 U.S. 1 (1967)]. Similarly, a state law
prohibiting interracial cohabitation was held invalid [McLaughlin v.
Florida, 379 U.S. 184 (1964)].
(2) Deliberate de jure segregation violates equal protection [Brown
v. Board of Education, 347 U.S. 483 (1954); Plessy v. Ferguson,
163 U.S. 537 (1896)].
(a) Various plans to hinder desegregation have been deemed
unconstitutional, including the closing of all public schools
[Griffin v. County School Board of Prince Edward County,
377 U.S. 218 (1964)].
(b) The Supreme Court has held that public aid to private,
segregated schools—such as tuition grants and the exclu-
sive use of public facilities—was unconstitutional [Norwood
v. Harrison, 413 U.S. 455 (1973)].
(c) School boards have an affirmative duty to eliminate the
intentional racial segregation of schools.
1) Court-ordered busing is constitutional where it is imple-
mented to remedy past discrimination in a particular school
system, rather than to attract nonminority students from
outside districts to achieve integration. Court-ordered
busing is a temporary measure that must be terminated
once the vestiges of past discrimination have been elimi-
nated [Board of Education v. Dowell, 498 U.S. 237 (1991)].
(3) Where it can be shown that race was the “predominant factor” in
defining the borders of new election districts (rather than contiguity,
compactness, or community interest), then such a plan will be
subject to strict scrutiny [Miller v. Johnson, 515 U.S. 900 (1995)].
b. The Court will generally apply strict scrutiny when a state law discrimi-
nates against aliens.
EXAMPLE: A state law prohibiting aliens from owning land was invali-
dated [Oyama v. California, 332 U.S. 633 (1948)].
EXAMPLE: A state law denying commercial-fishing licenses to resident
aliens who were ineligible for citizenship was held invalid [Takahashi v.
Fish & Game Commission, 334 U.S. 410 (1948)].
EXAMPLE: The Supreme Court invalidated a state law that excluded
financial assistance for higher education to aliens who were eligible for

132
CONSTITUTIONAL LAW

U.S. citizenship as not furthering any “compelling state interest” [Ny-


quist v. Mauclet, 432 U.S. 1 (1977)].
(1) States may discriminate against aliens in activities where partici-
pation in the functioning of government is involved.
(a) A New York statute requiring state police officers to be citi-
zens was held valid, under what is called the “government
function test” [Foley v. Connelie, 435 U.S. 291 (1978)].
(b) Aliens also may be denied positions as public school
teachers because they influence students’ views toward
government and the political process [Ambach v. Norwick,
441 U.S. 68 (1979)].
(2) The Supreme Court has not held illegal aliens to be a suspect
class; the rational basis analysis will apply.
(a) However, in Plyler v. Doe [457 U.S. 202 (1982)], the Court
determined that illegal alien children have a right to free public
elementary and secondary education. The Court used a strin-
gent version of the rational basis scrutiny test and held that the
discrimination against the children “can hardly be considered
rational unless it furthers some substantial goal of the state.”

NOTE Federal laws that discriminate against aliens are not subject to the strict scru-
tiny test because Congress has broad plenary power to regulate immigration.

2. Intermediate Scrutiny
a. Distinctions drawn between legitimate and illegitimate children are
subject to an intermediate, or “quasi-suspect,” standard [Mathews v.
Lucas, 427 U.S. 495 (1976)]. As a result, it is now close to the “almost
suspect” standard used for gender discrimination [Mills v. Habluetzel,
456 U.S. 91 (1982)].
(1) Classifications must be “substantially related to an important
state interest” [Id.].
(2) Classifications that favor legitimates and disfavor illegitimates are
generally struck down because the overriding government interest
in this area is not to punish the offspring of an illicit relationship.
EXAMPLE: In Levy v. Louisiana [391 U.S. 68 (1968)], the Supreme
Court struck down a state law that permitted legitimate children, but
not illegitimate children, to maintain a wrongful death action.
EXAMPLE: Similarly, the Supreme Court invalidated a state law
that excluded illegitimate children from sharing equally with other
children in worker’s compensation death benefits [Weber v. Aetna
Casualty and Surety Co., 406 U.S. 164 (1972)].
EXAMPLE: Likewise, illegitimate children are entitled to welfare
benefits [New Jersey Welfare Rights Organization v. Cahill, 411
U.S. 619 (1973)].

133
OUTLINE

b. As with federal laws that discriminate against aliens, the Supreme


Court has been more lenient in applying an intermediate standard of
scrutiny to illegitimacy under federal law than under state law. The
Supreme Court upheld a federal law granting immigration preferences
to legitimate children as within Congress’s plenary power to regulate
immigration [Fiallo v. Bell, 430 U.S. 787 (1977)].
(1) In 2012, the Court permitted to stand a law that required officers
to verify the immigration status of anyone they stopped, detained,
or arrested who they reasonably suspected was in the country
illegally [Arizona v. United States, 567 U.S. 387 (2012)].
c. Classifications based on gender are “quasi-suspect” and violate the Equal
Protection Clause unless they “serve important governmental objectives
and [are] substantially related to achievement of those objectives” [Craig v.
Boren, 429 U.S. 190 (1976)]. In Craig, an Oklahoma statute permitted the
sale of beer to females who were 18 years of age or older, but prohibited the
sale of beer to males under 21 years of age. The statute was held unconsti-
tutional because it was not substantially related to any important government
objectives. (Where separate treatment of men and women is found to be
proper, a state may offer different, but equivalent, facilities to each gender.)
(1) Intermediate review applies whether the classification is invidious
(intended to harm) or benign (intended to help or to remedy past
discrimination). Intentional, or purposeful, discrimination is required to
trigger middle-tier scrutiny; discriminatory effect alone is insufficient.
(2) Statutes that reinforce archaic, gender-based stereotypes will
almost certainly be struck down.
EXAMPLE: The Supreme Court required a state to show an “ex-
ceedingly persuasive justification” for its gender discrimination in
admissions to the Virginia Military Institute. This language reflects
the rigorous nature of the Craig v. Boren intermediate-level test
[United States v. Virginia, 518 U.S. 515 (1996)].
(3) In recent decisions, the Court has held unconstitutional, under
equal protection, all laws discriminating against women.
EXAMPLE: A state law giving preference to men over equally
qualified women to be administrators of decedents’ estates was
held unconstitutional [Reed v. Reed, 404 U.S. 71 (1971)].
EXAMPLE: Discrimination in military benefits to servicewomen
was held invalid [Frontiero v. Richardson, 411 U.S. 677 (1973)].
(4) Similarly, the Supreme Court has held unconstitutional under an
equal protection analysis laws discriminating against men [Craig
v. Boren, 429 U.S. 190 (1976)].
EXAMPLE: The Court struck down a law authorizing alimony
payments upon divorce to women but not to men [Orr v. Orr, 440
U.S. 268 (1979)].

134
CONSTITUTIONAL LAW

EXAMPLE: A New York law that permitted an unwed mother, but


not an unwed father, to block the adoption of their child was held
to be invalid [Caban v. Mohammed, 441 U.S. 380 (1979)].
EXAMPLE: The Supreme Court held that the exclusion of males from
a state nursing school violated a male applicant’s right to equal protec-
tion [Mississippi University for Women v. Hogan, 458 U.S. 718 (1982)].

EXAM TIP In some cases, laws discriminating against men have been upheld when they are
“substantially related to the achievement of an important government interest.”

EXAMPLE: The Supreme Court has upheld the registration of


males but not females for conscription by the military because
Congress, pursuant to its military powers, had determined that
this was necessary to further important government interests
[Rostker v. Goldberg, 453 U.S. 57 (1981)].
EXAMPLE: Statutory rape laws that punished the male participant,
but not the female accessory, were upheld because they furthered
the important state interest in preventing teenage pregnancy [Mi-
chael M. v. Sonoma County Superior Court, 450 U.S. 464 (1981)].
3. Affirmative Action
a. Affirmative action measures based on race must pass strict scrutiny.
(1) The only justifications for affirmative action that have been upheld are:
(a) remedying the effects of past or present discrimination in a
particular institution; and
(b) achieving a diverse student body in an institute of higher education.
(2) A school district may not assign individual students to schools
based on race in order to achieve “voluntary integration” or “racial
balancing” when school segregation is caused by social factors (de
facto segregation) and not caused by past or present government
action (de jure segregation) [Parents Involved in Cmty. Schools v.
Seattle School Dist. No. 1, 551 U.S. 701 (2007)].
(a) However, a school district is permitted to carry out such
voluntary racial balancing by “structural” measures, such as
redrawing school zones or building new schools.
(3) Affirmative action measures based on race must be narrowly
tailored to achieve their goals. The institution that adopts such a
program must show that available, workable, non-racial alterna-
tives do not suffice.
(4) In 2014, the Court upheld a Michigan voter initiative that banned
taking account of race in admissions to the state’s public
universities [Schuette v. Coalition to Defend Affirmative Action,
572 U.S. 291 (2014)].
b. Affirmative action based on gender need only pass intermediate scrutiny.

135
OUTLINE

EXAMPLE: Social Security statutes and tax exemptions that entitle


women to greater benefits have been upheld [Califano v. Webster,
430 U.S. 313 (1977)].

EXAMPLE: A U.S. navy discharge procedure that required male


officers twice denied promotion to be automatically discharged,
whereas female officers were not, was upheld because in the past,
men had been afforded greater promotional opportunities than women
[Schlesinger v. Ballard, 419 U.S. 498 (1975)].
c. In Richmond v. Croson [488 U.S. 469 (1989)], the Supreme Court first
held that minority set-asides (“affirmative action”) established by state
or local governments for construction projects—i.e., programs where
a fixed percentage of publicly funded money is awarded to minority-
owned businesses—are subject to strict scrutiny review, and must be
narrowly tailored to justify a compelling interest.
(1) In Adarand Construction, Inc. v. Pena [515 U.S. 200 (1995)],
the Supreme Court set forth a clear rule that any race-based
affirmative action program designed to remedy past discrimina-
tion—whether enacted by a state municipality or even the federal
government—is subject to strict scrutiny.
(2) This rule applies to any “benign,” or compensatory, program by any
government entity that either favors or discriminates against racial
or ethnic minorities. Other general principles include the following:
(a) remedying past discrimination in a particular government institu-
tion is generally viewed as a compelling interest, but attempting to
remedy general societal injustice through affirmative action is not;
(b) race or ethnic origin may be considered as a “plus” factor in
admissions programs, for the purpose of achieving a diverse
student body [Regents of University of California v. Bakke,
438 U.S. 265 (1978)]; and
1) Bakke was affirmed in Grutter v. Bollinger [539 U.S. 306
(2003)], whereby the Supreme Court held that a school
may take race into account as one of many factors in
making admissions decisions.
2) However, a school may not use an admissions system
that gives a set number of admissions points to each
minority applicant, even if the purpose is to create a
diverse student body [Id.].
(c) discrimination by private employers is not subject to equal
protection review.
4. Other Classifications
a. Age is neither a “suspect” nor a “quasi-suspect” classification. Thus,
laws and other governmental actions against the elderly are judged by
the traditional (or rational basis) test.

136
CONSTITUTIONAL LAW

EXAMPLE: The Supreme Court upheld the validity of a Massachusetts


statute requiring police officers to retire at age 50 even though they
may be as physically fit as younger officers [Massachusetts Board of
Retirement v. Murgia, 427 U.S. 307 (1976)].
b. The Supreme Court has held that “poverty standing alone is not a
suspect classification” [Harris v. McRae, 448 U.S. 297 (1980)].
c. Mental retardation is not a “quasi-suspect” classification, and the
rational basis standard of review is applicable [City of Cleburne v.
Cleburne Living Center, 473 U.S. 432 (1985)].

EQUAL PROTECTION CHALLENGES


Rational Basis Intermediate Scrutiny Strict Scrutiny

• Age • Gender • Alienage


• Alienage (some) • Illegitimacy • Domestic travel
• Disability • Undocumented • National origin
• Sexual orientation alien children • Race
• Wealth • Voting
• All else

137
OUTLINE

VII. PRIVILEGES AND IMMUNITIES CLAUSES

A. Privileges or Immunities under the Fourteenth Amendment


1. The Fourteenth Amendment provides that no state shall make or enforce any
law which abridges the privileges or immunities of citizens of the United States.
2. The Supreme Court ruled early on, in the Slaughter-House Cases
[83 U.S. 36 (1873)], that the Privileges or Immunities Clause of the
Fourteenth Amendment had very limited application. Among the
protected privileges and immunities of national citizenship are the rights
[Twining v. New Jersey, 211 U.S. 78 (1908)]:
a. to travel from state to state;
b. to petition Congress for redress of grievances;
c. to vote for national offices;
d. to enter public lands;
e. to be protected while in custody of U.S. Marshals; and
f. to assemble peaceably.

NOTE Corporations and aliens are not protected under the Fourteenth Amendment
Privileges or Immunities Clause.

EXAM TIP When an in-state citizen is discriminated against on the basis of the
citizen having only resided in the state for a limited time (i.e., a durational
residency requirement), this citizen can assert that the Fourteenth
Amendment Privileges or Immunities Clause has been violated, insofar
as an aspect of a citizen’s right to travel (the right to elect to become a
permanent resident of a new state) has been violated.

B. Privileges and Immunities under Article IV, Section 2


1. Sometimes referred to as the Comity Clause, Article IV, Section 2 provides:
“The Citizens of each State shall be entitled to all Privileges and Immunities
of Citizens in the several States.”
2. This clause prohibits states from discriminating against citizens who are
nonresidents with respect to rights and activities that are fundamental to
the national union. Corporations and aliens are not “citizens” for purposes
of the Article IV Privileges and Immunities Clause.
a. The following have been held to be invalid forms of nonresident discrimination:
(1) a state statute requiring a nonresident commercial fisherman to pay
a $2,500 license fee to fish offshore, while a resident fisherman paid
only a $25 license fee [Toomer v. Witsell, 334 U.S. 385 (1948)];
(2) a commuter tax applied to nonresidents who entered New
Hampshire to work, while New Hampshire residents were exempt
from the tax [Austin v. New Hampshire, 420 U.S. 656 (1975)];
(3) a state statute imposing residency requirements on women
seeking an abortion, which was held invalid because an individual

138
CONSTITUTIONAL LAW

has a fundamental right to seek medical care [Doe v. Bolton, 410


U.S. 179 (1973)]; and
(4) a state law requiring employers to give hiring preference to state
residents [Hicklin v. Orbeck, 437 U.S. 518 (1978)].
b. The Privileges and Immunities Clause does not protect a nonresident
against all forms of discrimination. The following types of nonresident
discrimination have been upheld:
(1) a state statute requiring a nonresident to pay $225 for a recre-
ational hunting license, while a resident hunter paid only a $9
license fee, because it is within a state’s police power to regulate
recreational, noncommercial activities [Baldwin v. Montana Fish
and Game Commission, 436 U.S. 371 (1978)]; and
(2) discrimination against nonresidents will be given special consid-
eration if its purpose is the preservation of natural, state-owned
resources [Sporhase v. Nebraska, 458 U.S. 941 (1982)].

EXAM TIP Discrimination against nonresident citizens in regards to an essential eco-


nomic right or liberty triggers the Article IV Privileges and Immunities Clause,
whereas general economic discrimination against a business or entity is more
often viewed using Commerce Clause analysis.

139
OUTLINE

VIII. RETROACTIVE LEGISLATION

A. The Contract Clause


1. “No state shall…pass any…Law impairing the Obligation of Contracts” [U.S.
Const. art. I, Section 10].
a. The principal reason for the inclusion of the Contract Clause in the
Constitution was to prevent state legislatures from passing retroactive
laws impairing an existing contractual obligation. Therefore, the Contract
Clause applies only to state legislation and not to state court decisions.
Similarly, the Contract Clause does not apply to the federal government.
b. A private contract can be modified by the legislature under its police
power when it is necessary to serve an important and legitimate public
interest and the regulation is a reasonable and narrowly tailored means
of promoting that interest.
(1) Thus, during the Great Depression, a statute imposing a mora-
torium on mortgage foreclosures was upheld [Home Building &
Loan Association v. Blaisdell, 290 U.S. 398 (1934)].
(2) In determining whether a contract may be modified, the Court will
consider [Allied Structural Steel Co. v. Spannaus, 438 U.S. 234 (1978)]:
(a) the severity of the impairment; and
(b) the importance of the public interest to be served.
EXAMPLE: New York and New Jersey violated the Contract
Clause by permitting the Port Authority to use funds to subsi-
dize public transportation in violation of a previous statutory
covenant to private bondholders [United States Trust Co. v.
New Jersey, 431 U.S. 1 (1977)].
EXAMPLE: The Supreme Court recognized the validity of
police power limitations on the Contract Clause and invali-
dated state pension reform legislation which increased the
obligation of companies under preexisting pension plans [Al-
lied Structural Steel Co. v. Spannaus, 438 U.S. 234 (1978)].

B. Ex Post Facto Laws


1. There are two ex post facto clauses in the Constitution that prevent both
the state and federal governments from passing retroactive criminal laws.
Article I, section 9, clause 3 provides: “No…ex post facto Law shall be
passed”; and Article I, Section 10, Clause 1 provides: “No State shall pass
any…ex post facto law.”
a. In general, a statute retroactively alters the criminal law if it:
(1) makes criminal an act that was not a crime when committed;
(2) prescribes greater punishment for a crime after its commission;
(3) decreases the amount of evidence required for conviction; or

140
CONSTITUTIONAL LAW

(4) extends the statute of limitations for a crime as to which the previ-
ously applicable statute of limitations has already expired.

C. Bills of Attainder
1. A bill of attainder is a legislative act that inflicts punishment without a judicial
trial upon named individuals or an easily ascertainable group for past conduct.
2. Article I, Section 9, Clause 3 states: “No bill of attainder…shall be passed”;
and Article I, Section 10, Clause 1 provides: “No state shall pass any Bill of
Attainder.” These two provisions prevent both federal and state legislatures
from passing bills of attainder.
a. The Supreme Court held that a provision in the Landrum-Griffin Act
making it a crime for a member of the Communist Party to act as an
officer of a labor union was the equivalent of legislative punishment, and
hence a bill of attainder [United States v. Brown, 381 U.S. 437 (1965)].
b. However, the Court later held that legislation authorizing government
control of various presidential papers and tape recordings did not
constitute a bill of attainder, because the act was nonpunitive [Nixon v.
Administrator of General Services, 433 U.S. 425 (1977)].

141
OUTLINE

IX. FIRST AMENDMENT FREEDOMS

A. Freedom of Religion and Separation of Church and State


1. The First Amendment provides that “Congress shall make no law respecting
an establishment of religion, or prohibiting the free exercise thereof.”
2. Establishment Clause
a. Where a government program prefers one religion or religious sect
over others, strict scrutiny analysis will be applied [Board of Education
v. Grumet, 512 U.S. 687 (1994)].
b. Where the legislation or government program contains no religious or
sect preference, the Supreme Court will follow the following three-part
test under Lemon v. Kurtzman [403 U.S. 602 (1971)]:
(1) the statute must have a secular legislative purpose;
(2) the principal or primary effect or purpose must neither advance
nor inhibit religion; and
(3) the statute must not foster an excessive government entangle-
ment with religion.
c. Religious Activities Conducted at Public Schools
(1) As a general rule, religious activities conducted in public schools
violate the Establishment Clause because their primary purpose
is to promote religion.
(2) The following practices in public schools have been held to be invalid:
(a) prayer and bible reading in public schools [Engel v. Vitale,
370 U.S. 421 (1962)];
(b) an Alabama law authorizing a period of silence “for meditation
or voluntary prayer” [Wallace v. Jaffree, 472 U.S. 38 (1985)];
(c) posting the Ten Commandments on the walls in public school
classrooms [Stone v. Graham, 449 U.S. 39 (1980)]; and
(d) a public school sponsoring a rabbi or other cleric to conduct
even a nondenominational prayer as part of a graduation
ceremony [Lee v. Weisman, 505 U.S. 577 (1992)].
1) A state legislature may employ a chaplain to conduct
an opening day prayer [Marsh v. Chambers, 463
U.S. 783 (1983)].
2) However, a state court judge may not conduct such a
daily prayer [North Carolina Civil Liberties Union Legal
Foundation v. Constangy, 947 F.2d 1145 (4th Cir. 1991)].
(3) High school students being allowed to vote on whether a student-
delivered “invocation” or “message” would take place at school
football games is impermissible, as such an activity would be
forced on all those present at a school-sponsored event [Santa
Fe Independent School District v. Doe, 530 U.S. 290 (2000)].

142
CONSTITUTIONAL LAW

(4) A state university regulation prohibiting the use of school facilities


by a registered student religious organization (whereas facilities
were available to other student groups) was held to be in viola-
tion of the freedom of free speech under the First Amendment
[Widmar v. Vincent, 454 U.S. 263 (1981)].
(5) The Supreme Court has held that prohibiting a religious club from
meeting in a public school amounted to religious viewpoint discrimi-
nation and violated the Free Speech Clause of the First Amendment.
Furthermore, the Court ruled, permitting the club to meet on school
grounds would not be in violation of the Establishment Clause [Good
News Club v. Milford Central School, 533 U.S. 98 (2001)].
(6) The University of California required student groups to open their
membership to “all comers.” The university denied funding to the
Christian Legal Society on the grounds that it refused member-
ship to gays and lesbians. The Court upheld the university’s rule
against a First Amendment challenge on the grounds that it was a
viewpoint-neutral restriction. The Court ruled that, in the academic
context, the rule was reasonable in light of the goals of ensuring
all students are included in the student group program, avoiding
the need for the University to inquire into the groups’ motives
for excluding members, bringing together diverse students, and
complying with state antidiscrimination rules [Christian Legal
Society v. Martinez, 561 U.S. 661 (2010)].
(7) Other decisions regarding religious activities in public schools include:
(a) a prohibition against showing religious films at a school
violated free speech [Lamb’s Chapel v. Center Moriches
Union Free School District, 508 U.S. 387 (1993)];
(b) a university’s refusal to fund a student publication because it
addressed issues from a religious perspective violated free
speech [Rosenberger v. Rector and Visitors of the University
of Virginia, 515 U.S. 819 (1995)]; and
(c) a state university’s viewpoint-neutral allocation of
mandatory student activities fees to political and ideological
groups as part of a program to facilitate extracurricular
student speech did not violate the First Amendment rights
of students who objected to subsidizing those groups’
activities [Board of Regents of University of Wisconsin
System v. Southworth, 529 U.S. 217 (2000)].
d. Public School Curriculum
(1) Anti-evolution laws prohibiting the teaching of Darwinian princi-
ples in public schools are unconstitutional [Epperson v. Arkansas,
393 U.S. 97 (1968)].
e. Other Government Endorsement of Religion
(1) Displays that celebrate the holiday season without favoring one
religion over another are generally upheld. The government

143
OUTLINE

cannot permit the type of display that a reasonable observer


would conclude constitutes an endorsement of religion. The
context surrounding the display is a key factor in determining its
validity [Lynch v. Donnelly, 465 U.S. 668 (1984)].
(a) The display of a creche as part of a Christmas display in a park
passes the Lemon test, but a creche with no other symbols
nearby—i.e., a “Season’s Greetings” banner or Chanukah
menorah—and prominently displayed by a private religious
group in the county courthouse violated the Establishment
Clause [County of Allegheny v. ACLU, 492 U.S. 573 (1989)].
(2) Placing a cross in a state-owned park immediately in front of the
State Capitol was held to not violate the Establishment Clause;
there was no endorsement of religion because the park had long
been used by a variety of groups to conduct expressive activities
[Capitol Square Review Board v. Pinette, 515 U.S. 753 (1995)].
(3) Opening town board meetings with sectarian prayers, where
there was no discrimination in the selection of prayer-givers or
content, was held to be permissible, with the Court explaining
that “[l]egislative bodies do not engage in impermissible coercion
merely by exposing constituents to prayer they would rather not
hear and in which they need not participate” [Town of Greece v.
Galloway, 572 U.S. 565 (2014)].
(4) Placement of the Ten Commandments on the walls of court-
houses was unconstitutional because there was no secular
purpose [McCreary County v. ACLU, 545 U.S. 844 (2005)].
(a) However, having the Ten Commandments carved on a six-
foot-high granite monument in a 22-acre park surrounding
the Texas State Capital, and surrounded by 37 other monu-
ments, was held to be constitutional [Van Orden v. Perry,
545 U.S. 677 (2005)].
(b) The crucial difference between the cases was the extent
to which an objective observer would have perceived each
display of the Ten Commandments as a governmental
endorsement of religion.
(5) Sunday closing laws have been upheld as a governmental action
with a mere incidental benefit to religion [McGowan v. Maryland,
366 U.S. 420 (1961)].
(a) Blue laws or state statutes giving churches and schools
the power to veto applications for liquor licenses within a
500-foot radius of the church or school were held unconsti-
tutional because of “excessive government entanglement”
[Larkin v. Grendel’s Den, Inc., 459 U.S. 116 (1982)].
f. Tax Deductions for Religious Institutions
(1) The Supreme Court has upheld the validity of a property tax
exemption for religious institutions as the equivalent of exempting

144
CONSTITUTIONAL LAW

other charitable organizations [Walz v. Tax Commission of New


York, 397 U.S. 664 (1970)].
(2) A tax exemption from sales and use taxes available only
for the sale of religious magazines and books violates the
Establishment Clause as an endorsement of religion [Texas
Monthly v. Bullock, 489 U.S. 1 (1989)].
g. Government Aid to Religious Schools
(1) Most government programs providing direct aid to parochial
elementary and high schools have been held to violate the
Establishment Clause because they involve “excessive
government entanglement” with religion.
(2) Programs that provide aid to all elementary and secondary
students (including parochial school students) have been held
to “pass” the three-prong test.
(3) Providing bus transportation to and from school for all students
(including those attending parochial schools) was held valid
[Everson v. Board of Education, 330 U.S. 1 (1947)].
(a) It was also held that necessary municipal services (e.g.,
police and fire protection) can be provided to churches and
church-related institutions.
(4) If a tax deduction is given to all parents based on actual expendi-
tures for children attending any public, private, or religious school,
it will be upheld [Mueller v. Allen, 463 U.S. 388 (1983)].
(5) The Supreme Court permitted the state to furnish textbooks to all
students, including those attending parochial schools, because
there was little entanglement, and the primary effect was secular
[Board of Education v. Allen, 392 U.S. 236 (1968)].
(6) Furnishing standardized secular examinations to parochial school
students was held valid because the materials did not contain
any religious content [Wolman v. Walter, 433 U.S. 229 (1977)].
(7) Public health services, including a school lunch program, can
be provided to all students, since their purpose and effect are
secular [Lemon v. Kurtzman, 403 U.S. 602 (1971)].
(8) The Supreme Court stated that Chapter 2 of the 1981 Education
Consolidation and Improvement Act, which channels federal funds
to local education agencies to acquire, for use in public and private
schools, instructional and educational materials, including library and
media materials and computer software and hardware, had a secular
purpose and did not have the effect of advancing religion either by
resulting in governmental indoctrination or by defining recipients by
reference to religion, and therefore did not violate the First Amendment’s
Establishment Clause [Mitchell v. Helms, 530 U.S. 793 (2000)].
(9) A public school may pay for an interpreter for a deaf child at
a religious high school in a program that aids both public and

145
OUTLINE

private school disabled students without reference to religion


[Zobrest v. Catalina Foothills School District, 509 U.S. 1 (1993)].
1) Grants to parochial schools for salaries of teachers of
secular subjects have been held invalid, because of the
risk of “excessive government entanglement” [Lemon v.
Kurtzman, 403 U.S. 602 (1971)].
(a) In contrast, grants to church-related colleges are permissible
because their instructors are thought to be restrained by the
various academic disciplines (e.g., sociology, biology, etc.),
and their students are older, more mature, and less suscep-
tible to indoctrination.
1) A state statute permitting all parents, in computing their
state income tax, to deduct expenses incurred in providing
“tuition, textbooks and transportation” for their children in
elementary and secondary schools, including parochial
schools, was upheld [Mueller v. Allen, 463 U.S. 388 (1983)].
2) Giving tax deductions or tuition grants only to parents
whose children attend parochial schools was held unconsti-
tutional as violating the Establishment Clause [Committee
for Public Education v. Nyquist, 413 U.S. 756 (1973)].
3) A voucher program that allows parents to send their
children to parochial or religious schools with state aid
instead of to failed public schools is allowed because
the aid is neutral with respect to religion to a broad class
of citizens, the program is defined without reference to
religion, and parents direct aid to religious schools as
a result of their own independent and private choice
[Zelman v. Simmons-Harris, 536 U.S. 639 (2002)].
h. Providing Public Services through Religious Institutions
(1) Federal grants to church-affiliated hospitals for maintenance
and care of indigent patients have been held valid [Bradfield v.
Roberts, 175 U.S. 291 (1899)].
3. Free Exercise Clause
a. A person’s religious beliefs are absolutely protected. The government
may not punish an individual by denying benefits or imposing burdens
based on religious belief [Cantwell v. Connecticut, 310 U.S. 296 (1940)].
(1) A state cannot require a person to carry a message on his license
plate (e.g., New Hampshire’s “Live Free or Die”) which offends
his religious belief [Wooley v. Maynard, 430 U.S. 705 (1977)].
(2) The government may not determine the truth or falsity of a person’s
religious beliefs, but it may determine a person’s sincerity in his claim
of religious belief [United States v. Ballard, 332 U.S. 78 (1944)].
(3) Public employment may not be conditioned on taking an oath
based on a religious belief.

146
CONSTITUTIONAL LAW

b. Where an individual’s conduct is motivated by his religious beliefs, the


state may regulate or prohibit the activity if the regulation is neutral in
respect to religion and is of general applicability.
(1) In Employment Division v. Smith [494 U.S. 872 (1990)], the State
of Oregon criminalized the possession of peyote, and no exemp-
tion was made for American-Indians who use peyote for their
religious rituals. The Supreme Court upheld the law, and held that
a neutral law of general applicability does not violate the Free
Exercise Clause, even if it prohibits or punishes conduct engaged
in as a religious observance or practice.
c. The approach taken by the Supreme Court in Smith is consistent with
all but two of the earlier cases:
(1) a state law outlawing polygamy was upheld [Reynolds v. United
States, 98 U.S. 145 (1878)];
(2) the Court struck down a free exercise challenge by a Jewish Air
Force doctor who violated uniform dress requirements by wearing a
yarmulke while on duty [Goldman v. Weinberger, 475 U.S.503 (1986)];
(3) the Social Security tax applied to an Amish employer was held
constitutional even though his religious beliefs prohibited him
from making payments and from receiving benefits [United States
v. Lee, 455 U.S. 252 (1982)];
(4) a state cannot deny unemployment compensation benefits to
a person whose religious faith commands the observance of
Saturday as the Sabbath [Sherbert v. Verner, 374 U.S. 398 (1963)];
(a) This approach by the Supreme Court to denials of unem-
ployment compensation benefits is inconsistent with, but not
overruled by, Smith. In light of Smith, it should be viewed as
precedent only for cases involving very similar facts.
(5) the Court required that Amish children be exempt from a state law
requiring all children to attend high school [Wisconsin v. Yoder,
406 U.S. 205 (1972)];
(a) This case is also inconsistent with Smith, but has not
been overruled.
(6) the Court upheld Sunday closing laws because they further the
state interest in providing a common day of rest [Braunfeld v.
Brown, 366 U.S. 599 (1961)]; and
(7) a state-funded scholarship program for which devotional theology
majors were ineligible did not suggest animus toward religion or
impose more than a relatively minor burden on program partici-
pants [Locke v. Davey, 540 U.S. 714 (2004)].
d. Where the government purposely interferes with particular conduct because
it is dictated by religious beliefs, strict scrutiny analysis will be applied
and the law will be held unconstitutional unless it can be justified through
proof of a compelling state interest served by narrowly tailored means.

147
OUTLINE

EXAMPLE: A city ordinance banned all animal sacrifice done in a public


or private ritual not for the primary purpose of food consumption (“ko-
sher” slaughter was permitted). The primary purpose of the law was not
to prevent cruelty to animals, but to abolish the sacrificial rituals of a par-
ticular Cuban Santeria religious sect. This purposeful interference with
religious observance and practice was held unconstitutional by the Court
[Church of the Lukumi Babalu Aye, Inc. v. Hialeah, 508 U.S. 520 (1993)].

B. Freedom of Expression
1. The First Amendment provides: “Congress shall make no law…abridging
the freedom of speech, or of the press; or the right of the people peaceably
to assemble, and to petition the government for a redress of grievances.”
a. The First Amendment was held applicable to the states through the
Due Process Clause of the Fourteenth Amendment [Gitlow v. New
York, 268 U.S. 652 (1925)].
2. The government may neither censor all categories of speech nor engage in
content-based discrimination among different categories of speech (even if
that speech is offensive), with some exceptions.
3. Exceptions to Freedom of Speech
a. Strict Scrutiny
(1) The regulation of speech is allowable if it passes strict scrutiny.
EXAMPLE: A provision of the District of Columbia Code prohibiting
signs within 500 feet of a foreign embassy that tended to bring the
foreign government into “public odium” or “public disrepute” was held
unconstitutional as a content-based regulation of political speech that
could not be justified by proof of a compelling governmental interest
served by narrowly tailored means [Boos v. Barry, 485 U.S. 312 (1988)].
EXAMPLE: Congress banned corporations and unions from
using their general treasury funds to advocate the election
or defeat of a candidate within 30 days of an election. Citi-
zens United wanted to run a television show that was critical
of presidential candidate, Hillary Clinton, shortly before the
primary election. The Court ruled that corporate spending
on speech merits the same First Amendment protection as
speech by natural persons. Hence, Congress could not cen-
sor such corporate speech [Citizens United v. Federal Elec-
tion Commission, 558 U.S. 310 (2010)]. However, the Court
has held that a state ban on the use of payroll deductions to
fund political campaigns is not an abridgement of speech. The
unions may engage in political speech, but are barred from
enlisting the state’s support through payroll deductions [Ysursa
v. Pocatello Education Association, 353 U.S. 553 (2009)].
EXAMPLE: In Washington State, citizens may put a referendum
on the ballot if they can get a certain number of signatures on

148
CONSTITUTIONAL LAW

a petition. Washington publicly disclosed the signers of a peti-


tion seeking a referendum on the rights of same-sex partners.
The Court ruled that the First Amendment rights of the signers
were not violated, at least in the absence of any showing that the
disclosure caused specific harm to the signers. The disclosure
requirement served the goal of maintaining the integrity of the
electoral process [Doe v. Reed, 561 U.S. 186 (2010)].
b. Conduct Regulation
(1) A law which regulates conduct but creates an incidental burden on
speech is allowable if [United States v. O’Brien, 391 U.S. 367 (1968)]:
(a) the regulation furthers an important or substantial government
interest that is unrelated to the suppression of free expression; and
(b) the incidental restriction on speech is no greater than is
essential to the furtherance of that interest.
EXAMPLE: Students wearing black armbands to protest
the Vietnam War was held to be protected speech [Tinker v.
Des Moines Independent Community School District, 393
U.S. 503 (1969)].
EXAMPLE: A Texas prohibition against desecrating an Ameri-
can flag was held unconstitutional as applied to an individual
who burned a flag as a form of political expression. Strict
scrutiny was applied because the Texas statute was intended
to control expression [Texas v. Johnson, 491 U.S. 397 (1989)].
EXAMPLE: Requiring students to salute the American
flag and say the Pledge of Allegiance in school was held
unconstitutional [West Virginia State Board of Education v.
Barnette, 319 U.S. 624 (1943)].
EXAMPLE: A federal statute, the Flag Protection Act of
1989, was subjected to strict scrutiny and held unconsti-
tutional as a content-based regulation of political speech
[United States v. Eichman, 496 U.S. 310 (1990)].
EXAMPLE: Virginia’s prohibition of cross-burning with the in-
tent to intimidate was not unconstitutional since it banned con-
duct rather than expression. While cross-burning could consti-
tute expression, such expressive conduct was not proscribed
unless it was done with the intent to intimidate, and targeting
cross-burning was reasonable because burning a cross was
historically a particularly virulent form of intimidation. A plurality
of the Supreme Court, however, found the law to be a facially
unconstitutional restraint on speech, to the extent it had the
effect that any cross-burning could, prima facie, be sufficient
evidence from which the jury could infer the required intent to
intimidate [Virginia v. Black, 538 U.S. 343 (2003)].

149
OUTLINE

EXAMPLE: An ordinance to keep the streets clean was held


as not permissible to justify prohibiting people from handing
out literature to others willing to receive it [Schneider v. State
of New Jersey, 308 U.S. 147 (1939)].
c. Government as Speaker
(1) Where the speaker is the government rather than a private actor, the
government may discriminate based on the content of the speech.
EXAMPLE: A city erects several permanent monuments in a
public park. Some of the monuments were donated by private
organizations. A religious organization asks the city to erect a
monument containing the Seven Aphorisms of Summum. The city
refuses. The organization sues the city, alleging the violation of
the organization’s right of free speech. The Supreme Court held
that there was no violation of the First Amendment. The perma-
nent monuments constituted speech by the government, even if
the monuments were donated by private organizations. When the
government speaks, it may favor some views and disfavor others
[Pleasant Grove City v. Summum, 555 U.S. 460 (2009)].
EXAMPLE: The Court held that the state of Texas was free to reject
specialty license plates bearing the Confederate battle flag [Walker v.
Texas Division, Sons of Confederate Veterans, 135 S. Ct. 2239 (2015)].
EXAMPLE: If private individuals or organizations engage in direct
communication in a public park (for example, making speeches
or distributing leaflets, as opposed to donating permanent monu-
ments) then, of course, the government may not discriminate
among the speakers based on the content of their message, since
the communication is by private actors and not by the government.
EXAMPLE: Pursuant to the First Amendment, a city must allow
the KKK to temporarily erect a cross on a public park when other
groups have been permitted to temporarily erect installations.
Temporary installations by private organizations constitute private
speech, while permanent installations constitute government
speech even if donated by private organizations.
d. Unprotected Speech
(1) A regulation which relates to unprotected speech is permissible.
(2) Unprotected speech includes:
(a) speech that advocates violence or unlawful action;
1) The constitutional guarantees of free speech do not
permit state regulation of the advocacy of the use of
force or of violation of law, “except where such advo-
cacy is directed to inciting or producing imminent
lawless action and is likely to incite or produce such
action” [Brandenburg v. Ohio, 395 U.S. 444 (1969)].

150
CONSTITUTIONAL LAW

EXAMPLE: A Virginia statute which stated, “Any such


burning of a cross shall be prima facie evidence of an intent
to intimidate a person or group of persons,” was found to
be unconstitutional. The Court held that this provision was
unconstitutional because of its “indiscriminate coverage.”
However, it ruled that a state “may ban cross burning carried
out with the intent to intimidate.” The state must prove the
intent to intimidate [Virginia v. Black, 538 U.S. 343 (2003)].
(b) fighting words;
1) Words likely to incite an ordinary citizen to acts of
immediate physical retaliation may be punished
[Chaplinsky v. New Hampshire, 315 U.S. 568 (1942)].
2) To fall within this category, the speech must be more
than annoying or offensive; there must be a genuine
likelihood of imminent violence by a hostile audience.
3) “Fighting words” statutes are subject to facial invalidity
if the conduct proscribed is vague (e.g., a law
prohibiting “opprobrious words”) or overbroad.
4) “Fighting words” statutes designed to punish certain
viewpoints are unconstitutional [R.A.V. v. St. Paul,
505 U.S. 377 (1992)].
(c) hostile audience speech;
1) Speech which elicits an immediate violent response
against the speaker by an audience may be grounds
for prosecution. The police, however, must make
reasonable efforts to protect the speaker, to guard
against a “heckler’s veto” of unpopular speech.
(d) obscene speech; and
1) For speech to be considered obscene, the following
three-part test must be satisfied [Miller v. California,
413 U.S. 15 (1975)]:
a) the average person, applying contemporary
community standards, would find that the work,
taken as a whole, appeals to the prurient interest
(a national standard is not used);
i) The manner in which allegedly obscene
materials are advertised and sold may be
probative of their “prurient” appeal [Ginzburg
v. United States, 383 U.S. 463 (1966)].
b) the work depicts or describes, in a patently
offensive way, sexual conduct specifically defined
by the applicable state law; and
c) the work, taken as a whole, lacks serious literary,
artistic, political, or scientific value.

151
OUTLINE

i) A national standard is used to determine this


value [Pope v. Illinois, 481 U.S. 497 (1987)].
2) Child pornography is unprotected speech. It may be regu-
lated even without satisfying the Miller test because of the
state’s compelling interest in protecting minor children.
a) The sale and distribution of visual depictions of
sexual conduct involving children may be prohibited
[New York v. Ferber, 456 U.S. 942 (1982)].
b) A state may criminalize the private possession of
child pornography in one’s home [Osborne v. Ohio,
495 U.S. 103 (1990)].
c) Recently, the Supreme Court held that a statute
which outlawed not only actual child pornography
but also virtual child pornography (in which no
real children were depicted) was overly broad and
violated the First Amendment [Ashcroft v. Free
Speech Coalition, 535 U.S. 234 (2002)].
i) However, in a follow-up case, the Court upheld
a congressional statute that punished those who
solicited or distributed pornography while believing
it depicted real children, even if in fact it did not
[United States v. Williams, 553 U.S. 285 (2008)].
(e) defamatory speech.
1) Constitutional restrictions apply to defamatory speech
where the plaintiff is either a public official or public
figure, or where the defamatory statement involves a
matter of public concern.
2) When the plaintiff is a private person and the subject of the
statement is a matter of purely private concern, such as the
operations of a credit-reporting agency, the plaintiff need not
prove actual malice or negligence, and may recover according
to common law defamation principles [Dun & Bradstreet,
Inc. v. Greenmoss Builders, Inc., 472 U.S. 749 (1985)].
3) When the plaintiff is a private person and the subject
of the statement is a matter of public concern, the
plaintiff need not prove actual malice, but must prove
negligence about the truth or falsity of the statement.
4) When the plaintiff is a public official or public figure, the
plaintiff must prove the state law requirements of defa-
mation plus actual malice, defined as knowledge of the
falsity or reckless disregard of the truth or falsity of the
statement. This rule holds whether the allegedly defam-
atory statement is a matter of public or private concern
[New York Times v. Sullivan, 376 U.S. 254 (1964)].

152
CONSTITUTIONAL LAW

5) The Supreme Court held that a private plaintiff suing


a media defendant for false-light invasion of privacy
concerning a matter of public interest must prove
actual malice [Time, Inc. v. Hill, 385 U.S. 374 (1967)].
a) Note that this standard is higher than in cases
where a private plaintiff sues for defamation over
matters of public interest; in the latter cases, the
plaintiff need only prove negligence.
b) In addition, the Court said that a newspaper or
broadcaster cannot be held liable for publishing
truthful information obtained from the public
record [Cox Broadcasting Co. v. Cohn, 420 U.S.
469 (1975)].
6) Pure opinion may be actionable if it reasonably implies
a false and defamatory fact [Milkovich v. Lorain Journal
Co., 497 U.S. 1 (1990)].
(3) Certain categories of speech receive lower levels of protection,
however, including:
(a) commercial speech;
1) A state may prohibit commercial advertising of
matters that are illegal (e.g., prostitution) or advertising
that is untruthful, misleading, or deceptive [Pittsburgh
Press Co. v. Pittsburgh Commission on Human
Relations, 413 U.S. 376 (1973)].
2) Commercial speech is protected by the First
Amendment if it is not false or deceptive and does not
relate to unlawful activity. If commercial speech satis-
fies these requirements, government regulation of
the speech must satisfy the three-part test set forth in
Central Hudson Gas v. Public Service Commission
[447 U.S. 557 (1980)]. The regulation must:
a) serve a substantial governmental interest;
b) directly advance the substantial governmental
interest; and
c) not be more extensive than is necessary to serve
that interest.
3) A state cannot place an absolute ban on the
advertisement of drug prices by a pharmacist
[Virginia State Board of Pharmacy v. Virginia Citizens
Consumer Council, Inc., 425 U.S. 748 (1976)].
4) A federal law making it a crime to send unsolicited
advertisements for contraceptive devices was held
unconstitutional [Bolger v. Youngs Drug Products Corp.,
463 U.S. 60 (1983)].

153
OUTLINE

5) A state cannot prohibit attorneys from advertising


routine legal services at stated fees [Bates v. State Bar
of Arizona, 433 U.S. 350 (1977)]
a) Similarly, a state supreme court rule prohibiting
an attorney from mailing new-office-opening
announcements to persons “other than lawyers,
clients, friends or relatives” was held to be uncon-
stitutional [In re R. M. J., 455 U.S. 191 (1982)].
6) A state may discipline lawyers for “in-person”
solicitation of clients for personal gain because of the
potential for overreaching [Ohralik v. Ohio State Bar
Association, 436 U.S. 447 (1978)].
7) A state can prevent doctors from claiming to be “board
certified” in a medical specialty unless the certifying
organization meets certain state standards because
the commercial speech doctrine does not protect
fraudulent speech, and because such a statute
reasonably advances the state’s interest in protecting
the public [American Academy of Pain Management v.
Joseph, 353 F.3d 2020 (9th Cir. 2004)].
8) It is within a state’s police power to prohibit billboards
carrying commercial advertising [Metromedia, Inc. v. City
of San Diego, 453 U.S. 490 (1981)]. It is unclear, however,
whether a total ban on billboards would be upheld.
9) A city ban on newsracks placed on sidewalks to
distribute commercial publications (while allowing
newsracks to sell newspapers) was held invalid,
because there was no reasonable “fit” between the
type of publication being regulated and the state interest
in reducing litter and promoting aesthetics [Cincinnati v.
Discovery Network, Inc., 507 U.S. 410 (1993)].
10) Clearly, advertising of unlawful products can be prohibited.
However, as to advertising of harmful, yet lawful,
products—i.e., “vice advertising,” such as advertising for
cigarettes, liquor, and gambling—it seems unlikely that the
government can completely ban truthful advertising.
a) The Supreme Court held that a ban prohibiting
all advertisement of liquor prices except for price
tags displayed on the merchandise, enacted in
order to protect the public and decrease alcohol
consumption by discouraging price wars, was
held unconstitutional [44 Liquormart, Inc. v.
Rhode Island, 517 U.S. 484 (1996)].
b) The Twenty-First Amendment, which allows the states
to regulate liquor and liquor establishments within the

154
CONSTITUTIONAL LAW

state’s borders, may not be used as a constitutional


basis to override the protection on commercial speech
afforded by the First Amendment.
c) A federal ban prohibiting beer bottle labels from
displaying alcoholic content was held unconstitu-
tional as a violation of the brewer’s free speech
rights. Because the law applied only to labels, not
to advertising, and only to beer, not to wine and
spirits, it did not directly advance the government’s
interest in preventing “strength wars” [Rubin v.
Coors Brewing Co., 514 U.S. 476 (1995)].
(b) speech by public employees; and
1) A person does not forgo his or her right to freedom of
speech merely by accepting public employment [Beilan v.
Board of Public Ed., School Dist. of Philadelphia, 357 U.S.
399 (1958)]. The employee retains a First Amendment
right to speak out on matters of public concern.
a) However, the government, as an employer, has a legiti-
mate interest in regulating the speech of its employees
to promote efficiency of its public services [Mandell
v. County of Suffolk, 316 F.3d 368 (2d Cir. 2003)].
2) There are two inquiries that determine the level of
protection afforded to a public employee’s speech
[Pickering v. Board of Ed. of Tp. High School Dist. 205,
Will County, Illinois, 391 U.S. 563 (1968); Connick v.
Myers, 461 U.S. 138 (1983)].
3) First, the court will determine whether the employee
spoke as a citizen on a matter of public concern. If not, the
employee has no First Amendment cause of action based
on the government employer’s reaction to the speech.
4) If the employee did speak on a matter of public concern,
the question then becomes whether the government
employer has an adequate justification for treating the
employee differently from any other member of the
general public [Garcetti v. Ceballos, 547 U.S. 410 (2006)].
5) In weighing the public employee’s First Amendment rights
against the public employer’s interest in promoting public
efficiency, the courts will balance the following factors
[Calef v. Budden, 361 F. Supp. 2d 493 (D.S.C. 2005)]:
a) impairs discipline by superiors;
b) impairs harmony among coworkers;
c) has a detrimental impact on close working relationships;
d) impedes the performance of the public
employee’s duties;

155
OUTLINE

e) interferes with the operation of the agency;


f) undermines the mission of the agency;
g) is communicated to the public or to coworkers in private;
h) conflicts with the responsibilities of the employee
within the agency; and
i) makes use of the authority and public
accountability which the employee’s role entails.
6) While the government entity has broader discretion
to restrict speech when it acts in its role as employer,
those restrictions imposed must be directed at speech
that has some potential to affect the entity’s operations
[Garcetti v. Ceballos, 547 U.S. 410 (2006)].
(c) sexual speech.
1) Regulation of sexual speech must serve a substantial
government interest and leave open reasonable
alternative channels of communication.
EXAMPLE: A ban on public nudity, including nude
dancing in adult entertainment establishments, was
held constitutional, with a plurality opinion focused on
the need to protect the important government interests
in societal order and morality [Barnes v. Glen
Theatres, Inc., 501 U.S. 1030 (1991)].
EXAMPLE: A majority opinion of the Supreme Court held
that a similar ban on nude dancing was content-neutral
and constitutional because its purpose was to deal with the
secondary effects of establishments where there is nude
dancing, such as prostitution and other forms of criminal
activity [City of Erie v. PAP’s A.M., 529 U.S. 277 (2000)].
e. Time, Place, Manner
(1) Reasonable regulation of the time, place, or manner of
speech is allowed.
(2) The government may place reasonable restraints on the
time, place, and manner of speech in public areas, such as
streets, sidewalks, and parks—places historically associated
with expressive conduct (e.g., picketing, leafleting, and
broadcasting). The focus is not on the “content” or “message”
of the speech, but rather its “conduct” or “method.”
(a) A three-part test is used to determine the constitutionality
of time, place, or manner regulations of speech and
assembly in public places. The regulation must:
1) be content-neutral as to both subject matter and
viewpoint (i.e., the regulation cannot prefer some
messages over others);

156
CONSTITUTIONAL LAW

2) be narrowly tailored to serve a significant (important)


government interest; and
3) leave alternative channels of communication open.

NOTE This test is very much like the test for intermediate scrutiny under
equal protection.

(b) The Court has held that a complete ban on door-to-door


solicitation is unnecessary, because a homeowner can
protect his privacy by posting a “No Solicitors” sign [Martin
v. Struthers, 319 U.S. 141 (1943)].
(c) An ordinance requiring door-to-door solicitors, or
canvassers, to identify themselves to local authorities was
upheld in the interests of crime prevention [Hynes v. Mayor
of Oradell, 425 U.S. 610 (1976)].
(d) A city may not require persons canvassing door to door to
register with the mayor’s office and receive a permit where the
canvassing consists of religious proselytizing (such as that
done by Jehovah’s Witnesses), anonymous political speech,
or the distribution of handbills [Watchtower Bible & Tract Soc.
of N.Y., Inc. v. Village of Stratton, 536 U.S. 150 (2002)].
(e) Charitable solicitations for funds in residential neighbor-
hoods are generally protected [Schaumburg v. Citizens for
a Better Environment, 444 U.S. 620 (1980)].
(f) Face-to-face solicitation of funds at an airport terminal, which
is a non-public forum, may be prohibited, but a total ban on
distribution of literature may not be [International Society for
Krishna v. Lee, 505 U.S. 672 (1992)].
(g) The Supreme Court has upheld the constitutionality of a federal
law that permits the post office, upon a householder’s request,
to order a mailer to stop all future mailings to that addressee
[Rowan v. United States Post Office, 397 U.S. 728 (1970)].
(3) Speech-related activities at non-public forums, such as military
bases, jails, government workplaces, and mailboxes, can be regu-
lated by reasonable time, place, and manner regulations. The test
used by the Court requires a government regulation to be:
(a) viewpoint-neutral; and
(b) reasonably related to a legitimate interest.
(4) A state may prohibit demonstrations on jailhouse grounds
[Adderley v. Florida, 385 U.S. 39 (1966)].
(5) Military bases may be closed to political speeches and distribu-
tion of leaflets [Greer v. Spock, 424 U.S. 828 (1976)].
(6) A city may sell space for commercial advertising on city buses
but refuse to sell such space for political advertising [Lehman v.
Shaker Heights, 418 U.S. 298 (1974)].

157
OUTLINE

(7) A public school may not deny use of its facilities to religious
groups if other public and private groups are allowed similar
access [Lamb’s Chapel v. Center Moriches Union Free School
District, 508 U.S. 672 (1992)].
(8) The owner of a private shopping center is not required by the
Constitution to allow access for purposes of picketing and/or leaf-
leting [Hudgens v. NLRB, 424 U.S. 507 (1976)].
(a) However, a state’s constitution may be interpreted to protect
such expressive activity [Pruneyard Shopping Center v.
Robins, 447 U.S. 74 (1980)].
4. Freedom of Association
a. There is a close “nexus” between the freedoms of speech and associa-
tion. The Supreme Court has acknowledged that “state action which
may curtail (or have the effect of curtailing) the freedom to associate is
subject to the closest scrutiny.”
(1) Under the freedom to associate, the Court has struck down laws
that prevented the NAACP from assisting individuals and that
prevented a labor union from assisting its members in retaining
lawyers [NAACP v. Button, 371 U.S. 415 (1963); Brotherhood of
Railroad Trainmen v. Virginia, 377 U.S. 1 (1964)].
(2) It is unconstitutional for the government to order the Boy Scouts
of America to allow homosexuals to participate in the organiza-
tion. The organization’s view that homosexual conduct was incon-
sistent with the values it sought to instill in youth members fell
within its First Amendment right of expressive association [Boy
Scouts of America v. Dale, 530 U.S. 640 (2001)].
(a) The Supreme Court refused to extend First Amendment
protection to a high school student’s lewd and offensive
campaign speech delivered at a school assembly [Bethel
School District No. 403 v. Fraser, 478 U.S. 675 (1986)].
5. Public Employment
a. In general, an individual cannot be denied public employment based
upon membership in a political organization unless the position is a
high-level policy-making position [Keyishian v. Board of Regents,
385 U.S. 589 (1967)].
(1) The Court invalidated political patronage dismissals by the Democratic
sheriff of Cook County [Elrod v. Burns, 427 U.S. 347 (1976)].
(2) The Court held that party affiliation is not an appropriate
requirement for the position of public defender [Brand v.
Finkel, 445 U.S. 507 (1980)].
b. An individual may be deprived of public employment for political asso-
ciation if [Scales v. United States, 367 U.S. 203 (1961)]:
(1) he is an active member of a subversive organization;
(2) such membership is with knowledge of the illegal aims of the
organization; and

158
CONSTITUTIONAL LAW

(3) he has a specific intent to further those illegal ends (e.g., violent
overthrow of the government).
EXAMPLE: A provision of the Subversive Activities Control Act
was held unconstitutionally overbroad by denying members
of the Communist Party “employment in any defense facility”
[United States v. Robel, 389 U.S. 258 (1967)].
c. In the past, the Court has chosen to deal with loyalty oath
qualifications under the “vagueness” and “overbreadth” doctrines.
EXAMPLE: An Arkansas statute requiring teachers to file an affidavit
listing “every organization to which they have belonged or regularly
contributed within the preceding five years” was invalidated as
overbroad [Shelton v. Tucker, 364 U.S. 479 (1960)].

EXAMPLE: Similarly, the Supreme Court invalidated a Florida


statute requiring public employees to swear, “I have not and will not
lend my aid, support, advice, counsel or influence to the Communist
Party” as vague and ambiguous [Cramp v. Board of Public
Instruction, 368 U.S. 278 (1961)].
(1) On the other hand, an oath that public employees will “support the
Constitution of the United States and will oppose the overthrow of
the government of the United States by force, violence, or by any
illegal or unconstitutional means” was held valid in part; the portion
of the oath requiring the employees to swear that they would
support the Constitution was held constitutionally valid. However,
the part of the oath requiring them to swear that they do not believe
in the violent overthrow of government was held invalid as providing
for the employee’s dismissal without inquiry or hearing as required
by due process [Connell v. Higginbotham, 403 U.S. 207 (1971)].
6. Prior Restraint
a. As a general rule, the government cannot suppress or restrain
speech in advance of its publication or utterance.
b. There is a strong presumption against the constitutional validity of
any system of prior restraint of expression.
(1) The Supreme Court refused to permit the government to enjoin
the publication of the Pentagon Papers [New York Times v.
United States, 403 U.S. 713 (1971)].
(2) Confiscation by the post office of mailed materials determined
by the postmaster general to be “obscene” was held invalid
[Bantam Books, Inc. v. Sullivan, 372 U.S. 58 (1963)].
(3) A restraining order issued by a trial judge to limit media reporting
of a criminal trial was held unconstitutional [Nebraska Press
Association v. Stuart, 427 U.S. 539 (1976)].
c. There are some exceptional cases in which prior restraints are
allowed, including:

159
OUTLINE

(1) a government agency can require prepublication review of writ-


ings related to employment of past or present employees where
such a review is necessary to protect national security [Snepp v.
United States, 444 U.S. 507 (1980)];
(2) classified military information; and
(3) any case involving a search and seizure is governed by the
Fourth Amendment.
(a) Thus, any “large scale” seizure of allegedly obscene mate-
rials must be preceded by a full adversary hearing, and a
judicial determination of obscenity [Quantity of Books v.
Kansas, 378 U.S. 205 (1964)].
(b) However, the seizure of a single book or film to preserve it as
evidence in a criminal proceeding need only be made pursuant
to a warrant based on a determination of probable cause by a
neutral magistrate [Heller v. New York, 413 U.S. 483 (1973)].
d. The Supreme Court required trial judges to consider three factors in
determining if a restraining order against pretrial publicity is appropriate:
(1) the nature and extent of the pretrial publicity;
(2) the availability of other measures to mitigate the effects of
pretrial publicity; and
(3) the likely effectiveness of the restraining order.
e. Another form of prior restraint is the censorship or licensing of
motion pictures prior to their exhibition. The Court has held that
statutes requiring films to be submitted to a Board of Censors before
showing them are constitutional if the following requirements are met
[Freedman v. Maryland, 380 U.S. 51 (1965)]:
(1) the standards for the denial of a license are narrowly drawn
and reasonable;
(2) when a license is denied, the censor promptly seeks an
injunction [Teitel Films v. Cusack, 390 U.S. 139 (1968)];
(3) the burden of proving that the material is “obscene,” or
otherwise unprotected, is on the censor; and
(4) a prompt judicial determination is provided.
f. A method the government frequently uses for regulating the time,
place, and manner of speech is to require a license or permit for such
activities as a parade, demonstration, or rally.
(1) Such a licensing statute is valid provided that it:
(a) is content-neutral as applied (i.e., for protected speech,
the message of the petitioners cannot be considered in
granting or denying the permit); and
(b) does not give licensing officials “unfettered discretion” to
determine who may receive a permit.
EXAMPLE: A parade permit that required applicants to pay
up to $1,000 per day to cover anticipated costs of police

160
CONSTITUTIONAL LAW

security was held void on its face as a prior restraint. Also,


as applied to a white supremacy group, the ordinance was
not content-neutral, because unpopular speech is met with
greater hostility, thus requiring more police protection [For-
syth County v. Nationalist Movement, 505 U.S. 123 (1992)].
(2) Where a statute is facially void (i.e., it gives the licensing
officials unrestricted discretion), a speaker need not even apply
for a permit. In this case, one may exercise his First Amendment
rights on the public property without a permit [Staub v. Baxley,
355 U.S. 313 (1958)].
(3) On the other hand, if a statute is valid on its face (i.e., it contains
narrowly defined standards as to time, place, manner, and
duration), then the applicant must seek a permit. If the permit is
denied, even arbitrarily, the applicant must appeal the adverse
ruling to the proper administrative or judicial body [Poulos v.
New Hampshire, 345 U.S. 395 (1953)].
g. As a general rule, if one is enjoined from speaking, he must either obey
the injunction or appeal it, unless the licensing statute is facially void
or the timing is such that an appeal would effectively frustrate the exer-
cise of his rights [Walker v. City of Birmingham, 388 U.S. 307 (1967)].
7. Overbreadth
a. When a state has the power to regulate an area dealing with free speech, it
must not do so “by means which sweep unnecessarily broadly and thereby
invade the area of protected freedoms” [NAACP v. Alabama, 377 U.S. 288
(1964)]. Thus, the wording of a statute must be narrow and specific, and
not overly broad so as to have a “chilling effect” upon protected speech.
(1) The Supreme Court has held that “the overbreadth of a statute
must not only be real, but substantial as well” [Broadrick v.
Oklahoma, 413 U.S. 601 (1973)].
(2) The Supreme Court further explained that “there must be a realistic
danger that the statute itself will significantly compromise recog-
nized First Amendment protections of parties not before the Court
for it to be facially challenged on overbreadth grounds” [Members
of City Council v. Taxpayers for Vincent, 466 U.S. 789 (1984)].
EXAMPLE: An ordinance making it unlawful “to curse or revile or
to use obscene or opprobrious language toward or with reference”
to a police officer performing his duties was held invalid because
the ordinance effectively punished all vulgar and offensive speech,
even though some of this speech may be protected by the First
Amendment [Lewis v. City of New Orleans, 415 U.S. 130 (1974)].
EXAMPLE: An ordinance prohibiting speech that “stirs the public
to anger, invites dispute, brings about a condition of unrest, or
creates a disturbance” was held to be overbroad and unconstitu-
tional [Terminiello v. City of Chicago, 337 U.S. 1 (1949)].

161
OUTLINE

8. Vagueness
a. The vagueness doctrine is closely related to the overbreadth
doctrine. In NAACP v. Button [371 U.S. 415 (1963)], the Court
admonished that governmental regulations must be drawn “with
narrow specificity.” The following statutes have been ruled “void for
vagueness” under due process inquiries:
(1) a statute making it a crime to “publicly mutilate, trample upon,
deface or treat contemptuously the flag of the United States”
[Smith v. Goguen, 415 U.S. 566 (1974)];
(2) a municipal vagrancy ordinance defining vagrants to include
“rogues and vagabonds,…lewd, wanton, and lascivious
persons,…[and] persons wandering or straying around
from place to place without any lawful purpose or objective”
[Papachristou v. City of Jacksonville, 405 U.S. 156 (1972)]; and
(3) a city ordinance that defined loitering as “to remain in any
one place with no apparent purpose” and gave police officers
absolute discretion to issue dispersal orders to groups of two
or more persons seen loitering in a public place if the officer
reasonably believed that one of them was a criminal street gang
member, and which made it a criminal offense to disobey such
order [City of Chicago v. Morales, 527 U.S. 41 (1999)].
9. Press
a. The press has no greater freedom to speak than any ordinary
member of the general public does. Also, the press has no special
right of access to government information.
(1) The First Amendment guarantees both the public and the press
a right to attend criminal trials. However, this right is not absolute
and may be outweighed where the judge finds an “overriding”
interest that cannot be accommodated by less restrictive means
[Richmond Newspapers v. Virginia, 448 U.S. 555 (1980)].
(2) A gag order—a pretrial order prohibiting the press from
publishing certain types of information—will almost never
be held constitutional because the trial judge has other
alternatives at his disposal (e.g., a change of venue,
postponement of the trial, careful voir dire, or restricting
statements of the lawyers, police, and witnesses) [Nebraska
Press Association v. Stuart, 427 U.S. 539 (1976)].
(3) A newsperson has no First Amendment right to refuse to testify
before a grand jury [Branzburg v. Hayes, 408 U.S. 665 (1972)].
(4) In general, radio and television broadcasting can be more closely
regulated than the press, due to the limited number of airwaves
available. A radio broadcast of “patently offensive sexual and
excretory speech” (even if not “obscene” under the Miller test) can
be sanctioned to protect the privacy interests of children likely to
be listening [FCC v. Pacifica Foundation, 438 U.S. 726 (1978)].

162
CONSTITUTIONAL LAW

(5) The Court has held that cable television receives First
Amendment protection somewhere between that of broadcast
television and newspapers.
(6) Where cable television operators are subjected to content-
neutral regulations, intermediate scrutiny is applied [Turner
Broadcasting System, Inc. v. FCC, 512 U.S. 622 (1994)].
(7) On the other hand, where content-specific regulation is imposed,
the Court has yet to select a standard of review. However, the
Court allows cable television operators the right to ban “indecent”
programming that describes sexual activities in a patently
offensive manner when in programming on channels that are
leased outright to unaffiliated third parties (but not to public
access channels) [Denver Area Educational Telecommunications
Consortium, Inc. v. FCC, 518 U.S. 727 (1996)].
(8) The Supreme Court held that where a radio commentator played
a tape on his talk show that he had legally obtained, but which had
been recorded by another person in violation of the law, such a
publication was protected under the First Amendment. The Court
reasoned that a stranger’s illegal conduct does not suffice to
remove the First Amendment shield from speech about a matter
of public concern [Bartnicki v. Vopper, 535 U.S. 514 (2001)].
(9) As a general rule, any prior restraint of publication will be
deemed illegal. This does not mean that, after publication, there
may not be civil or criminal consequences from the publication
itself. Consider the timing of the action in question and
determine if it takes place before or after publication.
10. Bar Admission
a. The state is permitted, under the Due Process Clause, to inquire into
the qualifications and fitness of candidates for admission to the bar.
(1) A candidate cannot be denied admission for past membership
in the Communist Party [Schware v. New Mexico Board of Bar
Examiners, 353 U.S. 232 (1957)].
(2) However, the state can refuse bar membership to an applicant
who refuses to answer questions (e.g., regarding past
Communist Party membership) if his refusal obstructs the bar
examiner’s investigation of his qualifications [Konigsberg v.
State Bar of California, 366 U.S. 36 (1961)].
(3) Reaffirming the validity of Konigsberg, the Court held that a state
can inquire into knowing membership in subversive organizations
in screening its applicants for the bar. Thus, the First Amendment
does not extend unlimited protection to a bar applicant who
refuses to disclose his political affiliations [Law Students Civil
Rights Research Council, Inc. v. Wadmond, 401 U.S. 154 (1971)].
(4) A residency requirement by the New Hampshire state bar
requiring a Vermont resident to establish a home address in

163
OUTLINE

New Hampshire prior to being sworn in was held to violate of


the Privileges and Immunities Clause of Article IV, Section 2
[Supreme Court of New Hampshire v. Piper, 470 U.S. 274 (1985)].

164
Contracts
TABLE OF CONTENTS

I. SOURCES OF CONTRACT LAW

In General....................................................................................................................................168
The Uniform Commercial Code....................................................................................................168
The Common Law of Contracts...................................................................................................169
Basic Definitions and Concepts of Contract Law.........................................................................170

II. CONTRACT FORMATION: OFFER AND ACCEPTANCE

In General....................................................................................................................................172
The Offer......................................................................................................................................172
Acceptance under Common Law.................................................................................................182
Acceptance and the UCC.............................................................................................................185

III. CONSIDERATION AND PROMISSORY ESTOPPEL

Consideration and Bargain...........................................................................................................192

IV. STATUTE OF FRAUDS

In General....................................................................................................................................201
The Statute of Frauds in Operation..............................................................................................201
Evaluating Statute of Frauds Issues............................................................................................201
Analyzing Problems under the Statute of Frauds.........................................................................202
Analyzing Problems under the UCC Statute of Frauds................................................................208
Effect of Satisfying the Statute of Frauds.....................................................................................215
No Mutuality of Obligation under the Statute of Frauds...............................................................215
Limitations on the Statute of Frauds............................................................................................216

V. GAP-FILLERS, INTERPRETATION, AND THE PAROL EVIDENCE RULE

In General....................................................................................................................................217
Filling in the Gaps with Default Provisions...................................................................................217
Interpreting Ambiguous Language...............................................................................................221
Trade Usage, Course of Dealing, and Course of Performance....................................................223
The Parol Evidence Rule.............................................................................................................224

VI. PERFORMANCE, MODIFICATION, AND EXCUSE

Obligations under the UCC..........................................................................................................231


Modification..................................................................................................................................234
Excusing Performance Due to Faulty Assumptions.....................................................................236
Excusing Performance by Agreement of the Parties....................................................................243
Conditions....................................................................................................................................245

166
VII. DEFENSES

Incapacity.....................................................................................................................................256
Misrepresentation.........................................................................................................................259
Duress..........................................................................................................................................264
Undue Influence...........................................................................................................................267
Unconscionability.........................................................................................................................268
Public Policy.................................................................................................................................270

VIII. REMEDIES

Monetary Remedies for Breach of Contract.................................................................................274


Equitable Remedies.....................................................................................................................286
Other Possible Remedies............................................................................................................290

IX. THIRD-PARTY BENEFICIARIES

In General....................................................................................................................................293
Classification of the Third-Party Beneficiary.................................................................................293
Rights of Parties to Enforce the Contract.....................................................................................295

X. ASSIGNMENT OF RIGHTS AND DELEGATION OF DUTIES

Assignment of Rights...................................................................................................................298
Delegation of Duties.....................................................................................................................303

167
OUTLINE

I. SOURCES OF CONTRACT LAW

A. In General
1. There are two principal sources of contract law: Article 2 of the Uniform
Commercial Code (“UCC”) and state common law.
2. In deciding which source of contract law applies, first look for the subject matter
covered by the UCC. If the UCC does not apply, then common law will apply.

B. The Uniform Commercial Code


1. The UCC governs “sales of goods.” If the UCC would apply to a contract,
then UCC provisions will trump any contrary common law rules.
a. Sales are any transactions in which the seller transfers title of
goods to the buyer.
(1) Leases, bailments, and any other types of transactions that
involve goods are not considered sales.
b. Goods are broadly defined under the UCC to mean any “movable item.”
(1) Examples of goods are ball bearings, Hummers, processed foods and
produce, the Mona Lisa, clothing, carpeting, furniture, and everything
on the shelves of Wal-Mart. The UCC also specifically defines the
following as goods: growing crops, unborn animals, and “identified
things attached to realty,” such as uncut timber or tobacco.
(2) Intangibles (e.g., the “goodwill” of a business), currency, stock,
bonds and other securities, the assignment of a legal claim, real
property, services (e.g., construction and repair projects, land-
scaping, dry cleaning) are not goods.
c. Under UCC Section 2-204, a contract for sale of goods may be made
in any manner sufficient to show agreement, including conduct by both
parties which recognizes the existence of such a contract.
d. In hybrid cases, which involve both the sale of goods and a services contract,
the question arises as to which source of contract law should apply.
(1) The majority rule is that the appropriate source is determined by
the predominant purpose of the transaction.
(a) To determine the predominant purpose of a contract, the
following factors are used:
1) the language of the contract;
2) the nature of the supplier’s business; and
3) the relative values of the good versus the service.
(2) The minority rule involves partitioning the contract, applying the
UCC to the sale of the goods portion of the contract, and then
applying common law to the services portion of the contract.
EXAMPLE: Both the sale and installation of an audio system
for an automobile and the sale of a car with a long-term service
agreement involve goods and services.

168
CONTRACTS

(3) Services Contract with Incidental Goods


(a) A rendition of services with goods incidentally involved,
such as a contract with an artist to paint a portrait, would be
governed by common law.
(4) Sale of Goods with Incidental Services
(a) A sale of goods with services being incidentally involved,
such as the sale and installation of a water heater, would be
governed by the UCC.
e. Computer Software and Other Electronic Information
(1) The majority of courts treat computer software and other elec-
tronic information as goods governed by the UCC.
(2) In Maryland and Virginia, the UCC has been displaced by the
Uniform Computer Information Transactions Act for transactions
involving electronic information.

EXAM TIP The UCC governs the sale of all goods, not just the sale of goods priced at
$500 or more. The UCC’s Statute of Frauds [§ 2-201] applies only to the sale
of goods for a price of $500 or more, requiring that a writing be signed by the
party against whom enforcement of the contract is sought, but the rest of the
UCC applies to the sale of all goods.

EXAM TIP The UCC governs all sales of goods, not just sales of goods involving mer-
chants. The UCC has a number of provisions that create special rules for
merchants (e.g., Section 2-205—only a merchant can make a firm offer; Sec-
tions 2-314 and 2-315—the merchant warranty provisions). Although the sale of
goods by and between consumers is beyond the scope of the special merchant
provisions, such sales are governed by all the other provisions of the UCC.

C. The Common Law of Contracts


1. Contracts that do not involve the sale of goods are covered by the common
law of contracts. Frequently encountered examples are service contracts,
contracts involving real property, and assignments of legal claims.
2. Interplay Between Sources
a. Common Law Filling in the Gaps of the UCC
(1) In cases involving the sale of goods, the UCC supplies the
primary law and displaces contrary common law rules. However,
where the common law rules are not displaced, those rules will
supplement the UCC and fill in its “gaps.”
EXAMPLE: An example of a UCC provision displacing a common
law rule would be Section 2-205, the firm offer rule. The firm offer
rule eliminates the common law requirement of consideration by
allowing a merchant to make a firm offer that will remain open by
requiring a signed writing to that effect.

169
OUTLINE

EXAMPLE: Another example of UCC provisions displacing the


common law rules would be Section 2-207, the battle of the forms
provision which overrules the common law’s mirror image rule,
and Section 2-209, the rule governing midterm modifications
which eliminates the common law preexisting duty rule.
EXAMPLE: Some examples of common law gap-filling include
the common law doctrines of fraud, duress, and incapacity of
minors and/or the mentally infirm. These common law rules apply
even where the sale is otherwise governed by the UCC.
b. UCC by Analogy
(1) For cases that do not involve the sale of goods, the UCC does
not apply. However, courts will frequently use its provisions “by
analogy” in developing common law.

D. Basic Definitions and Concepts of Contract Law


1. Contractual Obligations
a. There are three general categories of contractual obligations:
(1) Express contractual obligations are found where the parties
make oral or written “expressions” of their commitments.
EXAMPLE: A written contract, an exchange of e-mails, or a face-
to-face oral agreement.
(2) Implied-in-fact contractual obligations are consensual agree-
ments that fail to express the agreement of the parties in its entirety.
EXAMPLE: The most common implied-in-fact contractual
obligation is the duty to pay the “reasonable value” of services
rendered or of goods delivered where price is not discussed by
the parties in advance.
EXAMPLE: Homeowner hires a plumber to fix a leak, but be-
cause of the urgency of the service, the parties do not discuss
the price of the work performed. Upon completion of the work,
Homeowner has an implied-in-fact obligation to pay the plumber
the reasonable value of the services rendered.
(3) An implied-in-law contractual obligation arises where there
is an equitable imposition of a would-be contract. It is an equi-
table remedy available to prevent unjust enrichment, and arises
in situations where one party bestows a benefit on the other. An
implied-in-law obligation is also commonly referred to as a quasi-
contractual obligation or a restitutionary obligation.
(a) In many cases, the courts will conclude that the bestowal of
the benefits unjustly enriched the recipient and will accordingly
impose an implied-in-law obligation on the part of the recipient
to restore the value of the benefits conferred to the other party.

170
CONTRACTS

EXAMPLE: Emergency services are a typical quasi-con-


tract area, i.e., a surgeon who performs emergency surgery
on an unconscious patient creates an implied-in-law obliga-
tion to the patient.
EXAMPLE: Another common situation is where contractual
benefits have been conferred by mistake, i.e., a merchant
who mistakenly delivers goods to the wrong party may cre-
ate an implied-in-law obligation on the benefiting party.
EXAMPLE: A third frequent quasi-contract situation is where
contractual benefits have been conferred via an unenforceable
contract, i.e., a would-be buyer of real property who makes im-
provements on the land before closing and whose oral contract
turns out to be unenforceable under the Statute of Frauds may
create an implied-in-law obligation on the seller.
b. Quantum meruit is a means of enforcing both implied-in-fact and
implied-in-law contractual obligations.
EXAMPLE: The plumber who was hired to fix Homeowner’s leak
without first discussing the price may enforce Homeowner’s implied-in-
fact obligation to pay the reasonable value of the goods delivered via a
quantum meruit action.
EXAMPLE: A supplier mistakenly delivers goods intended for his buyer
to another merchant, who in turn sells them to her customers. The sup-
plier may seek to enforce the merchant’s implied-in-law obligation to pay
a reasonable value of the goods delivered via a quantum meruit action.
c. Signed Writing Requirements
(1) A number of rules in contract law require a “signed writing” in
order to create an enforceable legal obligation.
EXAMPLE: The Statute of Frauds under both the common law
and the UCC bars enforcement of certain contracts unless the
party against whom enforcement is sought has signed a writing
evidencing the underlying agreement.
EXAMPLE: The UCC requires that firm offers must be made in
writing and signed by the offeror in order to be enforceable.
(2) Electronic signatures are legally effective in the vast majority
of U.S. jurisdictions under the terms of the Uniform Electronic
Transactions Act.
(3) The majority of courts have held that e-mail and the like are suffi-
cient to satisfy the writing requirements of contract law.

171
OUTLINE

II. CONTRACT FORMATION: OFFER AND ACCEPTANCE

A. In General
1. In many cases, parties to a would-be contract negotiate in advance of
reaching agreement, engaging in a communications “volley” where inqui-
ries, proposals, and counterproposals are exchanged. The law of “offer
and acceptance” provides the rules for determining the point at which the
parties have a legally enforceable contract.

B. The Offer
1. In order to constitute an offer, a party’s communication must meet
two requirements:
a. an outward manifestation; and
(1) The manifestation can be oral, written, or made via conduct.
EXAMPLE: A newspaper seller stands on the corner with a stack of
newspapers and hands them to people who pay the posted price.
(2) Inward thoughts or subjective intentions are irrelevant unless they
are reasonably apparent to the other party.
EXAMPLE: A seemingly serious offer to sell real property made
in secret jest is nonetheless an offer.
EXAMPLE: A proposal to sell at a price that a reasonable per-
son would regard as “too good to be true” (e.g., “new HDTVs for
$8.99”) does not constitute an offer.
b. the signal that acceptance will conclude the deal.
(1) An offer must signal to the would-be offeree that the latter’s
agreement will conclude the deal. The key inquiry is whether
the party making the communication expressed a willingness to
commit without further assent.
EXAMPLE: “I will sell you my car if you’ll pay me $2,000 cash.”
This is an offer because it expresses a willingness to conclude
the deal if the other party pays the required $2,000.
EXAMPLE: “Yes, I’d be willing to sell you my car, but what are
you willing to pay for it?” This is not an offer because the commu-
nicating party is obviously reserving the right to decide whether
she likes the price suggested by the other party. She is thus with-
holding the privilege of further assent, or she is reserving the right
to assent (or not assent) to the other party’s proposal.
(2) Communications that withhold the privilege of further assent fall
short of constituting an offer.
(a) A preliminary negotiation is a generic term that applies to
the give-and-take that occurs during bargaining.

172
CONTRACTS

EXAMPLE: “We’re willing to consider that price if you can be


more flexible on the warranty and the payment terms.”
(b) An invitation for an offer is an advanced stage of prelimi-
nary negotiations, where the communicating party is closing
in on a deal but wants the other party to commit first.
EXAMPLE: “Okay, we’re in agreement on everything but
price now. What’s the best price you can give me?”
2. Multiple Offerees
a. When a question involves a party’s communication proposing a deal to
two or more persons at the same time, a multiple offeree issue arises.
b. There are three situations where this can occur:
(1) commercial advertisements, where possible offers occur in
catalogs, price lists, and circulars;
(a) Under the American advertising rule, advertisements
addressed to multiple recipients are generally treated as
invitations for offers rather than offers. The reason is that
responses from recipients of the advertisement may exceed
the available supply of goods or services; allocating the
goods or services among the responding parties will neces-
sitate further assent from the advertising party.
EXCEPTION: Language in the advertisement such as
“first-come, first-served,” or “first 10 customers only,” iden-
tifying the means by which the goods or services will be
allocated in the event of an excess of demand, eliminates
the need for further assent from the advertiser and, thus,
constitutes an offer.
(2) reward offers, which are general offers that name a price for
a service but do not specify an offeree, such as “Will pay $100
reward to the person who finds my lost dog”; and
(a) Generally, reward offers are treated as offers because they
are considered communications that promise a bounty in
exchange for the performance of a specified task. There are
two types of reward offers:
1) Self-limiting rewards are reward offers that indicate
the task to be rewarded can possibly be performed only
once. They are typically treated as offers that can only
be accepted by the first person to render the requested
performance in accordance with the terms of the offer.
EXAMPLE: “Will pay $100 for finding my lost dog.”
2) Open-field rewards are reward offers that indicate that
the task to be performed can potentially be performed
by multiple parties.

173
OUTLINE

EXAMPLE: The Carbolic Smoke Ball case, where the


purveyor of a medical preparation promised to pay $100
to anyone who used the preparation as directed and,
nonetheless, came down with the flu.
a) Unless otherwise specified in the offer, open-field
rewards can be accepted by any and all persons
who render the requested performance in accor-
dance with the terms of the reward.
EXAMPLE: “Free Joe Six-Pack Action Figure!
Send us proof of purchase labels from six cases
of your favorite Anheuser-Busch product together
with a self-addressed, stamped envelope. Offer
good while supplies last!” Because of the limitation
stated in the offer, the power of acceptance exists
only so long as supplies last.
(3) auctions, which are situations where an item is sold to the
highest bidder.
(a) The general rule is that the auctioneer is inviting offers, and
the responsive bids are the offers.
EXCEPTION: Where the auction is “without reserve,” then
the auctioneer is making an offer to sell to the highest bidder.
3. The Legal Effect of an Offer
a. An offer creates the power of acceptance in an eligible offeree. This gives
the offeree the power to create a contract simply by accepting the offer.
b. When an eligible offeree exercises the power of acceptance before that
power is terminated, a legally binding contract is formed between the parties.
c. Generally, there are four ways to terminate the power of acceptance:
lapse of time, the death or incapacity of either party, revocation by the
offeror, and rejection by the offeree.
(1) Lapse of Time
(a) An offeree’s power of acceptance terminates at the time
stated in the offer. If the offer does not specify the time of
termination, then the power of acceptance will terminate
after a reasonable time. The reasonable time determination
is based on the following factors:
1) subject matter and market conditions; and
a) What is considered a reasonable time will vary
depending on the subject matter and the relevant
market conditions.
EXAMPLE: A reasonable time would be shorter for an
offer to sell stock in a wildly fluctuating market than for
an offer to sell real estate in a dormant market.

174
CONTRACTS

2) the degree of urgency communicated by the means


of transmission.
EXAMPLE: A reasonable time period would be shorter for
an offer sent via e-mail than for an offer sent via postal mail.
(b) Face-to-Face Conversation Rule
1) An offer made by one person to another in a face-to-
face conversation is ordinarily deemed to remain open
only until the close of the conversation.
(2) Death or Incapacity of Either Party
(a) The supervening death of either the offeror or offeree will
terminate the power of acceptance with respect to the offer.
(b) The supervening incapacity of either the offeror or the
offeree, as evidenced by adjudication or the appointment of
a guardian, will terminate the power of acceptance.
(3) Revocation by Offeror
(a) Under the rule of free revocability in American contract law,
the offeror is free to revoke an outstanding offer, terminating
the offeree’s power of acceptance, at any time and for any
reason, so long as the revocation:
1) occurs prior to acceptance; and
2) is effectively communicated.
(b) There are two methods of communicating a revocation.
1) Direct Revocation
a) Typically, revocation occurs and terminates the
offeree’s power of acceptance when the offeror
communicates directly with the offeree and advises
the latter that the offer has been revoked.
2) Indirect Revocation
a) In some cases, the offeree will learn of the offeror’s
intention to abandon the deal from a third-party
source, and this will also terminate the offeree’s
power of acceptance where two conditions are met:
i) the offeror has taken definite action inconsis-
tent with the intention to enter the proposed
contract (such as by selling offered real estate
to a third party); and
ii) the offeree acquires reliable information of the
offeror’s inconsistent action (such as learning
of the sale from a real estate broker).
(c) Revocation of an Offer Made to Multiple Offerees
1) Under the Second Restatement of Contracts, where
an offer is made by advertisement in a newspaper or
other general notification to the public or some segment

175
OUTLINE

thereof, the power of acceptance is terminated when the


notice of revocation is communicated by advertisement
or other general notification equivalent to that used for
the offer and no better means of notification is reason-
ably available (the functional equivalents rule).
2) In such a case, a particular offeree loses the power of
acceptance, even if he is utterly unaware of the revocation.
(d) Option Contracts and Firm Offers
1) An offeror is free to revoke an offer any time before
acceptance. Under the common law rule of Dickenson
v. Dodds (1876), the offeror can revoke even if he has
expressly promised the offeree that he would hold the
offer open. Under contemporary law, there are two ways
to prevent revocation of an offer: a common law option
contract or a firm offer under the UCC.
2) Three elements are required for an enforceable
option contract:
a) an offer;
b) a subsidiary promise to keep the offer open; and
i) A “sell-by” date in an offer is not necessarily a
promise to keep the offer open. It can also constitute
an express lapse provision, establishing a deadline
for when the power of acceptance will terminate.
EXAMPLE: “I hereby offer to sell you Black-
acre for $10,000. This offer expires in 30
days.” This language would not indicate an
option contract but instead simply an offer with
an express date of termination.
c) some valid mechanism for securing enforcement of
the subsidiary promise.
i) The most common way to secure enforcement
of the subsidiary promise to keep the offer
open is by giving consideration in return.
ii) Consideration can be in the form of a perfor-
mance (e.g., payment for the option) or a
promise of performance (e.g., where a buyer of
real estate promises his best efforts to obtain
financing in exchange for the option).
EXAMPLE: An offer stating, “I hereby offer to
sell you Blackacre for $10,000, and in consid-
eration for the $100 received, I hereby grant
you a 30-day option on the deal,” would meet
these three requirements as follows: the offer
is the proposal to sell Blackacre for $10,000;

176
CONTRACTS

the grant of a 30-day option on the deal; and


the payment of $100 to the offeror.
3) A minority of courts have held that revocation of an
offer can be prevented where the offer is in writing and
signed by the offeror, recites the purported consideration
for the promise to keep the offer open, and proposes an
exchange on fair terms within a reasonable time.
EXAMPLE: “I hereby offer to sell you Blackacre for
$10,000, and in consideration for the $100 received, I
hereby grant you a 30-day option on the deal” will create
an enforceable option contract under the minority rule,
even if the offeree never paid the $100.
4) Under promissory estoppel, courts will sometimes enforce a
subsidiary promise to keep an offer open where the offeree
has foreseeably and reasonably relied on the option, and
injustice can only be avoided by enforcing the promise.
5) Special Rule for Construction Contracts
a) A general contractor bidding on a construction project
solicits bids from different subcontractors, such
as plumbers, electricians, or glazers. The general
contractor must then use these bids in formulating his
own bid, but cannot as a practical matter accept those
bids unless and until the general contract is awarded.
b) The problem occurs when a subcontractor revokes
his bid (offer) after the general contractor has relied
on it to make his own bid on the primary contract,
and the general contractor is then awarded the
contract based on that bid.
c) The majority rule is that where a general contractor uses
a particular subcontractor’s bid to formulate his own,
an implied option contract is created via promissory
estoppel. This prevents the subcontractor from revoking
the bid despite the fact that the subcontractor hasn’t
promised to keep the bid and the general contractor
hasn’t provided any consideration to keep the bid open.
6) The UCC “Firm Offer” Rule
a) Under the UCC, a merchant can make a firm offer
(an irrevocable offer) to either buy or sell goods
without consideration so long as [§ 2-205]:
i) the offer is made by a merchant;
ii) the offer is made in a writing signed by the
merchant; and
iii) the offer expressly states by its terms that it
will be held open.

177
OUTLINE

b) Note that the UCC defines a merchant in terms of


his special knowledge or skill with respect to the
practices or goods involved in a transaction. It is
important to note that a person may be considered
a merchant even if he only has knowledge of the
goods, or knowledge of the practices.
c) A firm offer that meets all of the above require-
ments becomes irrevocable either for the period of
time stated in the firm offer or for a reasonable time
if no time is specified.
d) Under the UCC, the shelf life of a firm offer can
be no more than three months. Any firm offers
that state a time longer than three months will only
be irrevocable for the first three months. The offer
would then become revocable, but still enforceable.
e) A merchant selling or buying goods can establish an
irrevocable offer for longer than the three-month period
stipulated in the UCC by creating a valid common law
option contract. This means that the requirements of a
subsidiary promise to keep the offer open, as well as
some consideration in return, must also be present.
(4) Rejection by the Offeree
(a) The power of acceptance can also be terminated if the
offeree refuses to accept the offer. There are three ways that
rejection can be effected:
1) outright rejection;
a) The offeree’s power to accept an offer is terminated
by his rejection of the offer. The offeree has the power
to accept the offer, but the offeree cannot change his
mind and try to accept the offer once he has rejected it.
2) rejection via a counteroffer; and
a) A counteroffer made on the same subject matter
operates to simultaneously reject the initial offer.
b) Not all statements or questions about an offer are
considered counteroffers. An offeree may test the
waters by making a “mere inquiry” about the offeror’s
willingness to negotiate without creating a counter-
offer and terminating the power of acceptance.
EXAMPLE: “I am not willing to pay $10,000 for the
car, but I would happily buy your car for $9,000.”
This statement would rise to the level of a counter-
offer because the terms of the initial offer, to pur-
chase the car at $10,000, have been modified by
the offeree to $9,000. As a result, the offeree made

178
CONTRACTS

a counteroffer which at the same time rejected the


offeror’s initial offer.

EXAMPLE: “$10,000 isn’t out of the question, but it


is a little high, given the age of the car. Would you
be willing to consider a lower offer?” This language
would not rise to the level of a counteroffer because
the offeree is only asking if the offeror is willing to ne-
gotiate the terms, and not proposing a different deal.
3) rejection via nonconforming acceptance.
a) At common law, the mirror image rule requires that
acceptance must mirror the terms of the offer, and
any variation results in a counteroffer and rejection
of the initial offer. Despite the mirror image rule, a de
minimis variation in the terms of the acceptance may
likely constitute acceptance in some situations.
i) If a court concludes that the minor variation
was implicit in the offer, or merely a sugges-
tion for the addition of a new term, then the
response may constitute acceptance mirroring
the offer, creating a contract.
ii) If the purpose of raising the issue is to attempt
in bad faith to back out of the contract, a de
minimis variation in the terms of the accep-
tance may still constitute acceptance.
b) The UCC rejects the common law mirror image rule
and recognizes a binding contract despite the pres-
ence of a nonconforming acceptance in two sets
of circumstances: the shipment of nonconforming
goods and the battle of the forms. Each of those
circumstances is discussed below.
(b) Revival of the Offer
1) The maker is the master of the offer. Therefore, an
offeror has the power to revive an offer that the offeree
has rejected, and with it the offeree’s power of accep-
tance, and he can likewise revive an offer that has
lapsed; all he must do is communicate the revival to the
offeree. This can be accomplished by restating the offer
or giving the offeree more time to make a decision.
4. Offer and Acceptance under Unilateral Contracts
a. An offer can require acceptance by either a promise or a performance.
EXAMPLE: If A makes an offer to sell his car to B for $10,000, A could
request a promissory acceptance from B, which would require B to say
something to the effect of, “I accept your offer, and promise to pay you

179
OUTLINE

$10,000.” Alternatively, A could request a performance as acceptance


from B, which would require B to pay A the actual $10,000.
(1) An offer seeking a promissory acceptance is an offer to enter into
a bilateral contract. A promise is being exchanged for a promise.
Under a bilateral contract, the offeree can accept the offer by making
the requested promise. In this situation, both parties are bound by
the terms of the contract once the mutual promises are exchanged.
EXAMPLE: When A promises the car to B, B accepts by promis-
ing A the money.
(2) An offer seeking performance in return is an offer to enter into a
unilateral contract.
EXAMPLE: A promises B the car, and B can only accept the offer
by paying A the money directly.
(a) Under a unilateral contract:
1) the offeror is bound only when the offeree completes
performance in accordance with the terms of the offer; and
2) the offeree is never bound to perform because he has
never promised to perform. It is entirely up to him whether
to perform or not, which accounts for the description of
the contract being unilateral. However, he will not be
entitled to the benefits of the offeror’s promise unless and
until he renders the required performance.
(3) Where an offer does not specify whether it must be accepted
by a promise or a performance, the offeree is free to choose
the means of acceptance. However, the offeree must accept
via performance where the terms of the offer or the surrounding
circumstances make it clear that a performance is required.
EXAMPLE: A says to his old friend B, “I’ll sell you my car for $10,000.”
B may choose the form of acceptance in this case. B is free to accept
the car by either promising to pay A the money, or actually paying
A the money. In this case, B’s acceptance would dictate the type
of contract created, because the terms of A’s offer did not specify.
(a) In practice, true unilateral contracts, or those stemming
from a unilateral offer, are relatively rare and typically fall
into two categories.
1) A reward offer is a type of unilateral offer where the
offeror offers to pay a reward for the successful perfor-
mance of some act. Ordinarily, the act in question requires
some effort on the part of the offeree, and the latter’s ability
to perform it may be uncertain. The key factor is that the
offeror will not settle for a promise and is instead seeking
the actual performance before binding himself to the deal.

180
CONTRACTS

EXAMPLE: “I’ll pay you $100 if you find my lost dog”


or “We’ll pay $1,000,000 to the first person who proves
Fermat’s Last Theorem.”
2)Rather than a promise to sell a house, most real estate
brokerage agreements include a promise by the owner
to pay a fee in exchange for the actual sale of the prop-
erty, which would constitute performance.
(b) Revocation of the Offer in a Unilateral Contract
1) Under common law, the offeror was free to revoke the
unilateral offer up until the moment that the offeree actu-
ally completed performance. The modern majority rule,
which also appears in the Second Restatement, is that
once the offeree begins performance, an option contract
is created and the offeror may not revoke.
EXAMPLE: If A offers to pay B $100 to climb to the top
of the flagpole, then once B starts climbing the flagpole,
A cannot revoke her offer under the modern rule.
EXCEPTION: An offeror may revoke under the modern
rule where the offeree is engaged in “mere preparations” to
perform rather than the beginning of performance itself.
EXAMPLE: A offers B $100 if the latter can “ride my
horse Bronco” for a full minute, but A revokes when B is
saddling the horse. Revocation is permissible because
the required performance consisted of riding the horse,
and saddling was merely preparation.
2) Although the offeror cannot revoke once the offeree has
begun performance, acceptance of the offer is still effec-
tive only upon the completion of the performance. The
task must be completed in accordance with the terms of
the offer in order for the offeree to accept the offer and
bind the offeror to his performance obligation. In other
words, the offeree who gets close to the top of the flag-
pole but doesn’t actually reach it has not yet accepted
the offer, and the offeror need not pay him a penny.
3) The offeree of a unilateral contract is free to abandon
the performance at any time and even to not under-
take performance at all.

NOTE If an offer to enter a unilateral contract was mailed to the offeree, then the
offeror may revoke the offer even if the offeree has placed the acceptance
in the mail. The mailbox rule (discussed later in this outline) only applies to
bilateral contracts, and so dispatch could not constitute acceptance to create
a unilateral contract.

181
OUTLINE

C. Acceptance under Common Law


1. There are only two general requirements to constitute effective acceptance:
a. under the mirror image rule, the acceptance must mirror the terms of
the offer; and
b. the acceptance must be communicated to the offeror.
2. Communicating Acceptance
a. Once again, the maker is the master of the offer, and if the offer
stipulates a particular means of communicating acceptance, then the
offeree must utilize those means in order to make an effective accep-
tance. If the offer is silent as to the means of communication, then the
offeree is free to use any reasonable means of transmission.
(1) Unless the circumstances indicate otherwise, a means of trans-
mission is reasonable if it is:
(a) the means used by the offeror;
(b) the means customarily used in similar transactions; or
(c) a means of communication that is equivalent in expeditious-
ness and reliability to the means used by the offeror.
b. There are three instances where the requirement that acceptance be
communicated to the offeror may not apply:
(1) acceptance by silence;
(a) Generally, an offeree’s silence in response to an offer cannot
constitute acceptance except in the following three circumstances:
1) where the offeree takes the benefit of the offeror’s
services with a reasonable opportunity to reject them
and with reason to know the offeror’s intention;
2) where the offeror has given the offeree reason to
understand that acceptance may be communicated by
silence, in which case the offeree’s silence will operate
as acceptance if he intends as such; and
EXAMPLE: An insurance company notifies A that his
policy will be automatically renewed unless it hears
otherwise from him. A remains silent, intending to accept
the offer of renewal. Accordingly, his acceptance is af-
fected by his silence.
EXAMPLE: Same facts as above, except that A does not
intend to accept. In these circumstances, his silence will
not operate as acceptance. This is a rare instance in which
contractual liability depends on the subjective intentions of
the party rather than his outward manifestation.
3) where, because of previous dealings or other circum-
stances, it is reasonable that the offeree should notify
the offeror if he does not intend to accept, in which case
his silence will operate as acceptance.

182
CONTRACTS

EXAMPLE: A newspaper sends a written notice that it


will continue delivering the paper after the subscription
has expired unless it hears otherwise from A. When the
subscription has previously been renewed in this man-
ner without incident, A’s silence in the face of notice will
constitute acceptance.
(2) acceptance by performance; and
(a) The maker is the master of the offer and, accordingly, the
maker of an offer to enter a unilateral contract is free to make
communication of acceptance a part of the required perfor-
mance. But if he fails to do so, then acceptance is effective
upon the offeree’s completion of the requested performance
whether or not the offeree so notifies the offeror.
(3) acceptance by mail or other correspondence.
(a) Under the common law mailbox rule, which is the rule
in every American jurisdiction except the federal court of
claims, acceptance by mail is effective upon dispatch so
long as the acceptance is properly posted, with the correct
address and postage amount.
1) The mailbox rule applies only to acceptances and not to
any other communication between contracting parties.
Thus, offers, revocations, rejections, and counteroffers
are all effective only upon receipt by the other party.
2) Once the offeree dispatches his acceptance, he thereby
creates a binding contract.
a) The offeror may not revoke an offer once accep-
tance has been dispatched.
EXAMPLE: While B was considering A’s offer, A had
a change of heart and sent B a revocation. However,
B dispatched his acceptance before receiving A’s
revocation. The parties have a contract because A’s
revocation was not effective until receipt by B and
B’s acceptance was effective upon dispatch, which
was prior to his receipt of the would-be revocation.
b) Once the offeree dispatches his acceptance, the
parties have a binding contract and the offeree may
not withdraw his acceptance.
EXAMPLE: B dispatches her acceptance of A’s
offer and then has a change of heart, telephonically
notifying A of the fact while the acceptance is still in
transmission. The parties have a binding contract
because B’s acceptance was effective upon dis-
patch, and it is too late for either party to back out
because the contract has been formed.

183
OUTLINE

c) The parties are bound even if the acceptance is


lost in transmission and the offeror has no knowl-
edge of that acceptance.
3) The mailbox rule applies to acceptances sent by any
means of transmission which involves a foreseeable
delay between the time of dispatch by the offeree and
the time of receipt by the offeror.
a) This definition would include a private overnight
delivery service.
b) This would not include an instantaneous means of
communication, such as a facsimile.
c) Its application to e-mail is still undecided.
4) The mailbox rule is only a default rule, meaning that
it only applies if the offeror is silent on the question of
when acceptance will be effective. As always, the maker
is the master of the offer, and thus the offeror is free to
establish his own rules for effective acceptance.

EXAM TIP In a situation where the parties simultaneously dispatch identical offers, re-
member that offers are not effective until received, and the only effect an offer
has once received is to create the power of acceptance in the offeree. Ac-
cordingly, unless and until one of the parties receives and accepts one of the
offers passing in the night, there is no contract.

5) According to the Restatement, acceptance under an


option contract is effective upon receipt by the offeror,
but the case law on this question is limited. Some
authorities have rejected this approach and held that the
mailbox rule applies in this context as well.
6) Mailbox Rule Does Not Apply
a) Two situations under the mailbox rule involve
an offeree who dispatches two responses to an
offer: the first purporting to reject the offer, and the
second purporting to accept it. In these cases, the
mailbox rule does not apply.
b) The parties’ obligations will depend on which of the
offeree’s communications reaches the offeror first:
i) If the acceptance reaches the offeror first, then
the acceptance is effective upon receipt and
the parties are bound to the contract.
ii) If the rejection reaches the offeror first, then
the offeree’s power of acceptance is termi-
nated and the subsequently arriving accep-
tance becomes a counteroffer, which the
original offeror is free to accept or reject.

184
CONTRACTS

D. Acceptance and the UCC


1. Unless the contract language or circumstances unambiguously indicate
otherwise, acceptance may be made in any manner and by any medium
reasonable under the circumstances [§ 2-206(1)(a)].
2. The UCC rejects the common law mirror image rule and recognizes a binding
contract despite the presence of nonconforming acceptance in two situations:
the shipment of nonconforming goods and the so-called “battle of the forms.”
3. Seller’s Shipment of Conforming and Nonconforming Goods
a. Under the UCC, a seller can accept a buyer’s offer to purchase goods
for prompt or current shipment in one of three ways:
(1) a promise to ship goods in conformity with the terms of the offer,
such as an acknowledgment of order form sent to the buyer;
(2) a prompt or current shipment of the goods in conformity with the
terms of the offer; or
(3) the seller can also accept the buyer’s offer by shipping
nonconforming goods.
(a) In contrast to common law, under which the nonconforming
shipment would have constituted a counteroffer which the
buyer would have been free to accept or reject, the UCC
specifies that the shipment constitutes a valid acceptance
and creates a binding contract between the parties.
EXCEPTION: Under the UCC, the shipment of nonconforming
goods will not constitute acceptance if the seller notifies the
buyer that the shipment is offered only as an “accommoda-
tion” to the buyer. In such circumstances, the shipment consti-
tutes a counteroffer which the buyer is free to accept or reject.
EXAMPLE: Buyer orders 1,000 widgets from Seller for imme-
diate delivery. Seller responds by shipping 800 widgets with
an accompanying notice to Buyer explaining that Seller did not
have adequate inventory to ship 1,000 widgets and was thus
shipping 800 widgets as an accommodation to Buyer in light
of Buyer’s urgent need. The nonconforming shipment would
not constitute an acceptance of Buyer’s offer in this case.
(b) Absent accommodation, the seller’s shipment of noncon-
forming goods constitutes acceptance of the buyer’s
offer under the UCC. It also constitutes a breach of the
resulting contract under the perfect tender rule, which is
described in detail later.
4. The Battle of the Forms
a. UCC Section 2-207 is designed primarily to deal with difficulties
created by the application of the common law mirror image rule to
commercial settings, in which parties typically transact business via the
use of preprinted forms.

185
OUTLINE

b. A so-called “battle of the forms” may occur when a buyer places an


order (the offer) and the seller’s acceptance form contains terms which
differ from the buyer’s order or are not addressed in the order at all.
(1) Note that these rules apply regardless of which party (buyer or
seller) is the offeror or offeree.
c. Because the seller’s acceptance contains terms that vary from the
buyer’s offer, the seller’s response constitutes a nonconforming
acceptance. The UCC deals with two issues in a manner different
from the common law.
(1) First, in many circumstances the UCC rejects the common law
mirror image rule and treats a nonconforming acceptance as
a legally effective acceptance, thus binding both parties to the
contract despite the differing forms.
(2) Second, the UCC rejects the common law last shot doctrine
in determining the terms that will govern a contract formed via
mismatching forms.
d. Unless acceptance is expressly made conditional on assent to the
additional or different terms (a conditional acceptance), the noncon-
forming acceptance will operate as an effective acceptance of the offer,
forming a valid and enforceable contract. Once the contract is formed,
the next step is to determine the effect of the new or additional terms.
EXAMPLE: Buyer sends Seller a purchase order for 1,000 widgets at
the advertised price of $10 each. Seller sends Buyer an “Acknowledg-
ment of Order Form” that promises delivery of the widgets at the stated
price, but also contains boilerplate language negating warranties and
limiting remedies in the event of breach. Seller’s form will operate as
acceptance of Buyer’s offer and create a binding contract despite the
presence of terms that vary from Buyer’s purchase order.
(1) Dickered versus Boilerplate Terms
(a) Typically, the forms will be in agreement as to the dickered
terms—those specific to the transaction—but may vary
with respect to the boilerplate terms—standard terms that
appear on the parties’ respective forms regarding issues
such as arbitration and warranties.
e. Nonconforming Acceptance or Confirmation
(1) When a contract is formed by an offer followed by a noncon-
forming acceptance, the treatment of additional or different terms
depends on the identities of the parties.
(2) Transaction Involving a Consumer
(a) When at least one party to the transaction is not a
merchant, the additional or different terms are construed
as proposals for addition to the contract. Thus, they are not
part of the contract unless the offeror expressly agrees to
the additional terms [§ 2-207(2)].

186
CONTRACTS

(3) Transactions Where Both Parties Are Merchants


(a) In a transaction between merchants, there is a distinction
between additional terms and different terms.
1) A nonconforming acceptance contains additional
terms when its provisions address an issue or topic not
addressed in the original offer.
EXAMPLE: Buyer’s offer is silent on the question of
arbitration, and Seller’s acceptance purports to require
arbitration of all claims.
2) A nonconforming acceptance contains different terms
when the offer says one thing about a particular issue
and the would-be acceptance says something else.
EXAMPLE: Buyer’s offer requires arbitration of claims,
and Seller’s acceptance precludes it.
(b) Additional Terms
1) Additional terms become part of the contract
unless [§ 2-207(2)]:
a) the offer expressly limits acceptance to the terms
of the offer;
EXAMPLE: “This order expressly limits accep-
tance to the terms stated herein.”
b) the offeror objects to the additional terms within a
reasonable time after receiving notice of them; or
EXAMPLE: This could be accomplished through
language to the effect of: “We do not accept the
binding arbitration provision set forth in your Ac-
knowledgment of Order.”
c) the additional terms would materially alter the contract.
2) Materially Altering Terms
a) Under Section 2-207, terms that materially alter
the contract are those that would result in surprise
or hardship if incorporated without the express
awareness of the other party [§ 2-207, cmt. 4].
b) Examples of clauses that would materially alter
the contract include [Id.]:
i) a clause negating standard warranties, such
as that of merchantability or fitness for a
particular purpose in circumstances in which
either warranty normally attaches;
ii) a clause requiring that complaints be made
in a time materially shorter than customary or
reasonable; and

187
OUTLINE

iii) any clause that would vary in a significant way


an established usage of trade or a course of
past dealing between the parties.
EXAMPLE: Buyer sends Seller a purchase
order for 1,000 widgets at the advertised price
of $10 each. Seller sends Buyer an Acknowl-
edgment of Order that promises delivery of the
widgets at the stated price and includes boiler-
plate language that negates all warranties and
requires payment for the order to be complete
within 30 days of delivery, which is standard in
the widget trade. The negation of warranties will
not be considered part of the contract because
it would serve to materially alter the contract.
The 30-day payment deadline would be consid-
ered part of the contract because it would not
materially alter the terms of the contract.
(c) Different Terms
1) With respect to different terms, the majority of decisions
employ the knockout rule and omit both the offeror’s
original provision and the offeree’s differing provision
from the resulting contract.
EXAMPLE: Buyer’s purchase order contains a choice-
of-law provision stating that California law will govern
disputes arising from the transaction, and Seller’s order
acknowledgment states that New York law will govern.
Neither provision will be part of the parties’ contract; if
the parties desire a choice-of-law provision, they will
have to negotiate one from scratch.
2) A minority of decisions treat a different term appearing
in the acceptance as a mere proposal for alteration of
the contract, which the offeror is free to accept or reject.
EXAMPLE: On the same facts as immediately above,
the buyer’s California law provision would govern the
contract unless he expressly agreed to the seller’s
contrary provision.
(4) Written Confirmations
(a) Another situation governed by the battle of the forms rules
occurs when parties enter a contract—typically in real-time
via face-to-face or telephonic communications—and then
one or both of the parties follow up with a written confirma-
tion containing terms additional to or different from the terms
of the original deal.

188
CONTRACTS

(b) As is the case for a contract formed by an offer followed by a


nonconforming acceptance, the treatment of the additional or
different terms depends on the identity of the parties.
1) If at least one of the parties to the transaction is a
consumer, then any additional or different terms are
mere proposals for addition to the contract which the
receiving party is free to accept or reject.
2) If the transaction is between merchants:
a) any additional terms are automatically part of the
contract unless:
i) they would materially alter the contract; or
ii) the receiving party objects to them within a
reasonable time;
b) any terms in a confirmation that differ from the
terms of the prior agreement are proposals for
inclusion in the contract, which the receiving party
is free to accept or reject; and
c) if both merchants send written confirmations and
those confirmations contain conflicting terms, then
the knockout rule applies and neither party’s term
is in the contract.
i) All jurisdictions agree that the knockout rule
applies to written confirmations.
(c) Because the parties already have a binding contract in
this situation—formed prior to the sending of any written
confirmation(s)—a written confirmation cannot constitute
a conditional acceptance and, accordingly, the parties are
bound to the contract irrespective of any differences between
their prior agreement and the forms.
f. Conditional Acceptance
(1) Under the UCC, a definite and seasonable expression of accep-
tance will operate as an acceptance even though it states additional
or different terms, unless acceptance is expressly made conditional
on assent to the additional or different terms [§ 2-207(1)].
(2) However, if the acceptance is made expressly conditional on
assent to the additional or different terms, the nonconforming
acceptance will not be effective to form a contract.
(a) In this situation, no contract is formed until the offeror
expressly assents to the additional or different terms.
(3) What Constitutes a Conditional Acceptance?
(a) Under the majority rule, a would-be acceptance which tracks
the language of the exception as set out in Section 2-207
constitutes a conditional acceptance if the language in ques-
tion is clear and conspicuous.

189
OUTLINE

EXAMPLE: Buyer sends Seller a purchase order for 1,000


widgets at the advertised price of $10 each. Seller sends
Buyer an Acknowledgment of Order form that promises deliv-
ery of the widgets at the stated price, but also contains boiler-
plate language negating warranties and limiting remedies in
the event of breach. Seller’s form also contains the following
language in large bold font: “Our acceptance of your order is
expressly conditional on your assent to the additional or differ-
ent terms that appear in this acknowledgment.” Under the ma-
jority rule, Seller’s form constitutes a conditional acceptance of
Buyer’s order, and thus does not operate as a legally effective
acceptance. Thus, the exchange of forms does not create a
contract. Either party is free to back out of the deal prior to the
buyer’s assent to the additional or different terms.
(b) Under the minority rule, a would-be acceptance constitutes
a conditional acceptance only if it clearly communicates that
the offeree is unwilling to do business with the offeror unless
and until the offeror agrees to the offeree’s terms.
EXAMPLE: Buyer sends Seller a purchase order for 1,000
widgets at the advertised price of $10 each. Seller responds to
Buyer’s order with an e-mail saying, “We are in receipt of your
order for 1,000 widgets at the advertised price of $10 each, and
we will be happy to fill it. However, before we can do so, we
must require you to agree to the following terms.” The message
then sets forth terms negating warranties and limiting remedies
and concludes, “Upon receipt of your reply to this message
stating your agreement to these terms, we will immediately ship
your order.” Under the minority rule, this exchange of forms
would constitute conditional acceptance. By contrast, the pro-
viso tracking language in the form used in the previous example
would not constitute conditional acceptance.
g. Other Situations Governed by the Battle of the Forms Rules
(1) Contracts Formed by Conduct
(a) The UCC also provides that the parties’ conduct in recog-
nizing the existence of a contract is sufficient to establish a
contract, even though their writings do not otherwise estab-
lish a contract [§ 2-207(3)].
EXAMPLE: Buyer sends a purchase order to Seller, Seller
responds with a conditional acceptance containing a negation
of warranties, and the parties have no further communica-
tions. Seller nevertheless ships the ordered goods, and Buyer
accepts and pays for them. Although the parties’ writings do
not form a contract—because Seller sent a conditional accep-
tance—a binding contract is formed by their conduct.

190
CONTRACTS

(b) If a contract is established in this way, the terms of the contract


will be the terms on which the writings of the parties agree,
together with the implied terms or default rules established
by the UCC for warranties, remedies, party performance, and
other topics. (The UCC’s default rules are discussed at length
below in the section on contract interpretation.)
EXAMPLE: On the same facts as above, the parties’ con-
tract will include all the terms on which their respective forms
agree—typically, the dickered terms such as product descrip-
tion, price, and quantity. Seller’s negation of warranties would
not be included in the contract because it does not appear in
Buyer’s form. The UCC’s implied warranties would therefore
govern the transaction despite Seller’s attempt to negate them.
(2) Transactions without Preprinted Forms
(a) Although most situations governed by these rules will involve
transactions consummated via preprinted forms, the same
rules apply to any sale of goods in which an offer is followed
by a nonconforming acceptance (e.g., an exchange of trans-
action-specific e-mails or letters) or a real-time contract is
followed by confirmation(s) (e.g., a follow-up letter confirming
the details of a deal reached over lunch).
(3) Shrink-Wrap Contracts
(a) The authorities are divided on whether the rules of Section
2-207 apply to so-called shrink-wrap contracts (i.e., agree-
ments that consumers find “shrink-wrapped” to computers
and other high-tech goods once they open the boxes
containing them).
1) The courts that apply Section 2-207 typically conclude that
the boilerplate terms in such agreements are mere proposals
for additions to the contract, following the usual rules
governing transactions that are not “between merchants.”
2) Other courts have held that Section 2-207 does not apply
and that the consumer’s retention and use of the goods
constitutes an acceptance of the shrink-wrap terms.

191
OUTLINE

III. CONSIDERATION AND PROMISSORY ESTOPPEL

A. Consideration and Bargain


1. The general rule in American contract law is that a promise is unenforce-
able unless it is supported by consideration. The promise has some value
and must be exchanged for something else of value, such as a counter-
promise or performance given to the promisor by the promisee as quid pro
quo for making the promise.
2. The exchange of promise for consideration is called a bargain or
bargained-for exchange. The requirement of consideration is easily
fulfilled in most contract situations, since there is almost invariably a quid
pro quo (e.g., goods for money) establishing a bargained-for exchange.
3. However, in some promissory transactions, no exchange is contemplated
or the existence of quid pro quo is questionable. Many of these situations
involve intrafamilial transactions, but they occur in the commercial world as
well. The problem arises because the promisee has neither promised nor
given anything in return to the promisor, leaving the consideration require-
ment unsatisfied. These transactions are called gratuitous promises,
which are generally unenforceable under the doctrine of consideration.
EXAMPLE: A invites his newly widowed sister-in-law, who lives 60 miles
from A’s farm, to come and live in his guest house without charge so that
she’ll have a safe place to raise her family, explaining that he is eager to
help her in her time of dire need. Because the sister-in-law neither prom-
ised nor gave anything in return for A’s promise, A’s promise is gratuitous
and unenforceable.
4. Insufficient Consideration versus Failure of Consideration
a. Insufficient Consideration
(1) A promisor defending against an attempt to enforce a gratuitous
promise can state his defense in several ways:
(a) the promise is not supported by consideration;
(b) there is a want of consideration, or consideration is lacking; or
(c) there is legally insufficient consideration.

NOTE These terms are not to be confused with a failure of consideration.

b. Failure of Consideration
(1) A failure of consideration is a claim that the party has not
performed in accordance with his promise.
EXAMPLE: If A promises to deliver a horse to B in exchange
for B’s promise of payment and A then fails to deliver the horse,
then there is a failure of consideration. However, we would not
say that the consideration for B’s promise was lacking, since
A’s original promise of performance constituted consideration
for B’s promise of payment.

192
CONTRACTS

c. The Preexisting Duty Rule


(1) A promisor cannot provide consideration where that
consideration is a duty the promisor is already obligated to
perform. This is known as the preexisting duty rule.
(2) Preexisting duties are not deemed consideration because the
promisor must already perform that duty on the basis of the
original contract.
5. Bargained-For Consideration
a. The majority of jurisdictions evaluate consideration based on the
bargained-for exchange inquiry. A minority of jurisdictions use a
benefit/detriment analysis instead. Under this test, consideration is
evaluated based on whether there is a benefit to the promisor or a
detriment to the promisee.
(1) There are also jurisdictions that use a hybrid evaluation
combining the bargained-for exchange inquiry with the benefit/
detriment analysis to define consideration as requiring a
promise in exchange for a detriment.
(2) Whether courts are applying the benefit/detriment test or the
hybrid “promise in exchange for a detriment” test, the question
of whether a particular performance by the promise constitutes
a “detriment” is answered by the application of the so-called
legal detriment test.
(a) Under this test, the question is not whether the promisee will
actually suffer or end up with some sort of net loss due to the
transaction. Rather, the question is whether the promisee is
doing something he had a legal right not to do or is forgoing
some activity in which he had a legal right to engage.
EXAMPLE: Playing a concert at Carnegie Hall may well be
thrilling for a musician, but it would still constitute a legal
detriment because the musician has a legal right not to play
the concert. Likewise, giving up smoking and drinking may
well be good for one’s health, but it would still constitute a
legal detriment because the abstainer is forgoing activities
in which he has a right to engage. Accordingly, promises
to pay the musician and the abstainer for their respective
efforts would be enforceable under the legal detriment test.
These activities would likewise meet the majority bargained-
for exchange definition of consideration, for they would each
constitute a performance given in exchange for a promise.
6. Inadequacy of Consideration
a. The modern rule is that courts do not police the equivalence of
bargained-for exchanges via the consideration doctrine. In other
words, the supposed inadequacy of consideration is no defense to a
breach of contract claim.

193
OUTLINE

EXAMPLE: If A promises his birthright for a mess of pottage from B, B


has nonetheless provided consideration for the promised birthright no
matter how meager the value of the pottage.
EXCEPTION: If A promised to sell Blackacre to B for a meager sum
and then refused to deliver, some courts would refuse to grant B spe-
cific performance because of inadequate consideration and, instead,
limit B’s remedy for breach to money damages.
b. Do not confuse “insufficient consideration” with “inadequate consideration.”
(1) Insufficient consideration is an increasingly archaic way of
saying that there was no consideration for a particular promise.
EXAMPLE: A promises his sister-in-law a place to raise her fam-
ily and seeks nothing in return.
(2) By contrast, a claim of inadequate consideration assumes that
there is consideration but argues that it is too small in comparison
with the value of the promise for which it is exchanged.
c. Modern courts do not police the fairness of bargains via the consider-
ation doctrine. However, a party may be able to defeat the enforcement
of an excessively one-sided bargain through the defense of unconscio-
nability, discussed later.
7. Illusory Promises
a. A promise to perform that leaves performance to the discretion of the
promising party is an illusory promise and won’t constitute consideration.
EXAMPLE: Thus, if A says to B, “I’ll sell you as many widgets as you
want to order within the next two weeks for $5 a piece,” and B agrees
to buy at that price as many widgets as he decides to order from A, B’s
promise is illusory and will not constitute consideration for A’s promise.
8. Bargain
a. Consideration requires a bargain, but does not require bargaining.
Thus, there is no requirement that the parties actually negotiate before
reaching a deal. “Take it or leave it”-styled transactions are just as
enforceable as those following extensive rounds of negotiation. In other
words, the bargain requirement refers to the presence (or absence) of
an exchange between the parties, not the process of reaching the deal.
b. In contemporary language, bargain may mean that something of greater value
was acquired for lesser value, and implies that the transaction was beneficial
to the recipient. For purposes of the consideration doctrine, though, bargain
merely indicates an exchange, which could be beneficial or detrimental.
9. Gratuitous Promises
a. Executed Gifts Distinguished
(1) Although gratuitous promises are unenforceable under the
consideration doctrine, gratuitous transfers are legally binding
upon satisfaction of the requirements of a gift.

194
CONTRACTS

(2) In most states, delivery of the would-be gift combined with a


present intention to bestow the gift constitutes a legally binding
gratuitous transfer.
EXAMPLE: If John promises Kirsten a new car for her birthday,
that would constitute a gratuitous promise and would be unen-
forceable if John declines to follow through. However, if John gives
Kirsten a new car for her birthday, then the transfer is legally bind-
ing, and John cannot later change his mind and take back the car.
b. Bargains Distinguished
(1) Certain considerations will determine whether an exchange is a
bargain or a gratuitous promise.
(a) Recitals of Consideration
1) The requirement of consideration is not satisfied by a
false recital.
EXAMPLE: A wants to make a binding promise to give
his young granddaughter $10,000 on her 21st birth-
day and, accordingly, gives her a note that reads: “For
value received, I promise you $10,000 upon your 21st
birthday.” If no value was received in exchange for the
promise, the recital would not satisfy the consideration
requirement and the promise would be unenforceable.
EXCEPTION: Some courts will enforce option con-
tracts on the basis of a signed writing with a false
recital of consideration.
2) A condition on a gratuitous promise also does not
satisfy the requirement of consideration. A condition is
something a promisee must do to avail himself of the
promisor’s benevolence.
EXAMPLE: A invites his newly widowed sister-in-law, who
lives 60 miles away from his farm, to come and live in his
guesthouse without charge so she’ll have a safe place to
raise her family, explaining that he is eager to help her in
her time of need. The promisee here won’t be able to take
advantage of A’s offer unless she moves her family the 60
miles to A’s farm. However, the move is merely a condition
of the gratuitous promise, not consideration.
3) Distinguishing consideration from a condition on a gratu-
itous promise can be accomplished using three factors:
a) the language of the parties;
i) Words suggesting benevolence rather than
self-interest, such as “gift,” may indicate a
gratuitous promise that has a condition rather
than consideration.

195
OUTLINE

b) commercial versus charitable or familial context; and


i) Bargains are standard operating procedure
in the commercial context, and gratuitous
promises in that setting are rare. By contrast,
gratuitous promises are made in the family
and charitable context more regularly.
c) the benefit to the promisor.
i) In case of doubt, the most important factor in
determining whether a particular performance
is consideration or a mere condition on a
gratuitous promise is whether the promisor
benefits from the performance.
EXAMPLE: Consider again the situation in
which A invites his newly widowed sister-
in-law, who lives 60 miles from A’s farm, to
come live in his guest house without charge
so the she’ll have a safe place to raise her
family, explaining that he is eager to help
her in her time of need. The 60-mile move
is a condition on a gratuitous promise rather
than consideration because A’s explanation
suggests a benevolent purpose, the
transaction occurs in the familial context, and
it is the promisee-widow and her family, not A,
who will benefit from the move.
10. Forbearance of a Claim or Defense
a. Valid consideration exists when a party agrees to forbear a claim or
defense in exchange for a promise or performance by the other party.
This is so even if the claim or defense proves to be invalid. However,
forbearance to assert or the surrender of a claim or defense which
proves to be invalid is not consideration unless [Restatement (2d) of
Contracts § 74(1)]:
(1) the claim or defense is in fact doubtful because of uncertainty
as to the facts or the law; or
(2) the forbearing party believes that the claim or defense may be
fairly determined to be valid.
b. The execution of a written instrument surrendering a claim or
defense by one who is under no duty to execute it is consideration if
the execution of the written instrument is bargained for, even though
he is not asserting the claim or defense and believes that no valid
claim or defense exists [Id.].
11. Alternatives in the Absence of Consideration
a. There are two sets of circumstances in which a promisee may be
able to enforce a promise that is not supported by consideration.

196
CONTRACTS

(1) Past or Moral Consideration


(a) As a general rule, a promise given in exchange for
something already given or already performed will not
satisfy the bargain requirement.
EXAMPLE: In Mills v. Wyman [20 Mass. (3 Pick.) 207 (1825)],
the court refused to enforce the father’s after-the-fact promise
to compensate a Good Samaritan for nursing his dying son,
and the case would likely come out the same way today.
(b) Two exceptions to the past consideration rule were
available at common law and continue to apply today:
1) a written promise to pay a debt barred by the statute
of limitations is binding; and
2) a written promise to pay all or part of an indebtedness
that has been discharged in bankruptcy is binding.
(c) A minority of jurisdictions, supported by Section 86 of
the Second Restatement, will enforce a promise made in
recognition of a past benefit conferred so long as:
1) the promisee conferred the benefit on the promisor
and not on a third party; and
2) the benefit is material.
EXAMPLE: A sees that B is in grave danger and
heroically intervenes to save the latter, injuring himself
in the process. B gratefully promises to compensate A
for his efforts. These are the facts of Webb v. McGowin
[168 So. 196 (Ala. Ct. App. 1935)], where the court
enforced the promise because A’s efforts bestowed a
material benefit (the saving of a life) on B.
EXAMPLE: Because the Good Samaritan in Mills v.
Wyman bestowed the benefit of nursing services on
the promisor’s son rather than on the promisor, the
promise would not be enforceable.
EXCEPTION: Where the promisor makes a promise in
recognition of benefits that the promisor received under
the terms of a contract, the promise is not enforceable.
EXAMPLE: Employer promises a retiring employee a
pension worth half the latter’s salary in recognition of
the latter’s many years of hard and loyal work. Although
the employee’s past work no doubt bestowed material
benefits on the employer, the work was done under
contract (i.e., the employee was paid for it under the
terms of the employment agreement), and accordingly,
the employer’s promise is not enforceable.

197
OUTLINE

(2) Promissory Estoppel


(a) A promisee that reasonably relies to his detriment on a
gratuitous promise may be able to enforce that promise. The
doctrine of promissory estoppel allows for the enforcement of
certain promises even where there is no consideration in return.
(b) There are four requirements that must be met in order for
promissory estoppel to be available:
1) a promise;
a) Clearly, in order for promissory estoppel to be
available, there must be a promise on which the claim
is based. Statements of present intention or vague
statements about the future would not qualify as a
promise. The statement must contain an unambiguous
assurance of future action to be considered a promise.
EXAMPLE: Statements like “I would like to give you
some money to help you with college” would be
considered too vague to be an actual promise. Alter-
nately, “I will give $25,000 toward your tuition this fall”
would be specific enough to qualify as a promise.
2) foreseeable reliance;
a) The promisee’s reliance must be reasonably fore-
seeable to the promisor at the time of the promise
EXAMPLE: The widow’s move to her brother-in-law’s
farm on the faith of his promise would have been
reasonably foreseeable to the brother-in-law at
the time he made her the promise. However, if the
widow had purchased a high-end SUV to facilitate the
move, this would likely not have been foreseeable.
3) actual reliance; and
a) The actual reliance must be induced by the
promise. This means that the action or reliance
cannot be taken on account of other factors.
EXAMPLE: If the widow had already decided to
make the 60-mile move before her brother made her
the promise, then this would not be actual reliance.
4) injustice without enforcement.
a) A promise that meets the other elements required by
promissory estoppel will be binding if injustice can be
avoided only by the enforcement of the promise.
b) Although this requirement may sound vague and
open-ended, most courts take it seriously and are
willing to use promissory estoppel to depart from
the general rule that gratuitous promises aren’t

198
CONTRACTS

enforceable only where the prospect of injustice is


convincingly established.
c) There are several factors that a court analyzes.
i) The court will consider the strength of
the case as a whole, and where there is
substantial proof of the other elements, the
claim that injustice cannot otherwise be
prevented is strengthened. Alternately, where
there is weak evidence of the other elements,
it is less likely that the court will find that
injustice could not otherwise be prevented.
ii) The court will consider the blameworthiness
of the breach. Thus, a willful breach provides
a stronger case for promissory estoppel than
a breach occasioned by circumstances beyond
the promisor’s control.
iii) The court will consider the balance of equities
between the parties. Thus, the requisite
injustice is easier to establish the greater the
harm of nonenforcement to the promisee is,
and more difficult to establish the greater the
harm of enforcement to the promisor is.
EXAMPLE: An uncle’s promise to give his
nephew $100,000 upon graduation from a spe-
cific school is more likely to be enforced if the
nephew turned down scholarship opportunities
at other schools in order to attend the school
of the uncle’s choosing, or if the school would
have been out of the nephew’s reach but for
the uncle’s promise to pay. It is less likely that a
court would enforce an uncle’s promise to pay
$100,000 upon graduation from law school if
the nephew has a lucrative job and the uncle’s
family has in the meantime suffered serious
financial setbacks since making the promise.
iv) The more detrimental a promisee’s reliance,
the easier it is to establish the requisite injus-
tice. Some courts treat detrimental reliance as
an independent requirement to be established
in order to prove promissory estoppel.
EXAMPLE: Consider an alternative to the
preceding example of the uncle who promised
to pay the nephew $100,000 upon graduation
from law school. The nephew in reliance on

199
OUTLINE

the promise decided to borrow his way through


a top-ranked school in a distant state, giving
up a chance to attend a lesser-ranked, less
expensive state school. In this case, it is very
possible that the nephew’s job prospects and
earning potential upon graduation from the
high-ranked school are dramatically improved,
and a court might find that his reliance on his
uncle’s promise was, in fact, beneficial and not
detrimental. As a result, this would prevent the
promise from being enforced.
v) Some courts treat enforcement of a gratuitous
promise as a last resort. In those jurisdictions,
enforcement via promissory estoppel is barred
where other alternatives that could prevent
injustice exist. However, all courts would find
the existence of options short of enforcement
of the promise to be a highly relevant factor in
deciding the nature of the injustice at risk.

200
CONTRACTS

IV. STATUTE OF FRAUDS

A. In General
1. The general rule in contract law is that a contract need not be in writing and
that oral and written agreements are equally enforceable.
2. However, the Statute of Frauds provides an important exception to that
rule and makes some classes of contracts unenforceable unless reflected
in a signed writing. The Statute of Frauds was originally an English statute
and has been adopted in every U.S. jurisdiction through either legislation
or common law.
3. The Statute of Frauds can be divided into two general areas of application:
a. certain sales of goods; and
(1) The Statute of Frauds governs certain sales of goods in every
jurisdiction except Louisiana [§ 2-201].
b. certain transactions not involving the sales of goods.
(1) Other types of transactions that do not fall under Section 2-201
of the UCC may still be governed by the Statute of Frauds
where states adopted the original Statute of Frauds or some
modified version of it.

B. The Statute of Frauds in Operation


1. The Statute of Frauds applies to certain categories of contracts, such as
contracts for the sale of real estate or contracts lasting more than one year.
2. For these contracts, the Statute of Frauds requires that the contract be
evidenced by a writing signed by the party against whom enforcement is sought.
3. In the context of litigation, the Statute of Frauds is a defense which, if avail-
able, may be raised against a party who has claimed breach of contract.
a. The defense is available to the breaching party when the transaction in
question falls within one of the categories governed by the Statute of
Frauds, and the required writing is absent or deficient.

C. Evaluating Statute of Frauds Issues


1. The general rule is that a contract within the Statute of Frauds is not
enforceable absent a writing signed by the party against whom the enforce-
ment is sought. The most efficient way to analyze a Statute of Frauds
problem is by analyzing three questions in the following order:
a. Does the agreement fall within the Statute of Frauds?
(1) The Statute of Frauds only governs certain categories of
contracts/agreements; thus, if the subject matter of the agree-
ment does not fall under the Statute of Frauds, then the Statute
of Frauds will not apply.
(2) If the agreement does fall under the Statute of Frauds, it is neces-
sary to proceed further with the analysis.

201
OUTLINE

b. Is the Statute of Frauds satisfied?


(1) If an agreement is within the Statute of Frauds, then the key
question is whether the Statute’s writing requirement is met.
(a) If the signed writing requirement is met and the Statute
is satisfied, then the Statute of Frauds will not render the
contract unenforceable.
(b) If the contract is within the subject matter of the Statute of
Frauds but the Statute is not satisfied, then the party seeking
to enforce the contract will lose.
c. Is alternative enforcement available?
(1) If the contract is within the Statute of Frauds, but the Statute is
not satisfied, the party seeking to enforce the contract will have to
use another theory of enforcement, such as promissory estoppel
or quasi-contract, in order to protect its interests. The final step in
the Statute of Frauds analysis is to look for any of those alterna-
tive theories that could apply.

D. Analyzing Problems under the Statute of Frauds


1. Specific Coverage under the Common Law Statute of Frauds
a. In most states, six categories of agreements are governed by the
Statute of Frauds:

STATUTE OF FRAUDS

MY LEGS

Marriage, Year, Land, Executor, Guarantee, Sale of goods

(1) a contract upon consideration of marriage;


(2) a contract that will not be completed within one year of the forma-
tion of the contract;
(3) a contract for the sale of an interest in land;
(4) a contract of an executor or administrator to answer for a duty of
the decedent;
(5) a contract of guarantee or suretyship; and
(6) a contract for the sale of goods at a price of $500 or more.
(a) This category is governed specifically by Section 2-201 of the UCC.
b. Individual states have added additional categories to the six conven-
tional ones. Two common additional categories are real estate
brokerage agreements and nonmarital cohabitation agreements.
c. In the typical case, there is little difficulty in determining whether a
particular contract falls under the Statute of Frauds. However, some
categories have specific limitations within them that narrow the reach
of the Statute of Frauds.

202
CONTRACTS

d. Promise to Marry
(1) Most states have abolished the cause of action for breach of a
promise to marry, though it still exists in some jurisdictions. In
jurisdictions that recognize this claim, the coverage of the Statute
of Frauds has been narrowed as follows:
(a) A promise to marry that has been given in exchange for
dowry or other settlement is governed by the Statute, and
there must be a signed writing in order to secure enforce-
ment of either party’s obligations.
(b) An exchange of promises to marry is not governed by the Statute
of Frauds and does not need signed writing to secure enforcement.
e. Performance within One Year
(1) The year at issue under the one-year provision is measured from
the date of the contract’s formation rather than the date of the
beginning of performance. A contract that contemplates a dura-
tion of less than a year may nonetheless fall under the Statute of
Frauds if performance is not to be completed until more than one
year after the contract’s formation.
EXAMPLE: In June of her first year of law school, Law Student
enters an oral agreement with Law Firm to work for the firm dur-
ing June, July, and August of her second summer. Although the
duration of the contemplated performance is only three months,
the performance will not be complete until 14 months after the
making of the agreement. Accordingly, the contract is governed
by the one-year provision, and a signed writing is required in
order to secure enforcement.
(2) When a contract does not specify a date by which performance
is to be completion, the question of whether a particular contract
is to be performed within one year of the making thereof is
answered by determining whether it is at all possible to complete
the required performance within a year’s time.
EXAMPLE: It is highly unlikely that the construction of an oil-pro-
ducing facility in a war-ravaged country would be completed within
a year. But because the performance is possible within that time—
even if exceedingly unlikely—the agreement is not governed by the
one-year provision, and no signed writing is required.
EXAMPLE: One of the parties to an oral construction agreement
attempts to back out 14 months into the project and raises the
Statute of Frauds as a defense. Because the prospect of perfor-
mance is measured from the point of making the contract (when
completion within a year was possible) rather than from the time
of the dispute (when completion within the first year is clearly no
longer possible), the agreement is not governed by the one-year
provision and no signed writing is required.

203
OUTLINE

EXAMPLE: A three-year exclusive representation agreement


between a professional athlete and his agent cannot be
performed within a year, no matter how diligently the agent
works. Accordingly, the agreement is governed by the one-year
provision, and a signed writing is required.
(3) Virtually any contract can be breached or excused within a year
of its formation. However, courts typically hold that a breached
or excused contract would not be a fully performed contract for
purposes of the one-year provision.
EXAMPLE: Although a two-year contract for an employee’s services
might be breached by the employee’s poor performance at some
point during the first year, the contract is nevertheless governed by
the one-year provision because full performance will take two years.
(4) Under the majority rule, a lifetime or permanent contract of
employment is not governed by the one-year provision because
the employee’s death is possible within the first year, resulting in
termination of the contract without breach. The minority does treat
these types of contracts as falling within the one-year provision.
f. Land-Sale Contracts
(1) The land provision has been modified under some circumstances.
(2) The land provision governs contracts for the sale of an interest
in land, and most courts take the expression “contract for sale”
literally, thus distinguishing between a contract for a future sale
and a present conveyance.
(a) The contract for future sale is governed by the land provision
and requires a signed writing. The present conveyance of land
promised for money is held to be outside the land provision.
(3) Although real estate brokerage agreements certainly
contemplate the sale of an interest in land, most courts treat
brokerage agreements as falling outside of the land provision.
The reason is that the brokerage agreement is really a service
contract and not a contract for real estate.
(a) Some jurisdictions have resolved this by adding a sepa-
rate category to their Statute of Frauds specifically for
brokerage agreements.
(4) Leases are generally treated as contracts falling within the land
provision because a leasehold is an interest in land. Similarly,
leases for longer than one year also fall within the one-year
provision of the Statute of Frauds.
(a) However, most states except from the land and one-year
provisions of the Statute of Frauds short-term leases (for
one year or less) or contracts to lease. Where this is the
case, leases for greater than one year still need to be in
writing to satisfy the Statute of Frauds.

204
CONTRACTS

g. Guaranty / Suretyship Agreements


(1) The general rule that a promise to answer for the debt of a third
party—a suretyship or guaranty agreement—is subject to the
Statute of Frauds has two critical exceptions.
(a) When the creditor discharges the original debtor from his
obligation on the faith of a guarantee by a third party to pay
the debt, these agreements are not governed by the Statute of
Frauds and do not require a signed writing by the guarantor.
(b) Under the main purpose exception, if the main purpose
of the guarantor’s promise is to protect or promote his own
economic interests, rather than the interests of the debtor,
then the agreement is not within the Statute of Frauds and
no signed writing is required.
1) The mere presence of a selfish interest is not sufficient to
trigger this exception; it must be central to the promise.
h. Sale of Goods over $500
(1) This category is discussed under “Analyzing Problems under
the UCC Statute of Frauds.”
2. Satisfaction of the Statute of Frauds
a. If a contract falls within the Statute of Frauds, then the general rule
is that the contract is unenforceable unless evidenced by a writing
signed by the party against whom enforcement is sought. In essence,
there are two requirements for satisfaction of the Statute: a writing,
and that the writing be signed.
b. The Writing Requirement
(1) There is no requirement that the parties put their actual agree-
ment in writing; rather, all that is necessary is that the writing be
a memorandum thereof, which can be prepared before, during,
or after contract formation. Under the common law, the following
memoranda have been held to satisfy the writing requirement:
(a) a letter from one of the parties to a third party describing
the agreement;
EXAMPLE: “Dear Carville: I finally convinced Gregory to re-
paint my house, and he was willing to do so for only $2,000.”
(b) the written offer, acceptance of which formed the contract; and
EXAMPLE: “I will clean your house on the first Friday of
each month for the next six months in exchange for $500
per month, payment due by the 15th of each month when
such services occur.”
(c) a letter from one of the parties to the other party
repudiating, and so admitting, the agreement.
EXAMPLE: “Dear Tony: Unfortunately, I regret that I will be
unable to build your new shed. The $5,000 you agreed to

205
OUTLINE

pay to me was very generous, but I just don’t think that I


have the time right now, and so I must regretfully bow out.”
(2) The memorandum need not document the transaction in detail.
Only the following terms are required:
(a) the identity of the parties to the transaction;
(b) the nature and subject matter of the contract; and
(c) the essential terms of the unperformed promises in the agreement.
(3) Required Description in Cases under the Land Provision
(a) The case law is split on the kind of description you need to
satisfy the land rule under the Statute of Frauds, with some
cases holding that an address or its equivalent will do the
trick and others requiring a full legal description.
(4) Most courts treat electronic documentation, such as e-mail, as
satisfying the writing requirement.
c. The Signature Requirement
(1) The actual signature of the party against whom enforcement is
sought is not necessary. Any symbol, including initials; typed,
stamped, or preprinted signatures; or letterhead, if used with
the intention to authenticate the writing, will suffice.
(2) Most states have adopted the Uniform Electronic Transactions
Act (“UETA”), pursuant to which electronic signatures are
considered to satisfy any legal writing requirements.
(3) “Tacking Together” Multiple Documents
(a) The writing need not be a single document; a party may
satisfy the Statute by tacking together several documents
which, once combined, satisfy all the necessary
requirements for the Statute of Frauds.
1) If all of the documents are signed by the party against
whom the contract is being enforced, or if a signed
document incorporates unsigned documents by
reference, then the signature requirement is satisfied.
2) If unsigned documents are not incorporated by
reference in a signed document, “tacking together” the
signed and unsigned documents to satisfy the Statute
of Frauds is nevertheless permissible if:
a) there is at least one signed writing
unambiguously establishing a contractual
relationship between the parties;
b) the signed and unsigned documents clearly refer
to the same subject matter; and
c) there is clear and convincing evidence of
acquiescence to the unsigned documents by the
party against whom enforcement is sought.

206
CONTRACTS

d. Performance
(1) The Statute of Frauds may be satisfied with respect to some of
the categories of governed contracts via part performance.
(2) Land Contracts
(a) Part performance will make an oral contract for the sale of
land enforceable in two situations:
1) in an action by the buyer against the seller, but not in an
action by the seller against the buyer; and
2) in an action for specific performance, but not in an
action for money damages.
(b) Part performance requires a showing of any combination, or
all three, of the following:
1) payment of all or part of the purchase price;
2) taking of possession; and
3) making substantial improvements to the property.
(3) One-Year Contracts
(a) Full performance of an oral contract for services by the party
performing the services will make the contract enforceable
against the paying party. On the other hand, part perfor-
mance is not compensable on the contract. However, the
performing party may be able to recover for the reasonable
value of the services actually rendered via quantum meruit.
(4) Sale of Goods Contracts
(a) The UCC Statute of Frauds may be satisfied by part perfor-
mance of a sale of goods contract, and the rules governing
such satisfaction are discussed later.
3. Enforcement Where the Statute of Frauds Is Not Satisfied
a. If the contract is within the Statute of Frauds but the Statute is not satis-
fied because the breaching party never signed a written document, the
aggrieved party may nevertheless be able to secure some protection for
his interests via an action for restitution or promissory estoppel.
b. Recovery for Benefits Conferred
(1) Where one party bestows benefits upon another in connection
with an oral contract, even if the enforcement of that contract is
barred by the Statute of Frauds, the aggrieved party may recover
something. The party has the option of filing a cause of action for
restitution, seeking to recover the value of the benefits that he
conferred. Alternatively, if services are involved, the party may
recover on a theory of quantum meruit, meaning he could sue to
recover the reasonable value for the services he rendered.
EXAMPLE: A spends three months working for B under an oral
two-year employment contract. If B fires A and successfully raises
a Statute of Frauds defense against enforcement of the oral con-

207
OUTLINE

tract, A may nonetheless be able to recover from B the reason-


able value of the services rendered in quantum meruit.
c. Promissory Estoppel
(1) Where a party suffers losses in reliance on an oral contract, but the
enforcement of that contract is barred under the Statute of Frauds,
the party may be able to recover damages via promissory estoppel.
EXAMPLE: Where a party to an oral contract within the Statute
of Frauds promises the other party that he has created or will
create a signed writing evidencing the parties’ agreement, and
the other party relies on that assurance by failing to take other
steps to satisfy the Statute of Frauds, most courts will find prom-
issory estoppel against the party who falsely made assurances
of a signed writing.
EXAMPLE: In many cases, however, there is no assurance that a
writing has been or will be created, and the reliance at stake will in-
stead be based on the very contract whose enforcement is barred
by the Statute of Frauds—as would be the case, for example, for
an employee who leaves his current job and moves across the
country based on an oral two-year employment contract, only to be
dismissed before he starts his new job. Invoking promissory estop-
pel in this setting is both more controversial and more difficult.
(2) Some courts hold that promissory estoppel is available to protect
reliance on a contract whose enforcement is barred by the Statute
of Frauds. Other courts have held that the common law promissory
estoppel doctrine is preempted by the Statute of Frauds.
(3) Among those courts that do recognize promissory estoppel, some
will impose stiffer requirements on the claimant than those appli-
cable under an ordinary promissory estoppel action. In particular,
courts will look for:
(a) the definite and substantial character of reliance, and its rela-
tionship to the remedy sought;
(b) the extent to which the reliance is corroborated by the
evidence of the formation and terms of the contract; and
(c) the extent to which the formation and terms of the contract
are otherwise established by clear and convincing evidence.

E. Analyzing Problems under the UCC Statute of Frauds


1. Specific Coverage under the UCC Statute of Frauds
a. The UCC Statute of Frauds is Section 2-201, which by its terms governs
agreements “for the sale of goods for the price of $500 or more.”
(1) With regard to agreements for which the price is payable in some
form other than money (e.g., goods), the monetary value of the
consideration must be determined in order to apply the $500 rule.

208
CONTRACTS

(2) For agreements contemplating the sale of multiple goods, calcu-


lation of the price will depend on whether the transaction consti-
tutes a single contract or a series of separate contracts. When an
agreement falls within the UCC Statute of Frauds and the Statute
is not satisfied, the entirety of the contract is unenforceable, not
just the portion of the contract which exceeds $499.99.
b. The UCC Statute of Frauds also applies if a contract as modified falls
within the Statute (e.g., the contract is modified such that the price
of the goods has gone from $300 to $600) [§ 2-209(3)]. By the same
token, if the newly modified contract now falls outside of the Statute of
Frauds, it does not apply.
2. Satisfaction of the UCC Statute of Frauds
a. There are five ways to satisfy the UCC Statute of Frauds.
(1) Signed Writing
(a) The easiest way to satisfy the UCC Statute of Frauds, a
signed writing requires:
1) a writing;
a) Although the UCC defines writing as “any intentional
reduction to tangible form,” which clearly covers
everything from quill and parchment to computer
printouts, a more difficult question is presented by
electronic documentation (e.g., e-mail). A majority of
courts have concluded that electronic documentation
satisfies the writing requirement of the UCC.
b) The proposed revisions to UCC Article 2 replace the
word “writing” with the word “record” for the express
purpose of covering electronic documentation in the
UCC Statute of Frauds and other provisions.
2) that is signed by the party against whom enforcement is
sought; and
a) The UCC broadly defines “signed” as “any symbol
executed or adopted by a party with present inten-
tion to authenticate a writing,” which covers the
same variety of “signings” effective under the
common law Statute of Frauds.
EXAMPLE: The party’s initials; a typed, stamped,
or printed signature; or a memorandum written on
preprinted letterhead would all suffice.
b) Most states have adopted the UETA, under which
electronic signatures satisfy legal writing requirements.
3) which is “sufficient to indicate that a contract for sale
has been made between the parties.”
a) The Official Comment to UCC Section 2-201 states,
“All that is required is that the writing afford a basis

209
OUTLINE

for believing that the offered oral evidence rests on


a real transaction,” a very relaxed standard.
EXAMPLE: A enters an oral agreement with B to sell
the latter a speedboat for $10,000. B writes a check
that contains A’s name as the payee, B’s name as
payor, $10,000 as the amount, and the word “speed-
boat” in the notation space. Although the writing
does not use the word “agreement” or “contract,”
it is clearly “sufficient to indicate that a contract of
sale has been made” between A and B and would,
therefore, satisfy the writing requirement of the UCC
Statute of Frauds against B, the signing party.
(b) Satisfaction of the UCC Statute of Frauds requires the term
indicating the quantity of goods sold in the transaction.
1) The contract is unenforceable beyond the quantity of
goods shown in the writing, irrespective of the parties’
actual agreement.
2) If there is no quantity term, the contract is altogether
unenforceable, subject to two exceptions:
a) where other language in the writing provides an
unambiguous basis for measuring quantity; and
EXAMPLE: “My 2009 sweet basil crop” or “my
1998 Honda Odyssey.”
b) in the case of output and requirements contracts, the
expression “output,” “requirements,” or their equiva-
lent satisfies the quantity requirement under the UCC.
3) Apart from the quantity term, no other term is required
in the writing. Moreover, it doesn’t matter if a term is
incorrectly stated. However, a writing can have so few
terms that it will no longer be “sufficient to indicate that a
contract for sale has been made between the parties.”
EXAMPLE: A and B enter an oral agreement under the
terms of which A is to sell B five tobacco barns for a
total of $5,000. After they shake hands, A takes out a
piece of paper, writes “five barns,” signs it, and gives
it to B. If A subsequently backs out of the deal, he will
have a successful Statute of Frauds defense against B
despite the fact that he signed a writing with a quantity
term, because the writing, taken as a whole, is insuffi-
cient to indicate that a contract for sale has been made
between A and B.
(c) Despite the fact that the rules under the UCC Statute of
Frauds are generally more easily satisfied than the common

210
CONTRACTS

law Statute of Frauds rules, a written offer that will satisfy the
latter may not be enough under the UCC.
1) Because the UCC’s writing requirement says that the
writing must be “sufficient to indicate that a contract for sale
has been made between the parties,” the plain language
suggests that only a writing that is contemporaneous with
or subsequent to contract formation will satisfy the test.
2) However, a written offer that proposes a sale of goods
at a price of $500 or more that meets the requirements
of the UCC firm offer rule is enforceable against the
signing merchant. In such a case, the UCC Statute of
Frauds is irrelevant, despite the fact that without docu-
mentary proof of acceptance, the offer is not sufficient to
indicate that a contract for sale has actually been made.
(2) Merchant’s Confirmation
(a) The UCC Statute of Frauds may be satisfied when two
merchants enter an oral agreement and one of them sends the
other a written confirmation of the agreement. In such a circum-
stance, the Statute is satisfied against the recipient merchant if
the latter fails to object to the confirmation in a timely fashion.
(b) A valid merchant’s confirmation requires a writing that [§ 2-201(2)]:
1) is “sufficient against the sender”;
a) The question here is whether the confirmation
meets the requirements of Section 2-201(1) (i.e.,
it must be signed by the sender, it must contain a
quantity term, etc.).
2) is “in confirmation of the contract”;
a) The authorities are divided on whether this language
imposes a requirement on valid merchants’ confirma-
tions in addition to the requirement that the writing be
“sufficient against the sender.” Some courts hold that
a writing that is sufficient against the sender under
Section 2-201(1) is all that is necessary; other courts
stress the “and” term and require express language
confirming the existence of a prior oral agreement.
EXAMPLE: During a telephone conversation on
March 8, A places an order and B agrees to fulfill it.
A sends a signed letter to B that outlines the price,
quantity, and delivery terms “per 3/8 discussion,”
and ends with the following language: “We look
forward to you fulfilling our order per these terms.”
Because the signed writing would be sufficient to
satisfy the UCC Statute of Frauds against sender
A, some courts would hold that it would also be

211
OUTLINE

sufficient against the recipient. Other courts would


require an express reference to the parties’ preex-
isting “agreement” or “contract” in order to satisfy
the “writing in confirmation of the contract” require-
ment of Section 2-201(2).
3) is “sent within a reasonable time” of the making of the
oral agreement; and
4) the contents of which the receiving merchant has
“reason to know.”
(c) A confirmation that never reaches the intended recipient’s
place of business (e.g., because of postal misdirection)
would not be valid against the recipient, since the latter
would have no “reason to know” its contents. On the other
hand, a confirmation that reaches the intended recipient’s
place of business but remains unopened through some
neglect would meet the “reason to know” test.
(d) A confirmation that meets the foregoing requirements will
satisfy the UCC Statute of Frauds against the recipient unless
the recipient provides written notice of objection to the confir-
mation’s contents within 10 days of receipt, but a would-be
objection that is itself sufficient against the objector under
Section 2-201(1) will satisfy the Statute against the objector.
EXAMPLE: A and B enter an oral agreement under the
terms of which A is to sell B five tobacco barns for a total of
$5,000. The following day, B sends a written confirmation of
the agreement to A, and two days later B receives a signed
reply that reads in pertinent part: “My understanding is that
we don’t have a contract between us, for as I mentioned, I
am still entertaining other offers for the barns. My apologies
for any confusion that may have been caused.” Because A’s
reply objects to the confirmation’s contents, the confirmation
does not satisfy the UCC Statute of Frauds against A.
EXAMPLE: A’s signed reply to B’s confirmation reads: “I re-
gret that I can’t go through with the sale of my barns to you.
Although the $5,000 you agreed to pay is very generous,
my accountant has advised me of certain tax implications of
the transaction that I hadn’t considered, and I must there-
fore bow out.” Because A’s signed reply is itself sufficient to
indicate that a contract for sale had been made between the
parties—a contract from which A is now seeking to escape—
the reply satisfies the UCC Statute of Frauds against A.
(3) In-Court Admission
(a) A third means of satisfying the UCC Statute of Frauds occurs
when a party against whom enforcement is sought “admits in

212
CONTRACTS

his pleading, testimony, or otherwise in court that a contract


for sale was made.”
1) This exception applies whether the admission in ques-
tion is voluntary (as in pleadings) or involuntary (as on
cross-examination).
2) A number of courts have held that the existence of this
exception requires that Statute of Frauds claims be resolved
at trial rather than at the pleadings stage, since the party
against whom enforcement is sought might be persuaded to
admit to the existence of a contract in open court.
3) Other courts have concluded that it would defeat the
purpose of the Statute of Frauds to permit a party
attempting to enforce a contract to “badger the defen-
dant through discovery and trial simply because of the
possibility of obtaining an admission.”
4) The in-court admission exception will not secure
enforcement of the contract in question “beyond the
quantity of goods admitted.”
(4) Part Performance
(a) Despite the absence of a signed writing, an otherwise valid
contract is enforceable “with respect to goods for which
payment has been made and accepted or which have been
received and accepted.”
(b) The part performance exception does not apply unless
there are actions by both parties indicating that a contract
for sale exists. Thus, a payment can constitute part perfor-
mance only if the buyer makes the payment and the seller
accepts it, and delivery of goods can constitute partial
performance only if the seller makes the delivery and the
buyer accepts the goods.
1) If a buyer pays cash, there is seldom any ques-
tion whether payment has been made and accepted.
Payment by check, however, is more complicated, with
some courts holding that receipt of a check without
indorsement or negotiation does not constitute accep-
tance, and other courts holding that mere retention of the
check for a reasonable period constitutes acceptance.
2) Any act of dominion inconsistent with a seller’s
continued ownership of goods (e.g., moving them from
the warehouse receiving area to a retail store) will
constitute acceptance of the goods.
(c) Divisible versus Indivisible Goods
1) If the contract in question involves divisible goods (e.g.,
widgets), part performance secures enforcement for any
quantity that has already been paid for by the buyer or

213
OUTLINE

delivered by the seller; the contract is not enforceable


beyond that quantity.
2) If the contract in question involves an indivisible good
(e.g., a speedboat) the majority rule holds that partial
payment secures enforcement of the entire contract,
though a minority of courts refuses enforcement unless
full payment has been made.
(5) Substantial Reliance by the Seller of Specially Manufactured Goods
(a) A seller is unlikely to be able to sell specially manufactured
goods to other buyers if a would-be buyer backs out, thus
leaving the seller with a potentially large reliance loss. Also,
the fact of manufacture according to specifications attrib-
utable to the buyer provides strong evidence that an oral
contract actually exists between the parties.
(b) Accordingly, an aggrieved seller can secure enforcement of
an oral contract by establishing the following five elements:
1) the goods are to be specially manufactured for the buyer;
2) the goods are not suitable for sale to others in the ordi-
nary course of the seller’s business;
3) the seller has substantially begun to manufacture, or
made commitments to procure, the goods;
4) the actions undertaken to begin to manufacture or
procure occurred under circumstances that reasonably
indicate that the goods are for the buyer; and
5) the actions undertaken to begin to manufacture or
procure occurred before the seller received notice of the
buyer’s revocation.
b. Authorities are divided on whether a party seeking enforcement of a sale
of goods contract that is also within the common law one-year provision
must satisfy both the UCC and the common law Statute of Frauds.
EXAMPLE: Merchants A and B enter an oral installment contract calling
for A’s delivery to B of 1,000 widgets per month for two years at a price of
$1.00 per widget. A sends B a written confirmation of the oral agreement
that meets the requirements of UCC Section 2-201(2), to which B does
not respond. B backs out of the deal, A sues for breach of contract, and B
defends via the Statute of Frauds. Although the written confirmation clearly
satisfies the UCC Statute of Frauds, the contract will be nevertheless un-
enforceable in some jurisdictions because the agreement is also governed
by the Common law one-year provision, which is not satisfied because the
party against whom enforcement is sought (B) did not sign the writing.
3. Enforcement Where the UCC Statute of Frauds Is Not Satisfied
a. If a party has relied to its detriment on an oral contract whose enforce-
ment is barred by the UCC Statute of Frauds, the aggrieved party may
be able to secure a remedy via promissory estoppel.

214
CONTRACTS

(1) While some courts hold that promissory estoppel is available in


appropriate circumstances to secure enforcement of a promise
otherwise barred by the Statute of Frauds, other courts point to
the express language of the UCC Statute of Frauds (“[e]xcept as
otherwise provided in this section, a contract . . . is not enforce-
able”), and conclude that it precludes promissory estoppel, since
that doctrine is not mentioned in Section 2-201.
(2) Among those courts that do recognize promissory estoppel,
some impose enhanced proof requirements on the claimant, such
as proving unconscionable injury or that the other party would be
unjustly enriched without enforcement of the contract.
(3) Because of the unique vulnerability of construction contractors
to revocation by their subcontractors, virtually all courts have
protected a general contractor via promissory estoppel even if the
oral subcontract in question is for goods at a price of $500 or more.

F. Effect of Satisfying the Statute of Frauds


1. An otherwise perfectly valid contract, possessing offer, acceptance, and
consideration, can still be unenforceable under the Statute of Frauds. This
can happen if the contract is covered by the Statute of Frauds, but does not
satisfy the Statute.
2. On the other hand, satisfaction of the Statute of Frauds does not ensure a
“win”; the aggrieved party must still go on to establish both a valid contract
and a breach thereof to succeed on his claim.
3. Thus, satisfying the Statute of Frauds is necessary but not sufficient
alone to enforce a contract.
EXAMPLE: A sues B for breach of an oral contract for the sale of land, and
B raises the Statute of Frauds defense. A adduces the written offer signed
by B and thus establishes that the Statute is satisfied. A may still lose on his
breach of contract claim, however, if it turns out that the offer lapsed or was
revoked before A accepted it or if for any other reason A fails to persuade
the trier of fact that a valid contract existed between the parties.

G. No Mutuality of Obligation under the Statute of Frauds


1. It is possible for one party to prevail under a Statute of Frauds defense
while, for the same transaction, the other party would be fully liable under the
contract. This is a result of the fact that the Statute of Frauds does not require
that both parties have signed a writing, only that the party against whom
enforcement of the contract is sought has signed. If the writing evidencing the
contract is signed by only one of the parties, the contract will be enforceable
against the signing party but not against the non-signing party.
EXAMPLE: A and B enter an oral contract for the sale of land and the only
signed writing between them is an offer signed by B. If B breaches, he will
not have a Statute of Frauds defense because A will be able to adduce a

215
OUTLINE

writing signed by B. If A breaches, however, he will be able to raise a suc-


cessful Statute of Frauds defense because he is the party against whom
enforcement is sought and he didn’t sign the writing.
a. An exception to this principle is created by the merchant’s confirmation
provision of UCC Section 2-201, under which a signed written confir-
mation sent by one party to the other will defeat a Statute of Frauds
claim by the receiving party as well as by the sending party unless
certain conditions are met.

H. Limitations on the Statute of Frauds


1. The only effect of a successful Statute of Frauds defense is to defeat
enforcement of the contract against the non-signing party. The contract may
still be valid and enforceable for other purposes, such as:
a. the contract may provide evidence in establishing an element of a legal
claim apart from breach of contract;
EXAMPLE: A brings an action against C for tortious interference with
A’s oral land contract with B. Although B would have a successful Stat-
ute of Frauds defense in a breach of contract action brought by A, the
Statute will be irrelevant in A’s tortious interference claim against C.
b. the contract may provide evidence establishing a defense to a legal
claim apart from breach; and
EXAMPLE: Under an oral contract for the sale of A’s land to B, B
takes possession. A reneges and brings a trespass action against B.
Although A would have a successful Statute of Frauds defense in a
breach of contract action brought by B, the Statute would be irrelevant
to the validity of B’s defense to A’s trespass claim.
c. the contract may provide evidence of the value of the services
already rendered.
EXAMPLE: A spends three months working for B under an oral two-year
employment contract. If B fires A and successfully raises a Statute of
Frauds defense against enforcement of the oral contract, A may none-
theless be able to recover from B the reasonable value of the services
rendered in quantum meruit. Moreover, he may offer the price agreed to
in the oral contract as evidence of the value of the services rendered.

216
CONTRACTS

V. GAP-FILLERS, INTERPRETATION, AND THE PAROL EVIDENCE RULE

A. In General
1. Parties to a contract ordinarily reach explicit and unambiguous agree-
ments on the essential issues of the contract, such as quantity and price.
However, they frequently fail to be as explicit as to the details. This can
result in either gaps in the contract or contractual language that is subject to
more than one interpretation. This section deals with the rules courts use to
fill contractual gaps and resolve contractual ambiguities.

B. Filling in the Gaps with Default Provisions


1. One of the principal ways that contract law deals with gaps in a contract is
by supplying terms that govern areas where the contract is silent. Just as a
word processing program has default settings for the margins and layout,
the body of contract law has default provisions for contracts.
2. These provisions can be overridden where the parties provide for a different
procedure, but if they do not, the default contract terms will supplement
their contract. Standard rules automatically supply contract terms unless
they are set aside by party action or agreement.
3. UCC Default Rules for the Sales of Goods
a. Implied Warranties
(1) Warranty of Title and against Infringement
(a) For the sale of all goods, there is an implied warranty of
good title to the goods, of the rightful transfer of the goods,
and that no liens or other security interests are attached to
those goods [§ 2-312].
(b) This warranty can only be excluded or modified by [§ 2-312(2)]:
1) specific language; or
2) circumstances which give the buyer reason to know that
the seller does not claim unencumbered title.
(c) Unless otherwise agreed, a merchant regularly dealing in
goods of the kind warrants that they are free of any rightful
claim of a third person by way of infringement. However,
a seller is not liable for such a claim if the buyer provides
specifications to the seller and the claim arises out of compli-
ance with the specifications [§ 2-312(3)].
(2) Warranty of Merchantability
(a) If the seller of the goods is a merchant with respect to goods
of that kind under the UCC, then there is a warranty of
merchantability for the sale of goods. The warranty guaran-
tees that the goods are fit for the ordinary purposes for which
those goods would be used [§ 2-314].

217
OUTLINE

(3) Warranty of Fitness for a Particular Purpose


(a) This warranty grants that the goods being sold are fit for the
particular purpose for which the buyer intends to use them.
The warranty only applies where, at the time of contracting,
the seller has good reason to know [§ 2-315]:
1) the particular purpose for which the goods are required; and
2) that the buyer is relying on the seller’s skill or judgment
to select or furnish reasonable goods.
(4) Excluding or Modifying Implied Warranties of Merchantability
and Fitness
(a) The implied warranty of merchantability may be excluded
or modified by language mentioning “merchantability” and
need not be in a writing. However, if it is in a writing, it must
be conspicuous [§ 2-316(2)].
(b) An implied warranty of fitness may be excluded or modi-
fied by language in a writing that is conspicuous. Language to
exclude all implied warranties of fitness is sufficient if it states,
for example, that “There are no warranties which extend
beyond the description on the face hereof” [§ 2-316(2)].
(c) However, the above language is not necessary to exclude or
modify the warranty if [§ 2-316(3)]:
1) the contract includes an expression like “as is” or “with
all faults” or other similar language that in common
understanding calls the buyer’s attention to the exclu-
sion of warranties and makes plain that there is no
implied warranty;
2) the buyer has fully examined the goods before entering
into the contract, or has refused to do so, in which case
there is no implied warranty as to defects that should
have been apparent from the examination; or
3) course of dealing, course of performance, or usage of
trade indicate that the warranty is excluded or modified.
b. Express Warranties
(1) Note that the UCC also provides that a contract can create
express warranties. Express warranties by the seller are created
as follows [§ 2-313(1)]:
(a) any affirmation of fact or promise made by the seller to
the buyer that relates to the goods and becomes part of the
basis of the bargain creates an express warranty that the
goods will conform to the affirmation or promise;
(b) any description of the goods that is made part of the basis
of the bargain creates an express warranty that the goods
will conform to the description; and

218
CONTRACTS

(c) any sample or model that is made part of the basis of the
bargain creates an express warranty that the whole of the
goods will conform to the sample or model.
(2) The seller need not use formal words such as “warrant” or “guar-
antee,” nor must the seller have a specific intention to make a
warranty in order to create an express warranty. However, a mere
affirmation of the value of the goods or a statement of the seller’s
opinion does not create a warranty [§ 2-313(2)].
(3) Words or conduct relevant to the creation of an express warranty and
words or conduct tending to negate or limit warranty shall be construed
wherever reasonable as consistent with each other [§ 2-316(1)].
(a) Where an express warranty conflicts with a limitation or
disclaimer, the express warranty will prevail [Id.].
(4) The measure of damages for breach of warranty is the difference
at the time and place of acceptance between the value of the
goods accepted and the value they would have had if they had
been as warranted [§ 2-714(2)].
c. Missing Terms
(1) Under the UCC, where there are pertinent terms missing, the
following provisions will supplant the contract:
(a) The default rule for a missing price term is the reasonable price
at the time established by the contract for the delivery of goods.
(b) A missing time term exists if the contract is silent as to the
date of delivery or any other date by which action must be
taken under the contract. The default rule for a missing time
term is that contractual action must be performed within a
reasonable time.
(c) The default rule for a missing place of delivery term is that
the place of delivery will be the seller’s place of business
unless otherwise agreed.
4. Common Law Default Rules for Service and Employment Contracts
a. Missing Price Term
(1) If one party performs services at the request of another but
no price is discussed in advance, then the default rule under
common law will apply. The default rule for a missing price term
is the reasonable value of the services rendered. This is typically
available based on quantum meruit.
b. Missing Duration Term
(1) In practically every jurisdiction, the employment-at-will rule is
the default rule for duration of an employment contract; absent
an agreement to the contrary, an employer may dismiss, and an
employee may quit, at any time for any reason.
(2) In a majority of jurisdictions, oral or written assurances of job
security made to an individual employee, as well as assurances

219
OUTLINE

contained in policy documents distributed to the workforce, may


suffice to take the contract out of the default rule.
EXAMPLE: Statements made during a recruiting interview that
make assurances of job security would displace the at-will rule.
Alternatively, similar written assurances contained in a personnel
handbook would also displace the rule.
EXCEPTION: Most courts will enforce a clear and conspicuous dec-
laration in a personnel handbook or other employer policy statement
to the effect that the employment relationship is “at will.” However,
a conflict between such a disclaimer and other written or oral repre-
sentations by the employer will create a jury question on whether the
employer has successfully opted out of the at-will default.
5. The Obligation of Good Faith and Fair Dealing
a. Another source of gap-filling is the obligation of good faith and fair
dealing. Both the UCC and the Restatement impose an obligation of
good faith in the performance and enforcement of contracts. Good
faith is generally defined as “honesty in fact in the conduct or trans-
action concerned” and, in the case of a merchant, good faith means
“honesty in fact and the observance of reasonable commercial stan-
dards of fair dealing in the trade.”
b. The obligation of good faith may only be raised in regards to the perfor-
mance and enforcement of contracts, but not to negotiations or other
precontractual conduct. Challenges to unfair dealing in those settings must
be made via claims of fraud, duress, or other defenses, discussed later.
c. The good-faith obligation will also operate to ensure good faith where
the terms of the contract leave a critical term, such as the price, satis-
faction, or quantity, open to the determination of one party.
(1) Open Price Term
(a) If a contract leaves the price to be fixed by one of the
parties, then the specified party must fix the price in good
faith [§ 2-305(2)].
(2) Satisfaction Term
(a) A contract may contain a satisfaction clause, or similar term,
whereby the determination as to whether a party’s performance
obligation is complete is left to the discretion of the other party.
In those contracts, the good-faith obligation operates to require
the party making that determination do so in good faith.
EXAMPLE: A commercial developer and the seller of a piece
of land enter into a contract. The developer places a condi-
tion on his purchase of the land that the seller first acquires
“satisfactory leases” for the future occupancy of the land.
The developer must determine in good faith whether or not
the future leases are “satisfactory.”

220
CONTRACTS

(3) Open Quantity Term: Output and Requirements Contracts


(a) An output contract is a contract in which the buyer agrees
to purchase all of a seller’s output for a particular good.
(b) A requirements contract is a contract in which the seller
agrees to supply the buyer with all of the buyer’s require-
ments for a particular good.
(c) Under the UCC, the party entitled to determine the particular
quantity of goods to be sold—either the buyer demanding
delivery of his requirements or the seller demanding
purchase of his output—must make that determination in
good faith [§ 2-306].
(d) In addition to the good-faith requirement, the UCC prohibits
any unreasonably disproportionate demand or tender, if
there was either [§ 2-306(1)]:
1) a stated estimate made between the parties; or
2) in the absence of a stated estimate, any comparable
prior outputs or requirements.
EXAMPLE: Seller and Buyer are parties to a three-
year contract obligating Seller to supply Buyer with the
latter’s upsidasium bearing requirements for Buyer’s
turbojet engine manufacturing plant. The market price
for upsidasium suddenly skyrockets, and Buyer greatly
increases its demand for bearings with the intention of
selling them at great profit to third parties. Seller is not
obligated to fill the order because Buyer’s requirements
demand was made in bad faith.
EXAMPLE: The upsidasium market remains stable, but
because of a decline in the commercial aviation industry,
the demand for Buyer’s engines declines precipitously
and accordingly Buyer reduces its monthly demand for
bearings by 90% in comparison with the previous 24
months. Although Buyer’s reduced demand was made in
good faith, it is unreasonably disproportionate to its prior
requirements and, accordingly, violates Section 2-306.

C. Interpreting Ambiguous Language


1. Where the parties did not fail to address an issue, but instead drafted the
contract’s language in such a way that the language could be subject to
more than one interpretation, then a different type of analysis is needed in
order for the courts to properly determine the meaning.
a. Objective versus Subjective Meaning
(1) The objectively reasonable meaning of a term at the time of
contracting would control over a contrary subjective under-
standing of the term by one of the parties.

221
OUTLINE

EXAMPLE: Contractor and Homeowner enter a contract for reno-


vations to Homeowner’s home. The contract incorporates by refer-
ence “the specifications dated 1/11/07,” which had been prepared
by Homeowner and were attached to the contract before signing.
Contractor mistakenly thinks that the specifications referenced were
an earlier version prepared by Contractor, and he signs the contract
on that understanding. The specifications prepared by Homeowner
and referenced in the contract are binding on both parties.
(2) This rule is subject to two exceptions.
(a) Where one party harbors a subjective understanding of a term
that differs from the objectively reasonable meaning, typically
the objective understanding will prevail. However, if the other
party knows or has reason to know of the first party’s subjective
understanding, then that subjective interpretation will control.
EXAMPLE: The facts being otherwise as stated in the previous
example, just before the parties sign the contract, Homeowner
overhears Contractor tell his business partner that he was glad
Homeowner had agreed to Contractor’s version of the speci-
fications. Because Homeowner knew of Contractor’s under-
standing of the contractual terms, that understanding controls.
(b) Where the evidence demonstrates that both parties shared
the subjective understanding of the term at the time of
contracting, the mutual subjective understanding will control.
EXAMPLE: A general contractor is building a bridge. He hires
a paving subcontractor to do work at a price to be calculated
in accordance with the dimensions of the “concrete surface of
the bridge deck.” At the time of contracting, both of the parties
understand that the concrete work included the sides and bot-
tom as well as the “surface” of the bridge deck and, accord-
ingly, that interpretation would be the controlling interpretation.
b. Contra Proferentem and the Doctrine of Reasonable Expectations
(1) Under the rule of contra proferentem, if an ambiguous term
is included in the contract, then it will be interpreted against the
party who supplied the term during negotiations or drafting. Thus,
if more than one meaning is possible, then the meaning more
favorable to the other party will control.
(a) This rule of construction applies to contracts between
bargaining equals as well as to contracts of adhesion, which
are standard form (take-it-or-leave-it) contracts that a party
with superior bargaining power uniformly imposes on its
trading partners.
(2) Under the doctrine of reasonable expectations, even unambig-
uous terms may be interpreted against the drafting party if they
conflict with the reasonable expectations of the other party.

222
CONTRACTS

(a) Most courts will apply the doctrine of reasonable expecta-


tions to insurance contracts, but its application is potentially
available to all adhesion contracts.
(b) Under the doctrine, courts make a distinction between the
dickered terms, which discuss the transaction-specific terms
negotiated by the parties, and boilerplate terms, or those
terms appearing in fine print. The court will apply the doctrine
only to the boilerplate terms.
EXAMPLE: In an insurance contract, the coverage, deduct-
ibles, premium amounts, and policy periods are typically the
dickered terms. The rest of the agreement, including specifi-
cation of the claims process and exclusions, are typically set
forth in boilerplate and are therefore subject to the doctrine
of reasonable expectations.
(c) Under this doctrine, the parties’ obligations consist of the
dickered terms and only the boilerplate terms that would be
consistent with the reasonable expectations of the purchaser.
Unreasonable boilerplate terms are unenforceable.
EXAMPLE: Fertilizer Co. purchased from Allied Insurance Co.
an insurance policy covering burglary with fine-print boilerplate
language defining “burglary” as requiring “visible marks on”
or “physical damage to” the exterior of the insured building.
During the policy term, Fertilizer Co. was the victim of a theft in
which the perpetrators were able to force open a Plexiglas door
without leaving marks or damage, and Allied refused payment
on the policy. Because the definition of “burglary” was a boiler-
plate term inconsistent with the reasonable expectations of the
insured, the court interpreted the policy to cover the theft.

D. Trade Usage, Course of Dealing, and Course of Performance


1. Extrinsic evidence of trade usage, course of dealing, or course of perfor-
mance will be available as an additional means of filling contractual gaps or
resolving contractual ambiguities.
2. Definitions under the UCC
a. A usage of trade is any practice or method of dealing having such
regularity of observance in a place or trade as to justify an expectation
that it will be observed with respect to the transaction in question.
b. A course of dealing is a pattern of conduct concerning previous trans-
actions between the parties that is fairly to be regarded as establishing
a common basis of understanding for interpreting their subsequent
expressions and other conduct.
c. A course of performance is present when a particular contract involves
repeated occasions for performance by a party and the other party, with
knowledge of the nature of the performance and opportunity for objec-
tion to it, accepts the performance or acquiesces in it without objection.

223
OUTLINE

3. Evidence of trade usage, course of dealing, or course of performance is admis-


sible under the UCC and is typically available in common law cases as well.
a. Such evidence may be used to supplement the express terms of a
contract or to give meaning to a particular ambiguous term. However, it
is not admissible to contradict the express terms of a contract.
EXCEPTION: Course of performance evidence may be available to
establish the waiver or modification of an express term.
b. In the event of a conflict, course of performance prevails over course
of dealing and usage of trade, and course of dealing prevails over
usage of trade.
EXAMPLE: A is one of many suppliers of upsidasium, and B is one of
A’s long-standing customers. A fills an order by B and demands im-
mediate payment; B refuses. It is standard practice in the upsidasium
industry for purchasers to pay invoices upon delivery, and, therefore,
B’s refusal to do so is in violation of the parties’ contract.
EXAMPLE: Same facts as the previous example except that in the
past, A has regularly given B 30 days to pay for deliveries. Because
this course of dealing between the parties trumps the payment-upon-
delivery usage of trade, B has 30 days within which to make the re-
quired payment.
EXAMPLE: Same facts as the previous example except that the par-
ties’ contract contains a provision requiring payment within seven days
of delivery. Because the express terms of a contract override both
course of dealing and usage of trade, B has seven days within which to
make payment despite the past practice of paying in 30 days and the
industry practice of immediate payment.
EXAMPLE: Same facts as the previous example except that A and
B are parties to an installment contract and B has paid 30 days after
multiple deliveries without objection by A. This evidence of the parties’
course of performance is admissible to establish that A has waived its
contractual right to payment within seven days of delivery and/or that
the parties have mutually modified that obligation.

E. The Parol Evidence Rule


1. In General
a. Another source of evidence shedding light on the meaning of an agree-
ment is the negotiations of the parties leading up to the execution of a
written contract. The parol evidence rule governs efforts to introduce
such evidence [§ 2-202].
b. The rule governs:
(1) both oral and documentary evidence of negotiations and other
communications between the parties (e.g., an exchange of letters
between the parties); and

224
CONTRACTS

(2) communications that took place prior to or contemporaneously


with the execution of the written contract.

NOTE For ease of reference, courts refer to all such evidence as “parol evidence.”

2. Approaching a Parol Evidence Analysis


a. When a party seeks to introduce parol evidence, the court will deter-
mine the admissibility of the evidence based on two questions:
(1) First, what is the purpose for which the evidence is being introduced?
(2) Second, does the evidence relate to a term or contract which
is integrated?
(a) The integration analysis may focus on the term itself or the
contract as a whole, depending on the purpose for which the
parol evidence is being introduced.
3. Integration
a. While the first step in the analysis is to determine the purpose for
which the evidence is being introduced, the effects of these different
purposes will depend upon the level of integration in the contract.
b. Full versus Complete Integration
(1) When a particular contract is fully integrated (sometimes
referred to as partial integration), it means that the terms
contained within the contract are intended to be the final discus-
sion of the parties as to those specific terms. It does not,
however, mean that the parties have specifically excluded any
provisions not contained within the agreement.
(2) On the other hand, when a contract is completely integrated,
the parties intend the contract to represent a “complete and
exclusive statement of all the terms” [§ 2-202(3)]. When this is
the case, only the terms contained within the written agreement
are considered part of the contract.
c. Determining Full Integration and Complete Integration
(1) The question of whether the terms of a contract are fully inte-
grated (i.e., final) and whether an integration is complete (i.e.,
exhaustive) is for the judge, not the jury, to decide.
(2) In reaching decisions regarding whether the terms of a contract
are fully integrated, the judge will rely on the following evidence:
(a) The thoroughness and specificity of the written instrument
in connection with the terms at issue are strong indica-
tors that the parties intended the writing to represent their
final agreement.
(b) Parol and other extrinsic evidence are admissible. The
determination is not made on the basis of an examination
of the writing alone; “all the circumstances”—including the
proffered parol evidence and other evidence extrinsic to the
written contract—must be considered.

225
OUTLINE

(3) In reaching decisions regarding the complete integration of a


contract, the judge will rely on the following evidence:
(a) Merger Clause
1) The most important evidence that the parties intended their
written agreement to represent an exhaustive account of
their contractual obligations is the presence in the contract
of a merger clause reciting that the writing “contains the
complete and entire agreement of the parties” or other
words to that effect. The authorities are divided, however,
on whether a merger clause is conclusive or merely
persuasive on the question of complete integration.
(b) Other Evidence
1) Additional factors which are important for determining
whether a contract as completely integrated include the
detail with which the contract sets forth its provisions, as
well as the length of the agreement itself.
(c) Parol and Other Extrinsic Evidence Admissible
1) Once again, the question is subject to an “all the
circumstances” test, and the proffered parol evidence
as well as other extrinsic evidence may be considered
in that connection.
EXAMPLE: A and B have a written contract under the
terms of which A agrees to sell B “all the uncut timber
on my property.” Evidence of conversations between the
parties during a visit to the timber site is admissible to
explain that “property” means A’s country cottage and
not A’s home in the suburbs.
EXAMPLE: A and B have a written contract under the
terms of which A agrees to sell B “all the uncut timber on
my property.” Evidence of an exchange of letters between
the parties in which they agreed that their agreement
would not include the shady trees in the immediate vicinity
of A’s country cottage would contradict the written expres-
sion (i.e., that A had agreed to sell B “all the uncut timber
on my property”) and would therefore be admissible only if
the court concluded that the quoted provision not intended
by the parties to be the final word with respect to which
timber would be cut (i.e., that the writing was not integrat-
ed). In making that determination, the court is free to con-
sider the exchange of letters and other extrinsic evidence.
EXAMPLE: A and B had a written contract under the
terms of which A agrees to sell B “all the uncut timber
on my property,” and the contract contained detailed
terms with respect to the felling and transportation of

226
CONTRACTS

the timber as well as a merger clause stating that the


contract was “a complete and exhaustive account of the
obligations between the parties.” Evidence of an oral
agreement between the parties at the time of signing the
contract that A would pay to have the trees inspected for
termites before cutting would be admissible to supple-
ment the agreement only if the court concluded that the
contract was partially (rather than completely) inte-
grated. Some courts would consider the merger clause
to be conclusive on the question of complete integration
and thus bar the evidence, but other courts would treat
it as persuasive only, evaluate the proffered evidence
of the supplemental oral agreement, and admit it if they
concluded that the parties did not intend the previously
drafted merger clause to bar it, despite their subsequent
signatures on the contract.
EXAMPLE: Same facts as in the previous example,
except A and B are logging companies, and B offers
evidence that sellers doing business in the commercial
lumber trade invariably pay to have the trees inspected
for termites before cutting. This evidence would be
admissible even if the court concluded that the contract
was completely integrated because, under the UCC, un-
cut timber equals “goods,” and usage of trade evidence
is admissible to supplement a written agreement.
4. Purpose for Which Evidence Is Introduced
a. The effect of the rule depends on the purpose for which the parol
evidence is being introduced:
(1) to explain or interpret the terms of the written contract;
(2) to supplement the terms of the written contract; or
(3) to contradict the terms of the written contract.
b. To Explain or Interpret Terms of a Written Contract
(1) In a majority of jurisdictions, parol evidence is always admis-
sible for this purpose.
(2) A minority of jurisdictions follow the traditional common law rule
and require a threshold showing that the term in question is
patently ambiguous (i.e., ambiguous on its face) before admitting
parol evidence for this purpose.
c. To Supplement Terms of a Written Contract
(1) Parol evidence is admissible for this purpose unless the contract
is completely integrated.
(a) A completely integrated contract is intended by the parties
to represent an exhaustive as well as final account of the
parties’ obligations.

227
OUTLINE

(2) UCC Distinction


(a) Usage of trade, course of dealing, and course of perfor-
mance evidence are admissible in sale of goods cases to
supplement the terms of a written agreement.
d. To Contradict Terms of a Written Contract
(1) Parol evidence is admissible for this purpose unless the term(s)
in question are fully integrated.
(a) Integrated terms are terms intended by the parties to repre-
sent their final agreement on the subjects in question.
(2) UCC Distinction
(a) Course of dealing or course of performance evidence may
be admissible in sale of goods cases to “qualify” the meaning
of an integrated term.
5. When the Parol Evidence Rule Will Not Apply
a. The parol evidence rule will not apply to the following types of evidence:
(1) Subsequent Agreements
(a) The parol evidence rule only applies to oral or written
communications made prior to or contemporaneously with
the execution of a written agreement. This means that it will
not apply to subsequent agreements entered into after the
execution of the written document.
EXAMPLE: Buyer and Seller enter into a written agreement
for the sale of 1,000 widgets. Two months before delivery is to
be made, the parties orally agree to modify the agreement and
specify the goods as nonstandard type-Y widgets, with Buyer
agreeing to pay an additional cost. Evidence of this modifica-
tion would not fall within the parol evidence rule, as it was made
subsequent to the execution of the written agreement.
(2) Collateral Agreements
(a) The parol evidence will not affect agreements between the
parties that are entirely distinct from the written agreement
of the contract at issue. Evidence offered to prove or enforce
such “collateral agreements” is admissible.
EXAMPLE: A and B execute a detailed written contract,
complete with a merger clause, by which A agrees to sell A’s
car to B. As part of the transaction, the parties orally agree
that B may park the automobile in A’s garage for one year,
paying $25 per month. Despite the completely integrated
writing, either A or B may adduce evidence of the parking ar-
rangement under the collateral agreement rule.
(3) Attacks on the Validity of the Written Agreement
(a) The key predicate to the application of the parol evidence
rule is that there is a valid written agreement. As a result,

228
CONTRACTS

the parol evidence rule will not bar efforts to prove that the
written agreement is invalid or unenforceable. The absence
of an enforceable agreement may be proved by parol
evidence or any other extrinsic evidence.
(b) A party can assert such a claim in one of the following ways:
1) Failure of an Oral Condition Precedent to the Agreement
a) Evidence that the parties orally agreed to a condi-
tion precedent to the contract taking effect as well
as further evidence that the condition failed would
not be barred by the parol evidence rule where both
parties’ obligations would be discharged by the
failure of that condition.
EXAMPLE: The parties to a written contract for the
sale of artwork orally agree that the sale will not take
effect unless and until a certificate of authenticity is
issued by a named art expert. Because authentication
is a condition precedent to the sale, the parol evi-
dence rule will not bar proof of the oral agreement.
2) Absence of Consideration
a) Evidence that a written agreement lacks consider-
ation or that there was a false recital of consider-
ation is not barred by the parol evidence rule.
3) Mistake or Duress
a) Evidence that the written agreement was formed
from mistake or duress may be admissible to prove
that there was never a contract.
4) Fraud
a) The majority rule is that the parol evidence rule will
not bar extrinsic evidence of fraud.
b) The minority rule is that extrinsic evidence is inad-
missible to prove fraud where the written contract
expressly eliminates an element of the claim (e.g.,
“the parties agree that neither party relied in any
way on representations made outside the four
corners of this agreement”).
5) Reformation
a) The parol evidence rule is also inapplicable where
a party to a written agreement alleges facts entitling
him to reformation of the agreement. Reformation is
based on the premise that the parties had reached
an agreement concerning the instrument, but while
reducing their agreement to written form, and as the
result of mutual mistake or fraud, some provision or
language was omitted from, inserted, or incorrectly

229
OUTLINE

stated in the instrument intended to be the expres-


sion of the actual agreement of the parties.
b) For the plaintiff to obtain reformation, it must be shown:
i) that there was an antecedent valid agreement;
ii) which is incorrectly reflected in the writing
(e.g., by mistake); and
iii) proof of these elements is established by clear
and convincing evidence.

230
CONTRACTS

VI. PERFORMANCE, MODIFICATION, AND EXCUSE

A. Obligations under the UCC


1. In General
a. The seller’s obligation is to transfer and deliver, and the buyer’s obliga-
tion is to accept and pay in accordance with the contract [§ 2-301].
2. Seller’s Obligations
a. Non-Carrier Cases
(1) Non-carrier cases are contracts in which it appears that the parties
do not intend for the goods to be moved by common carrier.
(2) In non-carrier cases, the seller has an obligation to tender
delivery—that is, to put and hold conforming goods at the buyer’s
disposition and give the buyer any notification reasonably neces-
sary to enable him to take delivery [§ 2-503(1)].
(3) Tender must be at a reasonable hour, and the goods must be
kept available for the period reasonably necessary to enable the
buyer to take possession [§ 2-503(1)(a)].
(a) However, unless otherwise agreed-upon, the buyer must
furnish facilities reasonably suited to receiving the goods
[§ 2-503(1)(b)].
b. Carrier Cases
(1) Carrier cases are contracts in which, due to express terms or
due to the circumstances, it appears that the parties intend for
the goods to be moved by common carrier.
EXAMPLE: Common carrier may include such means as freight
train, boat, or parcel delivery service.
(2) Absent some other agreement, there is a presumption that the
contract is a shipment contract, under which the seller is not
obligated to deliver at a named destination, and bear the concur-
rent risk of loss until arrival, unless he has specifically agreed to
do so or the commercial understanding of the terms used by the
parties contemplates such delivery [§ 2-503, cmt. 5].
(a) Under a shipment contract, the seller need only put the
goods in the possession of a carrier and make appropriate
arrangements for them to be sent to the buyer, provide the
buyer with any document necessary to enable him to obtain
possession of the goods, and promptly notify the buyer that
the goods have been shipped [§ 2-504].
(b) The seller’s failure to notify the buyer of the shipment or to
make a proper contract for the shipment is only grounds for
rejection if material delay or loss ensues [Id.].
(3) In a destination contract, the seller has agreed to tender the
goods at a particular destination.

231
OUTLINE

(a) If the contract requires the seller to tender delivery of the goods at
a particular destination, the seller must, at the destination, put and
hold conforming goods at the buyer’s disposition [§ 2-503(1)].
(b) The seller must also give the buyer any notice of tender that
is reasonably necessary and provide the buyer with any docu-
ments of title necessary to obtain delivery. Tender of documents
through ordinary banking channels is sufficient [§ 2-503].
(4) In contracts that specify that delivery is free on board (“F.O.B.”),
the F.O.B. point is the delivery point.
(a) If the contract is F.O.B. the seller’s place of shipment, the
seller need only, at his expense and risk, put the goods in
the possession of the carrier.
EXAMPLE: Seller is a shirt manufacturer with a factory in
Baltimore. He contracts to sell 2,000 shirts to Buyer, who
runs a retail clothing store in Las Vegas. If their contract
states that the goods are “F.O.B. Baltimore,” Seller’s risk and
expense would end once he puts it in the possession of the
carrier (e.g., The Quick-Ship Co.).
(b) If the contract is F.O.B. the destination, the seller must, at his
expense and risk, tender delivery of the goods at the desti-
nation location [§ 2-319(1)].
EXAMPLE: If in the above example, the contract states that
the goods are “F.O.B. Las Vegas,” the Seller would continue
to bear the expense and risk while the goods are being
shipped by The Quick-Ship Co., until the goods are actually
tendered for delivery at the destination location.
(5) In contracts that specify the delivery is free alongside (“F.A.S.”), the
seller must deliver the goods alongside the vessel (in the manner
usual at the port of delivery) or on a dock designated by the buyer
and obtain and tender a receipt for the goods [§ 2-319(2)].
3. Buyer’s Obligations
a. Unless otherwise agreed-upon, the buyer’s tender of payment is a condi-
tion to the seller’s duty to tender and complete delivery [§ 2-511(1)].
b. A tender of payment is sufficient when it is made by any means or
in any manner current in the ordinary course of business, unless the
seller demands payment in cash and gives the buyer a reasonable
extension of time to procure it [§ 2-511(2)].
c. A buyer’s payment by check is conditional and will be defeated if the
check is not honored upon presentment [§§ 2-511(3); 2-511, cmt. 5].
d. Inspection of the Goods
(1) Generally, unless the parties agree otherwise, the buyer has a
right to inspect goods upon tender or delivery before making
payment or acceptance [§ 2-513(1)].

232
CONTRACTS

(2) If the contract requires payment before inspection,


nonconformity of the goods will not excuse the buyer from
making payment unless [§§ 2-512(1), 5-109(b)]:
(a) the nonconformity appears without inspection (i.e., it is
evident merely from taking delivery); or
(b) despite tender of the required documents, the documents
are forged or materially fraudulent such that an injunction
against the issuer would be justified.
(3) Note that payment before inspection will not constitute an
acceptance of the goods or impair the buyer’s right to inspect
or any of his remedies [§ 2-512(2)].
4. Risk of Loss
a. If the seller is required or authorized to ship the goods by carrier, the
risk of loss passes to the buyer [§ 2-509(1)]:
(1) when the goods are delivered to the carrier if the contract
does not require the seller to deliver the goods at a particular
destination (i.e., under a shipment contract, which is the default
presumption in carrier cases); or
(2) when the goods are tendered at a particular destination by the
carrier so that the buyer is able to take delivery if the contract
requires the seller to deliver the goods at a particular destination
(i.e., under a destination contract).
b. If the goods are held by a bailee to be delivered without being moved,
the risk of loss passes to the buyer [§ 2-509(2)]:
(1) when the buyer receives a negotiable document of title
covering the goods;
(2) when the bailee acknowledges the buyer’s right to possess
the goods; or
(3) after the buyer receives a nonnegotiable document of title or
other written direction to deliver the goods to the buyer, once
the buyer has had a reasonable time to present the document
or direction to the bailee.
c. In any other case [§ 2-509(3)]:
(1) if the seller is a merchant, the risk of loss passes to the buyer
when the buyer receives (i.e., takes physical possession of)
the goods; and
(2) if the seller is not a merchant, the risk of loss passes to the
buyer upon tender of delivery.
d. Effect of Breach on Risk of Loss
(1) If the seller breaches the contract by making a nonconforming
tender or delivery, the risk of loss remains on the seller until
cure or acceptance [§ 2-510(1)].
(2) If the buyer rightfully revokes acceptance (as described below),
the buyer may—to the extent of any deficiency in his effective

233
OUTLINE

insurance coverage—treat the risk of loss as having been on the


seller from the beginning [§ 2-510(2)].
(3) If the buyer breaches before the risk of loss passes to the buyer,
the seller may—to the extent of any deficiency in his effective
insurance coverage—treat the risk of loss as resting on the
buyer for a commercially reasonable time [§ 2-510(3)].

B. Modification
1. At common law, parties were free to enter a contract on virtually any terms
that they wished. Once they had entered the contract, however, the law
made it very difficult for them to modify those terms.
2. The contemporary rules allowing for modification under the UCC and
common law are more flexible. However, effective modification of a valid
contract is still a burdensome project.
3. Modification at Common Law
a. The Preexisting Duty Rule
(1) At common law, a promise to increase compensation under an
existing contract is an unenforceable modification to an existing
contract because there is no consideration offered for the
modification under the preexisting duty rule.
EXAMPLE: The captain of a fishing vessel promised to pay $100
to each of the sailors for their work on a fishing voyage. Midway
through the voyage, the sailors threatened to cease work unless they
were promised an additional $50 each, and the captain reluctant-
ly agreed. At the end of the voyage, the captain paid each of them
the originally promised $100 but refused to pay the $50 increase,
so the sailors sued to recover the additional amount. Because the
sailors were already obliged to perform the work in question under
the terms of their original contract with the captain, his promise of
an additional $50 is unenforceable under the preexisting duty rule.
b. Exceptions to the Preexisting Duty Rule
(1) Mutual Modification
(a) A promise to increase compensation under an existing contract
is enforceable as a mutual modification to the contract if:
1) both parties agree to a performance that is different
from the one required by the original contract; and
2) the difference in performance is not a mere pretense of
a newly formed bargain.
EXAMPLE: During the course of the fishing voyage, the
ship’s cook takes ill, and the captain instructs the young-
est of the sailors to perform the cook’s duties in addition
to his fishing duties. The sailor refuses to perform the
work unless the captain promises him an additional $50

234
CONTRACTS

in compensation, and the captain reluctantly agrees.


The promise of additional compensation is enforceable
as part of a “mutual modification” of the original contract.
EXAMPLE: Midway through the voyage, the sailors
threaten to cease work unless they are promised an ad-
ditional $50 each. The captain reluctantly agrees, and in
exchange, the sailors promised to wake him each morn-
ing with a chorus of “Rule Britannia.” Although there ap-
pears on the surface to be a “mutual modification” of the
original contract, it is a “mere pretense” of a new bargain
and, accordingly, the promised increase in compensation
is unenforceable under the preexisting duty rule.
(2) Unforeseen Circumstances
(a) Where a promise of increased compensation is given in
exchange for a performance, and that performance is
rendered substantially more burdensome than reasonably
anticipated by the parties when they entered the contract,
then the preexisting duty rule will not apply.
EXAMPLE: Same basic facts as in the previous examples,
but the fishing nets provided by the captain turn out to be
defective and thus increase the workload of the sailors in a
manner substantially in excess of what was reasonably con-
templated under the original contract. The sailors threaten
to cease the fishing unless they are promised an additional
$50 each, and the captain reluctantly agrees. The promise
is enforceable despite the preexisting duty rule in view of
circumstances not reasonably anticipated by the parties at
the time of contracting.
4. Modification and the Sales of Goods under the UCC
a. Under the UCC, the preexisting duty rule is abolished and an agreement
modifying an existing contract for the sale of goods needs no consider-
ation to be binding. Modifications must, however, meet the UCC’s good
faith test, and a failure to do so will render them unenforceable.
b. The good faith test for modifications applies even to modifications that
are supported by consideration. Similar to the contemporary common
law rule that requires more than mere pretense of a new bargain to
secure enforcement of a mutual modification, a bargained-for modifica-
tion is unenforceable under the UCC if the appearance of the mutual
bargain is merely a pretext to hide a bad faith change of terms.
5. Duress and Midterm Modifications
a. In addition to challenging a midterm modification under the rules just
discussed, a party who agrees to a contractual modification in commer-
cially extortionate circumstances may also be able to raise the defense
of duress, discussed later in this outline.

235
OUTLINE

b. The duress defense is available not only in common law cases, but
also in sales of goods cases governed by the UCC.
6. Effect of “No Oral Modifications” Clauses
a. Unless required by the Statute of Frauds, modifications can generally
be oral or written. However, the enforceability of an oral modification to
an agreement may depend upon whether the contract contains a “no
oral modifications” clause.
(1) There is no specific language required for this type of provision.
“No oral modifications,” “all modifications must be in writing,” or
any such similar language would suffice.
b. Common Law Cases
(1) Originally, there was a common law rule that made oral modi-
fication clauses invalid, and even today most courts will refuse
to enforce them where a party has reasonably relied on the oral
agreement at issue. However, their enforcement is becoming more
likely in modern cases, particularly in the construction context.
c. UCC Cases
(1) Under Section 2-209, clauses prohibiting subsequent oral modifi-
cations are presumptively valid.
(2) An oral modification made in violation of such a clause may
nevertheless be enforceable if the disadvantaged party relies on
the modification or the parties perform in accordance therewith.

C. Excusing Performance Due to Faulty Assumptions


1. A party entering a contract is susceptible to making countless assump-
tions about the present and future circumstances in which performance will
take place. When such assumptions turn out to be faulty, the doctrines of
mistake, impossibility, impracticability, and frustration of purpose may be
available to excuse the parties’ obligations.
EXAMPLE: A faulty assumption about the facts at the time of contracting would
be that the stone in a ring the seller is selling is a diamond and not a topaz.
EXAMPLE: A faulty assumption about the future facts in a contract would be the
assumption that the cost of fuel won’t increase beyond a certain range and triple
the costs of the busing service whose bid was accepted by the school district.
2. Faulty assumptions about present facts, or the facts as they exist at the
time of contracting, are dealt with under the doctrine of mistake. Faulty
assumptions about future facts are dealt with via the doctrines of impossi-
bility, impracticability, and frustration of purpose.
3. Mistake
a. When a party or parties make a faulty assumption about the present circum-
stances, and thus enter a contract on that basis, this is known as a mistake.
b. A mistake regarding the facts that exist at the time of contracting will
excuse performance only where the mistaken facts are material to that

236
CONTRACTS

contract. This requires that the mistaken facts will significantly impact
the value of the transaction to one or both parties.
c. When only one of the parties to a contract is operating under a faulty
assumption about material facts as they exist at the time of contracting,
the situation is governed by the rules of unilateral mistake. When
both parties labor under a common faulty assumption, the situation is
governed by the rules of mutual mistake.
(1) Unilateral Mistake
(a) Under the rules governing unilateral mistake, a party oper-
ating under a faulty assumption about material facts as
they exist at the time of contracting is not excused from his
contractual performance unless:
1) the other party knew or had reason to know of the mistake; or
EXAMPLE: A agrees to sell B a cow, which A knows to
be barren and, as the parties are writing up and signing
the agreement, B asks A a series of questions about the
care and feeding of pregnant cows. Because A has rea-
son to know that B is mistaken with respect to the cow’s
capacity to bear calves, B’s obligation to purchase the
cow is excused on account of unilateral mistake.
2) the mistake was based on a clerical error.
a) The clerical error exception is not available in all
jurisdictions, and where available, it is subject to
the following exceptions:
i) where the error was caused by extreme negli-
gence on the part of the party making the error; or
ii) where the other party has relied on the clerical error.
(2) Mutual Mistake
(a) Where both parties have labored under a common faulty
assumption regarding the present facts, there is a mutual
mistake. Under the rules of mutual mistake, the contract will
be voidable by the disadvantaged party where:
1) the fact about which the parties were mistaken is
essential to the contract (i.e., it goes to the very heart
of the exchange);
2) both parties were mistaken; and
3) the disadvantaged party did not bear the risk of mistake
under the parties’ agreement.
EXAMPLE: A agrees to sell B a cow at beef cow prices be-
cause at the time of contracting, both parties were under the
assumption that the cow was barren. A short time later, the
cow was discovered to be with calf, which greatly increased
her resale value. The contract is voidable at the option of A.

237
OUTLINE

EXAMPLE: A agrees to sell B a cow at beef cow prices.


At the time of contracting, both parties share the mis-
taken impression that the cow is barren. B tells A that he
is still going to try to breed the cow in any case. In this
case, A bears the risk of B’s efforts succeeding, because
if B succeeds, it will prove the parties’ assumption that
the cow was barren wrong. So A is accordingly bound
by the contract even if the cow turns out to be fertile.
(3) Application under the Common Law and the UCC
(a) The doctrine of mistake originated under common law, but is
also available in sales of goods cases governed by the UCC.
4. Impossibility, Impracticability, and Frustration of Purpose
a. Where the faulty assumptions of the parties relate to future facts,
as opposed to existing facts, these issues are dealt with under the
doctrines of impossibility, impracticability, and frustration of purpose.
b. These doctrines may be used as a defense to a breach of contract
claim. However, they can also be raised affirmatively for claims of
rescission or cancellation of the contract.
c. Impossibility
(1) The doctrine of impossibility excuses both parties from their
obligations under a contract if the performance has been rendered
impossible by events occurring after the contract was formed.
(2) Application of the doctrine of impossibility requires:
(a) objectively impossible performance; and
1) Objective impossibility occurs when the performance
under the contract becomes literally impossible because
of circumstances beyond the control of the parties.
EXAMPLE: If X promises to sell Y his horse, but the
horse dies before X can deliver it, then X’s performance
has become objectively impossible.
2) Subjective impossibility occurs when the performance
under the contract becomes impossible because of some
failure or fault on the part of the performing party. Under
those circumstances, the performance obligation is not
excused and will be considered as a breach of the contract.
EXAMPLE: The failure of a party to have enough mon-
ey either to make a promised payment or to obtain the
components required for the production of a promised
product would be subjective impossibilities.
EXAMPLE: Dealer promises to sell Buyer 200 bushels
of wheat, expecting to procure the wheat from Farmer.
Just before harvest, Farmer’s crop fails due to wheat
blight. The impossibility of Dealer’s performance is
subjective. While it may be impossible for Dealer

238
CONTRACTS

to deliver the promised bushels in a timely fashion,


because his expected source dried up at the last
minute, under the contract he was free to secure the
wheat from any source. It was Dealer’s selection of
Farmer, and his failure to have a backup plan, that
gave rise to the resulting situation. Accordingly, Dealer
does not meet the legal test of impossibility.
(b) the occurrence of the contingency must not be known to
the parties at the time of contracting. This can occur in one
of two ways:
1) it may be a supervening contingency, whereby the
performance was possible at the time of contracting,
but afterward a contingency occurs that renders the
performance impossible; or
2) it may be an existing contingency, whereby a contingency
existed at the time of contracting, but was unknown to
the parties until after the contract was formed.
(3) Exceptions
(a) The doctrine of impossibility will not apply where the parties
have allocated the risk of the contingency and provided
remedial measures in the event of its occurrence.
(b) If events render performance only temporarily impossible,
then this will typically only suspend the obligations of the
parties until the impossibility ends.
(4) Types of Impossibility
(a) There are three main categories of impossibility:
1) destruction of the subject matter of the contract;
a) Where the contract performance involves particular
goods, a building or structure, or any other tangible
item, the destruction of which occurs without the
fault of either party, then the contract is discharged.
EXAMPLE: Buyer promises to buy Farmer’s 2008
wheat crop, the entirety of which is destroyed just
before harvest by wheat blight. Farmer’s perfor-
mance is excused on the basis of impossibility.
2) death or incapacity; and
a) If the existence of a particular person is necessary
for the performance of a contract (e.g., personal
performance by an individual is required, or
performance is to be rendered to a specific
person) then that person’s death or incapacity
will trigger the doctrine of impossibility, and the
parties’ obligations will be dismissed.

239
OUTLINE

3) illegality.
a) The doctrine of impossibility will discharge the
parties’ obligations if performance is prohibited by a
change in a constitution, statute, administrative regu-
lation, or municipal ordinance, or by judicial order.
d. Impracticability
(1) Courts are reluctant to excuse performance for any reason other
than impossibility. However, under the doctrine of impracti-
cability, a promisor may be excused from performance where
unforeseen difficulties have made performance prohibitively
expensive or otherwise extremely burdensome.
(2) The doctrine of impracticability has its source in Section 2-615
of the UCC, but has since been adopted in the common law of
contracts as well.
(3) The following elements are required to show that performance
under a contract would be impracticable:
(a) the impracticability of the performance was caused by some
unforeseen contingency;
(b) the risk was neither assumed nor allocated by the parties; and
(c) the increase in the cost of performance would be far beyond
what either party anticipated.
1) Even the courts that have allowed relief due to
impracticability have held that increased cost alone is
not sufficient as an excuse.
(4) Impracticability under the UCC
(a) Under the UCC, a number of examples of contingencies that
would not excuse performance are listed.
1) Increased cost alone does not excuse performance
unless the rise in cost is due to an unforeseeable contin-
gency and alters the essential nature of performance.
2) A rise or collapse in the market will not justify
impracticability because those contingencies are
exactly the types of business risks that fixed-price
contracts are intended to account for.
(b) The UCC also lists the following examples of contingen-
cies that would trigger impracticability. All involve a severe
shortage of raw materials or supplies:
1) where the shortage is caused by:
a) war or embargo;
b) local crop failure; or
c) unforeseen shutdown of major sources of supply; or
2) the shortage either caused a marked increase in cost or
prevents the seller from securing the supplies necessary
for his performance.

240
CONTRACTS

(5) Contractual Allocation of the Risk of Non-Performance


(a) The parties are free to allocate the risk of a contingency or to
specify the remedial measure to dictate performance in the
event of a contingency. Where they do so, the performance
that would otherwise be excused by impracticability will be
governed by their contractual agreement.
(b) The doctrine of impracticability thus operates as a default rule
where the contract is silent as to the occurrence of a contingency.
(c) Whether the parties have provided for the contingency will depend
on all of the circumstances, but some specific guidelines apply.
1) If the contingency was foreseeable at the time of contract
and the contract is silent, it may suggest that the absence
of a term excusing the performance was an affirmative
decision to allocate the risk to the performing party.
2) The payment of a supra-market premium by the non-
performing party might also reflect that the parties
bargained for the contingency, and that the consider-
ation in exchange for the premium would be the perfor-
mance of the party in spite of the contingency.
3) Where evidence of a course of dealing or of trade
usage reveals that it is customary for the performing
party or others in a similar position to assume the risk
of such contingencies.
(6) A performance obligation may be excused when the failure to
perform is based on a good faith compliance with a foreign or
domestic regulation or order.
e. Frustration of Purpose
(1) Where a contingency occurs that dramatically reduces the value
of performance to the receiving party, the doctrine of frustration
of purpose may be available to excuse the receiving party from
its contractual obligations.
EXAMPLE: The doctrine of frustration of contractual purpose has
as its source the famous case of Krell v. Henry, which involved
the owner of a London flat with a “ringside” view of the forthcom-
ing coronation parade who agreed to lease the flat at premium
prices to a lessee eager to witness the festivities. The parade
was canceled when the king became ill, and the lessee’s con-
tractual obligations to the owner were excused on the grounds
that going through with the rental agreement in the absence of
its raison d’être “cannot reasonably be said to have been in the
contemplation of the parties at the date of the contract.”
(2) Modern Test
(a) The contemporary version of this rule will discharge a party’s
contractual obligations when the following three conditions are met:

241
OUTLINE

1) the party’s principal purpose in entering the contract


is frustrated;
a) This means that the frustration of incidental or
non-material purposes would not qualify.
2) there is substantial frustration; and
a) Thus, a variation on Krell v. Henry in which
the town erected bleachers that only partially
obstructed the parade view from the apartment
would not justify a finding of frustration.
3) non-occurrence of the event precipitating frustration
was a basic assumption of the contract.
a) Frustration of purpose does not apply where the
risk of the supervening event was reasonably
foreseeable and the parties could and should
have anticipated and made provision for it in the
agreement. In other words, the frustration must
be so severe that it is not fairly regarded as being
within the risks assumed under the contract.
(3) Contractual Allocation of the Risk
(a) Where the parties contractually allocate the risk of the
non-occurrence of the event, the doctrine of frustration of
purpose will not be available.
EXAMPLE: The facts otherwise being as they were in
Krell v. Henry, assume that the parties’ written agreement
contained the following language: “The parties understand
and agree that the flat owner will be arranging at great
expense to move himself and his family to the countryside
in order to permit occupancy by lessee and accordingly that
lessee will be obligated under this agreement irrespective
of whether the coronation parade takes place during the
lease period.” Since the contract allocated the risk of
the contingency to the lessee, his obligations under the
contract were not excused for frustration of purpose when
the parade was postponed.
EXAMPLE: The facts otherwise being as they were in Krell
v. Henry, assume that all of London was abuzz with news
of the new king’s possibly serious illness during the days
before the parties entered into their agreement. Because
a postponement of the coronation parade was therefore
foreseeable, the lessee’s failure to secure language in the
parties’ agreement discharging his duties in the event of
postponement constitutes evidence that the lessee bore
that risk and remained fully obligated under the contract
when it came to fruition.

242
CONTRACTS

(4) Application under Common Law and the UCC


(a) The frustration of purpose doctrine is followed under
common law.
(b) The frustration of purpose doctrine is available to supple-
ment the UCC in sales of goods contracts.

D. Excusing Performance by Agreement of the Parties


1. Rescission
a. The parties may also agree to discharge each other’s remaining duties
of performance under an existing contract that is at least partly execu-
tory on each side [Restatement (Second) of Contracts § 283(1)].
Typically, consideration will be provided by each party’s discharge of
the other’s duties [Restatement (Second) of Contracts § 283, cmt. a].
b. The Statute of Frauds will not prevent an oral agreement of rescission
that discharges unperformed duties from being enforced unless rescis-
sion of a transfer of property is involved [Restatement (Second) of
Contracts § 283, cmt. b, 148].
2. Accord and Satisfaction
a. The parties may make an accord, which is a contract under which the
obligee promises to accept substituted performance in satisfaction of the
obligor’s existing duty. Satisfaction—performance of the accord—will
discharge the original duty [Restatement (Second) of Contracts § 281].
b. Accord alone will not discharge the original duty, but once the accord
is made, the original duty will be suspended subject to the terms of the
accord until the obligor has the chance to make the substituted perfor-
mance [Restatement (Second) of Contracts § 281, cmt. b].
(1) If there is such a breach of the accord by the obligor as to
discharge the obligee’s duty to accept the substituted perfor-
mance in satisfaction, the obligee is no longer bound under the
accord, and he may choose between enforcing the original duty
or the duty under the accord [Id.].
(2) If the obligee breaches the accord, the original duty is not
discharged, but the obligor can seek specific performance of the
accord (in addition to damages for partial breach) [Id.].
c. Validity of Accord—Consideration Required
(1) General contract law applies to accords, such that consideration
is required [Restatement (Second) of Contracts § 281, cmt. d].
(2) There may be sufficient consideration if the substituted consider-
ation differs significantly from that required by the original duty, or
because the original duty was doubtful or the obligor believed it to
be doubtful [Id.].
(3) Generally, there may be an accord and satisfaction when a
creditor agrees to accept part payment of an unliquidated debt
that the debtor tenders in full satisfaction of the debt; however,

243
OUTLINE

in order for part payment to constitute sufficient consideration on


its own, there must be a bona fide or good faith dispute as to the
debt [see 1 Am. Jur. 2d. Accord & Satisfaction § 39].
3. Anticipatory Repudiation
a. Anticipatory repudiation occurs when, prior to the time that perfor-
mance is due under the contract, a party announces his intention not to
perform, or circumstances make such an intention reasonably clear to
the aggrieved party.
b. Technically, anticipatory repudiation is not the same as a breach of
contract because the party has not yet failed on its promise of perfor-
mance. However, the aggrieved party is given the option to immedi-
ately treat the anticipatory repudiation as a breach of contract and to
be entitled to the same rights and remedies.
c. Anticipatory repudiation may be established by:
(1) a party’s definitive statement indicating that he will commit a
breach of contract; or
(2) a party’s voluntary or affirmative act that renders the party unable
to perform or apparently unable to perform (e.g., the sale of the
contracted-for item to a third party).
d. Adequate Assurance of Performance
(1) If anticipatory repudiation cannot be established but there are reason-
able grounds for insecurity (i.e., reasonable grounds to believe that
the other party is unwilling or unable to perform), the insecure party
may make a demand for adequate assurance of performance.
(2) Adequate assurance of performance must be provided in the
following forms:
(a) the UCC requires that a demand for adequate assurance
of performance on a sale of goods contract be made in
writing [§ 2-609(1)]; and
(b) under the Restatement of Contracts, an adequate assurance
of performance may be made either:
1) by oral communication; or
2) in writing.
(3) Suspension of Performance by the Insecure Party
(a) Upon making a demand for assurances, a party with reason-
able grounds for insecurity may suspend its own contractual
performance so long as:
1) suspension is commercially reasonable; and
2) the insecure party has not yet received the agreed-upon
return for the performance in question.
EXAMPLE: If a seller reads about a planned bankruptcy
filing by a buyer, the seller may, after demanding assur-
ances, suspend delivery of goods for which the buyer
has not already paid.

244
CONTRACTS

(b) The insecure party may also suspend his own performance
where the performing party:
1) does not respond to a demand for assurances within a
reasonable time (30 days under the UCC); or
2) does not respond in a manner that provides reasonable
assurance to the other party.
a) In this situation, the reasonableness of an assur-
ance of performance is determined by:
i) the circumstances;
ii) the relationship between the parties;
iii) the parties’ past dealings; and
iv) the nature of the insecurity requiring assurance.
(c) The failure to respond with reasonable assurances constitutes
a repudiation of the contract by the non-responding party.
e. Rights of the Aggrieved Party upon Repudiation
(1) If the aggrieved party chooses to treat the anticipatory breach as
a breach of contract, he may:
(a) cancel the contract and terminate all rights and obligations
under it; or
(b) bring an action for damages or specific performance.
(2) The aggrieved party may immediately resort to one of these
remedies upon repudiation, or he may wait until performance
should have occurred.
(3) If an aggrieved party chooses to ignore a repudiation, then he is
prevented from continuing to perform on the contract if perfor-
mance would increase his damages from the contract.
(4) If the aggrieved party can prove to the court that he was willing,
ready, and able to render performance had the anticipatory repu-
diation not occurred, then the aggrieved party is relieved of:
(a) performance of contractual obligations; and
(b) performance of any conditions precedent.
f. Retraction of Repudiation
(1) A party who has made an anticipatory repudiation to the other
party may retract the repudiation unless and until the other party:
(a) acts in reliance on the repudiation;
(b) positively accepts the repudiation by signifying this to the
breaching party; or
(c) commences a suit for damages or specific performance.

E. Conditions
1. In some contracts, the obligation to perform is conditioned upon the
happening of some event or upon action by the other party. The law
regarding conditions governs two distinct issues: the order of parties’ perfor-
mances and the remedies where conditions to the contract have failed.

245
OUTLINE

2. Promissory versus Pure Conditions


a. A condition to contractual performance can be a condition controlled by
the parties or can be a condition outside of either party’s control.
(1) A promissory condition is where the contract performance is
conditioned on the occurrence of the promised performance by
the other party.
EXAMPLE: A publication contract contains the following lan-
guage: “Publisher’s duty of payment is expressly conditioned on
Author’s timely completion of the promised manuscript.” In this
case, Author’s timely delivery of the manuscript is a promissory
condition on Publisher’s contractual obligation of payment.
(2) Pure conditions are typically where contract performance is condi-
tioned on the occurrence of events beyond the control of either party.
EXAMPLE: Skyblasters, Inc. agrees by contract to provide a
fireworks display for a holiday celebration sponsored by the Vil-
lage of Remulak, and the contract contains a “weather permit-
ting” provision. In this case, good weather is a pure condition on
Skyblasters’ performance obligation.
3. Express versus Implied Conditions
a. Express conditions are those which the parties expressly include in
provisions of the contract.
EXAMPLE: “Publisher’s duty of payment is expressly conditioned on
Author’s timely completion of the promised manuscript.” This would be
an express condition to contract performance of payment because it is
listed within the contract.
b. Implied conditions are those created under common law or the UCC to
address order of performance and rights upon breach when the parties
haven’t done so expressly. Implied conditions are those fairly to be
inferred from evidence of the parties’ intentions. They are often referred
to as implied-in-fact, since their existence is determined by the process of
contract interpretation rather than the express language of the contract.
4. Rules Governing the Order of Performance
a. The rules regarding order of performance are dependent on whether
the contract falls under common law or the UCC.
(1) Cases Governed by Common Law
(a) If the contract contains express conditions specifying the
order of performance, then those specifications will control
the order of performance.
(b) If the contract is silent as to order of performance:
1) Where one party’s performance requires a period of
time to be completed and the other party’s performance
does not, then the performance over time is treated as
an implied condition of the latter.

246
CONTRACTS

2) Where the parties can exchange performance more or


less simultaneously, then the performances are treated
as concurrent conditions of each other.
(2) Sales of Goods under the UCC
(a) Under the UCC, the parties are free to specify the order of
performance, and where the contract contains those specifi-
cations, the performance obligations under that contract will
be performed according to the contractual terms.
(b) Because sales contracts most commonly involve delivery
and payment of goods, performance is treated as concur-
rent, and so each performance is conditioned on the perfor-
mance of the other.
5. Rules Governing the Failure of a Condition
a. The default rules for a failed condition of the contract performance are
dependent on whether the contract falls under the common law rules of
contracts or under the UCC.
b. Cases Governed by the Common Law
(1) In common law cases, the rights of the parties in the event of a failed
condition depend on whether the condition is express or implied.
(2) Express Conditions
(a) Where a party’s performance under the contract is subject
to an express condition, the failure of that condition will
discharge the party’s obligation to perform.
(b) Identifying Express Conditions
1) In some cases, the parties will use the magic words to
describe a particular performance or event as a condi-
tion on a return performance.
EXAMPLE: “The party’s duty to perform is expressly
conditioned on the promised performance.”
2) The term “expressly conditioned” does not have to be
included, but language conveying the same idea will
constitute an express condition as well.
EXAMPLE: “No obligation to proceed unless and until
franchisee takes the following steps” or “payment due
upon completion of second benchmark.”
3)In the absence of clear language, an express condition
may be established by trade usage, course of dealing,
or course of performance evidence.
(c) Excusing Failed Express Conditions
1) Generally, a failure of the occurrence of an express
condition will discharge the performance obligation of
the party who stood to benefit from the condition.

247
OUTLINE

2) Nevertheless, there are three situations in which


the failure of the condition may be excused without
discharging the beneficiary’s performance obligation.
3) Waiver
a) The party who has been discharged from performing
by the failed condition may waive the right to discharge
and perform anyway. When the party waives the condi-
tion, the waiving party’s obligation becomes absolute
because it is no longer subject to the condition.
b) Some courts treat a waiver of condition as a
midterm modification to the contract and require
that additional consideration be given by the other
party to give the waiver effect.
4) Bad Faith Conduct
a) A condition will be excused on the basis of bad faith
conduct by the beneficiary of the condition. Thus,
bad faith conduct will excuse the condition where
the benefitting party interferes with the fulfillment
of a contract, or where the benefitting party fails to
take steps necessary for the condition’s fulfillment.
EXAMPLE: In a contract for the sale of real proper-
ty, a buyer’s performance obligation is conditioned
on his success in securing financing. If the buyer
fails to apply for financing, this would be considered
conduct in bad faith, and the buyer’s performance
obligation would become absolute.
5) Avoiding Forfeiture
a) In some situations the fulfillment of a condition
may result in a forfeiture or great loss to one of
the parties. In those cases a court may excuse the
condition to avoid forfeiture.
b) In deciding to excuse a condition, the court will consider:
i) whether the party favoring excuse will suffer
a loss greatly disproportionate to the actual
prejudice to the other party;
ii) whether the failure of the condition is due to
willfulness or serious neglect;
iii) whether the other party played a role in
bringing that failure about;
iv) whether the condition relates to a minor term in
the contract as opposed to a material one; and
v) whether the fulfillment of the condition has not
failed completely but has merely been delayed.

248
CONTRACTS

NOTE There is another consideration that applies to insurance contracts only. In deciding
to excuse a condition, courts will consider whether, where a contract for insurance
benefits conditions benefits payments on timely notice of covered events, such
notice is not provided because of circumstances outside the insured’s control.

EXAMPLE: A failure to provide notice of a cov-


ered disability, where the disability itself has
caused the failure of timely notice, would be a
condition that would be excused.
EXAMPLE: Failure to provide timely proof of death,
where the body is not discovered for an extended
period of time, would be an excusable condition.
(3) Implied Conditions
(a) When the breach in question relates to an express condition,
courts will enforce the condition to the letter and discharge
the conditioned performance obligation, absent an excuse
such as waiver or bad faith.
(b) When the possibility of breach is not addressed by an express
condition, it is still a breach; however, under the law of implied
conditions, courts can treat that breach in one of two ways:
either as a material breach or as substantial performance.
(c) Material Breach versus Substantial Performance
1) If the breach is serious enough, the court will treat the breach
in the same way it would treat a breach of an express condi-
tion (i.e., as relieving the aggrieved party of his own perfor-
mance obligation). This is called material breach, and the
aggrieved party is free to walk away from his own obligations
and sue the breaching party for damages.
2) In certain cases, if the breach is less serious, the court will
treat the party’s performance as “close enough,” meaning
that the party has rendered substantial performance of
the condition. In these cases, the aggrieved party will not
be discharged of his performance obligation. The doctrine
of substantial performance applies to contracts for services
and, in particular, to construction contracts.
3) Even where the court finds substantial performance of a
condition, the aggrieved party may still sue for damages
as remedy for the breach.
4) The Restatement of Contracts lists five factors that
can help to distinguish between material breach and
substantial performance:
a) the extent to which the aggrieved party will be
deprived of the benefit, which he reasonably
expected under the terms of the contract;

249
OUTLINE

b) the extent to which the aggrieved party can


adequately be compensated via damages for the
defective performance;
c) the extent to which the breaching party will suffer
forfeiture if a material breach is found;
d) the extent to which the breach was willful or in bad
faith, rather than merely negligent or innocent; and
e) the likelihood that the breaching party will cure his
failure within a reasonable time and in a manner consis-
tent with the reasonable purposes of the contract.
i) The likelihood that the failure will be cured
is a significant circumstance in determining
whether the breach is material or non-material
[Restatement (Second) of Contracts § 241].
EXAMPLE: Contractor builds a house for Land-
owner and—unbeknownst to Contractor—the
plumbing subcontractor installs a brand of pipe
that differs from the brand specified in the con-
tract but is in every important respect the same
quality. It would be an extremely expensive and
burdensome task for Contractor to remove and
replace the pipe. The use of the substitute pipe
is a breach of the parties’ contract, but Con-
tractor has substantially performed because
Landowner is deprived of merely a de minimis
benefit since the substitute pipe is the functional
equivalent of the brand required by the contract.
Contractor will suffer an enormous loss if he is
required to remove and replace the errant pipe;
and the breach was at most negligent if the
Contractor did not monitor the subcontractor
closely enough, but in no way was the Contrac-
tor’s breach willful or in bad faith.
EXAMPLE: Contractor builds a house for the
Yuppies, a young couple who are foodies and
love to entertain and, accordingly, their plans
include a spacious kitchen. The Yuppies spend
the year in Paris while the house is under
construction, and in their absence and without
prior consultation, Contractor decides to devi-
ate from the plans and build a smaller kitchen,
thus enabling Contractor to save significant
construction costs. Contractor’s improvisation
is a material breach because it would deprive
the Yuppies of a substantial benefit for which

250
CONTRACTS

they had contracted; because money would


not adequately compensate them for the dimin-
ished kitchen that was to be the centerpiece of
their dream home; and because Contractor’s
breach was willful and in bad faith.
5) Total Breach versus Partial Breach
a) A material breach can be treated as either a partial
breach or a total breach.
i) A claim for damages for total breach is one
for damages based on all of the injured party’s
remaining rights to performance [Restatement
(Second) of Contracts § 236].
ii) A claim for damages for partial breach is one
for damages based on only part of the injured
party’s remaining rights to performance [Id.].
b) However, a non-material breach can only be a
partial breach.
(4) Failed Condition That Cannot Be Excused
(a) Where a condition has failed and cannot be excused, there
are other methods of enforcement available to mitigate the
consequences for the breaching party.
(b) If a contract requires both parties to render a series of perfor-
mances over a period of time, the contract may be divisible
in the event of a failed condition.
1) The failed condition must not be material to the contract
itself, but must relate to a small portion of the contract
performance in order for the contract to be severable
into separate transactions, allowing the breaching party
to recover for portions properly performed.
2) The legal test for determining whether a contract is divisible is:
a) Can the performance in question be apportioned
into corresponding pairs of parties’ performances?
b) Can each pair be properly regarded as agreed
equivalents?

NOTE A helpful way to look at divisibility is by asking whether the contract is nothing
more than the sum of its parts. If so, then the contract is divisible. If the con-
tract involves more than the sum of its parts, then it is unlikely to be divisible.

EXAMPLE: A one-year cleaning service contract would


be divisible if the services were provided and paid for
on a monthly basis, as it would be easy to apportion
the larger exchange into “pairs of part performance”
that are “properly regarded as agreed equivalents” (i.e.,

251
OUTLINE

a month’s worth of cleaning in exchange for a month’s


pay). To put it another way, the one-year contract is
merely the sum of the 12 monthly exchanges.

EXAMPLE: A contracts to build a house for B for


$100,000, with progress payments of $5,000 due in
monthly installments during construction and a bal-
loon payment upon the architect’s certificate of sat-
isfactory completion. The contract is not divisible,
for the performances and the progress payments
are not “agreed equivalents,” and the whole is
greater than the sum of the parts, since no reason-
able person would enter a freestanding contract for
a month of construction work.
(c) Where a party has failed to fulfill an express condition or
is in material breach of the contract, he may still be able
to recover in quantum meruit the value of benefits he has
conferred to the aggrieved party during performance.
1) The breaching party may recover the reasonable value of
the benefits conferred under a theory of quantum meruit.
2) The reasonable value of the benefits conferred will be
reduced by the damages suffered by the aggrieved
party due to the breach.
c. Sales of Goods Contracts under the UCC
(1) Express and Implied Conditions in Relationship to the
Perfect Tender Rule
(a) Under the perfect tender rule, the terms of a contract for
the sale of goods are enforced exactly. Every contract term
is thus treated as an express condition, and a breach of the
performance obligation by the seller will relieve the payment
obligation of the buyer.
1) As a result, there is no need to distinguish between
express and implied conditions because they will be
treated in the same fashion.
(b) The perfect tender rule enables a buyer to reject a seller’s
goods “if the goods or the tender of delivery fail in any
respect to conform to the contract.” Under this rule, any devi-
ation from the performance specified by the contract (e.g.,
regarding the quality or quantity of the goods, or the time or
manner of delivery) constitutes a breach by the seller.
(c) If the seller fails to make perfect tender, the buyer has three
courses of action available [§ 2-601]:
1) Reject the Goods
a) For a buyer to reject the goods, the buyer must
exercise the right of rejection within a reasonable

252
CONTRACTS

time after delivery and notify the seller of the rejec-


tion within a reasonable period of time [§ 2-602(1)].
b) If the buyer rejects the goods, the buyer may bring
an action for damages against the seller for imper-
fect tender, and the buyer must use reasonable
care with respect to holding the goods for a time
sufficient for the seller to remove them [§ 2-602].
i) If the seller gives no instructions within a
reasonable time after notification of rejection,
the buyer may store the goods for the seller’s
account, reship them to the seller, or resell
them for the seller’s account [§ 2-604].
ii) However, if the buyer is a merchant and the seller
has no agent or place of business in the place
where the goods were rejected, the buyer must
follow any reasonable instructions received from
the seller, and if none, must make reasonable
efforts to sell the goods if they are perishable or
their value will decline quickly [§ 2-603(1)].
iii) A merchant buyer who resells the goods after
a rightful rejection is entitled to reimbursement
for the reasonable expenses of caring for and
selling the goods [§ 2-603(2)].
c) If the buyer fails to state in connection with the
rejection a particular defect, the buyer will be
precluded from relying on the unstated defect to
justify rejection if the defect is ascertainable by
reasonable inspection, where [§ 2-605(1)]:
i) the seller could have cured the defect if it was
stated seasonably; or
ii) between merchants, the seller after rejection
made a written request for a written full and
final statement of all defects on which the
buyer proposes to rely.
d) If a buyer does not effectuate rejection in the
manner specified above, then he has made a failed
rejection. A failed rejection will be deemed to be
acceptance of the goods by the buyer.
2) Accept the Goods
a) The buyer may accept the goods, despite the
improper tender. Acceptance occurs when the
buyer has had a reasonable opportunity to inspect
the goods and signifies acceptance either through:
i) stating to the seller that the goods conform to
the contract;

253
OUTLINE

ii) taking the goods despite their non-conformance;


iii) failing to make an effective rejection of the goods; or
iv) taking any action that would be inconsistent
with the seller’s ownership of the goods.
EXAMPLE: Buyer puts the goods on display in
the showroom.
b) If the buyer accepts the goods, he has the following
rights and duties:
i) he must pay contract price for the goods;
ii) he may seek damages against the seller for the
nonconformity if he notified the seller of the noncon-
formity within a reasonable time after discovering
it, the seller is not prejudiced by the lack of
notice, or his rights would not be affected; and
iii) he may revoke his acceptance if the noncon-
formity substantially impairs the value of the
goods, and he was initially unaware of the
nonconformity because of the difficulty of
discovery or because his acceptance was predi-
cated on the seller’s assurances of conformity
or that nonconformity would be cured.

NOTE The buyer must revoke his acceptance within a reasonable time after the
buyer discovers or should have discovered the grounds for revocation, and
before there is any substantial change in the condition of the goods (unless
caused by their own defect). The revocation is only effective once the buyer
notifies the seller of it [§ 2-608(2)].

3) Reject Part and Accept Part of the Goods


a) A buyer who has received improper tender may accept
some of the commercial tender and reject the rest.
b) The buyer will then have the rights and duties of
acceptance for the goods he accepted, and the
rights and duties of rejection for the goods he
rejected (though this picking and choosing is limited
to “commercial units”—e.g., a buyer of bread can
reject individual loaves but not half loaves).
EXAMPLE: Buyer’s contract guaranteed that Seller
would deliver 10 gross of quart-sized Mason jars,
which are invariably boxed and sold by the gross
in the Mason jar trade. If Seller’s tender is imper-
fect (e.g., it is late or short), Buyer is free to accept
some of the jars and reject the rest, but he must do
so “by the gross.”

254
CONTRACTS

(d) Seller’s Ability to Cure


1) If a seller makes a nonconforming tender but the time of
performance has not yet expired under the contract, then
the seller may substitute conforming goods so long as:
a) the seller gives buyer seasonable notice of his
intention to substitute; and
b) the seller makes conforming delivery within the time
specified in the contract.
2) If a seller makes a nonconforming delivery and had
reasonable grounds to believe that delivery would
be acceptable to the buyer, then he may substitute a
conforming delivery if:
a) the seller gives buyer reasonable notice of his
intention to substitute; and
b) the seller makes conforming delivery within a
reasonable time.
3) Proof of Reasonable Grounds by Seller
a) A seller may prove that he had reasonable grounds
to believe that the buyer would accept nonconfor-
mity if he has evidence of:
i) express assurances to that effect from the buyer; or
ii) trade usage, course of dealing, or course of
performance evidence to that effect.
(2) Special Rules for Installment Contracts
(a) Under the UCC, installment contracts are contracts that
contemplate the delivery of goods in separate lots to be
separately accepted by the buyer.
(b) Because such contracts are performed in installments, the
occasion of a nonconforming tender will almost invariably
occur in connection with a particular installment. When that
occurs, the rights of the buyer are as follows:
1) If the nonconforming installment substantially impairs
the value of the whole contract, there is a breach of the
whole contract [§ 2-612(3)].
2) If the nonconforming installment substantially impairs
the value of that installment, and the seller cannot cure,
the buyer may reject the installment. However, if the
nonconforming installment does not substantially impair
the value of the contract as a whole, and the seller gives
adequate assurance that he will cure, the buyer must
accept the installment [§ 2-612(2)].

255
OUTLINE

VII. DEFENSES

A. Incapacity
1. Infancy
a. Infancy is the time period before a person reaches the age of majority.
An infant, commonly referred to as a minor, is any person who is under
the age of 18.
b. At common law, minors lacked the capacity to enter into a contract.
The modern rule is that a minor may enter into a contract, but the
contract is voidable at the option of the minor. However, in certain
circumstances the contract may be ratified, or alternately, the minor
may still be liable for the benefits received.
EXCEPTION: In most states, minors enjoy the power of avoidance
even if they are married or emancipated. However, a minority of juris-
dictions deprive married or emancipated minors of the power of avoid-
ance, making all contracts they enter into enforceable against them.
c. Power of Avoidance
(1) While minors have the capacity to enter a contract before they reach
the age of majority, they also have the power to disaffirm contracts
they enter into (the power of avoidance), with some exceptions.
(2) The power of avoidance means that the minor has the option
of voiding the contract. However, the contract is not void against
the other parties to a contract, and so is enforceable against
everyone but the minor.
(a) There is an increasing trend toward limiting minors’ use of the
power of avoidance to a defense only. The minor is allowed to
disaffirm a contract that the other party seeks to enforce against
him. However, he may not use the defense to bring suit against
the other party to secure restitution of monies already paid.
(3) Upon exercising the right to disaffirm a contract, the minor is obligated
to return to the other party any goods received under the contract.
(a) The minor must return the goods if they are in his possession when
he disaffirms the contract. However, he is not liable for damage,
wear and tear, or any other depreciation in value of the goods.
(b) If the minor is not in possession of the goods at the time of
disaffirming the contract because he sold the goods, then the
minor will be obligated to turn over the proceeds of the sale
of the goods to the other party.
(c) A contract between a minor and another party may be for
something that cannot be returned, such as a services
contract or a lease. In such a case, where the subject matter
of the contract is non-returnable, the minor is under no
further obligation to return or compensate the other party.

256
CONTRACTS

(d) In a minority of jurisdictions, a minor can only disaffirm a


contract by making the other party whole. In these jurisdic-
tions, the minor would be liable for depreciation, as well as
compensating the other party for the services or other non-
returnable items, in addition to returning possession or the
proceeds of a sale for returnable items.
d. Ratification
(1) For most contracts that minors enter into before reaching the age
of majority, the minor may ratify that contract upon reaching the
age of majority.
(2) A minor may ratify most transactions entered into during infancy
by making any manifestation to the other party of an intention to
be bound by the original contract.
(a) The minor’s silence regarding the contract after reaching
the age of majority is not sufficient to constitute a ratification
unless the minor continues to take advantage of goods or
services provided under the contract.
(b) In a minority of jurisdictions, if a minor fails to disaffirm the
contract within a reasonable time after reaching the age of
majority, then the contract will be deemed ratified.
(c) Once a minor disaffirms a contract, a majority of jurisdic-
tions consider the contract void, with title to the prop-
erty revesting in the other party (where possible). Once
disaffirmed, a minor can no longer ratify the contract.
Ratification must occur prior to the minor’s avoidance of
the contract.
e. Exceptions
(1) Necessaries
(a) A minor’s contract for necessaries is voidable. However, the
other party has the right in quasi-contract to recover for the
reasonable value of the goods or services provided.
(b) Necessaries are those items considered necessary for
survival. Most basic categories of necessaries include things
like food, clothing, shelter, and medical care.
1) In some jurisdictions, these items are only considered
necessaries if provided to an emancipated minor whose
parents cannot or will not provide them.
2) Courts are split on whether to include automobiles and/
or education in the category of necessaries.
(c) The other party may secure the reasonable value of the
goods or services only, not the contract price. Additionally,
if the minor paid more than the reasonable value, then he
is entitled to a refund of the difference between the amount
paid and the reasonable value.

257
OUTLINE

(2) Misrepresentation by Minor


(a) In a minority of jurisdictions, if a minor has misrepresented
his age to the contracting party in order to obtain the goods
or services of the contract, he may be equitably estopped
from proving his real age in court. This would deny the minor
the defense of infancy and make the contract enforceable.
2. Mental Incompetence
a. At common law, an insane person did not have capacity to enter a
contract, and any contracts he made were void. Under modern rules,
the bar for establishing mental incompetence is lower—a party need
not be “insane” to invoke the doctrine—and the consequences of invo-
cation depend on multiple factors.
(1) The key in all cases is whether the person was incompetent at
the time of contracting.
(2) The party who becomes incompetent after entering a contract
can escape contractual liability—if at all—via impossibility and
related doctrines, discussed earlier.
b. Requirements for Mental Incompetence
(1) If a party is adjudicated incompetent and a guardian is appointed,
then this adjudication will be sufficient to establish mental incom-
petence for contract cases.
(2) Where there is no adjudication of mental incompetence by a court,
then a party may still be declared mentally incompetent for the
purposes of contract formation under the following circumstances:
(a) Cognitive Defects
1) In almost every jurisdiction, a person will be deemed mentally
incompetent and lacking capacity to enter a contract if the
person is unable to understand in a reasonable manner
the nature and consequences of the transaction.
EXAMPLE: A person who is operating under the influ-
ence of delusions or hallucinations will be deemed men-
tally incompetent based on cognitive defects.
(b) Volitional Defects
1) In many jurisdictions, mental incompetence can be
established if:
a) a person is unable to act in a reasonable manner in
relation to the transaction; and
b) the other party has reason to know of this condition.
EXAMPLE: A person who is a manic depressive
would be able to avoid a contract if he was in a
manic phase at the time of contract formation and if
the other party had reason to know of that person’s
state based, for example, on his erratic behavior.

258
CONTRACTS

c. Legal Consequences
(1) In most jurisdictions, a contract entered into by an incompetent
person is voidable. This means that the contract is enforceable at
the option of the incompetent party, rather than void, which would
make it unenforceable by either party.
(a) Some jurisdictions, as well as the Second Restatement, hold that
a person adjudicated incompetent and under guardianship has no
capacity to incur contractual duties, making such contracts void.
(2) A party who is mentally incompetent at the time of contract
may ratify the contract if he becomes competent at a later time.
Ratification may be accomplished by an oral or written manifesta-
tion of the intention to be bound by the original contract.
(3) If the mentally incompetent person exercises his power of avoid-
ance and has received some benefit under the contract, then he
is required to make the other party whole by paying the reason-
able value of the goods or services rendered.
(a) A mentally incompetent party will be responsible for damage,
wear and tear, or any other depreciation in value of the goods.
EXCEPTION: If the other party takes unfair advantage of
the mentally incompetent person, and has knowledge of the
person’s incompetence, then the other party is entitled only
to a return “as is” of any goods still in the possession of the
mentally incompetent party and will have no right of recovery
for goods or services already consumed.
(4) The rules governing the liability of a mentally incompetent party
for contracts for necessaries are the same as for minors. The
mentally incompetent party will be liable to the other party for the
reasonable value of those goods and services.
(a) For mentally incompetent persons, necessaries include the
retention of legal services in connection with adjudicating
mental incompetence.

B. Misrepresentation
1. Misrepresentation can play two roles in contract cases: it may be a “sword,”
as the basis for affirmative relief, or a “shield,” a defense to enforcement.
a. Misrepresentation can be used as a basis for affirmative relief under
a tort claim brought by the aggrieved party in the form of an action for
rescission and damages.
b. Misrepresentation can be used as a defense to an effort to enforce
the contract.
2. Misrepresentations are untrue statements or assertions that relate to
existing facts.
a. Misrepresentations relate to existing facts, not to future conduct or
action. If one says, “I’m going to do something,” and doesn’t do it,

259
OUTLINE

that’s a broken promise, not a misrepresentation. But, if one says,


“I’m going to do X,” and at the time he makes that promise he actually
doesn’t intend to do X, that may be actionable as promissory fraud.
b. Misrepresentations must relate to facts, not opinions. A statement that
is based on a party’s mere opinion, guess, or supposition will not be
considered a misrepresentation.
(1) Where one party disguises a fact as an opinion, then this will
constitute a misrepresentation.
EXAMPLE: An automobile dealer who tells a buyer that he thinks
the car runs pretty well, even though he knows that the car does
not run well, would be disguising a fact as an opinion, and thus
be making a misrepresentation to the buyer.
(2) Where a party holds himself out to have knowledge or special
skill, and asserts an opinion on the basis of the skill or knowl-
edge, then his assertions are held to relate to the underlying facts
and, therefore, sufficient to establish misrepresentation.
EXAMPLE: A certified mechanic who tells a buyer that he thinks
the car runs pretty well, even though he knows that the car does
not run well, would, in effect, be asserting facts about the state of
the car and, thus, making a misrepresentation to the buyer.
3. Types of Misrepresentation
a. Fraudulent Misrepresentation
(1) To establish fraudulent misrepresentation, the following must be proved:
(a) the defendant must have made an assertion that was incon-
sistent with existing facts;
1) Most claims of fraudulent misrepresentation are based
on oral or written statements.
a) Statements that are “half-truths” can be misrepre-
sentations as well.
EXAMPLE: An advertisement for real estate “suit-
able for commercial development,” where applica-
ble zoning regulations would prohibit such use.
2) Misrepresentations can also be made through conduct.
EXAMPLE: When a payor issues a check for a speci-
fied amount despite insufficient funds, he is representing
that he has funds on deposit to cover the instrument.
a) Active efforts to “cover up” the truth (termed
fraudulent concealment) likewise constitute
misrepresentations.
EXAMPLE: A seller of real estate undertakes ef-
forts to hide termite damage from potential buyers.

260
CONTRACTS

(b) the state-of-mind element requires that there was both:


1) scienter; and
a) The scienter requirement relates to the defendant’s
state of mind with respect to the assertion. The defen-
dant will satisfy the scienter requirement if either:
i) he made the assertion knowing it to be false; or
ii) he made the assertion knowing that he had no
idea whether it was true or false.
EXAMPLE: Scienter would be present where the
auto dealer says, “This baby can go from zero to
70 in six seconds flat,” if he either knows that the
car can’t accelerate that quickly or has no basis for
knowing whether the car can accelerate that quickly.
2) intent to mislead.
a) The defendant will meet the requirement for
intent to mislead if:
i) he made the assertion for the purpose of
misleading the aggrieved party; or
ii) he made the assertion knowing there was a
substantial likelihood that the aggrieved party
would be misled.
(c) the misrepresentation must be material to the contract; and
1) Materiality can be shown by either an objective standard
or a subjective standard:
a) Under an objective standard, a misrepresentation is
material where such an assertion was likely to induce
a reasonable person to enter into the contract.
EXAMPLE: An assertion that a used car has been
carefully inspected by an independent auto me-
chanic would appeal to a reasonable person and,
thus, satisfy the objective materiality test.
b) A misrepresentation is material under a subjec-
tive standard if assertion of such a fact was likely,
for reasons known to the defendant, to induce the
aggrieved party specifically into entering the contract.
EXAMPLE: An assertion that the car was owned
by Salman Rushdie when he was living incognito in
the U.K. would be material if made to an individual
whom the seller knew to be a Rushdie enthusiast.
(d) there must have been reasonable reliance on the representation.
1) The standard for determining whether an aggrieved party’s
reliance on the defendant’s misrepresentation is reasonable

261
OUTLINE

is whether, under all of the same circumstances, a reason-


able person would have relied on the assertion.
EXAMPLE: Reliance would be unreasonable if the ag-
grieved party has independent knowledge or reason to
know that the perpetrator’s statement is false.
EXAMPLE: Reliance would be unreasonable if the aggrieved
party has reason to believe that the perpetrator is unreliable.
EXAMPLE: Reliance would be unreasonable if no rea-
sonable person would have believed the assertion.
EXAMPLE: Reliance would be unreasonable if the ag-
grieved party could have easily ascertained the truth by
cursory inspection of the goods.
2) Because of the “duty to read,” it is generally not reasonable
to rely on a trading partner’s characterization of a writing’s
content. However, two situations in which reliance on the
contents of a writing would be reasonable are:
a) if the defendant is a merchant and the aggrieved
party is a customer; or
b) if the defendant induces the aggrieved party not to
read the writing by some devious means.
EXAMPLE: The perpetrator induces the victim to
sign a contract that the perpetrator knows to con-
tain terms of which the victim is unaware and in
circumstances in which the perpetrator know the
victim won’t have time to read the contract.
b. Non-Fraudulent Misrepresentation
(1) Non-fraudulent misrepresentation may either be negligent or
innocent misrepresentation. For either claim, the following
elements must be established:
(a) the defendant must have made an assertion that was inconsis-
tent with existing facts;
(b) the misrepresentation must be material to the contract; and
(c) there must have been reasonable reliance on the misrepresentation.
(2) Because negligent or innocent misrepresentation is not intentional,
there is no need to make a showing of the defendant’s state of
mind. Instead, the following factors are required:
(a) For negligent misrepresentation, the aggrieved party must
show that the perpetrator would have known that the asser-
tion was false had he exercised reasonable care.
(b) For innocent misrepresentation, the aggrieved party need
only show that the perpetrator made an assertion not in
accord with existing fact.

262
CONTRACTS

(3) The same remedies are available to the victim of either negligent
or innocent misrepresentation, so the difference between them is
merely descriptive (i.e., what to call a particular misrepresenta-
tion) but has no further legal consequence.
c. Fraudulent Nondisclosure
(1) The wrong that must be committed under a claim of fraudulent
nondisclosure relates not to an assertion made by the defendant,
but to the defendant’s silence where there was a duty to disclose.
(2) To prove fraudulent nondisclosure, one must show that:
(a) the nondisclosure was material to the contract, under either
an objective standard or a subjective standard; and
(b) there was reasonable reliance on the nondisclosure.

NOTE Because fraudulent nondisclosure is based on the defendant’s silence rather


than an assertion, there is no requirement that the defendant make an assertion.

(3) Although there is generally no duty of disclosure to trading part-


ners, if a party is aware of material facts that are unlikely to be
discovered by the other party in the exercise of ordinary care and
diligence, then there will be a duty to disclose that information in
these circumstances:
(a) where the parties enjoy a relationship of trust and confidence;
EXAMPLE: A relationship between family members or a pro-
fessional’s relationship with a client would be a relationship
of trust and confidence.
(b) if a party has made an assertion that was true at the time but
has been rendered untrue by intervening events; or
EXAMPLE: If a person makes an assertion that an individual
plans to keep property in an undeveloped state and later
learns that the individual no longer intends to keep the prop-
erty undeveloped, he would need to disclose this change.
(c) if the obligation of good faith would require that the party
disclose the information.
EXAMPLE: A seller of residential real estate who fails to dis-
close a termite infestation to the buyer would have a good-
faith obligation to disclose the infestation to the buyer.
4. Remedies
a. Victims of fraudulent misrepresentation, non-fraudulent misrepresenta-
tion, and fraudulent nondisclosure may use those claims in two ways:
(1) avoidance; or
(a) All three claims give the victim the power of avoidance,
which may be exercised to defend against a breach of
contract claim brought by the misrepresenting party.

263
OUTLINE

(2) rescission and reliance damages.


(a) All three claims enable the victim to bring a tort action to
rescind the contract and collect damages for reliance on the
misrepresentation, including consequential damages.
b. Victims of fraudulent and non-fraudulent misrepresentation have a third
option: they may live with the contract and sue for the benefit of the
bargain.
(1) The victim of fraudulent misrepresentation may bring a tort action
seeking the benefit of the bargain. This would allow the victim to
recover the difference between the value of the goods or services
actually received and the value of the promised goods or services.
This tort remedy is the functional equivalent of expectation
damages, discussed later in this outline.
(2) This remedy of bringing a tort action for the benefit of the bargain is
not available for claims of negligent or innocent misrepresentation.
However, parties bringing such claims may be able to characterize
the assertion contrary to fact as a warranty and thus obtain expec-
tation damages by asserting breach thereof.
c. Because fraudulent misrepresentation is an intentional tort, the
aggrieved party may secure punitive damages against the defendant to
penalize the defendant on the basis of the fraudulent intention.
(1) Punitive damages are not available for claims of negligent or
innocent misrepresentation or fraudulent nondisclosure.

C. Duress
1. At common law, the defense of duress was available in two circumstances:
a. physical compulsion; and
(1) A party could claim duress as a defense to contract liability if the
party signed the contract under force of physical compulsion.
b. unlawful threat.
(1) A party could claim duress as a defense to contract liability if the
party signed the contract under threat of unlawful activity against
himself or his family.
2. Although the above two circumstances still constitute duress, the contem-
porary definition of the term has been relaxed to cover less egregious
instances of coercive persuasion. Thus, under modern law, there are three
elements to a defense of duress:
a. a threat;
(1) A threat made by the perpetrator is a manifestation of intent to
inflict harm on the other person, made in words or by conduct.
EXAMPLE: The words “your money or your life” would be consid-
ered a threat.
EXAMPLE: An example of conduct considered a threat is the scene

264
CONTRACTS

in “The Godfather” where the man wakes up with a horse’s head in


bed, which was placed there to induce him to sign the contract.
b. that is wrongful in nature; and
(1) A threat is wrongful in nature if it involves criminal or tortious
conduct, whether the threat is itself unlawful (e.g., extortion or
blackmail) or whether it is merely a threat to engage in future
unlawful action (e.g., causing harm to a party’s person, family
member, or property).
(2) A threat to pursue criminal charges constitutes a wrongful threat,
even if the perpetrator honestly believes that the target is guilty
of that crime.
(3) A threat to bring a civil action is wrongful only if made in bad
faith—that is, if the perpetrator does not honestly and reasonably
believe that the target has civil liability.
(4) A threat is sufficient to constitute duress if what is threatened is a bad
faith breach of contract. This is commonly known as economic duress.
(a) If a party refuses to perform unless he is awarded additional
benefits, this may constitute a bad faith breach of contract.
1) Economic duress will usually arise where the threatened
party acquiesces to the other party’s demands at the
time but then refuses to confer the additional benefits.
EXAMPLE: Sailors on a fishing voyage cease their work
midway through the voyage and refuse to resume their
duties unless the captain agrees to give them a raise.
At the end of the voyage, the captain refuses to pay the
higher amount, and the sailors sue.
(b) Not every demand for a change in contract price will be consid-
ered a bad faith demand. If the demand for a change in contract
terms is due to the fact that performance under the terms of the
contract has become extremely burdensome because of unan-
ticipated circumstances, then there is no duress.
EXAMPLE: The sailors’ demand for additional compensation
in response to an unanticipated expansion of their workload
due to faulty fishing nets would be a good-faith demand.
c. that leaves the aggrieved party with no reasonable choice but to
succumb to the threat.
(1) The third element of duress is that the wrongful threat must leave
the aggrieved party with the absence of reasonable choice except
to agree to the contract or modification on which the perpetrator is
insisting. This is seldom an issue when what is threatened is a crime
or a tort, a criminal prosecution, or a bad faith civil suit, for risking the
eventuality of any of those is scarcely a reasonable choice.
(2) Hard cases often arise in the context of economic duress (i.e.,

265
OUTLINE

threatened breach of contract). Three common situations where


this will arise are:
(a) when there are no adequate and reasonably priced substi-
tutes for the services or goods the perpetrator is threat-
ening to withhold;
EXAMPLE: The ship’s captain is not in a position to find re-
placement sailors when already at sea, and so no adequate
substitutes would be available.
(b) when the threatened breach would cause the aggrieved
party to break his own contracts, especially if there is a pros-
pect of substantial consequential or liquidated damages; or
EXAMPLE: If the purchaser of upsidaisium bearings needed
for use in the manufacture of turbo-jet engines would, on ac-
count of the breach, be forced to breach its contract with an
aircraft manufacturer.
(c) when the alternative of acquiescing to the threat and then
suing for damages is inadequate to redress substantial
harms to the aggrieved party.
EXAMPLE: Where the threatened breach would cause the
aggrieved party to renege on its own commitments and thus
harm its reputation and opportunities for future business,
damages would be inadequate as a remedy.

NOTE A minority of jurisdictions add another element to the requirements for economic
duress: the aggrieved party must protest the threatened breach rather than acqui-
esce without complaint in the demand for additional payment or other benefits with
the secret intention of refusing to make the newly promised payment at contract’s
end. The courts that impose this additional requirement hold that the obligation of
good faith requires the aggrieved party to speak up at the time of the threat.

3. Third-Party Duress
a. If the wrongful threat is made by a third party, rather than the other party to
the contract, the aggrieved party will still have a valid claim of duress unless:
(1) the other party gives value or relies materially on the transaction; and
(2) the other party is proceeding in good faith without reason to know
of the duress.
4. Remedies
a. Contracts made under physical compulsion are void.
b. Contracts entered into under other forms of duress are voidable at the
option of the aggrieved party.
c. The aggrieved party is:
(1) entitled to restitution of any benefits conferred under duress; and
(2) required to return excess value of benefits to the perpetrator.

266
CONTRACTS

5. The common law defense of duress can be used in cases arising under the
UCC as a supplementary provision.

D. Undue Influence
1. An aggrieved party may avail himself of the defense of undue influence
when the circumstances reveal a vulnerable, but not incapacitated, party
who succumbs to untoward bargaining tactics and pressures from the other
party, where those pressures or tactics fall short of fraud or duress.
2. Elements
a. There are two elements to the defense of undue influence:
(1) unfair persuasion was used; and
(a) Although the test for unfair persuasion is one that looks at all
of the circumstances, courts have looked in particular to the
following indicia of untoward pressure:
1) discussion of the transaction at an unusual or
inappropriate time;
2) consummation of the transaction at an unusual place;
3) insistent demands that the transaction or business be
finished immediately;
4) extreme emphasis on the untoward consequences of
delaying the transaction;
5) the use of multiple persuaders against the target of
persuasion;
6) absence of third-party advisers to the target of persuasion; and
7) statements that there is no time to consult financial
advisers or attorneys.
(2) the other party was vulnerable to such persuasion.
(a) A vulnerable party can be established in any of the following ways:
1) where the mental infirmity is due to age or illness but
falls short of mental incompetence;
2) where the vulnerability is due to some recent trauma
or event; or
3) where the vulnerable party is reliant on the other party
because of some relationship of trust or confidence with
the other party.
3. Remedies
a. Contractual obligations assumed under undue influence are voidable
at the option of the aggrieved party.
b. The aggrieved party is entitled to restitution of any benefits conferred
on the other party, but must return the value in excess of those benefits
to the other party.
c. Where the vulnerable party suffers undue influence at the hands of a third
party rather than the other party to the contract, the aggrieved party has the
power to avoid the contract at his option, unless two conditions are met:

267
OUTLINE

(1) the other party gives value or relies materially on the transaction; and
(2) the other party is proceeding in good faith without reason to know
of the undue influence.
4. Like the other defenses, the common law claim of undue influence is avail-
able in cases arising under the UCC as a supplementary provision.

E. Unconscionability
1. Just as with undue influence, the defense of unconscionability may be
available when a party uses inappropriate bargaining tactics to take unfair
advantage of a vulnerable party. Unconscionability, however, focuses not
only on the unfair process but also on the unfair results.
2. Elements
a. There are two elements to a defense of unconscionability. In most
jurisdictions, both of the following elements must be proved in order to
succeed on a defense of unconscionability. However, where there is
an extremely strong showing of one element, the party may be able to
succeed on a defense of unconscionability despite a weaker showing
of the other element. The elements are:
(1) procedural unconscionability; and
(a) This element can be met where the bargaining process
that produced the contract in question created an absence
of meaningful choice for the aggrieved party. The following
are circumstances that may establish the absence of a
meaningful choice:
1) near-miss cases;
EXAMPLE: When a party is vulnerable to the pressure
of the other party, such as because of language barri-
ers or advanced age, but the vulnerability falls short of
mental incompetence or duress.
EXAMPLE: When the victim is subjected to misleading or
coercive sales tactics that fall short of actual fraud or duress.
2) absence of bargaining power; and
EXAMPLE: When the party is forced to accept pro-
posed terms, as may be the case in standard form con-
tracts with “take it or leave it” provisions which sellers
and suppliers impose on all customers and that cannot
be varied by bargaining, greatly limiting the other party’s
power to bargain.
EXAMPLE: Where one party is weakened by poverty or
language barriers, which greatly limits his ability to shop for
alternative terms particularly with respect to necessaries.
3) fine-print terms.

268
CONTRACTS

EXAMPLE: Where the contract included complex or


arcane terms hidden in the maze of the fine print in a
lengthy standard form contract.
(2) substantive unconscionability.
(a) The second element of unconscionability is met when the contract
terms are unreasonably unfavorable to the aggrieved party.
(b) Typical examples include:
1) grossly excessive price;
EXAMPLE: Where a pay-over-time plan requires the con-
sumer to pay a total sum that is many times the value of the
purchased goods, or where a bank charges an overdraft
fee that is many times the bank’s actual processing costs.
2) grossly disproportional consequences for a minor breach;
EXAMPLE: An add-on clause, pursuant to which a merchant
is entitled to repossess multiple household items, including
furniture, bedding, and a stereo, when a consumer missed a
payment, despite the fact that the consumer had already paid
nearly 80% of the monies owed for the various purchases.
3) provisions binding one party but not the other; and
EXAMPLE: An arbitration provision in an employment con-
tract that requires the employee but not the employer to ar-
bitrate any disputes, or that permits the employer but not the
employee to amend the provisions of the agreement at will.
4) provisions which are grossly unfair.
EXAMPLE: In Brower v. Gateway 2000 [676 N.Y.S.2d
569 (N.Y. App. Div. 1998)], the court found unconsciona-
ble a binding arbitration clause that required consumers
pursuing disputes with a merchant to pay a nonrefund-
able advance fee that exceeded the purchase price of
the goods, travel to a distant city for the proceeding, pay
the merchant’s legal fees in the event of a loss, and file
all correspondence in a foreign country.
3. Remedies
a. Upon a finding of unconscionability, a court may:
(1) refuse to enforce the contract;
(2) excise the offending clause and enforce the remainder of the contract; or
(3) limit the application of the offending clause as to avoid any
unconscionable result.
4. Application
a. The question of whether or not unconscionability is present is a ques-
tion of law to be decided by the courts.

269
OUTLINE

b. The question of whether a particular contract or clause is unconscio-


nable is to be determined by the circumstances existing at the time of
contract formation, not at the time the dispute arose.
c. A party claiming or resisting a claim of unconscionability is entitled to
present evidence of the allegedly offending provision’s commercial
setting, as well as its purpose and effect.
(1) To refuse a party the opportunity to present such evidence before
granting summary judgment is a reversible error.
5. Unconscionability under the UCC
a. The defense of unconscionability is specifically recognized as a
defense under the UCC, and applies in the same manner as under the
common law [§ 2-302].

F. Public Policy
1. The last of the defenses is called the public policy defense. It is a claim
that courts should not enforce a contract because doing so would violate or
undermine some important public policy.
2. Public policy may be raised as a defense to the enforcement of a contract
in the following four contexts:
a. where the subject matter of the contract itself is specifically prohib-
ited by law;
EXAMPLE: In most jurisdictions, a contract for prostitution, gambling, or
bribery is illegal under the law and, accordingly, unenforceable in court.
b. where a contract is formed for the purpose of committing a crime or
violating a legal regulation;
EXAMPLE: A contract between an employer and a hired assassin
would be a contract for the commission of a crime.
c. where the contract performance would not constitute a crime, but
would constitute a tort; or
EXAMPLE: A publicist being hired for the express purpose of spread-
ing a defamatory story about a private citizen would be a contract for
the commission of a tort.
d. where the contract performance would violate certain values and free-
doms designated by the state or jurisdiction.
EXAMPLE: A contract that prohibits one party from marrying for an ex-
tended period of time would violate the public policy of promoting free
and consent-based marriages.
3. Sources of Public Policies
a. Legislation
(1) Legislation is frequently the source of the “policy” invoked under
the public policy defense.

270
CONTRACTS

EXAMPLE: Cases involving contracts prohibited by statutory law


and cases involving contracts for the commission of a crime or
the violation of a legal regulation.
b. Judicial Decision
(1) Where the subject matter of the contract is not specifically prohib-
ited under law, then the public policy defense may operate to:
(a) fill gaps where there is no direct legislative warrant on point; and
(b) in most jurisdictions, promote larger notions of the public good.
(2) Judicially developed public policies include:
(a) policies grounded on moral and social values, such as the
policy against the impairment of family relationships;
(b) policies based on economic considerations, such as the
policy against restraints of trade; and
(c) policies designed to protect governmental processes and
institutions, such as the policy of protecting the integrity of
voting and elections.
EXAMPLE: A contract under which a private citizen agrees
to vote a certain way in exchange for a year’s supply of beer
would violate the public policy of free and unrestricted exer-
cise of the right to vote.
4. Operation of the Defense of Public Policy
a. The public policy defense is almost invariably invoked as a defense
in an action by one of the parties seeking enforcement of a contract
against the other. When it is successful, the defendant will win irre-
spective of whether he was the party who promised to perform the
public policy violation or the party paying for it.
b. The idea here is not that the courts have any special solicitude for
defendants, but rather that courts will refuse to play any role what-
soever in the enforcement of contracts violating public policy, with
the result that the party attempting to enforce the contract will be
out of luck.
EXCEPTION: An exception to the public policy defense may exist
where one of the parties is more egregiously “in the wrong” than the
other party. In the language of the courts, in such cases the parties are
not in pari delicto (“equally at fault”), and the more innocent party may
be able to secure restitution of any benefits conferred on the guiltier
party despite the fact that the contract itself remains unenforceable.
(1) The in pari delicto defense is most commonly available when a
particular statute has been enacted to protect a particular class of
persons or actions.
EXAMPLE: A debtor may be entitled to restitution of insurance
premiums, where the creditor coerced the debtor to purchase

271
OUTLINE

credit insurance and there is a statute on point that is designed to


protect debtors from such creditor coercion.
c. A contract that is subject to the defense of public policy is not itself
void as a matter of public policy. The defense only operates to treat the
contract as voidable at the option of the defending party.
5. Contracts Frequently Falling under the Public Policy Defense
a. Noncompetition Agreements
(1) Many employment contracts, as well as contracts for the sale
of a business, contain a provision that prevents one party from
competing for a certain period of time or in a certain area. Where
this provision is reasonable, the court will generally enforce it.
(a) However, where the noncompete provision imposes an
unreasonable geographical barrier, duration, or term, then
the enforcement of such a provision may be considered
a violation of the public policy of promoting a citizen’s
freedom to work.
(b) Under the blue pencil rule, most jurisdictions remove
the offending portion of the noncompetition clause while
enforcing the remainder of it.
EXAMPLE: A court may enforce only the first two years of a
five-year noncompete clause.
(c) A minority of jurisdictions will hold that the inclusion of such a
provision invalidates the entire noncompetition clause.
b. Sales of Goods via Bribery
(1) If a contract for the sale of goods was based on bribery by any
party, then the public policy defense could be used by the victim
of the bribe to make the contract unenforceable.
EXAMPLE: Buyer sues Seller for breach of contract when Seller
refuses to supply the specified quantity to Buyer at the agreed-
upon price. Seller raises the defense of public policy because
after the fact, Seller learns that Buyer bribed Seller’s agent with a
car in order to secure the unreasonably favorable contract terms.
c. Sales of Goods Intended for Unlawful Use
(1) If a contract is for the sale of goods that the seller knows the
buyer intends for an unlawful use, then the public policy defense
would be available to defeat an action either by the seller seeking
payment or by the buyer seeking delivery.
d. Liability-Limiting Provisions
(1) When the provisions of a contract would limit a party’s liability
for tortious behavior by restricting the right of the injured party to
pursue claims against the reckless or intentional harms caused
by the party, the provision would not be upheld, because to do so
would violate public policy.

272
CONTRACTS

(a) Provisions that limit liability for negligent (rather than reckless
or intentional) tortious conduct are generally permissible.
e. Unlicensed Goods or Services
(1) When a contract is for unlicensed goods or services, then the contract
may be rendered unenforceable by the public policy defense.
EXAMPLE: An unlicensed contractor could be prevented from
recovering payment for his services if the homeowner invokes a
public policy defense.
EXCEPTION: Because the homeowner is not in pari delicto with
the unlicensed contractor, an action by the homeowner against the
contractor for shoddy work might survive the public policy defense.

273
OUTLINE

VIII. REMEDIES

A. Monetary Remedies for Breach of Contract


1. Monetary Damages at Common Law
a. A party aggrieved by a breach of contract may be able to recover
money damages calculated to protect that party’s expectation interest,
reliance interest, or restitutionary interest. A party may elect only one of
these three remedies.
(1) To compensate a party for his expectation interest, the court
will calculate money damages designed to put the aggrieved
party where he would be if the other party had fully and properly
fulfilled the contract terms.
EXAMPLE: A patient with a hand injury contracts with his doc-
tor for hand surgery. The doctor promises him a “100% perfect”
hand, but instead, the surgery makes the hand worse. The
expectation interest of the patient is measured by the difference
between the value of the promised “100% perfect” hand and the
value of the hand worsened by surgery.
(2) To protect an aggrieved party’s reliance interest, a court will
calculate the money damages designed to put the aggrieved
party in the position he was before the contract was made.
EXAMPLE: A agrees to buy B’s car. On the faith of the promise, A
pays B a $2,000 down payment for the car and enters a 12-month
lease agreement with C for a parking space for $2,400. The reli-
ance interest of A would be $4,400, the amount it would take to
compensate A for what he’s lost on the faith of B’s promise.
(3) To protect an aggrieved party’s restitutionary interest, a court
will calculate money damages designed to return to the aggrieved
party the value of the benefit he conferred on the breaching party.
b. Expectation Damages
(1) The default rule for the proper measurement of damages in
breach of contract cases is the expectation interest of the
aggrieved party. This means the aggrieved party will be entitled
to the amount that will restore him to the position he would have
been in had the contract been fully performed.
(2) The formula for expectation damages is:

Loss of value of the breaching party’s performance


+ any incidental and consequential costs generated by the breach
- any payments received from the breaching party
- any costs saved as a result of the breach
= Expectation Damages of the Aggrieved Party

274
CONTRACTS

EXAMPLE: General Contractor repudiates its contract with Subcon-


tractor when Subcontractor is halfway finished with its work under
the subcontract. Subcontractor is entitled to seek from the breach-
ing General Contractor an amount equal to: the contract price or the
unrealized value of General Contractor’s promised performance plus
costs associated with storing, insuring, and/or returning materials
and equipment secured by Subcontractor in the course of contrac-
tual performance (i.e., incidental costs generated by the breach),
minus any progress payments already made by General Contractor
and minus money Subcontractor may have saved on salaries and
equipment rental by not having to complete performance.
(3) Limitations on Recovery for Expectation Damages
(a) The aggrieved party may not be able to recover the full
amount of expectation damages in the following situations:
1) where the cost of performance greatly exceeds the
market value of the performance;
a) Cost of performance is the cost that would be incurred
in an effort to perform as promised under the contract.
b) Market value of performance is the net increase
in the market value of finding substitute perfor-
mance of the contract.
EXAMPLE: In the infamous case of Peevyhouse
v. Garland Coal & Mining Co. [382 P.2d 109 (Okla.
1962)], landowners entered into a contract with a
coal-mining firm permitting the latter to strip-mine their
farmland but requiring the firm to restore the land to its
original state once the mining project was complete.
When the firm breached by refusing to do the restora-
tion work, there were two ways to measure the result-
ing expectation damages: the cost of performance
(i.e., the costs that would be incurred in an effort to re-
store the land as promised in the contract), or the net
increase in the market value of the property that would
be realized if the restorative work had been completed
as promised. Because restoration would have cost the
mining firm $29,000 but increased the market value of
the land by only $300, the court limited the landown-
ers’ recovery to the latter figure. This was based on the
determination that the restoration work was incidental
to the main purpose of the contract (which was a lease
to strip-mine the land for coal), the main purpose had
been fully performed, and the cost of performance
would have been disproportionately costly.
c) Many courts would award cost of performance
despite the disparity with market value on the view

275
OUTLINE

that expectation damages should be calculated, in


the words of the Restatement Second, “on the basis
of the value of performance to the injured party
himself and not on its value to some hypothetical
reasonable person or on some market.”
d) Where the breach of the contract is willful and the perfor-
mance was not incidental to the agreement, damages
may be measured by cost of completion despite it
being disproportionate to the value of the performance.
EXAMPLE: The plaintiff operated a manufacturing
plant which, after decades of operation, the plaintiff
decided to close. The plaintiff agreed to convey the
structures and equipment to the defendant in ex-
change for $275,000 and the defendant’s agreement
to remove the equipment, demolish the structures,
and grade the property as specified. The defendant
made the cash payment, removed the equipment,
and demolished the structures, but did not grade the
property as required by the contract. The plaintiff sued
for $110,500, constituting the cost of performance.
The defendant argued that damages should be limit-
ed to $3,000, representing the difference between the
fair market value of the property and the price that the
plaintiffs sold the property for. The court determined
that the performance was part of the main purpose of
the contract (rather than incidental), the defendant’s
breach was willful, and the appropriate measure of
damages was the cost of performance, despite the
disproportionate difference [American Standard, Inc.
v. Schectman, 80 A.D.2d 318 (1981)].
2) where expectation damages cannot be calculated with
reasonable certainty;
a) While an aggrieved party does not have to calcu-
late with mathematical precision to receive expec-
tation damages, there must be some reasonable
basis for calculation.
b) Doubts in calculations will be resolved in favor of
the aggrieved party.
c) Requiring reasonable certainty of expectation
damages may leave the aggrieved party uncom-
pensated for losses that cannot be easily valued.
EXAMPLE: A new business is unable to open
because of a breach by a key supplier who has
not completed promised work. Because there is

276
CONTRACTS

no proven track record for the new business, it is


difficult to establish with any certainty what profits
would have been realized had it opened on time.
d) Some jurisdictions apply the “new business” rule
and automatically bar recovery, but most jurisdic-
tions will evaluate the facts on a case-by-case
basis and award expectation damages if a reason-
able calculation is possible (e.g., on the basis of
profit records for similarly situated ventures).
3) where damages are unforeseeable; and
a) Under the Hadley rule, a breaching party will be liable
for general damages (those damages that naturally
flow from the breach) but not for special or consequen-
tial damages, which are those damages that result
from the particular circumstances of the aggrieved
party unless, at the time of contracting, the breaching
party knew or had reason to know that the consequen-
tial damages would result from breach. A common type
of consequential damages is lost profits.
b) The Hadley rule operates as another limitation on
the right of an aggrieved party to recover expecta-
tion damages by denying recovery to the aggrieved
party for unforeseeable consequential damages, even
though those damages would not have occurred if the
breaching party had fully performed on the contract.
c) The rule barring recovery for unforeseeable conse-
quential damages is a default rule, and the parties
may opt out of its application to their contract and
instead allow liability for all consequential damages
whether foreseeable or unforeseeable.
4) where damages can be mitigated.
a) The aggrieved party may not recover for any losses
it could have avoided without unreasonable risk,
burden, or humiliation.
b) The aggrieved party’s duty to mitigate damages is
limited to taking reasonable efforts to mitigate. If such
reasonable mitigation efforts fail or cause additional
expense, then the breaching party will be fully liable.
EXAMPLE: The liability of a breaching buyer of
produce would be reduced if the seller permitted
the produce to rot after breach rather than selling
it to third parties; but if the seller is forced by the
breach into a “fire sale” because of the short shelf
life of the produce in question, then the breaching

277
OUTLINE

buyer will be fully liable for expectation damages—


that is, for any difference between what the seller
nets from the “fire sale” and what it stood to earn
but for the broken contract.
EXAMPLE: The liability of an employer is reduced
if an employee dismissed in breach of contract
takes an extended vacation rather than seeking
other work. But the aggrieved employee need
only make reasonable efforts to secure a position
that is reasonably equivalent to the job lost and,
accordingly, she need not accept substitute work
when it: is in a different field; offers significantly
lower pay or less desirable terms and conditions
of employment than those of the lost job; would
entail more burdensome responsibilities than
those of the lost job; or would damage the ag-
grieved party’s career prospects (e.g., an A-list
movie star would not be required to mitigate dam-
ages for a film producer’s breach of contract by
taking a lesser role in a B-grade film).
EXAMPLE: The liability of a breaching landowner
to a construction contractor hired to erect a struc-
ture on the property is reduced where the contrac-
tor continues the building project after the landown-
er has repudiated the contract. But the liability of
a buyer of goods ordered for special manufacture
would not be reduced if the seller continues the
manufacturing process after breach in order to fin-
ish the goods for possible sale to third parties.
c. Reliance Damages
(1) Reliance damages may be available where expectation damages
are not available.
(2) The measurement of damages for the reliance interest of an
aggrieved party is calculated by determining the amount of
money necessary to restore the aggrieved party to the position
he was in prior to the contract.
(a) The reliance interest is compensated by the value lost to
the breaching party, as well as any other value lost from the
aggrieved party’s reliance on the breaching party.
(3) The aggrieved party’s reliance interest is measured by any
expenditures made in preparation for performance or in actually
performing, less any loss which the breaching party can prove,
with reasonable certainty, that the aggrieved party would have
suffered even if the contract had been fully performed.

278
CONTRACTS

(4) The most common use of reliance damages arises when the
expectation damages would be uncertain or speculative, as
where the breach would deprive the aggrieved party of opportuni-
ties to enter potential transactions with third parties.
EXAMPLE: Inventor signs a contract with Railroad Co. for shipment
of a new stove Inventor plans to display at a manufacturers’ conven-
tion, and he pays fees to the convention hotel for a room and for dis-
play space, as well as an exhibition fee to the convention sponsor. If
Railroad Co. breaches the contract by failing to deliver the stove until
after the convention, Inventor will be unable to recover damages for
opportunities he may have lost due to his inability to showcase the
stove, as calculating the probability and value of those opportunities
would be too speculative. However, Inventor would be entitled to reli-
ance damages (i.e., a return of Railroad Co.’s fee, plus compensa-
tion for the amounts he paid to the hotel and convention sponsor).
d. Restitutionary Damages
(1) A party aggrieved by a breach of contract is entitled to restitu-
tionary damages rather than expectation damages at his option.
If chosen, the aggrieved party may choose to recover the value
of the benefits conferred on the breaching party by the aggrieved
party during the course of the contract.
(a) Restitutionary damages are available to both the breaching
and aggrieved parties.
(2) The aggrieved party’s restitutionary interest will be measured by either:
(a) the reasonable value of the benefit conferred upon the
breaching party; or
1) This is measured by the market value of the service(s)
rendered, and not the price established by the parties in
the contract.
2) The contract price may be admissible as evidence in
calculating the restitutionary damages, as evidence of
the price of retaining those services on the market.
EXAMPLE: Where General Contractor dismisses Sub-
contractor halfway through Subcontractor’s performance,
Subcontractor’s restitutionary interest is measured by
the market value of the services rendered thus far. The
contract price may be admitted as relevant evidence for
determining the market value for these services.
(b) the extent to which the breaching party’s property has increased
in value based upon the aggrieved party’s performance.
EXAMPLE: Contractor built a Turkish bathhouse on Home-
owner’s property. The market value of the services is the cost to
Homeowner of hiring another contractor to do the work, which

279
OUTLINE

equals $30,000. The value added to Homeowner’s property is the


net increase in the home’s value from the addition of the bath-
house, which equals $20,000. A court calculating Contractor’s
restitutionary interest could award him either amount, consider-
ing what relevant precedents permit and what justice requires.
(3) An aggrieved party is likely to make this election if the restitutionary
recovery would exceed the amount recoverable based on his
expectation interest, and that situation is most likely to arise in the
context of a “losing contract” (i.e., where the expectation interest
would be less than zero because the aggrieved party would have
actually lost money had the other party not breached).
EXAMPLE: General contractor breaches its contract with Subcon-
tractor when Subcontractor is halfway finished with its work under
the subcontract. Subcontractor is entitled to seek from the breach-
ing General either expectation damages or restitutionary damages
(i.e., the market value of the services already rendered at the time
of breach). Even if Subcontractor had a “losing contract,” it is nev-
ertheless entitled to recovery of its restitutionary interest.
(4) In order to secure restitutionary damages, the party seeking the
remedy must have conferred some benefit on the other party
through either part performance or reliance.
(a) If the aggrieved party has fully performed under the contract,
then the aggrieved party is limited to expectation damages.
EXAMPLE: Subcontractor completes contractual perfor-
mance and General Contractor refuses to pay. Subcontractor
has no right to seek restitutionary damages and is entitled to
recovery of the contract price only.
e. Liquidated Damages Provisions
(1) Parties are free to include among the terms of their contract a
liquidated damages clause designed to provide for damages of
their own choosing in the event of breach.
(2) Such a provision is enforceable if the court finds it to be a valid
liquidated damages clause, and unenforceable if the court finds
that it constitutes a penalty.
(3) The basic idea here is that the parties are free to establish a
regime of “home-made” damages designed to reflect the likely
consequences of a breach in a manner suited to the peculiarities
of their transaction, but that the parties are not free to threaten to
punish a breach with harsh measures designed to give a poten-
tially breaching party no choice but to perform.
(a) A basic axiom of American contract law is that a party has a
right either to perform on a contract or to pay damages, and
a penalty clause is seen as interfering with that right.

280
CONTRACTS

(4) The test for determining whether a clause in a particular contract


is a valid liquidated damages provision has three prongs:
(a) Did the parties intend for the clause to operate as a liqui-
dated damages clause or as a penalty?
1) Evidence of specific or camouflaged effort to punish
breach would support a claim that the clause was
intended as a penalty.
2) Modern decisions downplay the importance of this
prong and emphasize the other two.
(b) Was the clause reasonable at the time of contracting in rela-
tion to the anticipated harm?
1) The key question is whether there was an anticipated
harm that would be difficult to prove or for which no
adequate remedy would be available.
EXAMPLE: Repudiation of a commercial lease by an “an-
chor” tenant in a shopping mall is likely to cause reduc-
tions in pedestrian traffic to other stores as well as harm
to the mall’s reputation and attractiveness as a shopping
“destination.” Because the extent and monetary value of
those harms would be difficult to establish in court, a liqui-
dated damages clause designed to award the mall owner
a sum that represents an inexact but reasonable forecast
of those harms would be appropriate.
(c) Was the clause reasonable in relation to the harm and
losses that actually occurred due to the breach?
EXAMPLE: If the shopping mall tenant repudiates the
contract shortly after entering it, and the mall owner is
able to secure a substitute “anchor” with minimal delay, a
clause guaranteeing the owner significant liquidated dam-
ages would be unreasonable in relation to the actual harms
caused by the breach.
(5) Anticipated versus Actual Harm
(a) When a particular clause satisfies the second prong of the
test (it was reasonable at the time of contracting in relation
to anticipated harms) but not the third prong of the test (it
wasn’t reasonable in relation to the harms actually suffered),
some courts will uphold the challenged clause as a valid
liquidation of damages with the view that a clause that satis-
fies either one of the prongs will be valid.
1) Those courts would uphold the clause in the example
immediately above because it was reasonable in relation
to anticipated harms even though it wasn’t reasonable in
relation to the actual harms caused by the breach.

281
OUTLINE

(b) Other courts, however, will conclude that the clause is a


penalty if it specifies damages that are “grossly dispropor-
tionate” to the actual harm, even if the clause was reason-
able at the time of contracting.
1) Those courts would declare the clause in the example
immediately above to be a penalty, because it was
excessive in relation to the actual harms, despite the
fact that it was reasonable in relation to the harms
anticipated at the time of contracting.
(6) The party attempting to show that the liquidated damages clause
is actually an unenforceable penalty has the burden of proof.
(a) If the party fails to offer evidence of the unreasonableness
of the provision, then the liquidated damages provision will
be upheld.
(7) If the court concludes that the liquidated damages clause is, in
fact, a penalty, then the clause will be stricken from the contract.
(a) If the clause is stricken from the contract, the aggrieved party
is still entitled to recover whatever legal or equitable relief it
would be entitled to under law.
2. Monetary Damages under the UCC
a. Most of the rules governing remedies under the UCC reflect the same
underlying principles that apply in common law cases—in particular,
the right of an aggrieved party to expectation damages and the duty of
an aggrieved party to mitigate damages—but they differ in their details
because they are specifically designed for application to the sales of
goods context.
b. Seller’s Remedies
(1) Upon a buyer’s breach of a contract for the sale of goods, the
seller is free to cancel the contract and to withhold delivery of any
yet to be delivered goods. In addition, the seller may have a right
to recover money damages from the breaching buyer.
(2) The nature of the seller’s right to recover depends on whether the
goods have been delivered.
(a) Action for the Price
1) If some or all of the goods have been delivered and
accepted, the seller is entitled to collect the contract
price for those goods [§ 2-709].
(b) Damages for Nonacceptance or Repudiation
1) If some or all of the goods have not been delivered—
either because the buyer has rejected them, or in the
context of anticipatory repudiation—the seller can
recover damages with respect to them.
2) The measure of recovery will depend on whether the
seller resells the goods to a third party.

282
CONTRACTS

a) Seller Resells
i) If the seller resells, he can recover the
contract-resale differential—that is, the
difference between the contract price and the
resale price [§ 2-706].
ii) In order for the seller to recover, the resale
must be made in good faith and in a commer-
cially reasonable manner.
EXAMPLE: A seller may not make a “sweet-
heart” deal to sell the goods at a below-market
discount to a friend or a relative.
iii)
The seller is not accountable to the buyer for
any profit made on any resale [§ 2-706(6)].
b) Seller Does Not Resell
i) If the seller does not resell, he can recover
the contract-market differential—that is, the
difference between the market value of the
goods at the time and place of the promised
delivery and the contract price [§ 2-708].
3) Whether or Not the Seller Resells
a) Whether or not the seller resells, he is also entitled
to recover incidental damages—that is, the costs
associated with getting stuck with goods the seller
thought he had sold as well as the costs of resale.
EXAMPLE: Incidental costs associated with the
seller remaining in possession of the goods include
such things as storage and insurance. Inciden-
tal costs associated with the resale of the goods
include costs incurred in the form of new advertise-
ments and sales commissions.
b) However, the seller’s damages will be reduced by
an amount reflecting expenses avoided on account
of the breach.
EXAMPLE: If the seller had expected to pay ship-
ping costs in connection with the original contract,
and instead incurs substantial savings by reselling to
a local buyer, the seller’s recovery will be reduced.
(3) Lost Profits for Lost Volume Sellers
(a) In the ordinary case, a seller’s profit is built into the contract
price and, accordingly, either the contract-market differen-
tial (if there is no resale) or the contract-resale differential
(if there is a resale) will fully protect the seller’s right to that
profit in the event of breach by the buyer.

283
OUTLINE

EXAMPLE: If Seller contracts to sell a boat to Buyer, Buyer


subsequently reneges, and Seller’s resale results in a lower
price, Seller will get his anticipated profit by requiring the
breaching Buyer to pay the difference between the contract
price and the resale price.
(b) A different problem arises if the seller is a volume seller (i.e.,
if, instead of having a single object to sell, it makes repeated
sales of the same goods, such as widgets). When a partic-
ular buyer breaches in this context, the seller has a “lost
volume” problem—even if he sells the particular widget he
expected to sell to the breaching buyer to another customer;
but for the breach, at the end of the day, the seller would
have made and profited from two sales rather than only one.
1) A lost volume seller is one whose supply of goods
exceeds the demand for the same; that is, the seller can
satisfy all potential buyers who may seek to deal with him.
EXAMPLE: BigBox, an electronics store, purchases from
the manufacturer a particular brand of new televisions for
$200, and resells them for $400. Buyer visits the store and,
after shopping around, decides to purchase one of these
new televisions as a gift for a friend. Buyer requests that the
television be gift-wrapped, and the parties agree that Buyer
will pay for the television two hours later, when he returns
to pick up the television after it has been gift-wrapped.
Buyer never returns. BigBox, the near the end of the day,
sells a television of the same model to another purchaser
for the same price. But for Buyer’s breach, BigBox would
have sold two televisions that day, rather than one.
(c) The UCC accordingly permits lost volume sellers to recover the
profit they would have made on the lost sale rather than relegating
them to either the contract-market or contract-resale differential.
(d) To recover lost profits, the seller must be able to show:
1) that he could have made the sale to both the breaching
buyer and resale buyer;
2) that it would have been profitable for the seller to make
both sales; and
3) that he probably would have made the additional sale to
the resale buyer even absent the buyer’s breach.
c. Buyer’s Remedies
(1) Upon a seller’s breach of a contract for the sale of goods, a buyer
can either recover damages or seek specific performance.
(2) There are two ways to measure a buyer’s damages under the
UCC, and the key factor is whether the buyer has covered
(purchased replacement goods).

284
CONTRACTS

(a) If Buyer Covers


1) If the buyer covers, his damage measure is the contract-
cover differential—that is, the difference between what
the buyer would have paid under the contract and what
he actually paid to secure cover [§ 2-712(2)].
2) Cover must be made “in good faith and without unrea-
sonable delay” [§ 2-712(1)].
EXAMPLE: An aggrieved buyer cannot purchase sub-
stitute widgets from his brother-in-law at premium prices
nor wait around until the market price of widgets has
skyrocketed and purchase then.
(b) If Buyer Does Not Cover
1) If the buyer does not cover, he is entitled to the
contract-market differential—that is, the difference
between what the buyer would have paid under the
contract and the market price of the goods at the time
the buyer learned of the breach [§ 2-713].
(c) Whether or Not Buyer Covers
1) Whether or not the buyer covers, he is also free to seek
incidental damages—the costs associated with securing
cover—and consequential damages, as discussed
earlier [§§ 2-712, 2-713].
2) However, the buyer’s damages will be reduced by an
amount reflecting expenses avoided because of the breach.
(3) Difference in Value Damages
(a) This measure of damages available under the UCC is similar
to the common law’s expectation damages. It is available
if the buyer receives nonconforming goods from the seller.
The buyer is entitled to recover damages for nonconforming
goods based on the following:
(b) The difference in value damages does not require a showing
of foreseeability—only that the goods did not conform to the
goods specified in the contract.

Value of Goods Contracted For


– Value of Goods Received
= Buyer’s Recovery under Difference in Value Damages

(4) Deduction of Damages from Price Still Due


(a) A buyer may deduct all or any part of the damages resulting
from the seller’s breach of contract from any part of the
price still due under the same contract. In order to make
the deduction, the buyer must give notice to the seller of his
intention to withhold all or part of the price [§ 2-717, cmt. 2].

285
OUTLINE

B. Equitable Remedies
1. The typical remedy for contract claims is to award monetary damages.
However, where an award of money damages would be inadequate to compen-
sate the aggrieved party, a court may instead award an equitable remedy.
2. The most common equitable remedies in contracts cases are specific perfor-
mance (which requires the breaching party to take some particular action),
negative injunctions (which prohibit the breaching party from a particular
action), and rescission (which amounts to a cancellation of the contract).
3. Specific Performance
a. Specific performance is an extraordinary remedy that is available to
order a breaching party’s performance only where a monetary award
would be inadequate to grant relief to the aggrieved party.
b. As a practical matter, specific performance is a remedy for paying,
rather than performing, parties (e.g., buyers who pay money for goods,
purchasers who pay money for real estate, and parties who pay money
for services rendered).
(1) When the paying party breaches, the remedy of money damages
will, in the ordinary case, give the performing party the promised
money, and so the availability of specific performance is a nonissue.
(2) When the performing party breaches, however, the paying party
may prefer to secure the promised performance itself rather than
money damages; this raises the question of whether specific
performance is available.
c. Cases Where Specific Performance is Generally Available
(1) At common law, money damages are presumed to be inadequate
when a party is purchasing either:
(a) unique objects; or
EXAMPLE: Works of art and precious heirlooms.
(b) real property.
EXAMPLE: At common law, Blackacre was presumed to be
“unique,” and such factors as location, natural resources, cli-
mate, and permissible uses justify that characterization even
in contemporary real estate markets.
(2) Equity Considerations
(a) Specific performance is an equitable remedy, and a decision
to grant or deny it is, accordingly, a decision for the court and
not the jury.
(b) Whether to grant or deny specific performance is committed
to the court’s remedial discretion based on the competing
equities in particular cases, and is not bound by hard-and-
fast rules (such as those governing money damages).
(c) The court’s determination is likely to take into account some
or all of the following factors:

286
CONTRACTS

1) whether the aggrieved party has clean hands (has


dealt fairly and in good faith with the breaching party);
2) whether the terms of the contract in question are suffi-
ciently definite;
a) Because a decree of specific performance is
enforceable via contempt, courts are reluctant
to issue an order that doesn’t give the parties
adequate guidance as to the conduct required.
3) whether performance by the aggrieved party can be
reasonably assured;
a) For example, performance could be assured by
requiring tender of the aggrieved party’s perfor-
mance as a condition of the breaching party’s
obligations under the decree.
4) whether the terms of the contract are fair; and
a) A court may deny specific performance where the
underlying exchange is unfair even if the unfairness
falls short of unconscionability or other doctrines
that would excuse contractual performance alto-
gether by the breaching party.
b) In some jurisdictions, a party seeking specific
performance of a contract for real property must
plead and prove adequate consideration, despite
the general rule that courts don’t police the
adequacy of contractual consideration.
5) whether specific performance would be in the public interest.
a) Specific performance may be denied, for example,
where the decree might have anticompetitive results.
d. Contracts for Which Specific Performance is Not Available
(1) Contracts for Personal Services
(a) Specific performance is not available to require a breaching
employee or contractor to perform. Specific performance in
such a case would violate the public policy prohibiting invol-
untary servitude, and it would be difficult to enforce.
(2) Contracts Requiring Ongoing Cooperation between Parties
(a) Specific performance is likewise disfavored in the context contracts
that are not capable of immediate enforcement. Contracts that
require a continuing series or acts or continuing cooperation
between the parties for successful performance would present
courts ordering specific performance with daunting problems of
supervision similar to those in the personal services context.
e. Sale of Goods Contracts under the UCC
(1) The UCC liberalizes the rules governing the availability of specific
performance for sales of goods in two ways:

287
OUTLINE

(a) Specific performance may be permitted if the goods are


unique or in “other proper circumstances” [§ 2-716(1)].
1) Thus, the subject matter of a contract does not have to be
unique in order to obtain specific performance; if the buyer has
adequately searched but is unable to cover the breach, “other
proper circumstances” may be present [§ 2-716, cmt. 2].
(b) Specific performance is available even where ongoing coop-
eration would be required between the parties (e.g., in output
or requirements contracts) so long as the requisite inability of
a party to cover can be established.
(2) Replevin
(a) The buyer also has a right of replevin—which is an
action to repossess property—for goods identified in the
contract if [§ 2-716(3)]:
1) after reasonable effort, the buyer is unable to cover;
2) the circumstances reasonably indicate that an effort to
cover will be unavailing; or
3) if the goods have been shipped under reservation (i.e.,
the seller has reserved a security interest in the goods)
and satisfaction of the security interest in them has been
made or tendered.
4. Negative Injunctions
a. Negative injunctions are orders by the court prohibiting the breaching
party from taking particular action. The limitations that apply to specific
performance also apply to negative injunctions. Most commonly, nega-
tive injunctions are used to prevent employees from going to work for a
competitor or competing with their former employer.
b. The availability depends on whether the former employer is seeking
mid-term or post-employment relief.
(1) Midterm Relief
(a) When an employee is under contract for a specified period of time
and the employee breaches the contract by departing before the
end of that period, a negative injunction will be available to prevent
the employee from competing directly or indirectly with his former
employer if the employee’s services are unique or extraordinary.
(b) Most of the cases finding that services meet this test involve
professional athletes and entertainers.
1) Although the presence of a specific contractual provision
establishing exclusive employment during the period of
time in question will aid the employer in securing a nega-
tive injunction, most courts will imply such a term for the
period of employment specified in the contract.
EXAMPLE: In the famous case of Lumley v. Wagner, an
opera singer under contract to sing at Her Majesty’s Theatre

288
CONTRACTS

for a three-month period was persuaded to depart mid-con-


tract to begin a concert series at a competing venue. Be-
cause her services were unique and extraordinary, the court
granted a negative injunction barring her from performing at
any competing venue for the duration of the contract term.
(2) Post-Employment Relief
(a) Enforcement of employment contract provisions that prohibit
post-employment competition against the employer have
become a common source of negative injunctions.
(b) The validity of such noncompetition clauses will depend on
three factors:
1) most courts require a significant business justification
for enforcing post-employment restraints;
a) An employer’s need to protect confidential informa-
tion would qualify, but some courts will allow little or
no justification for enforcing such a provision.
2) most jurisdictions limit the scope of the enforcement of
noncompetes to reasonable duration and geographical
restrictions; and
a) However, the advent of the information age has
prompted courts to broaden the definition of a
reasonable geographical restraint, but at the same
time, to lessen the amount of time considered to be
a reasonable duration of restraint.
3) most courts will issue a negative injunction against a
competing employee only if the employment contract
contains an express noncompetition agreement, but
some courts have granted relief in the absence of such
a provision based on the tort theory of inevitable disclo-
sure of trade secrets.
5. Rescission
a. Rescission is an equitable remedy that cancels the contract (and
forms the basis for restitution). Both parties are placed back where
they were before the contract was executed.
b. Rescission of a contract is available:
(1) by consent of both parties;
(2) for mistake (either unilateral or mutual);
(3) for fraud, misrepresentations, and nondisclosures;
(4) for duress or undue influence;
(5) for illegality; or
(6) for failure of consideration (which would also be a material breach).
c. A party seeking rescission must be ready to return to the other party
all benefits received.

289
OUTLINE

C. Other Possible Remedies


1. Promissory Estoppel
a. The type of interest a party may recover under a claim of promissory
estoppel depends on the jurisdiction in which it brings suit. Courts
are split:
(1) some courts award expectation damages;
(2) other courts, instead, favor reliance damages; and
(3) other courts award damages on a case-by-case basis and tailor
the remedy to the injustice at issue, where possible.
(a) In doing so, the court will focus on:
1) the strength of the proof for each of the individual
elements of the claim;
2) the blameworthiness of the breach;
3) the extent of the detrimental reliance by the aggrieved party;
4) the relative positions of the parties; and
5) the availability of alternative options to granting full
enforcement of the promise.
2. Restitution and Unjust Enrichment
a. A party who bestows benefits on another may seek to recover the
value of those benefits in an action for restitution. A party aggrieved by
a breach of contract may have the option of electing a restitutionary,
rather than expectation, recovery. However, there are many other
contexts in contract law in which a party may be entitled to restitution
pursuant to the policy of avoiding unjust enrichment.
b. The following are contexts where restitution might be available:
(1) Benefits Conferred under a Failed Contract
(a) When a party bestows benefits on his trading partner in
connection with what turns out to be a “failed” contract
(e.g., unenforceable because of the Statute of Frauds or a
failed condition, or voidable because of incapacity, fraud,
duress, undue influence, or unconscionability), the party
bestowing the benefits may ordinarily recover their value
via restitution, subject to offset by any benefits received
from the other party.
EXAMPLE: A enters an oral contract to sell land to B, and
prior to closing, B makes improvements on the land. If A re-
neges, B may be unable to enforce the contract because of
the Statute of Frauds. However, B is entitled to recover the
value of the improvements via an action for restitution.
(2) Benefits Conferred by a Breaching Party
(a) At common law, the breaching party was not entitled to receive
any recovery for the benefits it may have conferred under the
contract. The modern rule is that such a recovery is possible.

290
CONTRACTS

1) A breaching party may recover the amount of benefit he


conferred on the aggrieved party less any damages to
the aggrieved party as a result of the breach (otherwise
known as the right of offset).
EXAMPLE: A works for B under a 12-month contract
and, without advance notice, breaches the contract
after 11 months in order to accept other employment.
A is entitled to restitution of the reasonable value of
the services rendered, subject to offset for B’s dam-
ages—that is, the cost of hiring a replacement for A at
the last minute.
(3) Emergency Benefits Conferred by a Health Care Professional
(a) The general rule is that a person who bestows benefits
without request from the benefiting party is considered a
volunteer or officious intermeddler who will not be entitled
to any recovery.
(b) However, an exception applies to doctors and other health care
professionals who provide emergency health care to a patient
who is unable to consent. These health care professionals are
entitled to the reasonable value of those services they have
rendered, even where their efforts were unsuccessful.

NOTE This exception does not apply to non-medical persons whose life-saving ef-
forts are deemed “gratuitous.”

(4) Benefits Conferred by Mistake


(a) A party who mistakenly confers benefits to another
party may be entitled to restitution. In this situation, the
following will be considered:
1) the blameworthiness of the error;
2) whether the recipient was aware of the error in time to
prevent it; and
3) whether the recipient availed himself of the benefits at issue.
EXAMPLE: After a hurricane, Owner hires Contractor
to perform repairs on his storm-damaged home. Be-
cause of confusion caused by damage to street signs
and mailboxes, Contractor performs the repairs on the
wrong house. Contractor is entitled to restitution from
the benefiting homeowner if the latter was aware of
Contractor’s mistaken efforts and remained silent.
3. Agreed-To Remedies
a. Parties to a contract may also contract out of the legal and equitable
remedies available under the law, by specifying agreed-to remedies
in the contract.

291
OUTLINE

b. These remedies typically take two forms: liquidated damages provi-


sions and provisions limiting or excluding damages.
c. Provisions Limiting or Excluding Damages
(1) The parties to a contract may limit or exclude the availability of
certain damages. Thus, parties can limit their remedial rights to
those provided for in the contract or exclude rights to remedies
that it would otherwise be entitled to under law.
(2) There are different types of exclusive remedies provisions, including:
(a) provisions that limit or alter the measure of damages
available; and
EXAMPLE: A provision limiting or excluding consequen-
tial damages.
(b) exclusive remedies provisions, in which the parties expressly
agree that only one remedy, or certain specified remedies,
will be available in the event of a breach.
EXAMPLE: A provision that limits the remedy available to the
buyer for replacement goods or a “do-over” of services.
(3) Such provisions are enforceable unless they are unconscionable.
Under the UCC, limitation of consequential damages for personal
injury in the case of consumer goods is prima facie unconscio-
nable [§ 2-719(3)].

292
CONTRACTS

IX. THIRD-PARTY BENEFICIARIES

A. In General
1. In a typical contract, the parties promise performances to each other. If one
party refuses to perform, the other has standing to bring a claim for breach
because the parties are in contractual privity. In some contracts, however,
one of the parties promises a performance that will benefit a third party (a
third-party beneficiary).
EXAMPLE: In exchange for B’s promise of payment, A promises to deliver
widgets to C. C is the third-party beneficiary of the contract between A and B.
2. The critical issue in third-party beneficiary law is the circumstances under
which the third-party beneficiary has standing to enforce the contract
against the promisor.
a. Under common law, courts traditionally held that a third-party benefi-
ciary did not have standing to sue the promisor because there was no
contractual privity between them; however, under contemporary law, the
privity requirement has been relaxed and third-party beneficiaries may
have standing to sue the promisor for breach, even though the promise
was made to the promisee and not to the third-party beneficiary.

B. Classification of the Third-Party Beneficiary


1. The right of an aggrieved third-party beneficiary to bring an action
for breach of contract against a breaching promisor or promisee will
depend upon the classification of the beneficiary. The First and Second
Restatement each classifies beneficiaries differently.
a. While the two Restatements use different terminology, the practical
effect is the same under both nomenclatures.
2. First Restatement
a. The First Restatement placed third-party beneficiaries into one of
three categories:
(1) when a promisee seeks a performance from the promisor that will
benefit a third party, and the promisee’s purpose is to satisfy a
debt or other obligation owed by the promisee to the third party,
the third-party beneficiary is a creditor beneficiary;
EXAMPLE: A owes B $100 and enters a contract with C to white-
wash C’s fence in exchange for C’s promise to pay B the $100. B
is a creditor beneficiary because A’s purpose in contracting with C
is to satisfy his debt to B.
(2) when a promisee seeks a performance from the promisor that will
benefit a third party, and the promisee’s purpose is to make a gift
of that performance to the third party, the third-party beneficiary is
a donee beneficiary; and

293
OUTLINE

EXAMPLE: Desiring to make a graduation gift to A, B agrees to


pay C $100 in exchange for C’s promise to whitewash A’s fence.
A is a donee beneficiary because B’s purpose in contracting with
C is to make a gift to A.
(3) third parties who will benefit from a promisor’s performance as
a practical matter, but who may not meet the test for creditor
or donee beneficiaries under the First Restatement are inci-
dental beneficiaries.
EXAMPLE: A enters a contract with B whereby B is to erect a
house on A’s land. C owns land next to A, and the value of C’s
land will be enhanced by the building. C is an incidental benefi-
ciary of B’s promise to construct the house, and C has no rights
under the contract.
3. Second Restatement
a. The Second Restatement eliminates the “creditor” and “donee” benefi-
ciary terminology, and instead puts third-party beneficiaries into one of two
possible categories: intended beneficiaries and incidental beneficiaries.
b. Intended beneficiaries broadly fall into the same two categories
utilized by the First Restatement, where the promised performance will
satisfy an obligation of the promisee to pay money to the beneficiary
(creditor beneficiaries) and where the promisee intends to give the
beneficiary the benefit of the promised performance (donee beneficia-
ries). Many courts continue to use the older terms, however.
(1) The key factor in determining whether a party is an intended
beneficiary is whether the promised performance is intended
to benefit the third-party beneficiary. Courts are divided over
whether the test for determining intent to benefit requires a
mutual intention of the parties (the Second Restatement test)
or merely a reasonably apparent purpose of the promisee (the
First Restatement test).
EXAMPLE: A hires Attorney B to draft A’s will in accordance with
A’s instructions, which include a wish to make A’s stepdaughter,
C, a residual heir. Attorney B breaches the contract by neglecting
to include the requested provision, and upon A’s death, C brings a
third-party beneficiary action against Attorney B for his breach. The
intention of A to benefit C was “reasonably apparent” to Attorney B
and, accordingly, would satisfy the “intent to benefit” test under the
First Restatement. But because the benefit to C was a matter of
indifference to Attorney B, there would be no “intent to benefit” her
under the Second Restatement “intention of the parties” test.
c. Incidental beneficiaries, as under the First Restatement, are third
parties who will benefit from a promisor’s performance as a practical
matter, but who may not meet the test for intended beneficiaries under
the Second Restatement.

294
CONTRACTS

C. Rights of Parties to Enforce the Contract


1. Third-Party Beneficiary’s Rights
a. Under both the First and Second Restatement, an incidental benefi-
ciary does not enjoy any right to seek enforcement of the contract from
either the promisor or promisee to the original agreement.
b. Third-Party’s Rights against the Promisor
(1) Any third-party beneficiary, other than an incidental beneficiary,
has a right to secure enforcement of the agreement from a
breaching promisor.
(a) This applies to creditor and donee beneficiaries under the
First Restatement and intended beneficiaries under the
Second Restatement.
c. Third-Party’s Rights against the Promisee
(1) Although a third-party beneficiary has rights against the promisor in
connection with the promised performance, he has no rights against
the promisee in the event that the performance is not forthcoming.
EXAMPLE: Desiring to make a graduation gift to A, B agrees to
pay C $100 in exchange for C’s promise to whitewash A’s fence.
If C fails to whitewash A’s fence, A has no recourse against B.
(2) Instead, a third-party beneficiary will only have rights against the
promisee resulting from the promisor’s failure to perform based
on whether or not there is an independent obligation between the
promisee and the third-party beneficiary.
(a) Thus, a First Restatement creditor beneficiary can elect to
sue either the promisee on the prior obligation between the
beneficiary and the promisee or the promisor on the third-
party beneficiary contract. However, satisfaction of the claim
by one operates as a release of the other’s obligation.
(b) Similarly, a Second Restatement intended beneficiary can
only sue the promisee if the beneficiary would have fallen into
the First Restatement’s definition of a creditor beneficiary.
That is, the intended beneficiary can only sue the promisor
on the third-party beneficiary contract or the promisee if the
promisee owes a separate obligation to the beneficiary, which
the third-party beneficiary contract was intended to satisfy.
EXAMPLE: A owes B $100 and enters into a contract with C
to whitewash C’s fence in exchange for C’s promise to pay
B the $100. In the event of nonpayment, B can elect to sue
either A on the preexisting obligation or C as a third-party
beneficiary, but payment of the $100 by either A or C will re-
lease the other from any further performance obligation to B.
(3) The promisee’s preexisting obligations to a creditor or intended
beneficiary are not discharged by the formation of a third-party

295
OUTLINE

beneficiary contract under which the promisor promises to fulfill


the promisee’s obligation to the beneficiary.
EXAMPLE: A owes B $100 and enters a contract with C to white-
wash C’s fence in exchange for C’s promise to pay B the $100.
C’s obligation to pay B the $100 does not discharge A’s preexist-
ing obligation to B.
d. Vesting of the Right to Sue
(1) A third-party beneficiary does not automatically have the right to
seek enforcement of the contract, or prevent the contract from
being modified by the original parties, purely by virtue of the third-
party beneficiary contract having been made.
(2) The parties to a contract are free to modify or rescind it by mutual
consent, and they may modify or rescind a third-party beneficiary
provision without the beneficiary’s consent unless and until the
beneficiary’s rights under the contract have vested.
(a) Once the third-party beneficiary’s rights vest, the ability of
the original parties to make any future rescissions or modifi-
cations will be terminated.
(b) If the third-party beneficiary’s rights under the contract vest
before he is notified of any modification or rescission of the
contract, the modification or rescission will be ineffective.
(3) Vesting occurs with regard to an intended beneficiary when:
(a) the beneficiary brings suit on the matter;
EXAMPLE: A contracts with B to pay B $2,000 in exchange
for B doing renovations to C’s house. B begins to perform,
but ceases midway and informs C that he does not intend to
complete the work. A and B agree to modify the original con-
tract to instead have B do the renovation work on A’s house,
thereby eliminating C from the arrangement. Before being
notified of the modification, C sues B to enforce the original
contract based on B’s anticipatory repudiation. C’s filing suit
vests his rights in the agreement, and A and B’s modification
will be ineffective.
(b) the beneficiary changes his position in justifiable reliance on
the contractual promise;
EXAMPLE: As a birthday gift for his son B, A hires Contrac-
tor C to renovate B’s home and advises B of the “gift” at a
family party. In reliance on his father’s promise, B lets an
option expire on a new home he and his wife were intending
to purchase. A and Contractor C can no longer modify B’s
rights under the contract.
(c) the beneficiary manifests his assent to the contract at the
request of either the promisee or the promisor; or

296
CONTRACTS

EXAMPLE: A owes B $100 and agrees to whitewash C’s


fence in exchange for C’s promise to pay B the $100. A asks
B to assent to the contract; once B does so, A and C can no
longer modify B’s rights.
(d) the rights of the beneficiary have vested under an express
term of the contract providing for such vesting.
EXAMPLE: Under the terms of his life insurance policy with
Hartford Insurance, A’s designations of beneficiaries become
irrevocable upon A’s demise or adjudication of incompetence.
The rights of C, a designated beneficiary, against Hartford
Insurance will vest upon the occurrence of either event.
e. Defenses Available to Promisor
(1) Because a third-party beneficiary’s rights are entirely dependent on
the underlying contract, any valid defenses the reneging promisor
would have to the enforcement of the contract, such as impracti-
cability, the failure of a condition, or a breach of the contract by the
promisee, would also be effective against the beneficiary.
(2) The promisor may not assert defenses based on separate transac-
tions with the promisee, such as a right to setoff arising in connec-
tion with a deal unrelated to the contract at issue, unless the contract
expressly subjects the rights of the beneficiary to such a claim.
2. Promisee’s Rights against the Promisor
a. When the promisor does not perform, the promisee has a claim for
breach of contract against the promisor.
b. If the promisor’s performance is intended to benefit a donee benefi-
ciary, the promisee ordinarily will not have suffered any economic loss
for the nonperformance and, therefore, may be unable to recover more
than nominal damages. As a result, however, some courts consider the
damage remedy inadequate and will order specific performance.
EXAMPLE: Desiring to make a donation to her alma mater, Laurie
Lawyer promised to provide legal services to Nellie Merchant, a seller
of computer hardware, in exchange for Merchant’s promise to provide
100 free laptops to Nutmeg Law School. If Merchant fails to deliver the
promised laptops, Lawyer can sue her for breach of contract, but be-
cause Lawyer has suffered no economic loss on account of the breach,
she may be able to secure specific performance.
c. If the promisor’s performance is intended to benefit a creditor beneficiary,
the promisee may secure specific performance of the promisor’s obligation.
(1) A claim for money damages would, however, expose the prom-
isor to the possibility of double liability, since the promisor is also
liable to the third-party beneficiary. Accordingly, some courts
refuse to allow the promisee to recover damages against the
promisor unless the promisee has already made payment to the
beneficiary to cover the default.

297
OUTLINE

X. ASSIGNMENT OF RIGHTS AND DELEGATION OF DUTIES

A. Assignment of Rights
1. An assignment is a transfer of a right to receive a performance under a contract.
2. Assignments typically have three relevant parties in a basic factual
scenario: A and B have a valid contract, and B subsequently assigns his
rights under the contract to C. In this scenario:
a. A is the obligor (the party with the obligation to perform);
b. B is the assignor (the party who assigned the right); and
c. C is the assignee (the party to whom the right was assigned).
EXAMPLE: A and B have a contract whereby B agrees to provide services
to A in exchange for A’s payment. B’s right to A’s payment may be assigned
to C, a third party. Upon assignment, A has a payment obligation to C. In
this transaction, B is the assignor, who assigned the right to receive pay-
ment; C is the assignee, to whom that right was assigned; A is the obligor,
the party with the obligation to perform (here, to make the payment).
3. To make an effective assignment of a contract right [Restatement (2d) of
Contracts §§ 324, 327]:
a. the owner must manifest an intention to make a present transfer of
the right without further action by the owner or the obligor; and
(1) There are no “magic words”—not even an express reference to
“assignment”—required to make a manifestation effective.
(2) Oral assignments are effective unless there is some independent
applicable writing requirement, such as an assignment of a right
to real property under a contract of sale, which must be in writing
under the Statute of Frauds.
(3) A promise to transfer a currently existing right at a future date is
not an effective assignment.
(4) A promise to transfer a right that the assignor expects to acquire
in the future is not an effective assignment.
EXAMPLE: The statement “I will assign you 10% of the royalties
when I obtain them” is not an assignment, as it expresses only
an intent to assign in the future.
b. a manifestation of assent by the assignee is essential, unless:
(1) a third person gives consideration for the assignment; or
(2) the assignment is irrevocable by virtue of the delivery of a
writing to a third person.
(a) An assignee who has not manifested assent to an assignment
may, within a reasonable time after learning of its existence and
terms, render it inoperative from the beginning by disclaimer.
EXAMPLE: A contracts to buy $200 worth of widgets from B.
B delivers the widgets, but requests that A pay C the $200 as

298
CONTRACTS

a gift, without C’s knowledge. When C learns of the gift, he


refuses it. The assignment is ineffective due to C’s disclaimer.

NOTE An “order” by an obligee directing an obligor to pay the debt to a third party is
not an assignment. The most common example of an order is a check, which,
under both common law and the UCC, does not operate as an assignment of
the drawer’s rights against the bank.

4. Partial Assignments
a. A partial assignment is valid. Hence, there can be an assignment of a
fraction of the assignor’s rights.
EXAMPLE: If A owes B $10,000 for goods B delivered to A, B may as-
sign his right to $5,000 of that money to C.
b. When suit is brought, however, all of the parties owning rights after the
assignment must be joined in the action, unless joinder is not feasible
and it is equitable to proceed without joinder.
5. Rights to Be Assigned
a. The general rule is that all rights are assignable, subject to the
following exceptions:
(1) a right is not assignable if the assignment would materially alter
the risks to or obligations of the other party to the contract;
EXAMPLE: A and B are parties to a requirements contract under the
terms of which A is obligated to supply B with B’s monthly require-
ments for widgets. B’s rights under the contract are not assignable to
C if C’s monthly requirements would greatly exceed B’s.
(2) a right is not assignable if the obligor has a personal interest in rendering
the performance in question to the obligee and not to a third party;
EXAMPLE: Nanny A’s obligation under a contract to provide child-
care services to B’s family is not assignable to another family.
(3) a right is not assignable if assignment would violate applicable
law or public policy; and
EXAMPLE: A state statute prohibiting assignment of wages.
(4) a right is not assignable if assignment is prohibited by the
contract. (Note that such provisions are strictly construed in both
scope and effect.)
(a) Although there is older precedent invalidating an assignment
when the contract expressly states that attempted assign-
ments are “void,” most courts will treat an assignment in
violation of contractual restriction as a breach of contract
by the assignor—making him liable for any damages to the
obligor—but not as a basis for nullifying the obligor’s perfor-
mance obligation to the assignee.

299
OUTLINE

(b) Absent contractual language or circumstances suggesting


a contrary intention of the parties, a contractual prohibition
against “assignment of the contract”:
1) will bar a delegation of contractual duties but not an
assignment of contractual rights;
2) will not apply to:
a) rights that accrue to the assignor as a result of a
breach of contract by the obligor; or
EXAMPLE: If A agrees to sell and B agrees to buy
widgets under a contract prohibiting assignment,
and A fails to deliver the promised widgets, giving
rise to a claim for breach by B, B can assign his
rights upon breach to C.
b) rights that accrue to the assignor upon complete
performance by the assignor; and
EXAMPLE: If A agrees to sell and B agrees to buy
widgets under the terms of a contract prohibiting
assignment, and A delivers the promised widgets, A
can assign his rights to B’s payment to C.
3) can be invoked by the obligor (to resist making perfor-
mance to the assignee) but not by the assignor (in an
action brought by the assignee).

NOTE An offer cannot be assigned, although an option can be.

EXAMPLE: If A has agreed to sell land to B and B has


paid consideration to keep the offer open for six months,
B can assign the option to C.

EXAM TIP The right of a partner to share in the management of the partnership is not assign-
able, but the right to profits and to the partner’s share in a dissolution is assignable.
6. Assignment for Value Contrasted with Gratuitous Assignment
a. An assignment for value, where the assignee acquires the assign-
or’s contractual right(s) in exchange for payment or a promise thereof
to the assignor, is valid against the obligor and cannot be revoked by
the assignor, although it may be modified like any other contract via
mutual consent of the parties.
b. A gratuitous assignment, where the assignor assigns contractual
rights to the assignee without consideration for such transfer, has the
following legal effects:
(1) A gratuitous assignment is valid and binding against the obligor,
who cannot raise the absence of consideration between the
assignor and the assignee as a defense to any breach of his
obligations to the assignee.

300
CONTRACTS

(2) Between the assignor and the assignee, the law of gifts governs
the question of whether the assignor can revoke the assignment.
A gift requires that there be donative intent and delivery.
(a) Because the assignment is a transfer of legal rights under
a contract, it is impossible to physically deliver an intan-
gible right. However, delivery of something representative or
symbolic of an intangible right, such as a savings account
book, is held to be sufficient to meet the delivery requirement.
(3) Even if there is no delivery, the assignment becomes irrevocable
once payment of the obligation is made to the assignee.
(4) The assignor will be estopped from revoking the assignment if the
assignee acts to his detriment in reliance upon the assignment.
7. Rights and Obligations of the Parties after Assignment
a. Rights of Assignee against the Obligor
(1) The basic rule is that:
(a) an assignee gets whatever rights to the contract his
assignor had; and
(b) the assignee takes subject to whatever defenses the obligor
could have raised against the assignor, such as a lack of
consideration, incapacity, fraud, duress, or mistake.
(2) Payment to Assignor
(a) If the obligor pays the assignor, this defense can be raised
against the assignee, provided that payment was made
before notice of the assignment was given to the obligor.
(b) Once notice of the assignment is given, payment to the
assignor is no defense.
(c) If the obligor doubts whether the assignment was made, he can
pay the money into court and interplead the assignor and assignee.
(3) Setoffs and Counterclaims
(a) If the obligor has a right of setoff that could be raised against
the assignor, such right can always be raised against the
assignee if the alleged setoff arises out of the same transaction.
(b) If, however, the setoff arises out of a separate transaction, it
is available against the assignee only if the transaction which
gave rise to the setoff arose before notice of the assignment
was given to the obligor.
(c) The obligor and the assignor may agree to an adjustment of
their rights without consent of the assignee up until the time that
the assignee has given notice of the assignment to the obligor.
(4) Waiver of Defenses
(a) If one of the original parties to the contract agrees that he will not
raise defenses against an assignee in the event that the rights
are assigned, the agreement is enforceable with two limitations:

301
OUTLINE

1) defenses that are in the nature of “real defenses” under


Article 2 of the UCC can still be raised; and
a) These defenses include infancy, other incapacity
that voids a contract, fraud in the execution, duress
(when it removes the parties’ capacity to contract),
discharge in bankruptcy, and any other discharge of
which the assignee has reason to know.
2) the agreement not to raise defenses is invalid if the
obligor who signed the waiver was the buyer or lessee
of consumer goods.
b. Rights of Assignee against Assignor
(1) Unless a contrary intention is manifested, one who assigns or
purports to make an assignment for value impliedly warrants to
the assignee:
(a) that he will do nothing to defeat or impair the value of the
assignment and has no knowledge of any fact that would do so;
(b) that the right as assigned actually exists and is not subject to
any limitations or defenses against the assignor other than
those stated or apparent at the time of the assignment; and
(c) that any writing evidencing the rights that are being delivered to
the assignee to induce him to accept the assignment is genuine.

NOTE An assignment does not, in and of itself, operate as a warranty that the obli-
gor is solvent or that the obligor will perform his obligation.

c. Rights among Successive Assignees


(1) A problem of rights among successive assignees may arise when
the assignor, who is owed money by the obligor, assigns his right
to the money to the first assignee, and then later assigns the
same right to a second assignee. The assignor is liable to both
assignees for assigning the same right twice.
(2) However, if the assignor is bankrupt or has fled the jurisdic-
tion and both assignees attempt to collect from the obligor, the
majority rule is that the first assignee prevails.
(a) There are various exceptions to the majority rule. In several
jurisdictions, and under the Second Restatement, a subsequent
assignee who has paid value and took the assignment in good
faith will prevail if he [Restatement (Second) of Contracts § 342]:
1) obtains payment from the obligor;
2) recovers a judgment on the debt;
3) enters into a new contract with the obligor; or
4) receives delivery of a tangible token or writing from
the assignor, the surrender of which is required by the
obligor’s contract.

302
CONTRACTS

B. Delegation of Duties
1. A delegation occurs when a third party agrees to satisfy a performance
obligation owed by one of the parties to a contract.
2. Delegations typically have three relevant parties in a basic factual scenario:
A and B have a valid contract, and B subsequently delegates duties under
the contract to C. In this scenario:
a. A is the obligee (the party for whom the performance obligation is owed);
b. B is the obligor (the party with a performance obligation), and is also the
delegator (the party who delegated his performance to a third party); and
c. C is the delegatee (the party to whom the performance obligation
was delegated).
3. Rights of the Obligee against the Delegator
a. A delegation does not relieve the delegator from his obligations under
the contract. Despite the delegator’s delegation to the delegatee of
the performance obligation owed to the obligee, the delegator remains
liable for the performance.
(1) Thus, while an assignment of rights effectuates a transfer of
those rights to a third party (upon assignment, the assignee and
not the assignor has rights under the original contract), a delega-
tion of duties does not operate as a “transfer” of those duties from
the delegator to the delegatee.
b. Novation
(1) If there is a novation, the delegator is relieved from the obligations
under the contract. This requires a clear promise by the obligee to
release the delegator in return for the liability of the delegatee.
(2) Simple assent to the delegation is not enough to effectuate a
novation; there must also be a promise to release the delegator.
4. Liability of the Delegatee
a. When the delegatee has agreed to perform the delegator’s contract
obligations, he is liable to the delegator if he does not do so.
b. Under the third-party beneficiary theory, the delegatee is also liable
to the obligee, because the obligee is an intended beneficiary of the
promise made to the delegator.
EXAMPLE: Joe Plumbing Co. enters a contract to provide plumb-
ing services for Mondo Condo Association and subsequently hires a
plumbing subcontractor, Elite Plumbers Inc., to perform the services in
question. If Elite Plumbers Inc. fails to perform the delegated duties, it
would be liable for breach of contract to both Joe Plumbing and Mondo
Condo. Elite Plumbers would be liable to Mondo Condo because it is a
third-party beneficiary of the Joe Plumbing-Elite Plumbers contract.
5. Delegable Duties
a. The general rule is that all obligations can be delegated.
b. The exceptions fall into the following two categories:

303
OUTLINE

(1) when the performance in question is personal and the recipient


must rely on qualities such as the character, reputation, taste, skill,
or discretion of the party who is to render the performance; or
EXAMPLE: Nanny A is under contract to provide childcare servic-
es to B’s family. Her performance obligation cannot be delegated
to a third party.
(2) when the contract prohibits delegation.
(a) Unlike prohibitions against assignments, contract provisions
barring delegation are fully enforceable.
c. The effect of a party’s attempt to delegate nondelegable duties
differs depending on whether the contract is governed by the
common law or the UCC.
(1) At common law, an attempted delegation of a nondelegable duty
operates as an immediate breach of the contract and gives the
other party an immediate right to sue.
(2) Under the UCC, the other party may treat any assignment which
delegates performance as creating reasonable grounds for inse-
curity, and the other party has a right to demand adequate assur-
ances from the assignee without prejudicing his rights against the
assignor [§ 2-210(5)].

304
Criminal Law
TABLE OF CONTENTS

I. GENERAL PRINCIPLES

Introduction and the Bases of Criminal Law (Common Law and Statutory Law).........................308
Types of Crimes...........................................................................................................................308
Constitutional Issues....................................................................................................................309
Elements of Crimes......................................................................................................................309

II. CRIMES AGAINST THE PERSON

Homicide......................................................................................................................................316
Assault and Battery......................................................................................................................323
Mayhem.......................................................................................................................................325
False Imprisonment......................................................................................................................326
Kidnapping...................................................................................................................................326
Rape.............................................................................................................................................328
Other Crimes against the Person.................................................................................................330

III. CRIMES AGAINST PROPERTY

Theft Crimes.................................................................................................................................331
Larceny........................................................................................................................................331
Embezzlement.............................................................................................................................333
Robbery........................................................................................................................................334
Obtaining Property by False Pretenses.......................................................................................335
Bad Checks..................................................................................................................................335
Credit Card Fraud........................................................................................................................336
Larceny by Trick...........................................................................................................................336
Extortion.......................................................................................................................................336
Receiving Stolen Property............................................................................................................337
Forgery.........................................................................................................................................338
Crimes against the Habitation......................................................................................................339
Possession Offenses....................................................................................................................341

IV. INCHOATE CRIMES

Solicitation....................................................................................................................................344
Attempt.........................................................................................................................................344
Conspiracy...................................................................................................................................346

V. PARTIES TO CRIME; ACCOMPLICE LIABILITY

Accomplice...................................................................................................................................349
Principal in the First Degree.........................................................................................................350
Principal in the Second Degree....................................................................................................350
Accessory before the Fact...........................................................................................................350
Accessory after the Fact..............................................................................................................351

306
VI. DEFENSES

Responsibility...............................................................................................................................352
Justification..................................................................................................................................355
Entrapment...................................................................................................................................358
Mistake.........................................................................................................................................358
Consent........................................................................................................................................359
Condonation.................................................................................................................................359

307
OUTLINE

I. GENERAL PRINCIPLES

A. Introduction and the Bases of Criminal Law (Common Law and Statutory Law)
1. The substantive criminal law is defined by statute in most jurisdictions. To that
end, some questions on the bar exam will include an excerpt from a statute.
2. Many jurisdictions follow the common law approach to criminal law when
writing criminal statutes; a smaller number of jurisdictions choose instead to
follow the approach of the Model Penal Code (“MPC”).
3. The Multistate Bar Exam (MBE) generally tests the law “common to the
states” (also referred to as “the generally prevailing view” or “the generally
accepted view”). On some points of law, there is a divergence between the
early common law approach and the modern majority trend among common
law jurisdictions. This outline notes where modern statutes diverge from
the common law. In all released questions available to date from the MBE,
the bar examiners have either specified the approach in the question (the
common law or the modern trend) or they have specified facts in the ques-
tion that eliminate any possible differences in the answer to the question.
EXAMPLE: At common law, burglary maintained the elements of the location
being a dwelling and the timing being at night. The modern majority approach
is to define burglary as including any structure and to include entries made
during the day as well as at night. The question specified that the structure
was a dwelling and was entered at night in order to eliminate the differences
between the common law and the modern approach on this point.

Unless otherwise instructed, directly or indirectly, applicants should apply


MAJORITY rules to criminal law questions on the MBE. In most cases, the
common law rule and the majority rule are the same. Where the outline
indicates that the majority rule is NOT the common law rule (e.g. burglary,
conspiracy), applicants should apply the MAJORITY rule on the exam.
4. With respect to essay questions, important distinctions relating to your jurisdic-
tion’s substantive criminal law will be covered in the state criminal law lecture.

B. Types of Crimes
1. Felony
a. A felony is a crime punishable by death or by imprisonment for more
than one year. At common law, burglary, arson, robbery, rape, larceny,
murder, manslaughter, and mayhem were considered felonies.
2. Misdemeanor
a. A misdemeanor is a crime punishable by imprisonment for less than
one year or by a fine only. At common law, crimes not considered felo-
nies were deemed misdemeanors.
3. Malum Prohibitum
a. Malum prohibitum is an act that is wrong only because it violates a
statute (e.g., speeding or failing to register a firearm).

308
CRIMINAL LAW

4. Malum In Se
a. Malum in se is an act that is inherently wrong or “evil”—an act that involves
a general criminal intent or moral turpitude (e.g., murder, theft, and battery).
5. An infamous crime at common law involves fraud or dishonesty.

C. Constitutional Issues
1. Void-for-Vagueness Doctrine
a. Under the Due Process Clause of the Fifth and Fourteenth Amendments
of the United States Constitution, people must be on notice that certain
conduct is forbidden. Therefore, the Supreme Court has required that
criminal statutes be specific and give a person of ordinary intelligence
“fair notice” of what conduct is prohibited. Furthermore, the void-for-
vagueness doctrine requires statutes to be fair and consistent in their
enforcement and not be arbitrarily or erratically enforced.
2. Ex Post Facto
a. Under the Constitution, ex post facto laws are also prohibited. An ex
post facto law is one that retroactively:
(1) makes conduct criminal;
(2) enforces a stricter punishment for the same conduct; or
(3) alters procedural or evidentiary rules in such a way that the
criminal defendant may be more easily convicted.

D. Elements of Crimes
1. Generally, the prosecution must prove the following elements:
a. actus reus (a guilty act);
(1) This element may be met by:
(a) a voluntary act that causes an unlawful result;
(b) an omission to act where the defendant is under a legal duty
to act; or
(c) vicarious liability where the defendant is responsible for the
acts of another party.
(2) Criminal liability can be imposed on a defendant for an omission
to act where:
(a) there is a legal duty to act; and
(b) the defendant can physically perform the act.
(3) Such a legal duty to act may arise in the following ways:
(a) by statute (e.g., failure to file a tax return);
(b) by contract (e.g., failure of a lifeguard, nurse, or guide on a
hiking or river-rafting expedition to rescue);
(c) based upon relationship (e.g., a parent for a child or a
spouse for a spouse);
(d) where a voluntary undertaking is begun (e.g., unreasonable

309
OUTLINE

abandonment of a rescue that could worsen a victim’s plight


is sufficient, even if done by a Good Samaritan); or
(e) where someone creates a risk of peril to another.
EXAMPLE: If a defendant pushes a victim into a swim-
ming pool as a joke, but then realizes that the victim can’t
swim when the victim begins to drown, the defendant can
be prosecuted for murder for failing to throw the victim a
life preserver.
(4) Acts that are reflexive, convulsive, performed while uncon-
scious, or otherwise involuntary are insufficient, as are mere bad
thoughts unaccompanied by action. However, habitual acts that
one is simply “unaware of” are considered conscious and volun-
tary (e.g., a chain smoker who lights a cigarette in a no-smoking
area without realizing it can be successfully prosecuted).
b. mens rea (a guilty mind);
(1) Virtually all crimes require some mental state (mens rea) with
respect to some element of conduct (actus reus), and the
definition of which mental state is required for which elements
of conduct is often the key to successfully answering a ques-
tion. In determining a defendant’s criminal liability, the jury
must look to the defendant’s state of mind at the time of the
commission of the crime. Except for a small category of strict
liability crimes, a crime is committed only when a criminal act
is coupled with a guilty mind; both the mental and physical
elements must coexist.
EXAMPLE: Grant took Foster’s sunglasses believing they were
his. No larceny has been committed since there was no intent to
take someone else’s glasses.
(a) A person acts intentionally when he desires that his acts
cause certain consequences or knows that his acts are
substantially certain to produce those consequences.
(b) A person acts knowingly when he knows the nature and/
or result of his conduct. Lack of knowledge can often
excuse criminal liability under the defense of mistake of fact.
Traditionally, intent has been defined to include knowledge.
(c) A person acts purposely when there exists a conscious
objective to engage in such conduct or to cause such a result.
(d) The term willfully encompasses the concepts of “intention-
ally” and “purposely,” as opposed to accidentally or negli-
gently, and has been used to imply evil purpose in crimes
involving moral turpitude.
(e) A person acts recklessly with respect to a material element
of an offense when he consciously disregards a substantial

310
CRIMINAL LAW

and unjustifiable risk that the material element exists or will


result from his conduct. The risk must be of such a nature and
degree that, considering the nature and purpose of the actor’s
conduct and the circumstances known to him, its disregard
involves a gross deviation from the standard of conduct that a
law-abiding person would observe in the actor’s situation.
(f) Criminal negligence usually requires that the defendant’s
conduct create a high degree of risk of death or serious
injury beyond the tort standard of ordinary negligence.
The degree of that risk, however, has no precise, objective
measure. It is more than mere ordinary negligence, but less
than wanton and willful misconduct.
(2) A specific intent crime involves more than the objective fault
required by merely doing the proscribed actus reus. A defendant
will possess specific intent if:
(a) he wants, hopes, or wishes that his conduct will bring about
a particular result, regardless of the objective likelihood of the
result occurring (unless the result is inherently impossible); or
EXAMPLE: Joe wants to kill his cousin Mike, so at noon he
tosses a brick off the 50-story building where Mike works,
knowing that Mike is usually walking somewhere for lunch
at that time. Despite the long odds, the brick actually hits
Mike and kills him. Joe has the requisite specific intent to be
charged with Mike’s murder.
(b) he expects (i.e., is substantially certain) that his purposeful
act will have that particular result, even though he does not
necessarily want a particular result.
(3) Specific intent crimes include first-degree murder; theft crimes
such as larceny, robbery, extortion, embezzlement, false
pretenses, and receiving stolen property; burglary (but not
arson, which is a “malice” crime); inchoate crimes (solicitation,
conspiracy, and attempt); and assault.
(a) The typical criminal defenses apply to specific intent crimes.
(b) Voluntary intoxication and unreasonable mistake may negate
requisite specific intent elements.
(4) A general intent crime merely requires commission of an
unlawful act (e.g., nonconsensual intercourse) without a specific
mens rea. A general bad state of mind will suffice where such a
criminal act is committed voluntarily and purposely.
(a) Negligence or recklessness is a sufficient mental state
for general intent crimes. While mere ordinary negligence
(defined, as in tort, as the failure to use due care) does not
amount to criminal negligence, negligence that causes a
greater risk of harm than ordinary negligence or ordinary

311
OUTLINE

negligence where a defendant is consciously aware of the


risk will amount to criminal negligence. Criminal negligence
may also be called gross or culpable negligence.
(b) General intent crimes include rape, battery, kidnap-
ping, false imprisonment, involuntary manslaughter, and
depraved-heart murder.

EXAM TIP If, on the exam, you encounter a criminal activity that does not appear by its
classification (see classification chart below) to involve specific intent, the
malice standard, or strict liability, and the question does not indicate or supply
a mental state requirement, you should assume that for criminal culpability,
general intent would be required. General intent is the “catch-all.”

(5) The requisite intent for malice is met when a defendant acts
intentionally or with reckless disregard of an obvious or known
risk that the particular harmful result will occur.
(a) Malice crimes are common law murder and arson.
(6) Under strict liability, culpability is imposed on a defendant
merely for doing the act that is prohibited by statute. In other
words, no particular mental state is required for at least some
element of a strict liability crime.
EXAMPLE: State A has a statute that makes having sex with a
minor under the age of 17 a strict liability crime. Joe meets Lolita
in a bar and assumes that she is 21 because 21 is the drinking
age in State A. Lolita also tells Joe that she is 21, and nothing
about her appearance suggests she may be lying. She also pres-
ents to Joe a completely convincing piece of false identification
that states a birth date that would make her 21. However, in real-
ity, Lolita is only 15. If Joe has sex with her, he is guilty of violat-
ing the State A statute.
(a) Strict liability crimes fall under the categories of:
1) regulatory offenses (e.g., traffic violations, vehicle
offenses, or administrative statutes);
2) public welfare offenses (e.g., regulation of firearms,
food, and drugs); and
3) morality crimes (e.g., statutory rape, and bigamy).
(b) The transferred intent doctrine preserves liability where
a defendant intends criminal conduct against one party but
instead harms another party, so that his actions bring about
an unintended, yet still criminal, result.

312
CRIMINAL LAW

CLASSIFICATION OF CRIMES
Specific Intent General Intent Malicious Strict Liability
Crimes Crimes Crimes Crimes

• Attempt • Battery • Arson • Regulatory offenses


• Solicitation • Rape • Common Law • Public welfare
• Conspiracy • Kidnapping Murder offenses
• Larceny • Involuntary • Morality crimes
Manslaughter (such as statutory
• Larceny by Trick
rape, or bigamy)
• False Pretenses • Depraved-Heart
Murder • Selling liquor to
• Embezzlement minors
• False Imprisonment
• Forgery
• Burglary
• Assault
• Robbery
• Intent-to-Kill Murder
• Voluntary
Manslaughter

c. concurrence in time between the act and the requisite mental state; and
(1) It is not only necessary that the defendant’s criminal intent occur
at the time he commits the criminal act, but the mental state
should also actuate, or put into action, the act or omission.
EXAMPLE: In burglary, the “intent to commit a felony or theft of-
fense therein” must exist at the time of the breaking and entering.
d. some (but not most) crimes require the occurrence of a result for the
crime to be complete (e.g., homicide crimes require that the victim die).
(1) For such crimes, causation between the defendant’s act and the
required result must be evident.
(2) The defendant’s conduct must be both the actual and the proxi-
mate cause of the specified criminal result.
(a) Actual cause (also called cause-in-fact) may be satisfied by
any of three tests:
1) if the criminal result would not have occurred absent the
defendant’s act, then the defendant’s act is the actual
cause of the criminal result;
a) In other words, were it not for, or “but for” the
defendant’s actions, the criminal result would not
have occurred.
2) when there are multiple causes or other parties respon-
sible for the criminal result, courts will still find a defen-
dant responsible if the defendant’s act was a substantial
factor causing the criminal result; or

313
OUTLINE

EXAMPLE: Defendant 1 stabs Victim in the heart with


a knife. Simultaneous with this stabbing, Defendant 2
shoots Victim in the head. Medical testimony conclusively
establishes that either the knife or bullet wound alone
was sufficient cause to instantly kill Victim. Defendant 1’s
act can be considered the actual cause of Victim’s death.
3) where a defendant’s conduct speeds up an inevitable
death, even by a very brief amount of time, the defen-
dant is considered an actual cause of the victim’s death
because he accelerated an inevitable result.
(b) To find proximate cause, the resultant harm must be
within the risk created by the defendant’s conduct in crimes
involving negligence or recklessness, or sufficiently similar
to that intended in crimes requiring intent, so as not to hold
the defendant liable for extraordinary results (such as acts
of nature or grossly negligent or intentional bad acts of third
parties, including intentional medical mistreatment).
1) The tort maxim “you take the plaintiff as you find him”
applies to crime victims as well. That is, if the harm results
from a special sensitivity (such as hemophilia or another
preexisting medical condition), the defendant’s act is a proxi-
mate cause of the harm regardless of whether the defen-
dant could have foreseen the unique medical condition.
(3) When the defendant’s actions alone cause the harm, the defen-
dant’s act is the direct cause. It is very likely that the defendant
will be held legally responsible.
(4) In indirect cause situations, the other force that combines with the
defendant’s act to bring about the harm is called an intervening cause.
(a) To relieve the defendant of liability and thus, in effect, break
the chain of proximate cause, the other force must be a
superseding intervening cause.
(5) The rules determining when an intervening cause is superseding
(and thereby one that breaks the chain of causation) differ
according to whether the intervening force is dependent on or
independent of the defendant’s act. An intervening force that
is a result of or response to the defendant’s act is a dependent
intervening cause. An independent intervening cause is one that
would have occurred regardless of the defendant’s act.
(a) A dependent intervening cause will supersede the defen-
dant’s act only when it is a totally abnormal response to the
defendant’s act.
EXAMPLE: Defendant intentionally runs over Victim with
Defendant’s automobile, causing Victim to suffer serious
(but not life-threatening) injuries. Victim is rushed to a hos-

314
CRIMINAL LAW

pital and given medical treatment. If the attending physician


negligently treats Victim, resulting in Victim’s death, and the
negligent treatment is not considered abnormal, Defendant
can be found to have proximately caused Victim’s death.
(b) An independent intervening cause will normally supersede
the defendant’s act, except when the independent inter-
vening force was foreseeable.
EXAMPLE: Defendant, having planned to kill his wife, re-
turns home late on a stormy night. Defendant drives into the
driveway and notes that the entire house is surrounded by
snow drifts from the storm. Defendant enters the home and
pulls a gun on his wife. The wife struggles and manages to
break free and get outside. She is dressed only in a night-
gown and decides to hide from her husband and sleep in the
doghouse rather than seek help in the bitter cold. She dies of
exposure during the evening. Defendant’s act might be suf-
ficient to impose him to criminal liability due to the foresee-
ability of the wife’s death from exposure.

315
OUTLINE

II. CRIMES AGAINST THE PERSON

A. Homicide
1. A homicide results when there is a killing of a human being caused by
another human being (i.e., the defendant). A more complete way of stating
the rule is that a criminal homicide results from some action or actions of
the defendant that cause the death of another human being, with criminal
intent, and without legal excuse or justification.
2. The difference between the varying homicide crimes typically depends on the
mental state the defendant had with respect to the conduct causing the death.
3. Murder
a. At common law, murder is defined as the unlawful killing of a human
being with malice aforethought.
b. The actus reus may be a voluntary act, an involuntary act arising from
a voluntary act (such as a person who has frequent seizures driving a
car), or an omission to act where there is a legal duty to act.
c. The act must actually and proximately cause the death of another
living person. The common law requirement for a living person was
one “born alive” (though a state may extend criminal liability to include
a fetus after the first trimester).
d. The death must be caused by someone other than the victim.
(1) Suicide is not homicide because the death must be caused by another.
(2) To persuade or aid another to commit suicide is a sufficient basis
for murder in some jurisdictions.
e. The defendant’s conduct must be both the actual cause and a legal
cause of the victim’s death.
(1) For common law murder, the “but-for” test applies. In other words,
the fact finder must determine that the victim’s death would not
have occurred but for the defendant’s actions. Even if the defen-
dant’s actions alone would be insufficient to cause the victim’s
death, but instead contributed to the death, a court may still find
actual causation.
(2) In situations when a victim is already dying, if the defendant’s
actions bring about the victim’s death more quickly than if the
defendant had not acted, the defendant’s actions would be an
actual cause of the killing.
EXAMPLE: Disconnecting life support to a dying patient is an actual
cause of the patient’s death if she dies more quickly as a result.
(3) Where the victim’s death was a “natural and probable” conse-
quence of the defendant’s conduct, the defendant may be guilty
of murder, even where he did not foresee the exact chain of
events that resulted in the victim’s death.

316
CRIMINAL LAW

(4) Note that where an intervening act occurs that is outside the
universe of foreseeable events caused by the defendant’s acts, such
an intervening act will sever the chain of causation, and the defen-
dant will be acquitted of murder. Additionally, remember that a dead
person cannot be killed. Thus, if an intervening cause kills the victim
before the defendant can complete his act, he will be acquitted.
(5) At common law, if the victim died more than one year and one
day after the defendant’s act, the courts would rule that the
defendant’s act was not the proximate cause of the killing. Most
states have either eliminated this rule or have extended the
period within which the defendant is held legally responsible.
(6) Defendants who do not personally commit any acts sufficient to
amount to actual cause may nonetheless be legally responsible
for a killing in the following circumstances:
(a) a defendant who is an accomplice to the killer may be held
liable for a homicide even though only the killer actually
acted to cause the victim’s death;
(b) where the reasonably foreseeable result of a conspiracy is
a homicide, and that homicide was committed in furtherance
of the conspiracy, then all members of the conspiracy can
be held liable for the homicide regardless of which of the
conspirators actually caused the killing;
(c) where both a third party and the defendant together cause a
victim’s death, the causation question varies depending on
whether the defendant’s act was a direct or indirect cause; and
1) When a victim would not have died “but for” the actions
of both the defendant and a third party, both will be
considered the direct causes of the death. The defen-
dant’s legal responsibility is superseded only by a
dependent intervening act that is totally abnormal or an
independent intervening act that is unforeseeable.
EXAMPLE: Defendant shoots Victim in the shoulder,
causing a serious but not life-threatening injury. Shortly
thereafter, Third Party stabs Victim repeatedly in both
feet, causing numerous bleeding wounds. Hours later,
Victim dies from excessive blood loss. Both the shoulder
and feet wounds caused the bleeding. Defendant and
Third Party caused Victim’s death.
(d) where a defendant causes the death of another, even if not
at his own hands, during the commission of or in an attempt
to commit a felony (i.e., felony murder)
(7) As in tort situations of nonfeasance, people who fail to prevent
injury or death are generally not criminally liable for the victim’s
condition unless they have a duty to act.

317
OUTLINE

(8) Where a victim has an unusual condition that contributes to his


death, a defendant can still be found guilty of murder. The defen-
dant is said to “take the victim as he finds him.”
f. At common law, the mens rea for murder was malice.
g. There are four distinct categories of mental states with respect to
common law murder, all of which are sufficient to meet the malice
standard required at common law. If any of the following mental states
exist, it supports a murder conviction at common law:
(1) intent to kill;
(a) Conduct where the defendant consciously desires to kill
another person or makes the resulting death inevitable
(absent justification, excuse, or mitigation to voluntary
manslaughter) constitutes an intent to kill.
(b) The defendant’s own statements may provide proof of
intent to kill.
(c) Under the deadly weapons doctrine, an inference of intent
to kill is raised through the intentional use of any instru-
ment which, judging from its manner of use, is calculated to
produce death or serious bodily injury.
EXAMPLE: A defendant’s intent to kill can be inferred from
deliberately swinging a baseball bat at the victim’s head.
(d) Intent-to-kill murder is a specific intent crime.
(2) intent to cause serious bodily harm;
(a) Serious bodily injury, also called “great bodily injury” or
“grievous bodily injury,” means significant but nonfatal injury.
Intent-to-inflict-serious-bodily-injury malice, like intent-to-kill
malice, can arise from a conscious desire or substantial certainty
that the defendant’s actions will result in the victim’s injury.
(b) Like intent-to-kill malice, intent to inflict serious bodily injury
must be proved by examination of all the surrounding
circumstances, including the words and behavior of the
defendant. Similarly, the intentional use of any deadly
weapon in a way that is likely to cause serious injury
provides evidence of intent to inflict serious bodily injury.
EXAMPLE: Defendant drives his car over Victim’s legs, intending
to break them. However, Victim ends up dying as a result of the in-
juries suffered. Defendant has the necessary mens rea for murder.
(3) depraved-heart murder; and
(a) Depraved-heart murder, sometimes referred to as extreme
recklessness murder, is an unintentional killing resulting from
conduct involving a wanton indifference to human life and
a conscious disregard of an unreasonable risk of death or
serious bodily injury.

318
CRIMINAL LAW

(b) A defendant may knowingly create a very high risk of death


or serious bodily injury for a logical and socially reasonable
purpose, in which case the conduct would not be considered
depraved-heart murder.
EXAMPLE: Defendant thought it would be amusing to drive
on the wrong side of the street, “like they do in England.” He
did so, at high speed, through a residential neighborhood and
veered into children crossing at a crosswalk (who were look-
ing the other way). Defendant likely will be found to have pos-
sessed the mens rea necessary for depraved-heart murder.
(4) felony murder.
(a) Felony murder is a killing proximately caused during the
commission or attempted commission of a serious or inher-
ently dangerous felony.
(b) Generally, it includes both intentional and accidental killings.
The mental state required is an intent to commit the underlying
felony, such as burglary, arson, robbery, rape, or kidnapping.
(c) The defendant must be guilty of the underlying felony. Any
defenses to the felony will negate the felony murder.
(d) Several limitations have been placed on the scope of the
felony-murder rule.
1) A majority of states have limited the felony-murder rule
by requiring that the underlying felony be collateral.
That is, the underlying felony must be independent
of the homicide so that every felonious attack upon a
victim (i.e., felonious assault) which is ultimately fatal
does not become escalated to murder by the rule.
2) The felony must be an inherently dangerous one (i.e.,
burglary, arson, robbery, rape, or kidnapping).
3) In determining whether an offense is inherently
dangerous, a majority of states apply an abstract test,
considering only the elements of the felony in the
abstract and not the factual circumstances of the felony
as committed. The minority of states apply a contextual
test, looking at the particular circumstances of the case
to determine whether the underlying offense should be
considered inherently dangerous.
EXAMPLE: Defendant pretended to be a doctor with a
cancer cure and induced Victim to pay $5,000 for a medi-
cally worthless ointment that Defendant claimed would
heal her. If Victim had received competent medical treat-
ment, the cancer would have been cured. If Victim dies of
cancer, Defendant can be charged with felony murder in
a minority jurisdiction. A majority of states would not ap-

319
OUTLINE

ply the felony-murder rule, though, as the act of selling a


worthless ointment was not inherently dangerous.
4) The resulting death must be a foreseeable outgrowth
of the defendant’s actions. Note that most courts have
generally been very liberal in applying the foreseeability
requirement. Therefore, most deaths are considered
foreseeable for purposes of felony murder.
5) The harm that results in the death of the victim must
occur during the commission or attempted commission
of the felony.
a) For purposes of felony murder, the felony starts
when the defendant could be convicted of
attempting the underlying felony. There is no
requirement that the felony must be completed.
b) The felony is deemed to have terminated when the
felon has reached a place of temporary safety. If
the killing occurs after this point, the defendant can
no longer be found guilty of felony murder.

NOTE If a killing occurs while the defendant is fleeing from the scene of the felony,
he may still be guilty of felony murder. The felony, and thus the possibility
of committing a felony murder, does not terminate until the felon reaches a
place of temporary safety.

(e) Felony Murder Based on Vicarious Liability


1) At common law, all felons were liable for any homicide that
occurred during the perpetration of the felony. The common
law did not make exceptions for homicides committed by
non-felons (i.e., victims, bystanders, police officers, etc.).
2) Under the majority (agency theory) rule, there is no
felony-murder liability when a non-felon causes the
death. The agency theory posits that felony murder
extends only when the killing is committed by one of the
agents of the underlying felony.
3) Under the minority rule, felony-murder charges can
be based on killings by non-felons, such as killings by
victims of the crime, bystanders, and police officers.
a) Among jurisdictions that follow the minority rule,
some extend an exception (the Redline limita-
tion). Under this limitation, a co-felon is not guilty
of felony murder where the killing constitutes a
justifiable or excusable homicide. Such would be
the case where, for example, the police or a victim
shoots one of the co-felons, but not where the
killing is done by one of the felons.

320
CRIMINAL LAW

EXAMPLE: A bank robber would not be charged


with felony murder under this limitation if his ac-
complice is justifiably killed by the police.
b) Other minority rule jurisdictions follow common law
felony murder principles.
4) A large minority of jurisdictions allow an affirmative
defense for nonviolent co-felons who were unarmed,
unaware that violence would occur, and did not
encourage the violence.
h. Murder by Degrees
(1) While degrees of murder were not recognized at common law,
most jurisdictions distinguish between different degrees of murder
based on criteria identified in the given state’s murder statute.
(2) First-Degree Murder
(a) First-degree murder includes intent-to-kill murder committed
with premeditation and deliberation, felony murder, and, in
some jurisdictions, murder accomplished by lying in wait,
poison, terrorism, or torture. Some jurisdictions require little or
nothing more than an intent to kill in order to find premedita-
tion and deliberation, but most jurisdictions require more. Most
jurisdictions require a reasonable period of time for premedi-
tation and some evidence of reflection in order to distinguish
first-degree murders from “spur-of-the-moment killings.”
EXAMPLE: Jerry walked into the ice cream shop and told Ben
he’d just “scored” with Ben’s new girlfriend. Ben, outraged,
decided in that split second to beat Jerry to death and began
pounding on Jerry’s head with an ice cream scoop. Jerry dies
as a result of the beating. Ben may have committed first-degree
murder in a minority jurisdiction, but not in a majority jurisdiction.
(b) If the defendant was voluntarily intoxicated—but still sober
enough to form the intent to kill—he may be able to avoid
liability for first-degree murder by proof that the intoxication
precluded him from acting with premeditation or deliberation.
(3) Second-Degree Murder
(a) Second-degree murder is any murder that does not meet the
requisite elements of first-degree murder. Examples include: a)
where the defendant’s malice is intent to inflict serious bodily
injury; b) where the defendant acted with wanton and willful
misconduct; or c) felony murder, where the underlying felony is
not specifically listed in an applicable first-degree murder statute.
4. Voluntary Manslaughter
a. Voluntary manslaughter is an intentional killing mitigated by adequate
provocation or other circumstances negating malice aforethought.
Voluntary manslaughter is commonly called a heat-of-passion killing.

321
OUTLINE

b. Adequate provocation, measured objectively, must be such that a


reasonable person would lose self-control.
(1) A causal connection must exist between the legally adequate
grounds for provocation and the killing.
(2) The time period between the heat-of-passion and the fatal act must
not be long enough that a reasonable person would have cooled off.
(3) Courts will commonly find that a defendant was adequately
provoked when he killed after he was the victim of a serious
battery or a threat of a deadly force or where he found his spouse
engaged in sexual conduct with another person.
(4) Where a defendant kills after an exchange of “mere words,”
courts generally will not find adequate provocation.
(5) Courts generally do not find that mitigating circumstances exist
(such that a defendant’s criminal liability will be reduced from
murder to voluntary manslaughter):
(a) where a defendant actually did cool off (even if a reasonable
person would not have cooled off); or
(b) where a defendant, for any other reason, killed the victim
while he was, subjectively, not in the heat of passion.
c. Other Mitigating Circumstances
(1) Imperfect self-defense may mitigate murder to voluntary
manslaughter where a defendant was either at fault in starting an
altercation or unreasonably, but honestly, believed that harm was
imminent or deadly force was necessary. Such mistaken justifica-
tion has been applied to self-defense, defense of others, crime
prevention, coercion, and necessity.
(2) A minority of states allow diminished mental capacity short of
insanity to reduce murder to manslaughter.
5. Involuntary Manslaughter
a. Involuntary manslaughter is an unintentional killing resulting without
malice aforethought caused either by recklessness, criminal negligence,
or during the commission or attempted commission of an unlawful act.
b. Examples of activities that may be deemed criminal negligence are
the mishandling of loaded weapons or dangerous operation of a motor
vehicle, including driving while intoxicated.
(1) Gross negligence or criminally negligent conduct is required, but
the majority of jurisdictions do not require that the defendant be
consciously aware of the risk created.
c. An unintentional killing that occurs during the commission or attempted
commission of a misdemeanor that is malum in se, or of a felony that is not of
the inherently dangerous type required for felony murder, is classified as invol-
untary manslaughter under the so-called misdemeanor-manslaughter rule.
(1) Whereas limitations do exist regarding the nature of the unlawful
act and the causation between the act and the killing, the malum

322
CRIMINAL LAW

in se misdemeanor need not be independent of the cause of


death, unlike felony murder.
d. Death resulting from a malum prohibitum crime can only be sufficient for
involuntary manslaughter when the killing is either a foreseeable conse-
quence of the unlawful conduct or amounts to criminal negligence.
e. Analogous in many respects to intent-to-kill or intent-to-cause-serious-
bodily-injury murders, where the defendant intends to injure the victim
to some “lesser” degree but this unexpectedly causes the death of the
victim, an involuntary manslaughter results. Since such action is either
a battery or an assault—both malum in se—application of the misde-
meanor-manslaughter rule renders any resulting homicide an involun-
tary manslaughter. Common examples include inflicting a superficial
cut upon a hemophiliac who dies from loss of blood or a simple battery
that results in death because the victim had an “egg-shell” skull.

B. Assault and Battery


1. Assault and battery were common law misdemeanors. Note that, at
common law, because assault and battery were considered only misde-
meanors, they could not be used as underlying felonies for purposes of
felony murder. However, modern statutes have created aggravated forms of
assault and battery, which are felonies.
2. Battery
a. Criminal battery is the intentional, reckless, or criminally negligent
unlawful application of force to the person of the victim.
b. Criminal battery is a general intent crime; a defendant may be guilty
of battery where he acts:
(1) recklessly;
(2) negligently; or
(3) with knowledge that his act (or omission) will result in criminal liability.
c. The defendant’s act of applying force may be direct or indirect.
(1) Where the defendant puts a force in motion, the force need not
be applied directly by the defendant.
EXAMPLE: If John tells his attack bull terrier to charge at a visi-
tor, John may be guilty of battery even though he did not person-
ally touch the visitor.
EXAMPLE: John, in a fit of rage, fires a gun at his ex-wife’s new
house. The bullet smashes the window and a shard of glass hits
the ex-wife’s new boyfriend in the eye. Regardless of whether
John intended to shoot someone, this act would likely rise at
least to the level of criminal negligence (creating a high degree
of risk of death or serious injury) or even recklessness (con-
scious disregard of a substantial and unjustifiable risk). John
would be guilty of battery.

323
OUTLINE

d. In most jurisdictions, certain circumstances cause a simple battery to


be elevated to an aggravated battery. Most commonly, these circum-
stances include:
(1) the defendant causing the victim serious bodily injury;
(2) the defendant using a deadly weapon to commit the battery; or
(3) the defendant battering a woman, child, or law enforcement officer.
e. Defenses
(1) Consent may be a valid defense where it is not coerced or
obtained by fraud, but it is no defense to a breach of the peace.
(2) Self-defense and defense of others are valid defenses to a
battery charge.
(3) Where the defendant commits an offensive touching to prevent
someone from committing a crime, this will be a defense to battery.
3. Assault
a. At modern law, a defendant may commit criminal assault by:
(1) attempting to commit battery; or
(a) At common law, this was the only way to commit
criminal assault.
(b) This type of assault requires an intent to commit a battery
(i.e., an intent to cause physical injury to the victim). Thus,
at common law, an intent merely to frighten (even where
accompanied by some fear-producing act, such as pointing
an unloaded gun at the victim) will not suffice.
(c) Because an intent to injure is required, recklessness or
negligence that comes close to causing injury (such as
driving a car recklessly but just missing a victim) will not
suffice for an assault.
(d) The fact that the victim was not aware of the attempted
battery is no defense to this type of assault.
(e) Most states do not permit a defendant to avoid liability for
“attempt” assault simply because the defendant lacked the
present ability to consummate the battery (e.g., the defen-
dant, with the intent to shoot the victim, pulls the trigger, but
unbeknownst to the defendant, there are no bullets in the
weapon). However, in a minority of states, there is an addi-
tional requirement imposed by statute or case law that the
defendant have the present ability to commit the battery.
(2) intentionally causing the victim to fear an immediate battery.
(a) In this type of assault, the defendant must act with threat-
ening conduct (mere words are insufficient) intended to
cause reasonable apprehension of imminent harm to the
victim. A conditional threat is generally insufficient unless
accompanied by an overt act to accomplish the threat.

324
CRIMINAL LAW

EXAMPLE: Defendant points a gun at Victim and says, “If


you don’t get over here right now, I am going to shoot you.”
Defendant has committed an assault.
(b) A defendant who is guilty of a “fear of battery” assault must
intend to either cause the actionable apprehension or cause
the victim to suffer bodily harm.
(c) Reasonable Apprehension
1) When a reasonable person would not expect imminent
bodily harm, there is no criminal assault.
EXAMPLE: Words alone are generally held insufficient
to constitute an “apprehension” assault.
2) The element of apprehension in this type of assault
connotes “expectation” more than “fear” (though the
term “fear” is employed frequently in assault crime stat-
utes). The victim does not have to actually be afraid but
rather to simply (and reasonably) anticipate or expect
that the defendant’s act(s) will result in immediate bodily
harm. Note that in this type of assault (unlike attempted
battery assault), the fear or apprehension piece is a key
element, so no assault has been committed where the
victim is unaware of the threat of harm.
(d) The threat must be to commit a present battery; a promise of
future action is generally not an assault.
(e) Any threatened contact, including one that is offensive or
insulting, is sufficient to constitute an “apprehension” assault.
There need be no actual pain or injury threatened.
(f) If the feared battery is accomplished, the assault and battery
merge and the defendant is found guilty only of the battery.
b. A simple assault may rise to the level of an aggravated assault under
certain circumstances. Most commonly, the circumstances include:
(1) where the defendant commits an assault with a dangerous
weapon; or
(2) where the defendant acts with the intent to seriously injure, rape,
or murder the victim.

C. Mayhem
1. At modern law, the felony of mayhem requires:
a. an unlawful act by means of physical force;
b. resulting in an injury which:
(1) deprives a human being of a member of his or her body;
(2) disables, disfigures, or renders such limb useless; or
(3) cuts or disables the tongue, puts out an eye, or slits the nose,
ear, or lip; and

325
OUTLINE

c. which was done maliciously.


(1) This element is usually defined as an unlawful intent to vex,
annoy, or injure another person.
2. At common law, the crime focused more on injuries which substantially
reduced the victim’s fighting capability, and required an intent to maim or do
bodily injury, accompanied by an act that either:
a. dismembered the victim; or
b. disabled his use of some bodily part that was useful in fighting.
EXAMPLE: In a California case, a middle school boy was convicted of may-
hem when he spat a spitball at a fellow student, hitting the other child in the
eye and disfiguring him.
3. Some states have abolished mayhem and treat it as a form of aggra-
vated battery.

D. False Imprisonment
1. False imprisonment is the intentional, unlawful confinement of one
person by another.
a. The confinement must be intentional and must be against the law. If
the defendant is privileged to confine the victim, such as a police officer
or a private citizen making a valid citizen’s arrest, no false imprison-
ment is committed.
b. The victim must be fully confined. In other words, blocking one exit but
leaving another open does not amount to false imprisonment.
c. False imprisonment is not limited to one method of restraining the
victim. For example, it may be accomplished by erecting physical
barriers, applying force or threatening to apply immediate physical
force, or invalidly asserting authority.
d. Victims are not required to try to resist or attempt escape where the
defendant has the apparent ability to effectuate threats. Victims are not
“confined” if they are aware of a reasonable means of escape, but are
not required to affirmatively search for potential escape routes.
EXAMPLE: After discovering Diane cheating on him with another man,
Jack left Diane in the bedroom and slammed the door, yelling, “Stay
there until you’re sorry. If you beg, I may let you out for breakfast.”
Though Jack intended to lock the bedroom door, he was so upset that
he accidentally released the lock before slamming the door. Diane
believed she was locked in and never checked the door herself. Jack
may be guilty of false imprisonment.

E. Kidnapping
1. Definition
a. At modern law, a person commits the offense of kidnapping when he:
(1) abducts or steals away any person;

326
CRIMINAL LAW

(a) This element can be met where the victim is taken from one
place to another or when they are secretly confined where
the person is not likely to be found.
(2) without lawful authority or warrant; and
(3) holds that person against his or her will.
b. At common law, kidnapping consisted of an unlawful restraint of a
person’s liberty by force or show of force so as to send the victim into
another country.
c. Under the Model Penal Code, a person is guilty of kidnapping if he or
she unlawfully removes another from his or her place of residence or
business, or a substantial vicinity where he or she is found, or if he or
she unlawfully confines another for a substantial period in a place of
isolation, with any of the purposes:
(1) to hold for ransom or reward, or as a shield or hostage;
(2) to facilitate the commission of any flight or felony thereafter;
(3) to inflict bodily injury on or to terrorize the victim or another; or
(4) to interfere with the performance of any governmental or
political function.
d. The federal kidnapping statute makes it unlawful to seize, confine,
inveigle, decoy, kidnap, abduct, or carry away and hold for ransom or
reward, or otherwise, any person, except in the case of a minor by the
parent thereof, when [18 U.S.C.S. § 1201(a)]:
(1) the person is willfully transported in interstate or foreign
commerce, regardless of whether the person was alive when
transported across a state boundary, or the offender travels in
interstate or foreign commerce or uses the mail or any means,
facility, or instrumentality of interstate or foreign commerce in
committing or in furtherance of the commission of the offense;
(a) The failure to release the victim within 24 hours after he
or she has been unlawfully seized creates a rebuttable
presumption that such person has been transported in inter-
state or foreign commerce [18 U.S.C.S. § 1201(b)].
(b) The intent by Congress was to criminalize only those abduc-
tions where the victim was transported in interstate commerce
against his or her will. As such, the consent of the victim at the
time of the crossing of state lines constitutes a valid defense
even if the victim was initially seized against his or her will
[United States v. Toledo, 985 F.2d 1462 (10th Cir. 1993)].
(2) any such act against the person is done within the special mari-
time and territorial jurisdiction of the United States;
(3) any such act against the person is done within the special aircraft
jurisdiction of the United States, as defined by statute;
(4) the person is a foreign official, an internationally protected person, or
an official guest, as defined by the specified statutory provision; or

327
OUTLINE

(5) the person is an officer or employee of the United States as


described by statute and such act against the person is done
while the person is engaged in, or on account of, the performance
of official duties.
2. Classification
a. Kidnapping was a misdemeanor at common law. However, at modern
law, it is generally classified as a felony.
b. Under the Model Penal Code, kidnapping is a first-degree felony
unless the actor voluntarily releases the victim alive and in a safe place
prior to trial, in which case it is a second-degree felony.
c. A number of states define certain types of kidnapping as being of
the aggravated type, and thus deserving of a higher punishment.
Examples of aggravated kidnapping requirements include:
(1) where the offense is committed and the actor uses or exhibits a
deadly weapon;
(2) where the victim suffers serious bodily injury or sexual assault;
(3) where the victim is held for ransom; or
(4) where the victim is a small child.
d. The federal kidnapping statute includes a minimum of 20 years’ impris-
onment where [18 U.S.C.S. § 1201(g)]:
(1) the victim is under the age of 18; and
(2) the offender is 18 or older, and is not:
(a) a parent;
1) The term “parent” here does not include a person whose
parental rights with respect to the victim have been
terminated by a final court order.
(b) a grandparent;
(c) a brother;
(d) a sister;
(e) an aunt;
(f) an uncle; or
(g) an individual having legal custody of the victim.

F. Rape
1. At modern law, rape is sexual intercourse against a victim’s will by force,
threat, or intimidation. Put another way, the crime of rape requires proof of
sexual intercourse with another compelled by force and against the victim’s
will or compelled by threat of bodily injury.
a. At common law, rape was the act of unlawful sexual intercourse by a
male person with a female person without her consent. While penetra-
tion was required, emission was not.
b. At common law, a husband could not rape his wife. This is no longer
the law in any state.

328
CRIMINAL LAW

c. The modern rule is that rape can occur regardless of gender (e.g., by a
woman to a man, or between people of the same sex).
d. Any penetration, however slight, will satisfy the requirements for rape.
(1) Rape generally refers to the anal or vaginal intercourse, regard-
less of degree of penetration. However, other acts of sexual inter-
course may be included in a statute’s definition of rape.
(2) Other nonconsensual sexual contacts are generally covered under a
separate crime of sexual assault, sexual contact, or sexual battery.
e. At common law, males under 14 were conclusively presumed inca-
pable of rape. Many modern jurisdictions maintain the presumption, but
make it rebuttable.
2. Resistance
a. Some rape statutes have been amended to eliminate any requirement
that the victim resist.
b. Where resistance remains as an element of rape, at common law, resis-
tance or opposition by mere words was typically not enough; the resis-
tance must be by acts [Mills v. United States, 164 U.S. 644 (1897)].
(1) Under the modern approach, verbal resistance is sufficient for
rape to occur.
c. The resistance must also be in good faith and not feigned.
d. Generally, the resistance required depends on the following factors
[C.M. v. Alabama, 889 So. 2d 57 (Ala. Crim. App. 2004)]:
(1) the parties’ relative strength;
(2) the degree of force manifested;
(3) the fear instilled in the victim;
(4) the victim’s age;
(5) the victim’s physical and mental condition; and
(6) all other circumstances, dependent upon the facts of the case.
3. Consent
a. If the victim is incapable of consenting, the intercourse is rape. Inability
to consent may be caused by the effect of drugs or intoxicating
substances or by unconsciousness.
b. There are two types of consent defenses: the consent defense and the
reasonable belief in consent defense.
c. A defendant’s reasonable and good-faith mistake of fact regarding
a victim’s consent to sexual intercourse is a defense to rape. This
defense has both an objective and subjective component.
(1) The subjective component asks whether the defendant honestly
and in good faith, albeit mistakenly, believed that the victim
consented to sexual intercourse.
(a) This component is met by the defendant introducing evidence
of the victim’s equivocal conduct which provided the basis on
which he erroneously believed there was consent.

329
OUTLINE

(2) The objective component asks whether the defendant’s mistake


regarding consent was reasonable under the circumstances.
(a) Regardless of how strongly a defendant subjectively believes
a person has consented, that belief must be formed under
circumstances society views as reasonable.
(b) If the prosecution’s proof does not raise a reasonable doubt
as to whether the defendant harbored a reasonable and
good faith but mistaken belief of consent, the defendant
bears the burden of raising such a reasonable doubt.
(3) Courts do not require that victims communicate their lack of consent.
d. Where the defendant claims the victim consented, the jury must weigh
the evidence and decide which of the two witnesses is telling the truth.
e. The jury will first consider the victim’s state of mind and decide whether
there was actual consent to the alleged acts. If there was not, the jury
will view the events from the defendant’s perspective to determine
whether the manner in which the victim expressed her lack of consent
was so equivocal as to cause the defendant to assume that there was
consent where, in fact, there was not.
4. Statutory Rape
a. Where a female is under the statutorily prescribed age of consent (usually
16), an act of intercourse constitutes rape despite her apparent consent.
(1) Some jurisdictions have expanded the scope of statutory rape to
include males under the age of consent.
b. A defendant’s mistake as to the age of consent is generally no
defense to statutory rape.
c. Where the parties are validly married, a person cannot be convicted
of “statutory” rape of their spouse.
d. A perpetrator can be guilty of both statutory and forcible rape.
However, in most jurisdictions, he would be sentenced to the more
serious crime of forcible rape, because the less serious crime of
statutory rape would merge.

G. Other Crimes against the Person


1. Bigamy is the crime of marriage by one individual to more than one
other person.
2. Incest is the crime of sexual relations between individuals who are closely
related to one another. The degree of relationship required varies by state.

330
CRIMINAL LAW

III. CRIMES AGAINST PROPERTY

A. Theft Crimes
1. Theft crimes are crimes that involve some taking of property from the victim
by the defendant. The key to analyzing theft crimes and to distinguishing
among them often depends on whether the defendant acquired custody,
possession, or title to the property—larceny requires that the defendant
obtain possession unlawfully, false pretenses that the defendant obtain title
falsely, and embezzlement requires that the defendant convert (misuse)
property that has already been entrusted to him.
2. A person is said to have custody of property when the property has been
left with them, but they have no rights over the property.
EXAMPLE: A bailee only has custody of goods. Low-level employees are
typically considered to only have custody of their employer’s property.
3. A person has possession of property when they have custody and the
authority to exercise discretion over the property, or limited rights to use the
property. When property is loaned to another person to use, that person is
considered to have possession.
EXAMPLE: An auto mechanic has possession over a car left with them for
repairs, because they have the right to open it, repair it, drive it around in
order to test it, etc.
4. A person has title to property when they are the legal owner of the property.

B. Larceny
1. At modern law, the crime of larceny is defined as:
a. the taking;
(1) The taking requires the assertion of dominion and control over
the property by a defendant who does not have lawful posses-
sion, generally through trespass (i.e., without consent). When the
taking is accomplished by trickery (i.e., where the victim consents
to the defendant taking possession, but such consent is induced
by misrepresentation), the crime is larceny by trick.
b. and carrying away;
(1) The carrying away (asportation) is complete upon even the
slightest movement (e.g., six inches will suffice).
c. of the property;
(1) Common law larceny was limited to tangible personal property.
(2) Modern statutes have expanded the kinds of property to include
theft of services and other intangibles (such as gas and electrical
power and written instruments that represent property rights).
(3) Abandoned property cannot be the subject of larceny, although
lost or mislaid property can.

331
OUTLINE

(a) In order to be guilty of larceny for lost or mislaid property,


there are two requirements. The finder:
1) must intend to permanently deprive the owner of it; and
2) must either know who the owner is or have reason to
believe (from earmarkings on the property or from the
circumstances of the finding) that he can find out the
owner’s identity.
d. of another;
(1) Because the property must be “of another,” a good-faith claim
of right is a valid defense. Larceny is a crime of possession, as
opposed to ownership. Therefore, under certain circumstances,
an owner can be guilty of larceny of his own property, such as
when another is in lawful possession of the owner’s property.
(a) Generally, employees are said to have custody over their
employer’s property; however, the employee has posses-
sion—and thus could commit embezzlement—if:
1) a third party gives property directly to an employee for
the benefit of its employer; or
2) the employee is in a high-level position (e.g., an office
manager, bank president, or corporate official).
(b) A bailee generally has possession.
1) When a bailee opens closed containers and then misappro-
priates property, under the “breaking bulk” doctrine, construc-
tive possession is said to exist in the bailor, and the bailee,
thus having only custody of the property, is guilty of larceny.
e. with the intent to permanently deprive the owner thereof.
(1) Larceny, a specific intent crime, requires that the intent to
permanently deprive the owner accompany the taking. Note
that the intent to keep, destroy, or hold property for ransom will
suffice to prove this element.
(a) Where a defendant recklessly exposes property to loss or
deals with property in a manner involving a substantial risk of
loss, the intent to permanently deprive is satisfied.
(2) If, at the time of taking, the defendant intends to return the prop-
erty to the victim unconditionally and within a reasonable time,
there is no intent to permanently deprive. The defendant must
have the ability to return the property, even if something unantici-
pated stops the actual return of the property.
(a) Intent is not conclusively established by proof that a defen-
dant actually returned the property to the victim. If the
taking occurred with the requisite intent to permanently
deprive, a defendant will not be relieved of liability for
larceny because the defendant later returns the property or
if the victim later forgives the defendant.

332
CRIMINAL LAW

(3) Under the doctrine of continuing trespass, a person who takes


another’s property without authorization and intending only to use
it temporarily before restoring it unconditionally to its owner may
nevertheless be guilty of larceny if he later changes his mind and
decides not to return the property after all.
(a) As a general rule, the initial taking must be “wrongful”—
i.e., without the owner’s authorization. If the initial taking
was wrongful, the trespass is said to continue until the
time the intent to steal is formed.
(4) The intent element will be satisfied where a defendant intends,
when taking the property, to recklessly use it temporarily and then
abandon it, hoping someone will return it to the victim, even if the
property somehow does return to the victim.
(5) Defendants who pawn property may negate the intent to perma-
nently deprive element by proof that, at the time of taking, they
intended to redeem the property and return it to the victim.
(6) When the defendant takes property with the intent to later pay for
it or later replace it, such intent may negate the intent to perma-
nently deprive element if, for example, the property is easily
replaceable (i.e., not unique).
EXAMPLE: Bill broke into Steve’s home and took a computer. Bill
took the computer and dumped it in a nearby alley, smashing it to
pieces. Bill is guilty of larceny.
EXAMPLE: Same case, but Bill left the computer in his yard
where it got rusted and dirty. One year later, Bill, feeling remorse,
returned the computer to Steve. Bill will be guilty of larceny.
(7) Where a defendant, at the time of the taking, has a good-faith belief
that he is entitled to possession, there is no intent to permanently
deprive, even if that belief is both incorrect and unreasonable.
EXAMPLE: Dana accidentally takes someone else’s umbrella upon
leaving a restaurant. She is not guilty of larceny if she honestly (even
if unreasonably) believed that it was not the property of another.
2. The taking and carrying away (asportation) must concur in time with the
intent to permanently deprive (animus furandi).

C. Embezzlement
1. Embezzlement is a statutory crime defined as:
a. the fraudulent conversion or misappropriation;
(1) Conversion is some action toward property (such as selling,
consuming, pledging, donating, discarding, heavily damaging,
or claiming title to it), which seriously interferes with the rights of
the owner. Slight movement or limited use, for instance, would
not suffice, but use that deprives the owner of a significant

333
OUTLINE

portion of the usefulness of the object would be sufficient.


Conversion is not fraudulent if the defendant honestly believed
he had a right to so use the property.
(2) No direct personal gain need result to the defendant.
(3) The specific fraudulent intent required may be negated by a claim
of right or by an intent to restore the exact property.
(4) A defendant who converts property but intends to later substitute
it for equivalent property is guilty of embezzlement.
b. of the property of another;
(1) Embezzlement deals with tangible personal property, not
services, although some modern statutes include real estate.
c. by one who is already in lawful possession.
2. In contrast to larceny, embezzlement involves misappropriation by a defen-
dant who has lawful possession (as opposed to custody). So defined,
embezzlement could not overlap with larceny.
3. Many embezzlement statutes require that the property be “entrusted” to
the defendant. Such statutes have been construed as not applicable to
defendants who come into lawful possession of property by finding it or by
mistaken delivery (i.e., the property was not “entrusted” to the defendant
because it was never intended that he be given possession). Fraudulent
conversion by a co-owner of property is not embezzlement.

D. Robbery
1. The felony of robbery consists of all the elements of larceny, plus two
additional elements:
a. the taking must be from the person or presence of the victim (meaning
an area within his control); and
b. the taking must be accomplished either:
(1) by force or violence; or
(2) by intimidation or the threat of violence.
(a) If the robbery is based on the threat of violence, the threat
must place the victim in actual fear at the time of the taking.

EXAM TIP If the victim is placed in fear, though baseless, there may still be a robbery. In
other words, fear based on lies qualifies as threats for constituting robbery.
(b) The use of force must be contemporaneous with the taking
(in other words, all part of one occurrence).
(c) This element will be satisfied by slight force, but must be some-
thing more than what is required to just move the property.
2. If any of the elements of larceny are not met, a robbery did not occur.
3. Larceny, assault, and battery are all lesser-included offenses of robbery
(which means that all of the elements of the lesser offense are included
within the greater offense).

334
CRIMINAL LAW

4. The Model Penal Code definition of robbery requires that the theft be
accompanied by serious bodily injury upon another, or by threatening or
placing another in fear of immediate serious bodily injury.

E. Obtaining Property by False Pretenses


1. The statutory crime of obtaining property by false pretenses consists of:
a. a false representation of a present or past material fact by the defendant;
(1) The representation must relate to a material fact, not opinion;
“puffing” is insufficient.
b. that causes the victim to pass title to his property;
(1) The victim’s reliance upon the representation must cause him to
pass title. However, it does not need to be the only reason that
the victim passes title.
(2) The distinguishing characteristic of false pretenses is that title passes
to the defendant even though it is voidable due to the defendant’s fraud.
(a) Where money is delivered to the defendant in a sale or trade
situation by the person defrauded, title generally passes and
the crime is false pretenses.
(b) When a victim pledges money for security, bails money for
safekeeping, or gives money to be used only for a specific
purpose, and the victim takes such action as a result of the
defendant’s misrepresentations, the defendant acquires only
possession and, thus, is guilty of larceny by trick.
1) If, however, in response to the defendant’s false repre-
sentations, the victim lends money or grants title, the
defendant is guilty of false pretenses.
2) A defendant who pays for property with a worthless
check is guilty of larceny by trick, since title does not
pass until the check is cashed.
3) Where money is exchanged by cheating at cards or
betting, the defrauding party is receiving possession,
not title, so the crime is larceny by trick.
(c) The modern scope of false pretenses includes written instru-
ments, stocks, bonds, notes, and deeds, as well as money
and credit cards.
c. to the defendant;
d. who knows his representation to be false; and
(1) The defendant must know that his representation is false at the
time the victim transfers title to him.
e. intends thereby to defraud the victim.

F. Bad Checks
1. All jurisdictions have enacted bad check legislation to deal with
no-account or insufficient-funds checks given with the intent to defraud.

335
OUTLINE

The giving of the check is an implied representation of sufficient funds,


absent postdating or some other means of notification of inadequate
credit by the drawer.
2. Bad check statutes generally do not require that any property be obtained
from the victim as a result of issuing the bad check.
3. However, the requisite mental state of the drawer is often required to be
that of knowledge of the insufficient funds and the intent to defraud.

G. Credit Card Fraud


1. Obtaining property by means of a stolen or unauthorized credit card is also
a statutory crime in most jurisdictions.

H. Larceny by Trick
1. Larceny by trick is a form of larceny whereby the defendant obtains
possession of the personal property of another by means of a representa-
tion or promise that he knows is false at the time he takes possession.
2. In larceny by trick, the defendant’s fraud is used to cause the victim to
convey possession, not title (as in false pretenses).
a. In some states, there must be an intent to steal at the time of the
induced delivery; a defendant who has some lesser (but still wrongful)
intent and then subsequently converts the property to his own use may
be guilty of embezzlement. In other states, the fraud will be construed
as a “trespass,” which continues until the time of “taking” and, there-
fore, a later misappropriation would still be considered larceny by trick.
3. Factual misrepresentations and false promises are typically deemed suffi-
cient for the “fraud” element of larceny by trick.

False Pretenses Larceny by Trick Credit Card Fraud Check Fraud

• Obtains title • Obtains possession • Obtains title • Obtains title


• By representation • By representation • By stolen or • By stolen check or
or promise he or promise he unauthorized on non-sufficient
knows is fake knows is fake credit card funds account

I. Extortion
1. Extortion was a common law misdemeanor involving the corrupt demand
or receipt of an unlawful fee by a public official under color of his office.
2. Under modern statutory law, extortion is commonly called blackmail.
Blackmail is defined as obtaining the property of another by the use of
threats of future harm to the victim or his property.
a. The threat is the essence of extortion, and includes threats to
expose a person or his family to disgrace and threats to accuse the
victim of a crime.

336
CRIMINAL LAW

3. Some statutes consider the crime complete upon the making of the threats
with the specific intent to obtain money or property, while other statutes
require the threats to actually cause the victim to part with his property.
4. Unlike robbery, extortion does not require threats of immediate or imminent
physical harm, and property does not need to be taken from the victim’s
person or presence.
5. The wrong that extortion statutes attempt to prevent is the taking of the
victim’s property by threat. Thus, even where a defendant’s threat is to
reveal factually correct information that the defendant is entitled to reveal,
the crime of extortion may nonetheless be committed.
6. In many jurisdictions, threats made for the purpose of obtaining payment
for a valid debt are not considered extortion. One rationale for this is that,
since the defendant is entitled to the property, the defendant lacks the
requisite intent to extort for gain. However, other states treat threats for debt
collection as extortion to discourage creditors from using such methods to
enforce their payment rights.

J. Receiving Stolen Property


1. Receiving stolen property was a common law misdemeanor.
2. As a modern statutory crime, it is defined as:
a. the receiving of stolen property;
(1) The receiving of physical possession of the property, while the
most common situation, is not required, as long as the defendant
exercises control over the goods.
EXAMPLE: Arranging for a sale, having the thief place the goods
in a designated place, or receiving the goods from another will all
constitute exercising control sufficient for meeting this element.
b. known to be stolen;
(1) The defendant must either know or actually believe that the property
is stolen. An honest but unreasonable belief that the property is not
stolen would likely prevent a conviction for receiving stolen property.
(2) However, actual and positive knowledge is not strictly
required. Constructive knowledge, through notice of facts
and circumstances from which guilty knowledge may fairly
be inferred, will suffice. Constructive knowledge will be found
where the facts and circumstances surrounding the receipt of
the property leave no reasonable doubt that the receiver must
have known, without further inquiry, that it was stolen.
c. with the intent to permanently deprive the owner.
(1) The defendant must, at the time he receives the stolen property,
have the specific intent to permanently deprive the owner of
that property. Where a defendant intends at the time of receipt
to unconditionally return property to its owner, that defendant is
not guilty of receiving stolen property. However, conditionally

337
OUTLINE

offering to return the property—such as upon payment of a


reward—may evidence an intent to permanently deprive. The
defendant does not have to be acting for personal gain.

K. Forgery
1. Forgery is defined as:
a. fraudulent making;
(1) The making element will be satisfied by the creation of a new
document, the altering of an existing document (including improp-
erly filling in a form), or by inducing someone to sign a document
knowing that the person is unaware of the document’s significance.
(2) Completion of the “making” element completes the crime; the
defendant does not have to actually use the forged document to
be found guilty of forgery.
(3) A defendant who actually acquires property by use of the forged
document may be guilty of another crime, such as false pretenses.
EXAMPLE: Dylan wrote a will and signed the name of his ail-
ing Aunt Bunny. He then signed the names of two other family
members whom Bunny would likely choose as witnesses. Dylan
intended to use the document after Aunt Bunny died to claim a
large part of her estate. The fake will is discovered before Bunny
dies. Dylan is guilty of forgery.
b. of a false writing with apparent legal significance; and
(1) Forgery is not merely including false information in an otherwise
genuine document, but must involve a writing that is itself false.
(2) An alteration must be material (that is, change the legal meaning or
effect of the document) to qualify the action as an element of forgery.
(3) For a writing to be one with apparent legal significance, the
writing must have purpose or value beyond the document’s own
existence, such as a contract, will, negotiable instrument, deed,
or mortgage. A document that is only valuable because of its
existence, even one that is highly valuable (such as a painting or
historical document), has no effect on legal rights or duties and
does not fall within the definition of forgery.
(a) However, attempting to obtain money in exchange for an
inherently valuable document may give rise to other crimes,
such as false pretenses.
c. with the intent to make wrongful use of the forged document.
(1) The defendant must act with an intent to make wrongful use, but
not necessarily for monetary gain.
(2) Note that acting with an intent to reimburse the victim or to collect
on a good faith claim of debt does not negate the intent element.

338
CRIMINAL LAW

L. Crimes against the Habitation


1. Burglary
a. At modern law, the crime of burglary consists of:
(1) the breaking;
(a) This element typically required a slight use of force to create
an opening. Today, some statutes have deleted the breaking
element altogether.
(b) Under both the common law and modern statutes, this
element may be satisfied where a defendant gains
entry by fraud, deception, or threat of force (known as
constructive breaking).
(2) and entering;
(a) Entry through an open door where the defendant later opens
an inside closet door is sufficient.
(b) Breaking into a trunk or safe, or breaking to exit rather than
to gain entry, is insufficient.
(c) Entry is achieved by placing any portion of the body inside
the structure.
(d) Insertion of a tool is sufficient for an entry if it is used to accom-
plish the felony, but insufficient if it is used merely to gain entry.
EXAMPLE: Inserting a tool merely to unlock the door is not
an entry, as opposed to shooting a bullet through a window
intending to kill the victim, which is an entry.
(e) Breaking and entering are two elements that may occur at
different times but must be related.
EXAMPLE: If the defendant breaks a window with his gun,
then sees an open window and climbs in, there would be no
“breaking and entering.” However, if that defendant breaks the
window with his gun, then goes to buy a heavy jacket to wear
so the broken glass will not hurt him, and comes back the next
night wearing the jacket and climbs through the same broken
window, there would be a breaking and entering.
(f) The breaking and entering must both be done without the
occupant’s consent (i.e., trespassory). Where someone with
the authority to enter breaks and enters, this element will not
be met. A business open to the public is generally deemed to
have extended an open invitation to enter during regular busi-
ness hours. There is a split of authority, however, regarding
people who enter businesses with the intent to steal or
commit other felonies. Some states find the entry nonetheless
consensual, while others view the undisclosed intent to steal
as a bar to consent and deem the entry a trespass.

339
OUTLINE

(3) of the structure;


(a) At common law, this element required that the structure be
a dwelling house, and this was interpreted literally. Only a
structure used as a home where people slept would meet the
definition. The dwelling could be occupied or unoccupied at
the time of entry. This element also encompassed structures
within the area immediately surrounding the dwelling (the
“curtilage”), such as a storage shed or greenhouse.
(b) Modern statutes have extended this element to include
most any structure regardless of whether it is a dwelling,
and in some jurisdictions regardless of whether it has a roof,
such as a fenced lumberyard.
(4) of another;
(5) with the intent to commit a felony or theft offense therein.
(a) Burglary requires that the breaking and entering be accom-
panied by a simultaneous intent to commit a felony or theft
offense therein. Under some modern statutes, an intent to
commit a misdemeanor will suffice. An intent formed after
entry is insufficient, absent an additional “entry” once inside.
EXAMPLE: A defendant who has no intent to steal when en-
tering a house but forms such intent once inside, and subse-
quently opens the bedroom door with the intent to steal items
inside the bedroom has committed an additional “entry.”
(b) A defendant who commits or attempts the intended felony or
theft offense can be convicted of both crimes. In addition, the
burglary is complete once the defendant enters with sufficient
criminal intent regardless of whether the defendant succeeds
or even continues the effort to complete the intended crime.
b. At common law, burglary had an additional element requiring that the
crime be committed “in the nighttime,” between sunset and sunrise.
However, modern statutes have largely eliminated this element.
2. Arson
a. At modern law, the crime of arson consists of:
(1) the malicious;
(a) The defendant need not have intended to burn the dwelling.
Malice requires only that the defendant intentionally took an
action that involved a substantial risk of burning.
(2) burning;
(a) With respect to the burning requirement, it was not necessary
that the dwelling be substantially or totally damaged. Although
a mere blackening of the surface was not enough, there must
have been some charring (i.e., slight burning) of the premises.
Additionally, mere burning of the furniture or other contents of the
dwelling, without fire damage to the structure itself, is not arson.

340
CRIMINAL LAW

(3) of property.
(a) At common law, arson required that the property burned be
a dwelling house. However, like burglary, the common law
requirement of a dwelling has been expanded in a majority
of states to include virtually any structure.
(b) The modern definition has been expanded to include most build-
ings, public or private, as well as vessels and personal property.
b. At common law, arson additionally required that the property burned be
owned “by another.” However, modern statutes have done away with
this and expanded the scope of arson to include situations where an owner
maliciously burns his own structure or property (such as for insurance fraud).

M. Possession Offenses
1. Controlled Substances
a. The Uniform Controlled Substances Act (“UCSA”) was first published
in 1970, and was adopted by almost every state as the basis for state
drug laws. The UCSA was most recently revised in 1994, and this
version has been adopted by more than half of the states.
b. Under the Uniform Controlled Substances Act, except as otherwise autho-
rized, a person may not knowingly or intentionally manufacture, distribute,
or deliver a controlled substance, or possess a controlled substance
with intent to manufacture, distribute, or deliver [UCSA § 401(a)].
c. At federal law, except as otherwise authorized, it is unlawful for any
person knowingly or intentionally [21 U.S.C.S. § 841(a)]:
(1) to manufacture, distribute, or dispense, or possess with intent to
manufacture, distribute, or dispense, a controlled substance; or
(2) to create, distribute, or dispense, or possess with intent to
distribute or dispense, a counterfeit substance.
d. It is also unlawful for any person knowingly or intentionally to possess
a controlled substance unless such substance was obtained directly,
or pursuant to a valid prescription or order, from a practitioner, while
acting in the course of his professional practice, or except as otherwise
authorized by statute [21 U.S.C.S. § 844(a)].
e. Both state and federal laws vary in level of offense and severity of
punishment depending on the substance and amount involved.
2. Firearms
a. The National Firearms Act [26 U.S.C.S. §§ 5801, et seq.] prohibits the
possession of certain firearms and accessories in violation of the registra-
tion, authorization, and notification requirements that the Act sets forth.
b. Registration of Importers, Manufacturers, and Dealers
(1) On first engaging in business and thereafter on or before the first
day of July of each year, each importer, manufacturer, and dealer
in firearms must register with the state in which the business is
located his name, including any trade name, and the address of

341
OUTLINE

each location in the district where he will conduct such business


[26 U.S.C.S. § 5802].
(2) An individual required to register under this section shall include a
photograph and fingerprints of the individual with the initial application.
(3) Where there is a change during the taxable year in the location of,
or the trade name used in, such business, the importer, manufac-
turer, or dealer shall file an application to amend his registration.
(4) Firearms operations of an importer, manufacturer, or dealer may
not be commenced at the new location or under a new trade
name prior to approval of the application.
c. Prohibited Acts
(1) It is unlawful for any person to [26 U.S.C.S. § 5861]:
(a) to engage in business as a manufacturer or importer of, or
dealer in, firearms without having paid the special (occu-
pational) tax required by Section 5801 for his business or
having registered as required by Section 5802;
(b) to receive or possess a firearm transferred to him in
violation of the provisions of this chapter;
(c) to receive or possess a firearm made in violation of the provi-
sions of this chapter;
(d) to receive or possess a firearm which is not registered to him
in the National Firearms Registration and Transfer Record;
(e) to transfer a firearm in violation of the provisions of this chapter;
(f) to make a firearm in violation of the provisions of this chapter;
(g) to obliterate, remove, change, or alter the serial number or
other identification of a firearm required by this chapter;
(h) to receive or possess a firearm having the serial number
or other identification required by this chapter obliterated,
removed, changed, or altered;
(i) to receive or possess a firearm which is not identified by a
serial number as required by this chapter;
(j) to transport, deliver, or receive any firearm in interstate commerce
which has not been registered as required by this chapter;
(k) to receive or possess a firearm which has been imported or
brought into the United States in violation of Section 5844; or
(l) to make, or cause the making of, a false entry on any
application, return, or record required by this chapter,
knowing such entry to be false.
(2) In order to be convicted, the defendant must be aware that he
or she in fact possesses something, whether the possession be
actual or constructive, and that the thing possessed is in fact
a “firearm” of some sort. However, it is unnecessary that the
accused know either that the item possessed is unregistered or

342
CRIMINAL LAW

even subject to registration [United States v. Freed, 401 U.S. 601


(1971); United States v. Javino, 960 F.2d 1137 (2d Cir. 1992)].
(3) Actual possession of a firearm exists when a person has direct
physical control over the firearm. Constructive possession is
established when a person, though lacking physical custody of
the firearm, still has the power and intent to exercise control over
the firearm [Henderson v. United States, 135 S. Ct. 1780 (2015)].

343
OUTLINE

IV. INCHOATE CRIMES

A. Solicitation
1. Solicitation was a common law misdemeanor including enticing, advising,
inciting, inducing, urging, or otherwise encouraging another to commit a
felony or breach of the peace.
2. The MPC defines solicitation broadly to include requesting another to
commit any offense (including misdemeanors and felonies).
3. The solicitor must intend that the solicitee perform criminal acts (thus the
solicitor must have a specific intent). Mere approval of the acts is not enough.
4. The offense is complete at the time the solicitation is made. In other words,
completion of the offense solicited is unnecessary.
5. It is also unnecessary that the person solicited enter into an agreement to
commit the requested crime. Indeed, a person solicited to do a crime may
not even respond, but the solicitor will still be guilty of solicitation.
6. A solicitor is treated as an accessory before the fact, and thus will be guilty
of any solicited crime (whether attempted or completed) by the solicitee.
7. The crime of solicitation, unlike conspiracy, merges with the target felony
upon completion of the crime.
8. A solicitor may not be guilty of the offense if he could not be convicted of the
underlying crime—for instance, where the solicitor is a member of the class of
persons the law seeks to protect (e.g., a minor, in relation to statutory rape).
9. Defenses
a. At common law, there were no defenses to solicitation if the elements
described above were present. However, under modern statutes,
specific intent defenses such as voluntary intoxication and unreason-
able mistake of fact are legitimate defenses to solicitation.
b. Even where it would be impossible for the solicitee to carry out the
crime, such impossibility is no defense to the crime of solicitation.
What matters is what the solicitor believes the circumstances to be,
not what they actually are.
c. Because the crime of solicitation is complete as soon as the solicitation
is made, withdrawal cannot be a defense to the crime of solicitation.
Even where the solicitor later changes his mind, the change of heart may
be a defense to the underlying crime, but not to the solicitation crime.

B. Attempt
1. The crime of attempt consists of two elements:
a. a specific intent to bring about a criminal result; and
b. a significant overt act in furtherance of that intent.
2. Once the target crime is committed, the attempt merges into the target crime.
3. The specific intent requirement for attempt liability—the require-
ment that the defendant specifically intended to bring about a criminal

344
CRIMINAL LAW

result—means that, in some cases, an attempt conviction requires more


proof than a completed offense.
EXAMPLE: A and B attack C. A intends to kill C, but B intends only to
seriously injure him. If C does not die, then only A can be convicted of at-
tempted murder. Even in a jurisdiction that recognizes murder based on a
mens rea, which itself does not require the specific intent to kill (e.g., mur-
der based on the intent to seriously injure), to be guilty of attempted murder,
one still must have a specific intent to kill.
4. The significant overt act must be an act (or act of perpetration) beyond an
act of “mere” preparation.
a. Several approaches are used to determine what constitutes a suffi-
cient overt act.
b. At common law, a defendant was required to have performed the “last
act” necessary to achieve the intended result.
c. Today, acts prior to the last act are usually sufficient. Many courts apply
a “proximity” test, which asks how close in time and physical distance a
defendant was to the time and place the target crime was to be committed.
Some courts use an “equivocality” test, which requires that the defendant’s
conduct be such that it can have no other purpose than the commission of
the crime attempted. Jurisdictions following the MPC require only an act that
constitutes a substantial step toward the offense that corroborates the crim-
inal intent required, such as scouting out the scene of the intended crime.
EXAMPLE: Defendant intends to kill Victim by tying him up and push-
ing him into the pool. At common law, Defendant likely would have to
push the tied-up Victim into the pool to be guilty of attempted murder.
Today, tying up the Victim next to the pool without pushing him in would
likely be sufficient under all of the tests discussed above.
5. Defenses
a. Abandonment
(1) At common law, abandonment is not a defense to attempt once
the attempt is complete.
(2) Many jurisdictions (including MPC jurisdictions) recognize a
voluntary and complete abandonment as a defense.
(a) Voluntary means a true change of heart, not simply giving up in
the face of difficulties or an increased likelihood of being caught.
(b) Complete means that the defendant is not merely post-
poning commission of the crime.
b. Legal Impossibility
(1) Legal impossibility is traditionally regarded as a defense to attempt
and involves the situation where the defendant did all those things
he intended to do, but his acts did not constitute a crime.
(2) Factual impossibility (e.g., shooting into an empty house) is not a
defense to attempt.

345
OUTLINE

C. Conspiracy
1. A conspiracy exists when there is:
a. an agreement between two or more persons to commit a crime; and
b. an overt act (committed by any conspirator) in furtherance of the conspiracy.
2. The agreement itself is the actus reus or conduct element of the offense
of conspiracy.
a. Actual agreement is required. Feigned agreement, as in the case of an
undercover police officer, is insufficient.
b. The agreement need not be express, but may be proved circumstan-
tially, such as by conduct where the conspirators demonstrate over
time that they intended to achieve the same objective and agreed to
work together toward that end.
(1) The agreement does not require a meeting of the minds, but
merely a shared intent to pursue a mutual goal.
c. The objective of the agreement must be to either commit a crime or to
commit a lawful act by unlawful means.
3. Conspiracy is a specific intent crime. The mental state required is both the
intent to agree and the intent to achieve the objective of the agreement.
4. Overt Act Requirement
a. The majority rule is that an overt act in furtherance of the conspiracy is
also required for conspiracy liability.
b. This prevailing view reflects a change from the traditional common law rule,
which had no such requirement (the agreement itself constituted the crime).
c. The overt act requirement for conspiracy need not be criminal or
unlawful, and need only be committed by one member of the conspiracy.
d. Mere preparation will suffice.
EXAMPLE: Purchasing matches in a conspiracy involving the burning
of a building or driving by a house to observe if someone is home in a
conspiracy involving theft.
5. Unlike attempt, conspiracy is a separate and distinct offense that does not
merge upon completion of the target crime because criminal combinations
are deemed to be dangerous apart from the underlying crime itself.
6. All jurisdictions recognize liability for crimes a conspirator aids and abets
under the doctrine of accomplice liability.
7. Where a conspirator does not have the sufficient mens rea for liability as an
accomplice, the Pinkerton doctrine provides that each conspirator is liable
for the crimes of all the other co-conspirators where the crimes were both:
a. a foreseeable outgrowth of the conspiracy; and
b. committed in furtherance of the conspiratorial goal.
8. Most jurisdictions have now accepted the Pinkerton doctrine and it is the
majority rule, imposing liability on a conspirator for the crimes of all the
other co-conspirators where the crimes were both:

346
CRIMINAL LAW

a. a foreseeable outgrowth of the conspiracy; and


b. committed in furtherance of the conspiratorial goal.
9. The nature of the agreement determines whether there is a single
conspiracy or multiple conspiracies.
a. In a chain relationship, where several crimes are committed under
one large scheme in which each member knows generally of the other
parties’ participation and a community of interest exists, one single
conspiracy results.
b. Alternatively, in the so-called hub-and-spoke relationship, where one
common member enters into agreements to commit a series of inde-
pendent crimes with different individuals, multiple conspiracies exist.
10. Procedural Issues
a. An acquittal of one co-conspirator traditionally results in the acquittal of
a single remaining co-conspirator because at least two guilty parties are
required for a conspiracy conviction. However, it is not necessary to try
more than one conspirator. Thus, a single conspirator could be tried and
convicted of conspiracy if his co-conspirators were missing or dead.
b. The common law rule is that no conspiracy exists if only one of the
conspirators truly agrees and the other conspirator(s) feign agreement
(i.e., there must be a “meeting” of at least two “guilty minds”).
c. The MPC, however, follows the unilateral approach to conspiracy and
dispenses with the requirement that two people actually agree. Under
the unilateral approach, all of the co-conspirators, having feigned
agreement or having been acquitted will not alone, prevent a defendant
from a conviction for conspiracy.
d. The Wharton Rule states that in crimes where two or more people are
necessary for the commission of the offense, there is no conspiracy
unless the agreement involves an additional person who is not essen-
tial to the definition of that crime.
EXAMPLE: If A and B engage in a duel, they are guilty of the crime of
dueling, but not of conspiracy to duel.
(1) Wharton Rule crimes include:
(a) bigamy;
(b) incest;
(c) gambling;
(d) giving and receiving bribes;
(e) adultery; and
(f) dueling.
e. Because a corporation and its agent are considered one person, no
conspiracy can exist between them.
f. Likewise, a member of a legislatively protected class cannot be guilty of
conspiracy to commit that crime (e.g., a minor female cannot conspire with
a male to commit statutory rape) or of being an accessory to a crime.

347
OUTLINE

g. A person can be guilty of conspiracy to commit a crime that he could


not commit himself.
11. Defenses
a. Impossibility is not a defense to conspiracy.
b. The general rule is that withdrawal is not recognized as a valid defense
to conspiracy because the conspiracy is complete as soon as the
parties agreed to commit the crime and an overt act is committed.
c. Withdrawal may cut off further liability for crimes committed in further-
ance of the conspiracy if, however, the withdrawing conspirator
communicates his withdrawal to each of the co-conspirators.
d. Under the MPC, withdrawal by a co-conspirator may be a valid affirma-
tive defense to the charge of conspiracy itself where the renouncing
party gives timely notice of his plans to all members of the conspiracy
and performs an affirmative act to “thwart” the success of the conspiracy.

348
CRIMINAL LAW

V. PARTIES TO CRIME; ACCOMPLICE LIABILITY

A. Accomplice
1. The common law traditionally distinguished between accessories before
and after the fact and principals in the first and second degree.
2. The modern approach is to distinguish only between accomplices to a crime
and accessories after the fact. Under the modern approach, an accomplice
to a crime is fully liable for the crime being committed.
EXAMPLE: An accomplice to a bank robbery is a bank robber.
3. An individual is criminally liable as an accomplice if he gives assistance
or encouragement or fails to act where he has a legal duty to oppose
the crime of another (actus reus), and purposefully intends to effectuate
commission of the crime (mens rea).
4. The Act Requirement
a. Even slight assistance or encouragement is sufficient, but the defen-
dant must actually assist or encourage. The assistance or encourage-
ment need not be a cause-in-fact of the commission of the crime.
EXAMPLE: A flashes B a thumbs-up sign just before B beats C. B
would have beaten C anyway, but A’s act is sufficient for complicity if B
saw it and felt even slightly encouraged.
b. Mere presence does not make one an accomplice.
c. Words alone may be sufficient if they assisted or encouraged.
d. The MPC recognizes complicity liability for one who attempts to aid or
encourage but does not actually aid or encourage.
5. Mental State Requirement
a. At common law, the person must intend to commit the acts of assis-
tance or encouragement and must further intend to encourage or assist
another to commit the crime charged.
b. A minority of modern statutes create accomplice liability with a lower
mental state—that of knowingly assisting or encouraging a crime, such
as in cases where a seller, knowing of the buyer’s intent to commit
arson, sells him an explosive device.
6. An accomplice is responsible not just for the crime assisted and encour-
aged but also for other crimes that are the natural and probable conse-
quences of the crime assisted or encouraged.
a. The MPC limits accomplice liability to only the crime assisted or encouraged.
EXAMPLE: An accomplice to an armed bank robbery can be held
responsible for the death of a bank teller who is shot by another defen-
dant during the bank robbery.
7. A defendant may be guilty as an accomplice for crimes he could not
commit alone.

349
OUTLINE

8. An individual is not liable as an accomplice for the acts of a “false accom-


plice” (i.e., an undercover police officer).
9. Withdrawal or Abandonment
a. An accomplice may sever liability for future crimes by withdrawal
or abandonment.
b. He must give no further assistance or encouragement.
c. He must communicate his withdrawal to his accomplices.
d. Many jurisdictions require him to make efforts to neutralize his prior
assistance or encouragement, but he does not have to try to thwart the
commission of the crime.
EXAMPLE: A had agreed to lend his car to B so that B could rob a
bank. A tells B that he changed his mind. A needs to get his car keys
back from B to cut off accomplice liability in the event that B goes
ahead with the robbery, but he does not need to call the police and tell
them of B’s plan.

B. Principal in the First Degree


1. The actual perpetrator who performs the criminal act with the requisite
mental state is known as the principal in the first degree. More than one
person can perpetrate the same crime.

C. Principal in the Second Degree


1. One who is present at the scene of the felony and aids, abets, or otherwise
encourages the commission of the crime with the requisite intent is guilty as
a principal in the second degree.
EXAMPLE: A getaway car driver who waits outside during a bank robbery.
a. Mere presence without assistance or assistance without intent, are
both insufficient.
b. A principal in the second degree may be punished to the same extent
as the perpetrator.

D. Accessory before the Fact


1. One who aids, abets, counsels, or otherwise encourages the commis-
sion of a felony, but is not present at the scene, is guilty as an accessory
before the fact.
2. An accessory before the fact may be punished to the same extent as a prin-
cipal for all crimes committed within the scope of the conspiracy.
3. Conviction of the Principal
a. Under the majority view, the principal need not be convicted in order
for the accessory before the fact to be convicted.
b. At common law, conviction of the principal was required for conviction
of an accessory.

350
CRIMINAL LAW

E. Accessory after the Fact


1. The following three requirements must be met for a person to be an acces-
sory after the fact:
a. a completed felony must have been committed;
b. the accessory must have known of the commission of the felony; and
c. the accessory must have personally given aid to the felon to hinder the
felon’s apprehension, conviction, or punishment.
2. Whereas modern law has abolished the distinction between principals in the first
and second degree, one who is an accessory after the fact is not an accomplice,
nor is he punished to the same extent as prescribed for the parties to the felony.
3. Modern statutes largely focus on punishing one for his own actions (which these
statutes classify as a separate crime called “obstruction of justice”), and not by
holding the “accessory after the fact” vicariously liable for the underlying felony.

351
OUTLINE

VI. DEFENSES

A. Responsibility
1. Insanity
a. If the defendant is insane at the time of his criminal act, no criminal liability
will be imposed. Four tests for insanity are followed in various jurisdictions.
(1) The M’Naghten test focuses on the defendant’s reasoning abili-
ties. Under this test, a defendant is relieved of criminal responsi-
bility upon proof that, at the time of commission of the act, he was
laboring under such a defect of reason from a disease of the mind
as not to know the nature and quality of the act he was doing, or if
he did know it, not to know that what he was doing was wrong.
(a) Disease of the mind includes all mental abnormalities, but
not a psychopathic personality.
(b) There is a split of authority in M’Naghten jurisdictions as
to whether “wrong” includes both legal and moral wrongs.
Some jurisdictions will find that a defendant is not insane
where, even though the defendant believes the act was
not morally wrong, he knows it to be illegal. Others will find
defendants who do not know an act was morally wrong to be
insane, regardless of their belief as to the act’s legality.
EXAMPLE: Following what Defendant actually believed
to be a higher power’s command to kill all law professors,
Defendant waited outside his criminal law final exam to kill
his professor. Defendant is considered insane under the
M’Naghten test.
(2) Under the irresistible impulse test, a defendant will be found
not guilty where he had a mental disease that kept him from
controlling his conduct.
(3) Under the Durham (or New Hampshire) rule, also known as
the “product” rule, a defendant is not criminally responsible if his
unlawful act was the product of a mental disease or defect (meaning
it would not have been committed “but for” the defect or disease).
(4) Under the Model Penal Code test (or substantial capacity),
a person is not responsible for criminal conduct if, at the time of
such conduct, as a result of mental disease or defect, he lacked
substantial capacity to appreciate the criminality (wrongfulness)
of his conduct or to conform his conduct to the requirements
of law. It is typically easier to establish MPC insanity because
it does not require the total or absolute loss of cognitive ability
or volition, only that the defendant lacked substantial capacity
in these areas. It also combines a cognitive component (not
knowing) with a volitional component (not controlling) and allows
a verdict of insanity for either one.

352
CRIMINAL LAW

(a) “Mental disease or defect” does not include abnormalities mani-


fested only by repeated criminal or otherwise antisocial conduct.
EXAMPLE: Kleptomania (a compulsion to steal) will not con-
stitute a mental disease or defect under the MPC test.
b. Procedural Issues
(1) A defendant is presumed sane until such time as he goes forward
by raising evidence as to his sanity.
(2) A not guilty plea at arraignment does not waive the right to raise
the insanity defense at a later time.
(3) Insanity Defense at Federal Level
(a) Pursuant to federal law, it is an affirmative defense to a
prosecution under any federal statute that, at the time of
the commission of the acts constituting the offense, the
defendant, as a result of a severe mental disease or defect,
was unable to appreciate the nature and quality or the
wrongfulness of his acts [18 U.S.C. § 17(a)].
(b) Mental disease or defect does not otherwise constitute a
defense [Id.].
(c) The defendant has the burden of proving the defense of
insanity by clear and convincing evidence [18 U.S.C. § 17(b)].
(4) Insanity Defense at State Level
(a) States have adopted varying approaches to the defense
of insanity for violations of state laws. The majority rule is
that the defendant has the burden of proving the affirmative
defense of insanity by a preponderance of the evidence.
(b) The federal approach for insanity has only been adopted in
a few states as the approach used for violations of state law.
2. Competency
a. Competency to stand trial is very different from a mental incapacity.
b. Incompetency typically refers to a doctrine that prevents defendants
from being tried, convicted, or punished unless they have the
sufficient present ability to consult counsel with a reasonable and
rational understanding of the proceedings.
c. If a defendant is found incompetent, he can later be tried and
punished if competency is restored.
(1) Insanity concerns the defendant’s mental state at the time the
offense is committed; competency is assessed at any time
during the pendency of the criminal case in court.
d. Burden of Proof
(1) The burden of proof in all courts is a preponderance of the evidence,
pursuant to 18 U.S.C. § 4241 and its interpretive case law.
(2) If, after the hearing, the court finds by a preponderance of
the evidence that the defendant is presently suffering from a

353
OUTLINE

mental disease or defect rendering him mentally incompetent


to the extent that he is unable to understand the nature and
consequences of the proceedings against him or to assist
properly in his defense, the court shall commit the defendant to
the custody of the Attorney General.
(3) In the federal system, the burden of proof falls on the party
making the motion to determine competence.
(4) However, the Supreme Court has ruled that the state may place
the burden of proof on the defendant when determining compe-
tency, but for due process reasons, it cannot increase the level of
evidence required to be proved. Therefore, whether in federal court
or state court, the burden of proof when determining competency is
a preponderance of the evidence [Cooper v. Oklahoma, 517 U.S.
348 (1996); Medina v. California, 505 U.S. 437 (1992)].
3. Diminished Capacity
a. Some jurisdictions allow the defense of diminished capacity, which is
short of insanity, to prove that as a result of a mental defect, the defen-
dant did or did not have a state of mind that is an element of the offense.
b. When pleading diminished capacity, the defense is used to negate a
specific mental state required for the particular crime.
4. Intoxication
a. Voluntary or involuntary intoxication (whether brought about by alcohol
or by narcotic drugs) is a defense to a crime when it negates the exis-
tence of an element of the crime.
(1) Under the prevailing view, voluntary intoxication may be a valid
defense for a specific intent crime if it negates the requisite mental
state, and may negate a purposeful or knowing mental state.
However, voluntary intoxication is not a defense to general intent
crimes and will not negate recklessness, negligence, or strict liability.
(2) Involuntary intoxication is a defense to a crime, even if it does
not negate an element of the crime, under the same circum-
stances as insanity.
(3) The excessive use of alcohol or drugs may bring about real insanity, in
which case the rules concerning insanity as a defense to crime govern.
EXAMPLE: Defendant voluntarily drank himself silly, then drove
down to the harbor and snuck aboard Victim’s yacht. Victim
ordered Defendant off the boat. In a rage-filled response, Defen-
dant grabbed some rope and strangled Victim to death. Defen-
dant will not be excused.
5. Infancy
a. At common law, a complete defense due to incapacity existed for chil-
dren under seven years of age. Children between the ages of seven
and 14 were rebuttably presumed to lack criminal capacity. Children
over age 14 were held responsible as adults.

354
CRIMINAL LAW

b. Many states have abolished the common law presumptions and estab-
lished a specific minimum age required for a criminal conviction.

B. Justification
1. Where an act is justified, no crime has been committed, notwithstanding
what might otherwise be a criminal result.
2. Self-Defense
a. If a person has a reasonable belief that he is in imminent danger of unlawful
bodily harm, he may use that amount of force in self-defense that is reason-
ably necessary to prevent such harm, unless he is the initial aggressor.
(1) Keep in mind that the force must be proportional to the initial
attack, and the initial attack must be wrongful.
b. Deadly force is that which threatens death or serious bodily harm.
(1) Deadly force can only be used in self-defense in response to an
attack that threatens death or great bodily injury.
c. Non-deadly force threatens only bodily harm.
EXAMPLE: If Abe takes a swing at Bob, Bob is legally justified in fight-
ing back with non-deadly force. If Abe swings with a butcher knife, Bob
is legally justified in shooting Abe with a gun if he cannot avoid the
potentially lethal harm through non-deadly force.
d. Reasonable force is that amount of force that is necessary to avoid
the threatened harm. In other words, if non-deadly force would stop a
deadly attack, responding with deadly force is not reasonable.
e. An aggressor is one who strikes the first blow or commits a crime
against the victim. The aggressor can regain the right of self-defense in
either of two ways:
(1) upon complete withdrawal perceived by the other party; or
(2) escalation of force by the victim of the initial aggression.
EXAMPLE: If Abe takes a swing at Bob, and Bob responds by
shooting at Abe with a gun, Abe is legally justified in swinging his
butcher knife at Bob.
f. The general rule is that a person who did not initiate conflict has no
duty to retreat in the face of a deadly attack.
g. However, a number of jurisdictions do require retreat if it is feasible and
can be done in safety before using deadly force.
(1) Those jurisdictions that require retreat before using deadly force
do not require retreat if it cannot be done safely or if the defendant
is in his own home, auto, or workplace. Even if in one of these
places, however, a defendant in a retreat jurisdiction must retreat
before using deadly force if the defendant is the aggressor.
(2) Retreat jurisdictions are split as to whether one must retreat in
one’s own dwelling if attacked by a co-dweller.

355
OUTLINE

h. It is never self-defense to kill in order to retaliate or seek revenge for a


wrongful attack once the attack is over.
3. Defense of a Third Person
a. Generally, a person is justified in using force to defend a third person
to the same extent that that person would be justified in using force to
defend himself.
b. Using lethal force is lawful when necessary to defend a third person who
is facing an immediate and wrongful deadly attack. As with self-defense,
however, the amount of force used must be proportional to the initial attack.
In other words, if something short of deadly force would effectively ward off
the attack, deadly force is not necessary and, therefore, not reasonable.
c. The majority rule focuses on the reasonableness of the defendant’s belief
that the third person was being unlawfully attacked. If the third person is
also the aggressor or a felon resisting lawful arrest, and the defendant
reasonably but mistakenly uses lethal force against the victim to protect
that third person, the defendant may nonetheless claim the justification
of defense of others. The minority rule is that the defendant has no more
right to use deadly force than the third person ostensibly being protected.
d. Under most modern laws, the third person does not have to have a
special relationship to the defendant; a defendant may use deadly
force to protect a total stranger.
EXAMPLE: For no reason, Greg pulled a knife and attacked Peter.
Peter struggled with Greg and succeeded in disarming Greg. Just as
Peter was obtaining control of the knife, Bobby walked around the cor-
ner, saw his friend Greg, and reasonably thought that Greg was about
to be stabbed by Peter. Bobby pulled a pistol and shot and killed Peter.
In a minority jurisdiction, Bobby would not be excused.
4. Defense of Property
a. Reasonable, non-deadly force is justified in defending one’s property
from theft, destruction, or trespass where the defendant has a reason-
able belief that the property is in immediate danger and no greater
force than necessary is used.
(1) This means that the use of non-deadly force is improper where a
request to desist would suffice.
b. Non-deadly force is also proper when used to reenter real prop-
erty or regain possession of wrongfully taken personal property upon
“immediate pursuit.”
c. Deadly force may never be used to merely defend property.
(1) However, deadly force may be used where the defender reason-
ably believes an entry will be made or attempted in his dwelling
by one intending to commit a felony therein.
5. Necessity
a. Reasonable force is justified to avoid imminent injury resulting from natural
(non-human) forces or where an individual reasonably believes that his

356
CRIMINAL LAW

criminal conduct is necessary to avoid a “greater harm” that would result


from compliance with the law (e.g., A kills B to save C and D).
(1) This defense also protects military personnel acting within their
duties and public executioners.
EXAMPLE: A pilot is generally legally justified in crash-landing his
disabled plane on a highway, even knowing it will kill several motorists.
b. There is no defense of necessity where the defendant is at fault in
creating the perilous situation.
EXAMPLE: If the pilot failed to ensure proper maintenance of his
plane, he may be criminally liable for harm to others in a crash landing.
6. Law Enforcement Defenses
a. Police
(1) A police officer may use that amount of non-deadly force that he
reasonably believes necessary to effect a lawful arrest or prevent
the escape of the arrestee.
(2) A police officer may use deadly force only to prevent the commis-
sion of a dangerous felony or to effectuate an arrest of a person
reasonably believed to have committed a felony where it reason-
ably appears necessary to the officer.
b. Private Citizens
(1) A private citizen is privileged to use that amount of non-deadly force
that reasonably appears necessary to prevent the commission of a
felony or a misdemeanor amounting to a breach of the peace.
(2) A private citizen may use non-deadly force to make an arrest if
the crime was, in fact, committed, and he reasonably believes the
person against whom he uses the force committed the crime.
(3) A private citizen may use the same amount of deadly force as a
police officer only if a dangerous felony is involved and the person
against whom he used the force is actually guilty of the crime.
(4) A proper mistake—meaning one that is both made in good faith
and reasonable—as to the use of lethal force may justify a homi-
cide committed in self-defense or to prevent the commission of
a dangerous felony. However, a private citizen-defendant who
mistakenly uses deadly force to prevent the escape of a fleeing
felon is not justified. Thus, if no dangerous felony has actually
been committed, or if the victim was not actually the felon, the
defendant will be criminally liable.

NOTE Unlike private citizens, police officers who mistakenly use deadly force in
such fleeing felon situations may be justified.

c. Resisting Unlawful Arrest


(1) A defendant may use reasonable, non-deadly force to resist an
unlawful arrest. A minority of jurisdictions do not permit even

357
OUTLINE

non-deadly force in order to resist an unlawful arrest. Most juris-


dictions permit an individual to use force to resist an arrest made
with excessive force, especially if the individual fears injury.
(2) An individual may only resist a lawful arrest by a police officer where
the individual does not know that the other person is a police officer.
7. Duress
a. The defense of duress justifies criminal conduct where the defendant
reasonably believes that the only way to avoid unlawful threats of great
bodily harm or imminent death is to engage in unlawful conduct. For duress,
the threat comes from human forces rather than from forces of nature.
b. Duress, or coercion, is not available as a defense to murder.
8. Public Duty
a. A public official, police officer, or private citizen offering assistance is justified
in using reasonable force against another or in taking the property of another
provided that he is acting within his authority pursuant to a law, court
order, or process that is valid or that he reasonably believes to be valid.
9. Domestic Authority
a. The parents of a minor child, or one in loco parentis, may justifiably use
reasonable force on the child to promote the child’s welfare.
b. Reasonableness is determined in light of the child’s age, sex, health,
and particular misconduct based on the totality of the circumstances.
c. This defense is available in some other situations where similar responsi-
bility lies, such as a ship’s captain for his crew or a warden for his prisoners.

C. Entrapment
1. The defense of entrapment exists where the criminal act is the product of
creative activity originating with law enforcement officials and the defendant
is in no way predisposed to commit the crime.
2. Government officials may, however, encourage criminal activity by providing
the opportunity or the equipment for the commission of a crime.
3. A defendant’s past criminal record is relevant in proving predisposition.

D. Mistake
1. Mistake of Fact
a. Mistake of fact is a defense where it negates the existence of a
mental state required to establish a material element of the crime. In
other words, there would be no crime if the facts were such as the
defendant thought them to be.
(1) To negate the existence of general intent, a mistake of fact must
be reasonable to the extent that, under the circumstances, a
reasonable person would have made that type of mistake.
(2) To negate the existence of specific intent, a mistake of fact need
not be reasonable. It may be unreasonable, provided it is honest.

358
CRIMINAL LAW

b. Mistake or ignorance of fact is not a defense to a strict liability crime,


which itself requires no mental state.
EXAMPLE: It is not a defense to statutory rape that the defendant
thought the victim was of age.
2. Mistake of Law
a. Generally, where the defendant is unaware that his acts are criminally
proscribed, such ignorance of the law is not a defense.
b. Exceptions include situations:
(1) where a statute proscribing the defendant’s conduct has not been
reasonably made available or where the defendant has reason-
ably relied on a statute or judicial decision that is later overruled
or declared unconstitutional;
(2) where a defendant relies in good faith upon an erroneous official
statement of law contained in an administrative order or in an
official interpretation by a public officer or department; and
(3) where some element of a crime involves knowledge or aware-
ness of the law by the defendant.

E. Consent
1. Consent of the victim is not a defense to a crime except when it negates a
specific element of the offense, such as in rape or kidnapping.

F. Condonation
1. Subsequent forgiveness by the victim is generally not a defense to the
commission of a crime.

359
360
Real Property
362
TABLE OF CONTENTS

I. OWNERSHIP OF REAL PROPERTY

Present Possessory Estates........................................................................................................364


Future Interests............................................................................................................................368
Cotenancy....................................................................................................................................379
Landlord-Tenant...........................................................................................................................382
Special Problems.........................................................................................................................394

II. RIGHTS IN LAND

Covenants at Law and Equity......................................................................................................403


Easements, Profits, and Licenses................................................................................................406
Fixtures........................................................................................................................................411
Zoning..........................................................................................................................................413
Support Rights.............................................................................................................................419
Water Rights.................................................................................................................................419

III. CONTRACTS

Creation and Construction...........................................................................................................421


Marketability of Title.....................................................................................................................424
Equitable Conversion and Risk of Loss.......................................................................................425
Options and Rights of First Refusal.............................................................................................425
Fitness and Suitability..................................................................................................................425
Merger..........................................................................................................................................427

IV. MORTGAGES/SECURITY DEVICES

Types of Security Devices............................................................................................................428


Some Security Relationships.......................................................................................................430
Transfers by Mortgagor................................................................................................................433
Transfers by Mortgagee...............................................................................................................433
Discharge.....................................................................................................................................433
Foreclosure..................................................................................................................................434

V. TITLES

Adverse Possession.....................................................................................................................438
Transfer by Deed..........................................................................................................................440
Transfer by Operation of Law and Will.........................................................................................447
Title Assurance Systems..............................................................................................................448
Special Problems.........................................................................................................................453

363
OUTLINE

I. OWNERSHIP OF REAL PROPERTY

NOTE An estate in land is an interest in real property that is presently or may


become possessory.

A. Present Possessory Estates


1. A freehold estate gives the owner of the estate title to or a right to hold
the property. A non-freehold estate gives mere possession.
2. Types of Freehold Estates
a. Fee Simple Absolute
(1) A fee simple absolute is the largest possible estate in land,
denoting the aggregate of all possible rights that a person may
have in that parcel of land, including:
(a) the unimpeded right to sell or convey all or part of the
property; and
(b) the unimpeded right to devise the property.

NOTE A fee simple absolute may last in perpetuity. If the owner of a fee simple ab-
solute dies intestate, the property will pass to the owner’s heirs by intestacy.

(2) The following words are traditionally used to create a fee simple
absolute: O conveys “to A and his heirs.”
(a) Words of purchase describe the persons, called
“purchasers,” who take an interest under a grant or device.
Words of limitation describe the nature of the estate taken
by the purchasers.
EXAMPLE: O conveys “to A and her heirs.” The words “to A”
are words of purchase because they indicate that A takes an
interest under the grant from O. The words “and her heirs”
are words of limitation because they describe the nature of
the estate taken by A—in this case, a fee simple absolute.
(b) At early common law, the words “to A and his heirs” were
required to create a fee simple absolute.
(c) However, under modern law, a fee simple absolute is
generally presumed when the words “to A” are used.
b. Defeasible Estates
(1) A defeasible estate is an estate that may terminate before its
maximum duration has run.
(2) Fee Simple Determinable
(a) A determinable estate terminates automatically on the
happening of a named future event.
(b) A determinable estate is described with the following words:
1) for so long as;

364
REAL PROPERTY

2) during;
3) while; or
4) until.
(c) A determinable estate is created:
1) in one clause; and
2) with a limitation built into that one clause.
EXAMPLE: O conveys “to A and his heirs for so long
as the premises are used for educational purposes.”
This creates a determinable estate.
(d) A determinable estate is followed by a possibility of reverter,
which may be implied.
(3) Fee Simple Subject to a Condition Subsequent
(a) An estate subject to a condition subsequent may be cut
short if the estate is retaken by the grantor or a third party
on the happening of a named future event.
1) The condition only gives the grantor the right to take the
estate, however. It does not automatically terminate the
estate (as distinguished from a determinable estate).
(b) An estate subject to a condition subsequent is described
with the following words:
1) provided, however;
2) however if;
3) but if;
4) on condition that; or
5) in the event that.
(c) An estate subject to a condition subsequent is created:
1) in two separate clauses; and
2) with a condition stated in the second clause.
EXAMPLE: O conveys “to A and his heirs, but if the
premises are not used for educational purposes, then
O has the right to reenter the premises and terminate
A’s estate.” This creates a fee simple subject to a
condition subsequent.
EXAMPLE: O conveys “to A for life; provided, however,
that if the premises are not used for charitable pur-
poses, O may reenter and retake the premises.” This
creates a life estate subject to a condition subsequent.
(4) Fee Simple Subject to Executory Interest
(a) A fee simple subject to an executory interest is an
estate that is automatically divested in favor of a third
person on the happening of a named event.

365
OUTLINE

EXAMPLE: O conveys “to A for so long as the premises are


used for charitable purposes, but if the premises cease to be
used for charitable purposes, then to B.”
EXAMPLE: O conveys “to A, but if the premises cease to be
used for educational purposes, then to B.”
1) A fee simple subject to an executory interest is followed
by a shifting executory interest.

NOTE This is subject to the Rule Against Perpetuities.

(5) Fee Tail


(a) At early common law, a fee tail was a freehold estate that
descended to the grantee’s lineal descendants (children) only.
(b) The following words are used to create a fee tail general:
O conveys “to A and the heirs of his body.”
1) In a fee tail general, children of the grantee take when
the grantee dies.
(c) The following words are used to create a fee tail male or female:
O conveys “to A and the heirs [male]/[female] of his body.”
1) In a fee tail male or female, descent is limited to the
male or female heirs of the grantee.
(d) The following words are used to create a fee tail special: O
conveys “to the heirs of his body by his wife B.”
1) In a fee tail special, descent is limited to the grantee’s
descendants by a specific spouse.
(e) The fee tail is followed by a reversion in the grantor or a
remainder in a third party. That future interest becomes
possessory if and when the grantee’s lineal line has failed.
(f) Fee tails are disfavored in modern law. Today, if the words
“to A and the heirs of his body” are used:
1) in most states, the grantee gets a fee simple absolute; and
2) in some states, the grantee has a life estate, with a
remainder per stirpes in the grantee’s lineal descen-
dants in being at the time of the life tenant’s death.
(6) Unclear Grants
(a) If the words of the grant are not clear:
1) a covenant is preferred over a defeasible estate because
the award is money damages rather than forfeiture; and
2) a fee simple subject to a condition subsequent is
preferred over a fee simple determinable because in
the former, forfeiture is not automatic.
c. Life Estate
(1) A life estate lasts for the duration of the grantee’s life.

366
REAL PROPERTY

(2) The following words are used to create a life estate: O conveys
“to A for life.”
(3) A life estate pur autre vie is an estate where the duration is
measured by the life of someone other than the grantee.
EXAMPLE: O conveys “to A for the life of B.” If B predeceases A,
A’s estate ends at B’s death. If A predeceases B, under the mod-
ern rule, A’s heirs are entitled to the use of the land until B dies.
(4) Life estates can be made defeasible.
(5) Duty to Repair
(a) A life tenant has a duty to maintain the property in a reason-
able state of repair, ordinary wear and tear excepted. This
duty is limited to the extent of the income derived or, if
he personally occupies the premises, to the extent of the
reasonable rental value of the land.
(b) A tenant for a term of years also has a duty to maintain the
property in a reasonable state of repair, ordinary wear and
tear excepted. However, this duty is not limited to the extent
of income derived or reasonable rental value discussed
above. Further, the lessee’s duty is now largely governed
by statutes that tend to relieve at least the residential
lessee of the duty to repair.
(6) Mortgages
(a) The life tenant has a duty to pay the interest on a mortgage
to the extent of profits derived from the property. A tenant for
years or a periodic tenant has no common law duty to make
mortgage payments. However, commercial leases often
require a nonresidential tenant to pay taxes, mortgages, etc.,
and to make specific repairs.
(7) Taxes
(a) The life tenant must pay all ordinary taxes, to the extent of
profits derived from the property. A tenant for years or a periodic
tenant has no common law duty to pay property taxes unless:
1) the lease is “perpetual” or for a long term with an option
in the tenant and his successors to renew “forever”;
2) the tenant holds without any obligation to pay rent; or
3) the tenant has erected improvements on the leased
premises for his own benefit.
(8) Special Assessments
(a) The life tenant must pay the full cost of special assess-
ments (apparently to the extent of income derived) if the life
of the public improvement is less than the duration of the
life tenant’s estate. Equitable apportionment is applied for
improvements likely to last longer (e.g., curbs and streets).

367
OUTLINE

A tenant for years or a periodic tenant has no common law


duty to pay carrying charges such as special assessments
or insurance premiums.
d. Non-Freehold Estates
(1) Leasehold Estate
(a) A leasehold estate is an estate that is limited in duration.
EXAMPLE: O transfers “to A for a term of 10 years.”
(b) Traditionally, the landlord was said to maintain the fee,
subject to the leasehold interest.
(c) The modern approach provides that the landlord holds a
reversion following the leasehold estate.

B. Future Interests
1. Reversionary Interest
a. A reversion is a future interest retained by the grantor when the
grantor transfers less than a fee interest to a third person.
b. Under the majority rule, a reversion is transferable, devisable, and
descendible.

NOTE A reversion is not subject to the Rule Against Perpetuities.

EXAMPLE: O conveys “to A for life, then to B for life, then to C for
life.” The entire fee simple absolute is not accounted for in this transaction.
Therefore, following all of these life estates, O has a reversion.
EXAMPLE: O conveys “to A for a term of 10 years.” A has an estate
for a term of 10 years. O retains a reversion.
EXAMPLE: O conveys “to A for life.” O retains a reversion. If O
conveys her reversion to C, C’s interest is still defined as a reversion,
even though it is now in a third party.
EXAMPLE: T devises “Blackacre to A for life.” When T dies, A gets a
present possessory life estate. T’s reversion passes by the terms of
his will or to T’s heirs by the laws of intestacy if there is no will.
EXAMPLE: T devises Blackacre “to A for life,” and T also devises “the
rest and residue of my estate to B.” When T dies, A gets a present
possessory life estate. B gets the residue of T’s estate, which
includes the reversion to Blackacre.
2. Remainders
a. A remainder is a future interest created in a third person that is intended
to take effect after the natural termination of the preceding estate.
b. Contingent Remainders
(1) A remainder will be contingent if:
(a) the takers are unascertained; or

368
REAL PROPERTY

(b) the interest is subject to a condition precedent and,


therefore, does not fall in automatically on the natural
termination of the previous estate.
(2) At strict common law, a contingent remainder could descend
and be devised but could not be transferred inter vivos.
(3) Under modern law, a contingent remainder is transferable,
descendible, and devisable (except, possibly, when the
contingent remainder is in an unascertained person).
(4) A contingent remainder is subject to the Rule Against Perpetuities.
EXAMPLE: T devises “to A for life, remainder to A’s widow and
her heirs.” If A is living at the time of creation, whether married or
unmarried, A has a life estate and A’s widow (if any) has a contin-
gent remainder in fee simple because the taker is not yet ascer-
tained or ascertainable. Therefore, T’s estate retains a reversion.
EXAMPLE: O conveys “to A for life, remainder to B and her
heirs if B reaches 21.” If, at the time of creation, B is already
21, A has a life estate and B has a vested remainder in fee
simple. If instead, at the time of creation, B is not yet 21, A has
a life estate and B’s remainder is contingent because B has
not yet fulfilled the condition precedent, which is B reaching 21.
Therefore, O retains a reversion.
(5) Doctrine of Destructibility of Contingent Remainders
(a) At common law, a contingent remainder in real property is
destroyed by any of the following:
1) if it fails to vest by the natural termination of the prior
vested estate;
EXAMPLE: O conveys “to A for life, remainder to B
and her heirs if B reaches 21.” A has a life estate. If B
is not yet 21, B has a contingent remainder (contingent
on B reaching 21), and O retains a reversion. Assume
that A dies and B is not yet 21. B’s contingent remainder
cannot fall in naturally at the termination of A’s life estate
because B has not yet met the condition precedent
to vesting. B’s contingent remainder is destroyed.
Therefore, O’s reversion will become possessory.
2) the doctrine of merger; or
a) Merger occurs, and the holder of the present
and future interest takes a fee simple absolute,
when one party who possesses a present or
future interest in the subject realty, by subsequent
transactions, obtains all outstanding present and
vested estates in that property. Situations in which
this may occur include:

369
OUTLINE

i) by surrender of the present estate to the


owner of a future estate;
ii) by release of a future estate to the owner of
a present estate; or
iii) when all holders of present and future vested
interests convey all of these interests to a
third party.
EXAMPLE: O conveys “to A for life, remainder
to B if B earns a law degree.” A has a life
estate. If B has not yet earned a law degree,
B has a contingent remainder in fee simple
absolute. O retains a reversion. Assume that,
one year later, O conveys her reversion to
A, and B still has not earned a law degree. A
now has both a life estate and a reversion. At
common law, A’s two interests merge because
they are not separated by a vested estate.
A gets a fee simple absolute. B’s contingent
remainder is destroyed.
EXAMPLE: O conveys “to A for life, remainder
to B’s first-born daughter.” A has a life estate.
If B does not yet have a daughter, B’s unborn
daughter has a contingent remainder. O re-
tains a reversion. Assume that O later devises
his reversion to A and that B still does not have
a daughter at the time of O’s death. At O’s
death, A receives O’s reversion. A also has her
life estate. A’s two interests merge because
they are not separated by a vested estate. A
gets a fee simple absolute, and B’s first-born
daughter’s contingent remainder is destroyed.
3) if the holder of the present possessory estate surren-
ders his interest before the contingent remainder vests.
EXAMPLE: T devises “to A for life, remainder to B’s
widower.” Assume that T’s will creates this devise but
does not expressly dispose of the reversion. However,
T’s will leaves the residue of T’s estate to A. Assume
further that, at the time the will takes effect (when T
dies), B is alive and married. At T’s death, A has a life
estate and a reversion. B’s widower has a contingent
remainder because a person cannot have a widower
until she dies. B’s widower’s contingent remainder is
not destroyed by merger because A’s interests were
created at the same time and by the same instrument.

370
REAL PROPERTY

EXAMPLE: Assume that one year after T dies, A


conveys her life estate and the reversion to C. C gets
a life estate and a reversion. The life estate and the
reversion merge. C gets a fee simple absolute. B’s
widower’s contingent remainder is destroyed.
(6) Under modern law, many jurisdictions have abolished the rule of
destructibility of contingent remainders. In those jurisdictions, if a
contingent remainder has not vested at the natural termination of
the prior vested estate, the contingent remainder will become an
executory interest, subject to the Rule Against Perpetuities.
(7) In jurisdictions that have retained the destructibility rule, it is
important to distinguish between contingent remainders, which
are destructible, and executory interests, which are not destruc-
tible. If a future interest is created such that it may in some case
fall in automatically at the natural termination of the previous
estate, an irrebuttable presumption will arise, causing the interest
to be treated for all purposes as a contingent remainder.
EXAMPLE: O conveys “to A for life, remainder to B and his heirs
if B reaches 21.” A has a life estate. If B is not yet 21 at the time
of creation, B may not be ready to take when A dies. However,
B could possibly turn 21 before A dies. Therefore, B’s interest
is treated as a contingent remainder rather than an executory
interest and thus may be destroyed in states in which contingent
remainders are still destructible.
EXAMPLE: O conveys “to A for so long as liquor is not served
on the premises, but if liquor is served on the premises then
to B and her heirs.” A has a fee simple subject to an executory
limitation. B will take only if A’s estate is cut short by the service
of liquor on the premises. Thus, B cannot possibly take at the
natural termination of A’s estate. Therefore, B has a shifting
executory interest rather than a contingent remainder.
c. Vested Remainders
(1) A vested remainder requires its takers to be ascertained
or ascertainable at the time that the remainder is created. A
vested remainder also must fall in automatically at the natural
termination of the previous estate, meaning there can be no
conditions precedent to taking.
(2) A vested remainder is transferable, descendible, and devisable.
(3) A fully vested remainder is not subject to the Rule Against
Perpetuities. However, a remainder that is vested subject to
open is subject to the Rule Against Perpetuities.
EXAMPLE: Testator devises “to A for life, remainder to B and
his heirs.” A has a life estate. B has a vested remainder in fee
simple because the taker is identified, and B’s interest falls in

371
OUTLINE

automatically at the natural termination of the previous estate


(when A dies).

EXAMPLE: O conveys “to A for life, remainder to B for life, re-


mainder to C and her heirs.” A has a life estate, B has a vested
remainder for life, and C has a vested remainder in fee simple.
(4) Special Types of Vested Remainders
(a) A remainder that is vested subject to open, sometimes called
vested subject to partial divestment, is a remainder that:
1) has been made to a class (e.g., “my children”); and
2) has at least one member who is ascertainable and
who has satisfied any conditions precedent to vesting,
but may have other members join the class later.
EXAMPLE: T devises “to A for life, remainder to A’s
children and their heirs.” Assume that at the time of
creation (T’s death), A is living and has one child, B.
A has a present possessory life estate and B has a
vested remainder subject to open, because A could
have more children. T retains nothing.
EXAMPLE: Assume further that one year later, a
second child, C, is born to A. A has a life estate, B and
C together have a vested remainder subject to open,
and T retains nothing.
(b) A vested remainder subject to total divestment is
presently vested but may be terminated on the happening
of a future event.
EXAMPLE: O conveys “to A for life, remainder to B for so
long as the premises are used for educational purposes.”
A has a life estate, and B has a vested remainder subject
to total divestment. O retains a possibility of reverter.
3. Executory Interests
a. An executory interest is a future interest in a third person that cuts
short the previous estate before it would have naturally terminated.
(1) Since a fee estate has the potential to last forever, any interest
created in a third party that follows the granting of a fee will
always be an executory interest.
(2) There are two types of executory interests.
(a) A shifting executory interest is an interest that cuts
short a prior estate created by the same conveyance. The
interest passes from one grantee to another.
(b) A springing executory interest is an interest that follows
a gap in possession or divests the estate of the transferor.
The interest passes from a grantor to a grantee.

372
REAL PROPERTY

b. An executory interest is transferable, descendible, and devisable.


c. An executory interest is subject to the Rule Against Perpetuities.
EXAMPLE: O conveys “to A, but if liquor is served on the premises,
then to B and his heirs.” A has a fee simple subject to an executory
limitation. B has a shifting executory interest because his interest
may cut short A’s estate.
EXAMPLE: O conveys “to A for life, remainder to B and her heirs one
month after A’s death.” A has a life estate. B has a springing execu-
tory interest because her interest cannot become presently possessory
until one month after the natural termination of A’s life estate. O retains
a reversion. After A’s death, O’s reversion becomes possessory. Then,
one month after A’s death, the fee simple springs out of O to B.
4. Possibility of Reverter and Powers of Termination
a. Possibility of Reverter
(1) A possibility of reverter is a future interest in the grantor that
follows a determinable estate.
(2) The creation of a fee simple determinable estate automatically
creates the possibility of reverter in the grantor.
(3) At strict common law, a possibility of reverter could descend through
intestacy but could not be devised or transferred inter vivos.
(4) Today, under the modern trend, a possibility of reverter is
transferable, devisable, and descendible (it can be transferred
by law, will, or intestacy).
(5) The statute of limitations begins to run on a possibility of reverter
as soon as the limitation occurs, because the property automati-
cally reverts to the grantor on the occurrence of the limitation.
(6) A possibility of reverter is not subject to the Rule Against Perpetuities.
EXAMPLE: O conveys “to A and her heirs for so long as liquor
is not served on the premises.” A has a fee simple determinable.
O has a possibility of reverter, which becomes possessory auto-
matically if liquor is served. If O then conveys her possibility of re-
verter to C, C’s interest is still defined as a possibility of reverter,
even though it is now in a third party.
b. Power of Termination
(1) A power of termination (formerly called a “right of re-entry”) is a
future interest in the grantor that follows a fee simple or life estate
subject to a condition subsequent.
(2) The creation of the power of termination requires a grantor
to reserve this right in the conveyance either expressly or by
necessary implication.
(a) While there is no definitive rule on this point, courts tend to
show a constructional preference against finding a defea-
sible estate where the language is ambiguous.

373
OUTLINE

(b) As a result, where the language is ambiguous or doubtful,


courts generally treat a provision in a deed as a covenant,
and not a condition subsequent.
(3) Upon the happening of the event, the property will not automatically
revert back to the grantor. The grantor has to exercise the power of
termination through an affirmative action to retake the property.
(4) At strict common law, a power of termination could descend through
intestacy but could not be devised or transferred inter vivos.
(5) Today, under modern law, all states agree that a power of
termination is descendible, and a majority of states agree that a
power of termination is devisable, but the majority rule is that a
power of termination is not transferable inter vivos.
(6) The statute of limitations does not begin to run against a power
of termination until the grantor attempts to exercise the right.
However, in some states, the statute of limitations begins to run
against a power of termination when the condition occurs.
(7) A power of termination is not subject to the Rule Against
Perpetuities.
EXAMPLE: O conveys “to A and his heirs, but if liquor is served
on the premises, then O and his heirs may reenter and terminate
A’s estate.” A has a fee simple subject to a condition subsequent,
and O has a power of termination. If liquor is served, O must take
affirmative steps before A’s estate is terminated. If O transfers his
power of termination to C, C’s interest is still defined as a power
of termination, even though it is now in a third party.
EXAMPLE: O conveys “to A; provided, however, that liquor may
not be served on the premises.” No power of termination has been
reserved. Therefore, O cannot take the property back if liquor is
served on the premises. The condition subsequent language will
be stricken. Therefore, A is left with a fee simple absolute.
5. Rules Affecting These Interests
a. Rule in Shelley’s Case
(1) Also known as the rule against remainders in grantee’s heirs,
the Rule in Shelley’s Case arises in grants such as the following:
O conveys “to A for life, remainder to the heirs of A.”
(2) The requirements for applicability of the Rule in Shelley’s Case are:
(a) A must get a freehold estate (a life estate or a fee tail);
(b) A’s heirs must get a remainder in fee (or in tail);
(c) the same instrument must create both A’s and A’s heirs’
interests; and
(d) both estates must be legal or both must be equitable.
(3) If all of these requirements are met:
(a) A gets both a life estate and a remainder; and

374
REAL PROPERTY

(b) by merger, A gets a fee simple and thus A’s heirs take
nothing by virtue of the grant itself.
(4) The Rule in Shelley’s Case is a rule of law, which the court must
treat as creating an irrebuttable presumption.
(5) The Rule in Shelley’s Case has been abolished by statute or
judicial decision in most states.
b. Doctrine of Worthier Title
(1) Also known as the rule against remainders in grantor’s heirs, the
Doctrine of Worthier Title arises in grants such as the following:
O conveys “to A for life, remainder to the heirs of O.”
(2) The requirements for applicability of the Doctrine of Worthier Title are:
(a) A must receive an estate less than a fee simple, such as a
life estate or a term of years;
(b) O’s heirs must receive a remainder (or an executory interest,
in the rare case);
(c) both interests must be created by the same instrument; and
(d) both interests must be legal or both must be equitable.
(3) The Doctrine of Worthier Title is a rule of construction, not law. The
language leaving a remainder to O’s heirs creates a rebuttable
presumption that O intended to retain that interest in himself as
a reversion. This presumption may be rebutted by clear express
evidence that O did intend to create a remainder in his heirs. If the
presumption is raised but not rebutted, A gets a life estate and O
retains a reversion (and thus O’s heirs take nothing on their own).
(4) Under modern law, the Doctrine of Worthier Title applies to inter
vivos transfers only (the testamentary branch of the doctrine no
longer is applied).
c. Rule of Convenience
(1) Under the Rule of Convenience, a class closes when a member
of the class is entitled to distribution.
(2) The Rule of Convenience applies to the following classes: “children,”
“grandchildren,” “brothers,” “sisters,” “nephews,” “nieces,” “cousins,”
“issue,” “descendants,” or “family” of a designated person.
(a) If the class is already closed at the time the gift takes effect (i.e.,
the time of conveyance for an inter vivos gift or time of testator’s
death for a will), all members of the class at that moment will
take. However, any individual later conceived or born will not be
a member of the class and will not share in the estate.
(b) If, at the time the gift takes effect, the class has members
entitled to take immediately but the class has not previously
closed, all members of the class conceived at the time the
gift is made will be included and may take; however, the
class will close to the exclusion of afterborn children, who
will not share in it.

375
OUTLINE

(c) If the class has no members at the time the gift is made, all
members of the class, whenever born, will be included and may
take. The class stays open until all possible members are born.
EXAMPLE: T devises “to the children of A.” If A is dead at
the time of T’s death, the class of A’s children will be closed;
therefore, all of A’s children will be included and will take. If
A is alive at the time of T’s death and has children at that
time, the children born at the time of T’s death are entitled to
immediate distribution. Therefore, the class will close, and
all of those children will be included and may take. However,
afterborn children will be excluded. If A is alive at T’s death
but does not have children at that time, no one is entitled to
immediate distribution. Therefore, the class should remain
open. All of A’s children, whenever born, will be included and
should be able to take. Some authorities suggest that the
first-born child takes the entire estate on her birth, subject to
partial divestment by the birth of later children; however, the
case law on this point is sparse.
d. Postponed Gift to a Class (with No Condition Precedent)
(1) If the class is already closed at the time the postponement ends
(e.g., at the end of a present possessory life estate), all members
of the class will be included and will take.
(2) If the class has members but is not yet closed at the time the
postponement ends, all members of the class conceived at
the time the postponement ends will be included and may
take. However, the class will close to the exclusion of afterborn
children, who may not take.
(3) If the class has no members at the time the postponement
ends, all members of the class, whenever born, will be included
and may take.
EXAMPLE: T devises “to A for life, remainder to the children of
B.” If B is dead at the time of T’s death, the class of B’s children
will be closed; therefore, all of B’s children will be included and
may take. If B dies after T but before A, the class of B’s children
will also be closed; therefore, all of B’s children will be included
and may take. If B is alive at the time of A’s death and B has
children at that time, all children then born are included and are
entitled to distribution at A’s death. However, the class will close
at that time to the exclusion of afterborn children. If B is alive at
A’s death but does not have children at that time, no child is then
entitled to immediate distribution. The class should remain open.
Therefore, all of B’s children, whenever born, should be included
and should be able to take. Again, some authorities suggest that
the first-born child takes the entire estate on her birth, subject to
partial divestment by the birth of later children.

376
REAL PROPERTY

e. Immediate Gift to a Class Coupled with a Condition Precedent


(1) An immediate gift to a class may be coupled with a condi-
tion precedent. In such a case, the class closes when the first
member of the class satisfies the condition.
(2) All then-born members of the class are included and may take if
and when they satisfy the condition.
(3) However, afterborn children will be excluded.
EXAMPLE: O conveys “to B’s children who reach 21.” If B is
dead at the time the conveyance is made, the class of B’s chil-
dren will be closed. Therefore, all of B’s children will be included
and may take if and when they reach 21. If B is alive at the time
the conveyance is made, and B has a child who has reached
21, the class will close because that child is entitled to immedi-
ate distribution. All of B’s then-born children will be included and
may take if and when they reach 21; however, all afterborn chil-
dren will be excluded. If B is alive at the time the conveyance is
made, and B has one child who is, say, 15, the class will remain
open. The class will close if and when a child of B reaches 21.
All children of B born at the time the first child reaches 21 will be
included and may take if and when they reach 21. All children
born after the first child reaches 21 are excluded. If B is alive
at the time the conveyance is made, and B has no children, the
class will remain open. The class will close if and when a child
of B reaches 21. All children of B born at the time the first child
reaches 21 will be included and may take if and when they reach
21. All children born after the first child reaches 21 are excluded.
f. Gift to a Class with a Combination of Postponements
(1) A gift to a class may be made with a combination of postpone-
ments combined with a condition precedent. In such a case, the
postponement is deemed to end, and the class closes when the
last condition is satisfied.
(2) All members of the class born before the class closes are included,
and they may take if and when they satisfy the condition.
(3) However, all afterborn persons are excluded.
EXAMPLE: O conveys “to A for life, remainder to B’s children who
reach 21.” If B is dead at the time the conveyance is made, the
class of B’s children will be closed. Therefore, all of B’s children will
be included and may take if and when they reach 21. If B is alive
when the conveyance is made but dies before A, the class of B’s
children also will be closed. Therefore, all of B’s children will be
included and may take if and when they reach 21. If B is alive at the
time of A’s death and B has a child who is 21, that child is entitled to
distribution at A’s death. Therefore, the class will close at that time.
All children born before A’s death will be included and may take if

377
OUTLINE

and when they reach 21. However, children born after the class
closes will be excluded. If B is alive at A’s death and either has no
children or has children, none of whom has reached 21, no child
is entitled to distribution at A’s death (which is the first postpone-
ment). Therefore, the class remains open until a child of B reaches
21 (which is the second postponement). When a child of B reaches
21, the class closes. All children conceived at the time a child of B
reaches 21 will be included and may take if and when they reach
21. However, children born after the class closes are excluded.
g. Waste
(1) The possessor of a life estate or a leasehold interest has the
right to possess, use, and enjoy the property during the duration
of his estate. However, he may not do anything that adversely
affects the future interest that follows the life estate. An act that
adversely affects the future interest is called waste.
(2) Voluntary waste is the voluntary commission of an act that has more
than a trivial injurious effect on or change in the property. Despite the
prohibition on voluntary waste, natural resources may be consumed:
(a) for the repair and maintenance of the property;
(b) with permission of the grantor; or
(c) under the open mines doctrine, which applies to both life
tenants and tenants for years.
1) If the grantor was exploiting the natural resources of
the mine, it is presumed that the grantee has the right
to continue that exploitation.
(3) Involuntary or permissive waste occurs if the life tenant or lease-
hold tenant permits the premises to fall into disrepair. Involuntary
waste may also occur if the life tenant fails to pay mortgage interest
payments, taxes, or the tenant’s share of special assessments.
(4) Ameliorative waste occurs if an act of a life tenant increases
the value of the premises by permanently altering it. Traditionally,
ameliorative waste was prohibited. However, under modern law,
a life tenant is allowed to commit ameliorative waste if:
(a) the market value of the remainderman’s interest is not
impaired; and
(b) either:
1) it is permitted by the remainderman; or
2) a substantial and permanent change in the neighborhood
has deprived the property of a reasonable current value.
(c) A holder of a reversion has standing to sue for past or future
waste. A vested remainderman also has standing to sue for
past or future waste. However, a contingent remainderman
has standing to sue to prevent future waste but not for
damages for past waste.

378
REAL PROPERTY

C. Cotenancy
1. A concurrent estate exists when two or more persons share concurrently
an interest in real property. A concurrent owner is called a co-tenant.
2. Tenancy in Common
a. Tenancy in common is a form of concurrent ownership where each
co-tenant owns an undivided interest in the whole of the property with
no right of survivorship.
b. Tenancy in common is now the presumed form of co-tenancy. Therefore,
no special words are required to create a tenancy in common.
EXAMPLE: O conveys “to A and B.” A and B hold a fee simple absolute
as tenants in common.
c. Tenancy in common requires unity of possession only. Unity of
possession means that each tenant in common has the right to
possess the whole of the property.
d. A tenant in common may transfer his interest inter vivos:
(1) voluntarily, through a conveyance, lease, mortgage, or other
transfer of a present possessory or future property interest; or
(2) involuntarily, through a foreclosure on a mortgage of the
tenant’s interest or an execution of a judgment creditor’s lien on
the tenant’s interest in the property.
e. A tenant in common may devise his interest, and the interest of a
tenant in common may also descend by intestacy.
3. Joint Tenancy
a. Joint tenancy is a form of concurrent ownership where each co-tenant
owns an undivided interest in the whole of the property and has a right
of survivorship.
b. Traditionally, the following words are required to create a joint tenancy:
“to A and B as joint tenants and not as tenants in common, with full
right of survivorship.”
c. A joint tenancy requires the following four unities:
(1) joint tenants must take at the same time (unity of time);
(2) joint tenants must take by the same instrument (unity of title);
(3) joint tenants must take equal shares of the same type (e.g., each
joint tenant takes a legal fee simple) (unity of interest); and
(4) each joint tenant has the right to possess the whole (unity of
possession).
d. Under the traditional rule, a sole owner of property cannot create a
joint tenancy between himself and another person by conveying a joint
interest directly to the other person. Instead, the owner must convey
the property to a straw man, who then conveys the property back to
the owner and the other person as joint tenants.
(1) Some jurisdictions now do not require the unities of time and title.
Under this approach, a straw man no longer is required to create

379
OUTLINE

a joint tenancy. Instead, a sole owner may create joint tenancy


between himself and another person by conveying a joint interest
directly to the other person.
e. Each joint tenant has a right of survivorship. At the death of one joint
tenant, the interest of the surviving joint tenant “grows” and absorbs
the interest of the deceased joint tenant.
EXAMPLE: O conveys Blackacre “to A and B as joint tenants and not
as tenants in common, with full right of survivorship.” Subsequently,
A dies, leaving C as her sole heir. B’s right of survivorship takes over,
and B becomes the sole owner of Blackacre. C takes no interest.
f. A joint tenant may transfer his interest voluntarily or involuntarily,
thereby severing the joint tenancy. However, the right of survivorship
prevents a predeceasing joint tenant from devising his interest.
In addition, the interest of the predeceasing joint tenant may not
descend by intestacy.
g. A joint tenancy is severed if one joint tenant conveys his interest volun-
tarily or involuntarily. An involuntary conveyance occurs if a creditor
forces a sale of a joint tenant’s interest to satisfy the debt owed to the
creditor by the joint tenant. If a joint tenancy is severed, the unities
of time and title are destroyed. Therefore, a tenancy in common is
created between the other tenant and the conveyee.
EXAMPLE: O conveys Blackacre “to A and B as joint tenants and
not as tenants in common, with full right of survivorship.” Subse-
quently, A conveys her interest in Blackacre to C. The conveyance
to C breaks the unities of time and title because C took by a differ-
ent instrument and time from B. Therefore, B and C hold Blackacre
as tenants in common.
h. If both joint tenants join in a mortgage of the subject property, the mort-
gage will not affect the joint tenancy. However, if only one of the joint
tenants mortgages his interest, the effect of the mortgage on the joint
tenancy will depend on whether the jurisdiction follows the title theory
or the lien theory of mortgages.
(1) In a title theory state, the execution of a mortgage by one joint
tenant causes the legal interest of that co-tenant to be transferred
to the mortgagee. Thus, the mortgage severs the joint tenancy
because the unities have been destroyed. Most jurisdictions no
longer follow the title theory.
(2) In a lien theory state, the mortgagee only receives a lien on the
property. Therefore, no severance occurs when the mortgage is
made because the unities remain intact. However, in a lien theory
state, foreclosure will sever the joint tenancy. Most jurisdictions
now follow the lien theory.

NOTE Jurisdictions are split on whether a lease severs a joint tenancy.

380
REAL PROPERTY

4. Tenancy by the Entirety


a. A tenancy by the entirety is a form of concurrent ownership reserved
for married couples, which gives each spouse an undivided interest in
the whole of the property and a right of survivorship.
b. Traditionally, to create a tenancy by the entirety, the following words
were required: “to H and W as tenants by the entirety and not as joint
tenants or tenants in common, with full right of survivorship.”
c. Today, most states have enacted statutes creating a presumption of
tenancy by the entirety if a conveyance is made to a married couple,
even if they are not identified as a married couple in the deed.
(1) Because a tenancy by the entirety is an older concept that has
begun falling out of favor, there has not yet been any case law on
whether this would apply equally to a same-sex couple.
d. In the states that recognize tenancy by the entirety, an attempted
conveyance by either spouse is wholly void.
5. Severance
a. A tenancy by the entirety cannot be terminated by the unilateral act of
either spouse. The tenancy can only be severed:
(1) by a joint conveyance of both spouses;
(2) by a conveyance of one spouse to the other of the property; or
(3) when the spouses divorce.
b. A joint tenancy may be severed by the unilateral conveyance of the
property by one of the joint tenants during the owner’s lifetime.
(1) However, the right of survivorship prevents the joint tenant from
devising the property, or from having it pass through intestacy.
c. A mortgage will not sever a joint tenancy when created except in a title
theory jurisdiction.
(1) In a lien theory jurisdiction, the creation of a mortgage does not cause
a severance of the joint tenancy. If the mortgagor dies before the mort-
gagee forecloses on the interest, the surviving joint tenant(s) will take
the property by right of survivorship and free of the mortgage interest.
(2) If the mortgagee forecloses on the property before the death of
the mortgagor, the foreclosure operates to sever the mortgaged
interest from the joint tenancy.
6. Partition
a. Each co-tenant has the right to seek partition of the property.
b. Voluntary partition is usually accomplished by an exchange of mutual deeds
among co-tenants or by sale of the property and division of the proceeds.
c. Involuntary (judicial) partition is accomplished by court action, usually
at the instance of one, but not all, of the co-tenants. Involuntary parti-
tion may be either in-kind (physical division) or by sale.
(1) Traditionally, physical partition has been preferred, unless a phys-
ical partition would be impractical and the interests of the parties
would be better served by a sale and division of the proceeds.

381
OUTLINE

(2) When partition cannot produce equal shares, the party receiving
the larger portion must make a cash payment to the other party.
7. Rights and Duties of Co-Tenants
a. Each co-tenant is entitled to possess the whole property.
b. Wrongful ouster occurs when one co-tenant wrongfully excludes
another co-tenant from possession of the whole or any part of the
jointly held property.
c. An out-of-possession co-tenant has the right to share in rents
and net profits derived by another co-tenant from third parties,
less operating expenses such as taxes, mortgages, interest, and
management fees. However, an out-of-possession co-tenant does
not have the right to demand rent from a co-tenant who is in actual
possession of the premises, unless the co-tenant in possession has
effected a wrongful ouster or has exploited the property in a manner
resulting in permanent depreciation.
d. The common law, which is still followed in some states, does not allow
a co-tenant to seek contribution from another co-tenant for repairs
performed on the property.
e. However, under the modern trend, contribution may be compelled for
necessary repairs if the repairs were requested by the repairing tenant
and refused by the other(s).
f. In a partition suit or an accounting suit, the costs of repairs may be
credited in favor of a co-tenant who repairs.
g. A co-tenant has no right to seek a contribution or setoff for improve-
ments made to the premises, unless the improvements generate
increased rents or profits. In that case, costs of improvements are
recoverable only in a partition suit.
h. A co-tenant who pays a mortgage or tax may seek contribution or may
recover in an accounting or partition suit. However, there is no sepa-
rate action to compel contribution. A co-tenant in sole possession will
receive reimbursement only to the extent that his payment exceeds the
value of the use and occupation of the property (i.e., the rental value).

D. Landlord-Tenant

NOTE A lease gives the tenant (lessee) exclusive possession of the premises for a
period of time.

1. Creation
a. A lease may be created expressly, either orally or in writing. A writing gener-
ally is required by the Statute of Frauds for a term of more than one year.
b. An implied lease may be created by the conduct of the parties when
the written lease is an invalid writing. In such a case, the resulting
tenancy will be periodic. An implied lease may also be created when a
holdover tenant pays rent and the landlord (lessor) accepts the rent.

382
REAL PROPERTY

2. Types of Leases

TYPES OF LEASES

Temporarily Pick Another Settlement

Term of Years, Periodic, At-Will, Tenancy at Sufferance

a. Term of Years
(1) A term of years has a definite beginning and end (e.g., until a
set date or for a set number of months or years). No notice is
required to terminate a lease for a term of years.
EXAMPLE: L leases to T for a period of one year.
b. Periodic
(1) A periodic tenancy has a set beginning date and continues from
period to period (e.g., from month to month) without a set termi-
nation date, until proper notice is given. While under the common
law, notice merely has to be sufficient to show intent, some
modern statutes require termination notice to be in writing.
(a) The modern rule is that notice can be oral unless the lease
specifically states otherwise.
(2) A periodic tenancy may be created expressly or by implication
with a holdover tenant.
(3) Under the traditional rule, a periodic tenancy could be termi-
nated only on the anniversary date. Improper notice was
deemed ineffective, and the notice period was measured by the
rent reservation clause.
(4) Under the modern statutory approach, termination may take
place on any date, and the notice period is measured by the rent
payment clause up to a maximum of six months.
c. At-Will
(1) An at-will lease has no fixed duration and lasts only as long as
the landlord and tenant desire. An at-will tenancy terminates if:
(a) either party dies;
(b) the tenant commits waste;
(c) the tenant attempts to assign his interest;
(d) the landlord transfers his interest; or
(e) the landlord transfers the premises to a third party for a
term of years.
(2) If the lease gives the landlord a unilateral right to terminate,
most courts find a tenancy at will that may be terminated by
either the landlord or the tenant.
(3) If the lease gives the tenant a unilateral right to terminate, some
courts find a tenancy at will that may be terminated by either

383
OUTLINE

the landlord or the tenant. However, under the modern view, the
tenant receives either a life estate or a fee simple, depending on
the language used in the lease.
d. Tenancy at Sufferance / Holdover Tenancy
(1) A tenancy at sufferance occurs when a tenant remains in posses-
sion of the leased premises (“holds over”) after the end of the lease
term. A tenancy at sufferance is not a true tenancy. In most states, if
a residential tenant holds over, the landlord may recover possession
and receive the reasonable rental value for the holdover period.
3. Possession and Rent
a. Tenant’s Duties
(1) Pay Rent
(a) A tenant has a duty to pay rent. Under the common law
estate theory, the tenant’s duty to pay rent was not relieved
if the premises were destroyed because the tenant would
still have possession of the actual land. There was an excep-
tion to this common law rule where the tenant leased only
a portion of the building. In that instance, if the building was
destroyed, the tenant was then excused from paying rent.
(b) Today, there is no distinction between leasing all or part of a
building. The tenant leasing all or a portion of a building may be
relieved from the duty to pay rent if the premises are destroyed.
1) An exception to this rule exists where the tenant is not
relieved from liability for rent when the tenant intention-
ally or negligently caused the destruction.
(2) Waste
(a) The tenant’s duties in respect to waste are similar to those
owed by life tenants. However, the tenant’s duty not to commit
permissive waste (primarily in respect to repairs) is not limited
to the extent of income derived from or the reasonable rental
value of the property when the tenant possesses personally.
(b) In addition, a tenant (except possibly a tenant for a long
term) may not commit ameliorative waste, which a life tenant
may be permitted to do.
(3) Repair
(a) Under the common law estate theory, the landlord has no duty
to repair the premises during the term of the lease. In contrast,
the tenant had a duty not to commit waste, which includes a
duty to repair the premises (i.e., not commit permissive waste).
This duty extended to minor repairs that keep the building’s
interior safe from water and wind, but absent a covenant to the
contrary, the tenant was not required to rebuild the premises if
they were destroyed by an act of God. (Under the estate theory,
the landlord also had no duty to rebuild.)

384
REAL PROPERTY

(b) Alternatively, landlords and tenants may also agree


on how to allocate repair duties. If a tenant assumes a
general duty to repair, then absent a covenant to the
contrary the tenant would in such a situation be respon-
sible for repairing the premises from casualties, such as
damages caused by fire and storms. The tenant would
also be responsible for rebuilding or replacing the prem-
ises if they are destroyed by such casualties.
(c) Under the modern contract theory, which is the majority rule,
the parties are free to covenant for duties that they would not
bear under the common law estate theory.
1) Absent any provision in the lease, the common law
rule as to the duty to repair still applies.
2) Under a general covenant to repair, the tenant is
required to repair the premises. Usually, this covenant
excepts ordinary wear and tear. The covenant may
also except damages caused “by act of God,” such as
fire and weather.
3) If the tenant covenants to repair without exception, the
tenant must make all repairs, regardless of how the
damage is caused.
4) If there is no covenant in place, the landlord is required
to repair damages resulting from casualties.
(d) Under the modern trend, in the absence of a contrary agree-
ment, the tenant is not required to rebuild if the destruction
was not his fault.
b. Landlord’s Duties
(1) The landlord has a duty to deliver possession of the premises
to the tenant at the start of the lease term.
(a) The majority (English) rule requires delivery of actual possession.
(b) The minority (American) rule requires delivery of the legal
right to possession only.
EXAMPLE: L and T execute a valid lease giving T a term
of one year. On the date on which the lease commences, T
discovers that the previous tenant is still on the premises be-
cause she has held over after the expiration of her term. Under
the American rule, L has properly delivered possession be-
cause L gave T the legal right to the leased premises when L
and T executed the lease. However, under the English rule, L
has not properly delivered possession. The presence of the old
tenant on the premises prevents L from delivering actual pos-
session to T on the date on which the lease commences.
(2) Typically, state law creates certain duties for the landlord, such as
the provision of running water and heat.

385
OUTLINE

(3) The lease document may also create certain duties for the land-
lord, such as the duty to repair within the leased premises.
4. Assignment and Subletting
a. Assignment
(1) An assignment occurs when the tenant transfers to a third
person (the assignee) all of his rights, title, and interest in the
leased premises. An assignee comes into privity of estate with
the landlord.
b. Sublease
(1) A sublease occurs when the tenant transfers to a third person
(the subtenant or sublessee) less than all of his rights, title, and
interest in the leased premises. A subtenant does not come into
privity of estate with the landlord.
c. Liability of Parties
(1) A party may be liable to a landlord through either privity of
contract or privity of estate.
(a) Privity of contract is the relationship that exists between
the original parties to an agreement. Privity of contract will
always exist between the original parties and only those
original parties, unless there is a novation.
EXAMPLE: L and T enter into a written five-year lease
agreement, with rent to be paid monthly. Because L and
T are both signatories to the lease agreement, they are in
privity of contract with each other.
(b) Privity of estate arises through the succession in rights to
the same property by multiple parties.
1) The landlord, by entering into the lease agreement,
is conveying a portion of his rights—specifically, the
right to presently possess and use the property for the
duration of the lease—to the tenant. In doing so, the
parties enter into privity of estate.
2) If the tenant then grants anything less than his full rights
and interest in the property to another party (i.e., if the
tenant subleases the property, rather than assigning
it), that subsequent party does not enter into privity of
estate with the landlord, only with the tenant.
a) This is because the subtenant is not acquiring
the portion of the property rights that the landlord
conveyed to the tenant, and so is not “stepping into
the shoes” of the original tenant.
b) Rather, the subtenant is only acquiring a portion of
the tenant’s rights, while the tenant retains some
right or interest to the property.

386
REAL PROPERTY

c) In this situation, the tenant and subtenant enter into


privity of estate, but the subtenant does not enter
into privity of estate with the landlord, because there
is no direct relationship. The landlord and original
tenant will instead remain in privity of estate.
EXAMPLE: L and T enter into a written five-year
lease agreement, with rent to be paid monthly. L
and T are in privity of estate with each other.
EXAMPLE: Two years into the five year lease
agreement, T subleases the property to Sub for
a period of one year. T and Sub are in privity of
estate with each other, but L and Sub are not. L
cannot sue Sub directly for unpaid rent or any
other breach of the lease agreement. Instead, L
must sue T, who can then sue Sub for breaching
the sublease agreement between T and Sub.
(2) Absent a novation, the original tenant always remains liable to the
landlord even after an assignment or sublease. This is because
the original tenant remains in privity of contract with the landlord.
(a) For the original tenant to be discharged from liability, there must
be a novation. This is a separate agreement between the land-
lord, original tenant, and assignee whereby the landlord agrees
to discharge the original contracting party (the original tenant)
from contractual liability in exchange for the assignee becoming
liable as if he had signed the original lease agreement.
(3) Because an assignee comes into privity of estate with the land-
lord, if the rent is not paid, the landlord may sue the assignee.
(a) However, the landlord may not sue the assignee after the
assignee has transferred the premises to another third person,
unless the assignee had assumed the duty to pay rent. This is
because, by assigning the premises to someone else, the first
assignee is no longer in privity of estate with the landlord. The
subsequent assignee is now the party in privity of estate.
EXAMPLE: L and T enter into a five-year lease agreement, with
rent to be paid monthly. One year later, T assigns the lease to
T2. If T2 fails to pay rent to L, L can sue either T (through privity
of contract) or T2 (through privity of estate) for the unpaid rent.
EXAMPLE: Same facts as above, only this time, six months af-
ter being assigned the property, T2 assigns the lease to T3. T3
fails to pay rent. L may sue T (through privity of contract) and T3
(through privity of estate). However, L may not sue T2, because
T2 is in neither privity of contract nor privity of estate with L.
EXAMPLE: L and T enter into a five-year lease agreement, with
rent to be paid monthly. One year later, T assigns the lease to

387
OUTLINE

T2. As part of the assignment, L, T, and T2 agree to a novation


of T in favor of T2. T2 fails to pay rent. L may only sue T2, be-
cause T2 is now in both privity of contract and privity of estate
with L. L may not sue T because the novation discharged T
from contractual liability, severing T’s privity of contract with L.
EXAMPLE: Same facts as above, except that six months
after the assignment and novation, T2 assigns the property
to T3. T3 fails to pay rent. L may sue T2 (through privity of
contract) or T3 (through privity of estate). L may not sue T,
because T is not in privity with L as a result of the novation.
d. Covenants against Assignments and Subleases
(1) Covenants against assignments and/or subleases are construed
strictly against the landlord.
(2) Under the Rule in Dumpor’s Case, once waived, a covenant
against assignment is unenforceable as to the next assignment.
(a) The Rule in Dumpor’s Case does not apply to subleases.
(b) The Rule in Dumpor’s Case also does not apply where
the landlord specifies that the waiver is “one-time only.”
In this situation, the covenant will remain against
subsequent assignments.
(3) Where the prohibition does not give a standard or condition for
the giving or withholding of consent, this is commonly known as a
“silent consent” clause.
(4) The traditional rule, which is still the majority rule today, is that
a silent consent clause gives a landlord the right to withhold
consent for any reason or for no reason at all, even if the with-
holding of consent is arbitrary and unreasonable. However, this
rule is subject to statutory housing discrimination laws.
(5) The minority rule is that silent consent clauses require that the
landlord be reasonable in withholding consent for an assignment.
5. Termination
a. Landlord’s Remedies for Tenant’s Breach of Duty
(1) If the tenant fails to pay rent or commits another material breach
of the lease, the landlord may seek:
(a) to evict the tenant; and/or
(b) to recover damages for the tenant’s breach.
(2) Traditionally, acceleration of rent was not available to the land-
lord for anticipatory breach of a lease by the tenant. However,
the modern rule permits the use of rent acceleration clauses
in leases.
(a) In an action for rent, the tenant may defend on the grounds
of non-compliance by the landlord or on the ground of
destruction of the premises.

388
REAL PROPERTY

(3) A tenant abandons the premises if he vacates the premises


without intent to return and fails to pay rent. Traditionally, if the
tenant abandons the premises, the landlord may:
(a) retake the premises;
(b) ignore an abandonment and continue to hold the tenant
liable for rent; or
(c) reenter and relet the premises.
1) Under the traditional view, if the tenant abandons
the premises, the landlord has no duty to mitigate by
attempting to relet the premises. However, under the
modern view, the landlord has a duty to mitigate if the
tenant abandons. If mitigation is required, the burden
of proof is placed on the landlord to show that he
attempted to mitigate his losses.
(4) Traditionally, a landlord was allowed to use self-help to remove a
breaching tenant from the premises. However, under the modern
approach, self-help is prohibited or carefully constrained. The
landlord must seek court action to reenter the premises.
(5) A security deposit may be retained by a landlord only to the
extent necessary to cover actual loss that the deposit was
intended to secure (e.g., unpaid rent, damage).
(6) Retaliatory eviction (e.g., eviction in response to tenant’s
complaints about landlord) is prohibited in most states.
b. Tenant’s Remedies for Landlord’s Breach of Duty
(1) A tenant may seek money damages for the landlord’s breach of
the lease.
(a) Traditionally, a tenant may vacate the premises and termi-
nate the lease if the tenant has been evicted by the landlord,
actually or constructively.
(b) Statutes in many jurisdictions provide the tenant with the
following statutory remedies:
1) withholding rent; or
2) repairing the premises and deducting the cost of repair
from subsequent rent payments.
(2) A landlord is not liable to the tenant or others on the premises
with the consent of the tenant for injuries caused by a condition of
the premises, except in the following situations:
(a) an undisclosed dangerous condition exists that is known or
should have been known to the landlord but is unknown to
the tenant;
(b) a condition exists that is dangerous to persons outside of
the premises;
(c) the premises are leased for admission to the public;

389
OUTLINE

(d) parts of land are retained in the landlord’s control but are
available for use by the tenant;
(e) the landlord has contracted to repair; or
(f) the landlord has been negligent in making repairs.
c. Anticipatory Repudiation
(1) When a tenant breaches a lease agreement, the common law
rule was that the landlord can only sue for rent as it accrues.
(2) However, under modern law, when the lease is for a tenancy for
years and the tenant abandons the property, the landlord may
treat this as an anticipatory breach of the entire lease, entitling
the landlord to sue immediately for all rent due on the entire
agreement (subject to the landlord’s duty to mitigate damages).
d. Security Deposits
(1) There are three types of security deposits:
(a) where the tenant deposits a sum of money against which the
landlord may draw to make up any default or to pay for damages;
1) This type of security deposit requires the landlord to
account to the tenant for anything the landlord with-
draws from the deposit. At the end of the tenancy, any
balance goes to the tenant.
(b) a deposit that will be forfeited if the tenant defaults; and
1) This type of deposit is essentially a liquidated damages
clause, and should be treated as such.
(c) a deposit that is denominated as advance rent.
1) This last type is not considered a “true” deposit, but
rather, an advance payment of rent. The tenant has the
right to have that advance payment attributed toward
rent due at the end of the lease.
(2) Today, security deposits are heavily regulated by statutes in most
jurisdictions, and vary greatly from one state to another.
e. Death of Either Party
(1) In general, a lease will not be terminated by the death of either
the landlord or the tenant.
(2) In the absence of a contrary provision, the death of a tenant for
years will not terminate the tenancy, and the leasehold will pass
as personal property through the tenant’s estate.
(a) This may not be the case if, for example, the terms of the
lease contain a provision that it terminates upon the death of
either party, or the nature of the lease and peculiar qualifica-
tions of the tenant make the lease personal to the tenant.
(3) A tenancy at will, however, will be terminated by the death of either party.
(4) If the landlord holds a life estate in the leased property, the lease
is considered void upon the death of the landlord.

390
REAL PROPERTY

f. Sale of the Property by the Landlord


(1) When a landlord sells the property, there is generally no effect on
the tenant, unless the lease provides otherwise, except for the
tenant paying rent to the new landlord.
(a) If the lease contains such a term, then the buyer may termi-
nate the lease upon the conclusion of the sale. Otherwise,
the buyer “steps into the shoes” of the original landlord as to
all contract obligations.
(2) If the buyer purchased the property without any notice of the
lease, and qualifies under the jurisdiction’s applicable recording
statute, they would take the property free of the lease.
(a) Leases are considered encumbrances on the property. If
the buyer knows about the lease and purchases the prop-
erty anyway, they have accepted that encumbrance and are
bound to the lease.
(b) On the other hand, if the buyer has no actual notice of the
lease, it is not recorded (no constructive notice), and the buyer
could not have discovered the leasehold by visiting the prop-
erty (no inquiry notice), then the buyer would not be bound by
the lease, and their purchase (free of the lease) would have
higher priority to the property than the tenant would have.
6. Habitability and Suitability
a. Implied Warranty of Habitability
(1) Under the modern trend, the landlord is deemed to have impliedly
warranted that the residential premises are fit for habitation. The
implied warranty of habitability guarantees that the landlord
will deliver and maintain premises that are safe, clean, and fit for
human habitation.
(a) To prove that the landlord has breached this warranty, the
tenant must show a defect in essential residential facilities.
1) In the majority of jurisdictions, the defect may be patent
(obvious) or latent (nonobvious).
2) Some jurisdictions exclude patent defects from the
scope of the implied warranty of habitability.
(b) To claim a breach of the implied warranty of habitability, the
tenant must:
1) provide the landlord with notice of the defect; and
2) allow the landlord reasonable time to make necessary repairs.

NOTE The implied warranty of habitability applies to residential leases. Commercial


leases are not covered by the warranty. The following leases may also be
excluded from coverage under the implied warranty of habitability: leases for
single-family residences, agricultural leases, long-term leases, and casual
leases made by non-merchant landlords.

391
OUTLINE

(2) In some jurisdictions, a housing code violation automatically


breaches the implied warranty of habitability. In other juris-
dictions, the housing code provides the standard; however,
substantial compliance with the code is sufficient so long as
habitability is not affected. In some jurisdictions, the standard
for the implied warranty is found in the common law, even in the
absence of a housing code. In those jurisdictions, a breach of
the warranty may be found if the premises are uninhabitable in
the view of a reasonable person.
(a) If the landlord breaches the implied warranty of habitability:
1) the tenant is excused from further performance under
the lease;
2) the tenant may, but need not, vacate the premises;
3) the tenant may treat the lease as canceled, and the
rent is abated;
4) the tenant may seek money damages; and
a) Some courts use the contract measure of
damages for breach of warranty, which is the
value of the premises as warranted less the value
of the premises as received, and agreed rent is
evidence of the rental value.
b) Other jurisdictions measure damages by finding
the difference between the agreed rent and the
fair rental value of the premises received.
5) the tenant may seek reformation or other traditional
contract or tort remedies.
b. Covenant of Quiet Enjoyment
(1) Every lease contains a covenant of quiet enjoyment
(express or implied). In the covenant of quiet enjoyment, the
landlord promises that the tenant will not be disturbed by the
landlord (or someone claiming through the landlord) during his
possession of the premises. A tenant may treat the lease as
terminated and withhold rent if the covenant of quiet enjoyment
has been breached by an actual or constructive eviction.
(a) A tenant may treat the lease as terminated and withhold
rent if the landlord or someone claiming through him
breaches the covenant of quiet enjoyment by actual
eviction of the tenant.
1) Under the majority rule, if a tenant is actually evicted
from part of the premises by the landlord, the
tenant is relieved of all liability for rent, even if the
tenant continues to occupy the rest of the premises.
However, the Second Restatement of Property rejects
the majority rule and calls for a rent abatement only.

392
REAL PROPERTY

2) Under the Second Restatement of Property minority


rule, when a tenant is partially evicted by an act of the
landlord (or his agent), rent abatement attributable to
the portion from which the tenant has been evicted
is appropriate.
(b) A tenant may treat the lease as terminated and with-
hold rent if the landlord breaches the covenant of quiet
enjoyment by constructive eviction of the tenant. For a
constructive eviction to exist:
1) the landlord’s act must substantially and permanently
interfere with the tenant’s use and enjoyment of the
premises; and
2) the tenant must move out.
(c) The following acts by the landlord constitute
constructive eviction:
1) withholding something essential to the full enjoyment
of the property that is included within the terms of the
lease, such as heat; or
2) withholding something required by statute, such as
hot and cold running water.
(d) A landlord probably constructively evicts the tenant if he
breaches one of the following landlord duties created at
common law:
1) making and keeping the premises habitable in a
short-term lease of a furnished dwelling;
2) disclosing latent defects existing in the premises,
known to the landlord, and undiscoverable by the
tenant at the time the lease was made;
3) maintaining common areas used by all tenants;
4) using due care in making promised or
volunteered repairs;
5) refraining from making fraudulent misrepresentations
concerning the condition of the leased premises; or
6) in some jurisdictions, abetting immoral conduct or
abetting nuisances on properties owned by the
landlord that may affect the leased premises.
(e) Under the majority rule, if a tenant is constructively evicted
from part of the premises, the tenant may receive a rent
abatement but is not relieved of all liability for rent.
(f) In some states, if a landlord’s breach of the covenant of
quiet enjoyment does not constitute actual eviction or is
not substantial enough to constitute constructive eviction,
the tenant who remains on the premises can seek money
damages from the landlord.

393
OUTLINE

CONSTRUCTIVE EVICTION AND BREACH OF IMPLIED WARRANTY OF HABITABILITY


Basis for Effects of Breach
Theory Remedies Available
Cause of Action on the Lease

Constructive The tenant must The tenant is The tenant may treat
Eviction show an interference excused from further the lease as canceled.
(Breach of Covenant with beneficial use performance under Rent is abated.
of Quiet Enjoyment) and enjoyment of the lease. The tenant
the premises serious must vacate the
enough to amount to premises within a
an eviction. reasonable time.

Breach of Implied The tenant must show The tenant is The tenant may treat
Warranty of a patent or latent excused from further the lease as canceled.
Habitability defect in essential performance under the Rent is abated. The
residential facilities. lease. The tenant may, tenant may seek
but need not, vacate money damages,
the premises. reformation, or other
traditional contract or
tort remedies.

E. Special Problems
1. Rule Against Perpetuities
a. The common law Rule Against Perpetuities provides that “[n]o
interest is good unless it must vest, if at all, not later than 21 years
after some life-in-being at the creation of the interest.”
b. To determine whether an interest violates the common law Rule
Against Perpetuities, the following analysis should be applied:
(1) Identify the interests subject to the Rule Against Perpetuities.
(a) Interests which are not subject to the Rule Against
Perpetuities include:
1) present possessory estates;
2) charitable trusts;
3) resulting trusts; and
4) interests that are fully vested at the time of creation.
The following interests are fully vested:
a) reversionary interests; and
b) completely vested remainders.
(b) Interests which are subject to the Rule Against
Perpetuities include:
1) options to purchase land not incident to a lease;
2) powers of appointment;
3) rights of first refusal; and
4) interests that are not fully vested at the time of
creation, including:

394
REAL PROPERTY

a) remainders that are subject to open;


b) contingent remainders; and
c) executory interests.
(2) Identify the life or lives in being, express or implied by the instru-
ment creating interests. There is no limit on the number of lives
in being, as long as they may be identified by or through the
creating document. A life-in-being may, but need not, receive an
interest in the document that creates the interest. However, to
serve as a measuring life, a life-in-being must in some way be
connected to the vesting of the interest under scrutiny.
(a) An express life-in-being is a person named in the document
that creates the interests under scrutiny. In the grant “O
conveys to A for life,” O and A are express lives in being.
(b) An implied life-in-being is a person who is not named in, but
may be implied from, the document that creates the interests
under scrutiny. In the devise “to my grandchildren,” the testa-
tor’s children are implied lives in being.
(c) A class cannot be used as measuring lives unless the class
is closed at the time the gifts are created.
(3) Determine whether the interest will vest or fail within 21 years of
the life or lives. Under the common law Rule Against Perpetuities,
interests are analyzed under the “might have been” rule. Under
the “might have been” rule, an interest violates the Rule Against
Perpetuities if there is any chance, however remote, that the
interest might vest more than 21 years after a life-in-being.
(a) Under the might-have-been rule, interests are scrutinized
at the time of creation. The time of creation is the date
that the creating instrument takes effect. If the interests
under scrutiny are created in a deed, the time of creation
is the date that the deed is delivered. If the interests under
scrutiny are created in a will, the time of creation is the
date of the testator’s death.
(4) Treat the part of the gift that violated the Rule Against
Perpetuities as void, leaving the remainder of the gift intact.
EXAMPLE: O conveys “to A for so long as liquor is not served
on the premises, but if liquor is served, then to B.” Because the
vesting of the future interest is unlimited in time, it is not certain
to vest or fail within 21 years of any life-in-being. Therefore, the
shifting executory interest in B is void. A is left with a fee simple
determinable, and O has a possibility of reverter.
EXAMPLE: O conveys “to A, but if liquor is served, then to B.”
Again, the shifting executory interest in B is void. However,
in this case, A is left with a fee simple. Therefore, O retains
nothing, and A is left with a fee simple absolute.

395
OUTLINE

c. Applications
(1) Fertile Octogenarian
(a) Imagine that O conveys “to A for life, remainder to A’s chil-
dren for life, remainder to A’s grandchildren.”
(b) A’s life estate is presently possessory, and therefore is not
subject to the Rule Against Perpetuities. The remainder for life in
A’s children is either contingent or subject to open because A is
conclusively presumed to be capable of having more children,
and therefore is subject to the Rule Against Perpetuities. The
remainder in A’s grandchildren is either contingent or subject to
open, and therefore is subject to the Rule Against Perpetuities.
(c) O and A are express lives-in-being. Even if children or grand-
children are already born, these persons are members of
an open class and therefore cannot serve as lives-in-being.
Therefore, O and A are the only possible measuring lives.
(d) A’s children will take (or not take) at A’s death; hence, their
interest is valid. However, A’s grandchildren may well take
more than 21 years after O and A die. Thus, their interest
violates the Rule Against Perpetuities.
(e) A retains her life estate. A’s children will take for life after A dies.
Because the interest in A’s grandchildren is void, O retains a
reversion that will become possessory after A’s children die.
(2) Unborn Widow
(a) Imagine that O conveys “to A for life, remainder to A’s widow for
life, remainder to A’s children living at the death of A’s widow.”
(b) A’s life estate is presently possessory, and therefore is not
subject to the Rule Against Perpetuities. The remainder for
life in A’s widow is contingent because there cannot be a
widow of a living person, and therefore is subject to the Rule
Against Perpetuities. The remainder in A’s children living
at the death of A’s widow is contingent because there is a
condition precedent to vesting, and therefore is subject to
the Rule Against Perpetuities.
(c) O and A are express lives-in-being. A’s widow is not a life-in-
being because she has not yet been identified. A’s children,
even if some are already born, may not be lives-in-being
because their class is open. Therefore, O and A are the only
possible measuring lives.
(d) A’s widow will take (or not take) at A’s death; hence, her
interest is valid. However, A’s children may well take more
than 21 years after O and A die. Thus, their interest violates
the Rule Against Perpetuities.
(e) A retains his life estate. A’s widow will take for life after A dies
if she survives A. Because the interest in A’s children is void,
O retains a reversion, which will fall in when A’s widow dies.

396
REAL PROPERTY

(3) Charity-to-Charity Rule


(a) Imagine that O conveys “to A Charity for so long as liquor
is not served on the premises, but if liquor is served on the
premises, then to B Charity.”
(b) As illustrated above, a shifting executory interest in fee will
violate the Rule Against Perpetuities, because an heir of
A could violate the condition more than 21 years after the
death of O, A, and B, causing the property to shift to B.
1) However, for policy public reasons, if both A and B are
charities, the shifting executory interest in B Charity will be
deemed to be valid under the Rule Against Perpetuities.
(4) Rule of Convenience
(a) Imagine that O conveys “to such of A’s children as shall
reach the age of 25.”
1) Assume A is living at the time of this grant and has two
children: B, age 26; and C, age 24. Because she is 26, B
is entitled to immediate distribution. As a result, the Rule of
Convenience will operate to close the class of A’s children
immediately. All children born as of the date the class closes
(B and C) will be allowed to take upon reaching the age of
25. However, no child born after the date the class closes
may take, even if he should eventually reach the age of 25.
(b) Assume that A is alive at the testator’s death and has three
children: B, age 24; C, age 23; and D, age 19.
1) Because no child of A has reached 25, A’s children have
a springing executory interest, which is subject to the
Rule Against Perpetuities.
2) O and A are express lives-in-being. The class of A’s
children cannot be measuring lives because the class
remains open; A is alive, and no child of A is entitled
to immediate distribution because no child of A has
reached the age of 25.
3) It is possible that a child of A could reach 25 more than
21 years after the deaths of O and A. Therefore, the
springing executory interest in the children of A violates
the Rule Against Perpetuities.
4) O is left with a fee simple absolute.
(5) Uniform Statutory Rule Against Perpetuities
(a) Some states have adopted uniform statutory revisions of
the Rule Against Perpetuities (“USRAP”). Under the USRAP, a
nonvested interest in real or personal property is invalid unless:
1) it satisfies the common law Rule Against Perpetuities; or
2) it vests or terminates within 90 years of its creation (this is
known as the “wait and see” test). Under the wait-and-see

397
OUTLINE

branch of the USRAP, the court waits until the end of the
prescribed period to determine whether the interest actu-
ally vested or failed within the prescribed period.
EXAMPLE: In 1990, O conveys “to A for life, remainder
to A’s children for life, remainder to A’s grandchildren.”
Assume that, at the time the conveyance is made, A is
55 and has one child, B, who is 16. In 1995, B marries.
In 2001, C is born to B. A dies in 2004 at the age of 69.
B dies in 2040 at the age of 66. C may take the property
in 2040. The remainder to A’s grandchildren would be
void under the common law Rule Against Perpetuities;
however, because C actually takes within 90 years of
the conveyance, her interest is valid under the USRAP.
2. Powers of Appointment
a. A general power of appointment gives the holder of the power the
right to appoint the property to anyone, including the holder of the
power. Thus, a general power of appointment is considered to be
equivalent to ownership of the property.
(1) To be valid under the Rule Against Perpetuities, a general power
of appointment must be exercisable, but not necessarily exer-
cised, during the period allowed by the Rule Against Perpetuities.
b. A special power of appointment gives the holder the right to appoint
the property to a limited class of persons.
(1) To be valid under the Rule Against Perpetuities, a special power
of appointment must be exercised in a manner that causes the
interest in property thereby created to vest within the period of
the Rule Against Perpetuities.
3. Restraints on Alienation
a. A restraint on alienation is a condition placed on the ownership of
real property that restricts the free conveyance of that property.
(1) Under a disabling restraint, A may not convey. Disabling
restraints are always void.
EXAMPLE: O conveyed Blackacre “to A and his heirs so long as A or
his heirs do not sell, mortgage, or otherwise transfer his interest in the
property.” This is a disabling restraint, and will be considered void.
A (or his heirs) may freely sell, mortgage, or convey the property.
(2) Under a forfeiture restraint, A loses his estate if he attempts to
convey. Forfeiture restraints are valid for life estates and future
interests, but are not enforceable for fee simple estates.
EXAMPLE: O conveys Blackacre “to A and his heirs, but if A or
his heirs should ever convey the property to anyone, then to B
and her heirs.” This is a forfeiture restraint on a fee simple estate,
and would not be enforceable.

398
REAL PROPERTY

EXAMPLE: O conveys Blackacre “to A for life, then to B and her


heirs, provided, however, that should A ever sell, mortgage, or
otherwise transfer the property, then A’s estate shall immediately
terminate in favor of B and her heirs.” This is a forfeiture restraint
on a life estate, and would be enforceable.

EXAMPLE: O conveys Blackacre “to A for life, then to A’s chil-


dren, provided, however, that should A’s children attempt to
sell, mortgage, or otherwise transfer the property, then to B and
her heirs.” This forfeiture restraint would be valid only while A’s
children possess a future interest in Blackacre, and would be
enforceable if they attempted to convey their future interest in the
property. Once their interest becomes possessory as a fee simple
estate, however, the restraint would no longer be valid.
(3) Under a promissory restraint, A promises not to convey.
Promissory restraints are valid for life estates and future
interests, but are not enforceable for fee simple estates. The
breach of a promissory restraint does not void the conveyance; it
just makes the promisor liable for breach of contract damages.
EXAMPLE: O conveys Blackacre “to A for life, then to B and his
heirs. A hereby promises not to convey the property during his
lifetime.” This is a promissory restraint, and is valid to the same
extent that a similar forfeiture restraint would be valid.
(4) Partial restraints may be enforced, with the court weighing the
duration and number of persons excluded.
(5) Some courts will uphold a reasonable restraint in a commercial
transaction on the theory that it appears in an agreement of
the parties, is the product of their bargaining, and presumably
serves a useful purpose in facilitating the parties’ objectives.
4. Fair Housing and Discrimination
a. The Fair Housing Act is Title VIII of the Civil Rights Act of 1968 [42 U.S.C.
§§ 3601, et seq.]. Under it, among other prohibitions, a person cannot:
(1) refuse to sell or rent a dwelling, or discriminate in sales or rental
terms, to any person based on race, color, religion, sex, hand-
icap, familial status (presence or anticipated presence of chil-
dren), or national origin;
(2) advertise the sale or rental of a dwelling indicating discrimina-
tion based on race, color, religion, sex, handicap, familial status,
or national origin; or
(3) coerce, threaten, intimidate, or interfere with a person’s exer-
cise of housing rights for discriminatory reasons or retaliate
against anyone who encourages the exercise of housing rights.
b. Religious organizations providing housing for members of their order
are exempt [42 U.S.C. § 3607].

399
OUTLINE

c. Housing specifically for older persons is exempt, subject to certain


requirements [Id.].
d. Enforcement by HUD
(1) An aggrieved person, within one year after an alleged
discriminatory housing practice has occurred, may file a
sworn complaint with the Secretary of Housing and Urban
Development (HUD) [42 U.S.C. § 3610].
(2) The Secretary may also file complaints [Id.].
(3) Within 10 days after the complaint is filed, the Secretary must
notify the complainant and the respondent, after which the
respondent has 10 days to file a sworn answer [Id.].
(4) The Secretary must complete the investigation into the complaint
within 100 days, unless it is impracticable to do so [Id.].
(5) The Secretary must encourage the complainant and the
respondent to create a conciliation agreement [Id.].
(6) The Secretary, based on reasonable cause to believe that a
discriminatory housing practice has occurred or is about to occur,
must either issue a charge on behalf of the complainant or
dismiss the complaint.
(7) If a charge is issued, the complainant may then elect
whether to have an administrative hearing or a judicial trial
[42 U.S.C. § 3612].
(8) At the conclusion of either an administrative hearing or a trial,
where the complainant has prevailed, equitable and legal
remedies may be awarded, including actual damages, statutory
monetary penalties, attorneys’ fees, and costs [Id.].
(9) The result of either an administrative hearing or a trial may be
appealed through the federal court system [Id.].
e. Enforcement by a Private Person
(1) An aggrieved person may file suit in a federal or state court no
later than two years after an alleged discriminatory housing
practice [42 U.S.C. § 3613].
(2) The aggrieved person may file suit without having exhausted
administrative remedies by first filing a complaint with the
Secretary of Housing and Urban Development [Id.].
(a) However, if a complaint has been filed and is in the hearing
stage, no suit may be filed [Id.].
(3) An aggrieved person may also file suit to enforce a conciliation
agreement reached through HUD [Id.].
(4) If the aggrieved person prevails in a civil action, the court may
award actual damages, punitive damages, and such other legal
and equitable relief as the court deems appropriate, including
attorneys’ fees and costs [Id.].

400
REAL PROPERTY

f. Enforcement by the United States Attorney General


(1) Upon reasonable cause to believe that a person or group is
engaged in a pattern or practice of housing discrimination, the
Attorney General may commence a civil action in federal court
enforce the Fair Housing Act [42 U.S.C. § 3614].
(2) The Attorney General may also enforce conciliation agree-
ments [Id.].
g. Intent Not a Required Element
(1) The United States Supreme Court has held that no proof of intent
to violate the Fair Housing Act is needed in order for a complainant
to prevail [Texas Department of Housing and Community Affairs v.
The Inclusive Communities Project, Inc., 135 S. Ct. 2507 (2015)].
(a) Thus, disparate impact claims are cognizable under the Act.
The complainant, if using such a theory, must show a dispa-
rate impact on a protected group caused by the respon-
dent’s actions. The burden then shifts to the respondent to
prove that the challenged practice is necessary to achieve a
substantial nondiscriminatory interest [Id.].
5. Conflicts of Law Related to Real Property
a. In property cases, two characterizations must be made.
(1) First, the problem must be characterized as to whether or not it
involves a property interest (e.g., an agreement to sell land might
be characterized as a contract, rather than a property, problem).
(2) Once it is determined that a property interest is involved, that
interest must be characterized as a “movable” or “immovable”
interest [Restatement (Second) of Conflict of Laws § 222, cmt. e].
(a) If an interest is closely connected with or related to land
(e.g., a leasehold or the right to rents), it is immovable; if it
is not, the interest falls into the movable category.
1) Characterization of the interest traditionally has been
made by the forum using its own internal law. However, the
Second Restatement suggests that characterization should
be made according to the law of the situs of the property
[Restatement (Second) of Conflict of Laws § 189, cmt. c].
(b) The law of the situs governs all rights in land and other
immovables. In applying the law, the forum state refers to
the whole law of the situs, including its choice-of-law rules
[Restatement (Second) of Conflict of Laws §§ 223; 223, cmt. b].
1) Generally, the validity and effect of a conveyance
(e.g., the form of the deed or the capacity of the grantor)
are governed by the law of the situs [Restatement
(Second) of Conflict of Laws § 223(1)].
2) Mortgages likewise are deemed so closely related to
the land that their creation, validity, and foreclosure

401
OUTLINE

are governed by the law of the situs [Restatement


(Second) of Conflict of Laws § 228].
3) However, the underlying contract or note is usually
governed by the law of the place of making [Restatement
(Second) of Conflict of Laws § 224, cmt. e].
4) Liens (such as materialmen’s or laborers’ liens) are also
governed by the law of the situs [Restatement (Second)
of Conflict of Laws § 230].
5) Some courts hold that executory contracts for the sale of
land are generally characterized as a contracts problem
and governed by the law of the place of making.
a) However, the Second Restatement provides that
in the absence of an effective choice of law by the
parties, the validity of a contract for the transfer of an
interest in land, and the rights created thereby, are to
be determined by the local law of the state where the
land is located, unless, with respect to the particular
issue, some other state has a more significant rela-
tionship to the transaction and the parties, in which
event the local law of the other state will be applied
[Restatement (Second) of Conflict of Laws § 189].

402
REAL PROPERTY

II. RIGHTS IN LAND

A. Covenants at Law and Equity


1. Nature and Type
a. A covenant that runs with the land is a promise that attaches to land.
In that covenant, the covenantor promises to do or refrain from doing
something on his land.
(1) In contrast to easements, most running covenants are
negative in nature.
EXAMPLE: Landowner promises that she will not build
anything other than a single-family residence on her premises.
(2) Running covenants include promises to pay rent, condominium
common charges, or maintenance fees.
b. Equitable Servitude
(1) A court may enforce a covenant as an equitable servitude if:
(a) the plaintiff can establish at law all of the elements for a covenant
that runs with the land, but the plaintiff seeks equitable relief; or
(b) the plaintiff cannot establish at law all of the elements for a
covenant that runs with the land, but the plaintiff can demon-
strate the relaxed requirements for an equitable servitude.
These elements relate closely to the elements required for a
covenant to run at law.
c. Implied Reciprocal Servitude
(1) If the owner of two or more lots, so situated as to bear the
relation, sells one with restrictions of benefit to the land retained,
the servitude becomes mutual and the owner of the lot or lots
retained may not do anything forbidden to the owner of the lot
sold, creating an implied reciprocal servitude.
2. Creation
a. Covenants
(1) For a covenant to run with the land, the following elements must
be satisfied:
(a) writing;
1) A covenant that runs with the land is a promise that attaches
to land. Therefore, a covenant that runs at law must be
embodied in a writing that satisfies the Statute of Frauds.
(b) intent;
1) The writing must include language that shows the
parties’ intent for the covenant to run.
2) Typically, the covenant will state that the covenantor
promises on behalf of himself and his heirs, successors,
and assigns to do or to refrain from doing something on

403
OUTLINE

his land for the benefit of the covenantee and his heirs,
successors, and assigns.
(c) privity;
1) Horizontal privity is the relationship that exists
between the original covenantor and covenantee.
2) In most jurisdictions in the United States, the horizontal
privity requirement is satisfied by a conveyance of land
between the covenantee and covenantor, which occurs
by the same deed that includes the covenant.
3) Horizontal privity may also exist when the covenant is
created in a lease or in the transfer of an easement.
a) Traditionally, horizontal privity was required for both
the benefit and the burden to run.
b) Under the modern rule, horizontal privity is not
required for the benefit to run.
4) Vertical privity is the relationship that exists between
an original party to a running covenant and the
successor in interest to the original party.
5) To demonstrate vertical privity, the plaintiff must show that
the successor “stepped into the shoes of” the original party
by taking the entire interest held by the original party.
a) For example, if the original covenantor took a fee
simple interest in land and then attempted to make
a conveyance to a subsequent party, vertical privity
would exist between the original covenantor and
the successor only if the successor received a fee
simple interest.
b) Traditionally, vertical privity is required for both the
benefit and burden to run.
c) For the burden to run, privity of estate will only exist
when the holder of the servient estate transfers all of
his interest in the servient estate to the new owner.
d) For the benefit to run, privity of estate can exist
when the holder of the dominant estate is transfer-
ring all or part of his interest.
(d) touch and concern; and
1) To touch and concern, a covenant must exercise direct influ-
ence on the occupation, use, or enjoyment of the premises.
2) Traditionally, both the benefit and the burden had to
touch for either side to run.
3) The benefit touches if it increases the value of the
benefited property.
4) The burden touches if it decreases the value of the
burdened property.

404
REAL PROPERTY

(e) notice.
1) Originally, notice was not a requirement for a covenant to run
with the land at law. However, the introduction of recording
statutes into American law caused a notice requirement to
be grafted onto the requirements for a running covenant.
2) The parameters of notice are determined according to
the terms of the relevant recording statute.
3) Notice under recording acts is necessary for the
burden to run.
b. Equitable Servitudes
(1) If the plaintiff cannot show that the covenant was embodied
in a writing, the plaintiff may prove the covenant through part
performance or estoppel.
(2) If the plaintiff cannot show intent in a writing for the covenant to run,
he may nevertheless establish intent by showing a common scheme.
(a) A common scheme typically exists in a subdivision. If a
sufficient number of lots in the subdivision are burdened
by the same covenant, the court may find that a common
scheme binds all of the lots in the subdivision, including
those that do not have the restriction written into the deed.
(b) The following factors may show a common scheme:
1) a large percentage of lots expressly burdened;
2) oral representations to buyers;
3) statements in written advertisements, sales brochures,
or maps given to buyers; or
4) recorded plat maps or declarations.
(c) In the absence of an express restriction in the deed, the
burden will not be imposed on a lot conveyed before the
conveyance of the first lot with an express restriction.
(d) The burden will be imposed on lots (even in the absence of
an express restriction in the deed to those lots) if a common
scheme was evident at the time of conveyance of those lots.
(e) The original subdivider may enforce a benefit unless he has
sold all of the property.
(f) Traditionally, a benefit could not be enforced by a buyer who
purchased his subdivision lot before the conveyance of the
burdened lot. In some jurisdictions, the existence of the common
scheme is sufficient to show that the subdivider intended to
benefit all purchasers, whether they took prior or subsequent
to the conveyance that created the burden. In other jurisdic-
tions, a previous grantee may sue as a third-party beneficiary.
(g) If a common scheme is present, suit may be brought by the
purchaser of a lot conveyed after the lot with the express
restriction (because that restriction was for the benefit of

405
OUTLINE

the grantor and his heirs and assigns, and the subsequent
purchaser is a successor to the grantor).
(3) Neither horizontal nor vertical privity are required for the
enforcement of an equitable servitude.
(4) The intent element must be demonstrated for both the benefit
and burden.
(5) The touch and concern element must still be demonstrated for
both the benefit and burden.
(6) The parameters of notice for the burden to run are determined
according to the terms of the relevant recording statute.
c. Implied Reciprocal Servitudes
(1) In most jurisdictions, a negative reciprocal easement may be created
by filing a declaration containing the restrictions (the “CC&Rs”, which
are covenants, conditions, and restrictions) before any lots are sold.
(2) A condominium or subdivision association may enforce a benefit
if the covenant is to benefit common land conveyed by the
developer to the association.
3. Termination
a. As with easements, covenants can be terminated by merger, release,
abandonment, or estoppel.
b. Covenants may also be terminated by changed circumstances (also known
as the “change of neighborhoods” doctrine). Under this concept, where the
neighborhood or circumstances have changed so greatly that it no longer
makes sense to enforce the restriction, it will be deemed terminated.
4. Property Owners’ Associations and Common Interest Ownership
Communities
a. A property owners’ association generally has standing to enforce a
restrictive covenant.
b. When a court is examining whether an association’s board has acted
properly, the standard applied appears to be similar to the business
judgment rule applied to corporations.
(1) Under this rule, there is a rebuttable presumption that an associa-
tion’s board members are honest, well-meaning, and acting through
decisions that are informed and rationally undertaken in good faith.
(2) As a result, a board will not have breached any duty of care for
making a good-faith error of judgment.

B. Easements, Profits, and Licenses


1. Nature and Type
a. Easements
(1) An easement is an interest in the land of another.
(2) Easements may be either affirmative or negative, and they may
be either appurtenant or in gross.

406
REAL PROPERTY

(a) Most easements are affirmative in nature. An affirmative


easement gives the holder the right to do something on
the land of another.
EXAMPLE: Alpha has the right to walk across Beta’s land.
(b) A negative easement gives the holder the right to
prevent a landowner from doing something on his land.
The common law recognized only the following four types
of negative easements: light, air, water, and lateral and
subjacent support. A writing is always required to create a
negative easement.
EXAMPLE: Alpha may prevent Beta from building a house
that blocks light to Alpha’s land.
(c) An easement appurtenant requires a dominant and
servient tenement.
EXAMPLE: A has the right to cross over B’s property from
the road to reach A’s property. A has the dominant estate
because the easement benefits A in the use of her property.
B has the servient estate because her land is burdened with
the easement. Therefore, A has an easement appurtenant.
(d) An easement in gross is personal in nature, resulting in a
servient but not a dominant estate.
EXAMPLE: A utility company has the right to lay its utility
cable across A’s land. The cable assists the company in
providing its utility services to its customers. A’s land is
the servient estate because it is burdened by the utility
company’s right to lay its cable across A’s land. However,
there is no dominant estate; the benefit of the easement
is personal to the utility company because the easement
does not benefit the company in the use of its land.
Therefore, the utility company has an easement in gross.
b. Profits
(1) A profit à prendre is a nonpossessory interest in land.
(2) The holder of the profit has the right to take resources from the
land of another.
(a) Profits, unlike easements, permit the right to remove some-
thing from the land, as opposed to mere use of the land.
(b) Examples of resources include, but are not limited to, soil,
timber, and minerals.
EXAMPLE: A has the right to take gravel from B’s land.
c. Licenses
(1) A license is a privilege, usually to do something on someone
else’s property.

407
OUTLINE

2. Methods of Creation
a. An easement may be created expressly through a writing, by implica-
tion, or by prescription.
b. Express
(1) An easement is an interest in land. Therefore, the Statute of
Frauds generally requires a writing to create an easement.
c. Implied
(1) An affirmative easement may be created by implication either by
prior (quasi-) use or by necessity.
(2) Implied by Prior or Quasi-Use
(a) An easement implied by prior use requires:
1) severance of title to land held in common ownership;
2) the use giving rise to the easement was in existence at
the time of the severance;
a) That use alone cannot constitute an easement
because an owner cannot have an easement
across his own property.
3) the use was apparent and could be discovered upon a
reasonable inspection; and
4) at the time of the severance, the easement was neces-
sary for the proper and reasonable enjoyment of the
dominant tract.
a) However, some courts may require strict necessity if
the implied easement is reserved and not granted.
(3) Implied by Necessity
(a) An easement implied by necessity requires:
1) severance of title to land held in common ownership; and
2) strict necessity for the easement at the time of severance.
EXAMPLE: O owns a large parcel of land. The parcel is
bordered by a two-lane road on the east and dense forest
on the north, south, and west. O lives in a house at the
northwest corner of the parcel and regularly drives to and
from his house on a gravel road that crosses the northeast
portion of the parcel and opens onto the road. O then con-
veys the west half of the parcel to A. When A attempts to
access the road by driving across the northeast portion of
the parcel, he finds O has erected a barrier. The question
is whether A has an implied easement across the portion
of the parcel retained by O. O initially owned the entire
tract, thereby satisfying the initial unity of ownership re-
quirement. He then severed that unity when he conveyed
the west half of the parcel to A. A has no direct access to
his parcel because it is surrounded by O’s parcel on the

408
REAL PROPERTY

east and dense forest on the north, south, and west. The
landlocked nature of A’s estate should satisfy the strict ne-
cessity requirement for an implied easement by necessity.
(4) Implied by Plat
(a) A purchaser who acquires a lot in a platted subdivision will
generally acquire an implied private easement to use any
streets or alleys (and possibly even parks) shown on the plat.
d. Prescription
(1) An affirmative easement may be created by prescription, which
requires proof of the use of property that is:
(a) open and notorious;
(b) actual;
(c) continuous (the traditional period for prescription is 20 years);
(d) hostile; and
(e) exclusive.
1) Some courts require prescriptive use to be “exclusive.”
In this context, exclusive means that the use is not
shared with the whole world. The user may share use of
the easement with the owner.
e. Profits and Licenses
(1) Profits can only be created expressly or by prescription.
(2) Neither a writing nor consideration is required to create a license.
3. Scope
a. If the language creating the easement limits its use, then that language
will be enforced.
b. However, when language is not specific as to use, the easement holder
can make reasonable use of the easement.
c. If an easement holder overuses the easement (i.e., beyond what is
reasonable), this is known as surcharging the easement. This does
not result in a termination of the easement, but will entitle the owner
of the servient estate to sue for damages caused by the surcharging,
and/or seek an injunction to stop future surcharging.
4. Transfer
a. Both the benefit and the burden of an easement may be transferred
according to the following rules:
(1) when the dominant tenement is transferred, the benefit of an
easement or a profit appurtenant follows the transferred estate,
even if the deed of conveyance does not specifically mention the
easement or profit; or
(a) Traditionally, the benefit of an easement in gross could not
be transferred. However, the modern rule permits transfer
of an easement in gross if the grantor of the easement so
intends, and the easement is commercial in nature.

409
OUTLINE

(2) in the case of an express easement, transfer of the servient


estate will result in an accompanying transfer of the burden, so
long as the holder of the dominant tenement has complied with
the provisions of the relevant recording statute.
b. All rules governing the alienation of easements are applicable to profits.
c. Because a license is personal, it is not transferable unless the licensor
so intends.
5. Termination
a. An easement may be terminated by any of the following methods:
(1) an easement may be created for a specific term, such as 10 years;
(a) When the time period ends, the easement is terminated.
(2) a release occurs when the holder of the dominant estate releases
his interest to the holder of the servient estate;
(3) an easement is terminated when the dominant and servient
estates come into common ownership;
(4) an easement may be terminated by abandonment;
(a) Abandonment requires proof of:
1) intent to abandon; and
2) an affirmative act in furtherance of the intent.
(5) an easement may be terminated by estoppel;
(a) Estoppel requires proof of:
1) an act or representation in respect to the easement;
2) justifiable reliance on that act or representation; and
3) damages.
(6) an easement may be terminated by prescription (use adverse to
the easement for the statutory period);
(7) an easement will be terminated when a governmental body
acquires the servient estate through an exercise of the eminent
domain power; or
(8) an easement may be terminated by the sale of the servient estate
to a bona fide purchaser for value without notice.
(a) Easements are interests in land, and so follow the same
recording rules as other interests. Thus, a subsequent bona
fide purchaser for value and without notice who satisfies the
requirements of the recording statute will take the servient
estate free of the easement.
(b) This applies to express easements. Courts are split as to
whether a servient estate subject to an implied easement,
such as an easement implied by necessity, would be trans-
ferred with or without the easement in such a situation.
b. All rules governing the termination of easements are applicable to profits.
c. A license expires on the death of the licensor or the conveyance of the
servient estate.

410
REAL PROPERTY

d. A license is generally revocable at the will of the licensor. However, a


license may become irrevocable in two situations:
(1) a license coupled with an interest exists when one person
owns personal property on the land of another and has a privi-
lege incidental to such personal property to come on the land to
use or recover the personalty; and
(2) an executed license is based on estoppel and often involves
substantial expenditure of funds in reliance on the promisor’s
promise to allow the promisee to use the land. An executed license
is sometimes deemed to be the equivalent of an easement.

C. Fixtures
1. In General
a. A fixture is a chattel that has become so connected to real property that a
disinterested observer would consider the chattel to be part of the realty.
EXAMPLE: If a homeowner attaches a bookshelf to the wall of her
home, it retains its separate identity after the attachment, and thus
is not a fixture. However, the same bookshelf built into the wall
becomes part of the realty, and thus is a fixture.
b. When one person owns a chattel and the land to which the chattel is
to be fixed, the chattel becomes a fixture if:
(1) it is annexed to the real property;
(2) it has been appropriated to the use of the land; and
(3) the annexor intends it to be a fixture.
c. A chattel may be deemed annexed to the land if:
(1) it is permanently attached to real property or to something
appurtenant to the real property;
EXAMPLE: A chain-link fence may be deemed annexed to the
land if it is erected around a parcel of land by placing posts into
holes in the ground.
(2) it would be difficult to move solely based on its own weight; or
EXAMPLE: A large, heavy statue may be deemed annexed to the
land if it is placed on the ground in a backyard garden.
(3) it is constructively annexed to the property by being specially
designed for the property or a fixture on the property.
EXAMPLE: Hurricane shutters custom designed for a house
may be classified as annexed to the house, even if they are re-
moved and stored in the garage during winter months.
d. An annexed chattel may become a fixture if the chattel is so
necessary or convenient to the use of the land that it is commonly
regarded as part of the land. In determining whether a chattel has
been appropriated to the use of the land, the court will consider the
nature of the land or structure to which the chattel is affixed.

411
OUTLINE

EXAMPLE: Chairs in a movie theater will be deemed appropriated to


the use of the land, while chairs placed in the kitchen of an apartment
may not be regarded as such.
e. Intent to annex a chattel is a question of fact that is judged objec-
tively, according to a reasonable person standard. The finder of fact
considers the nature of the chattel, the degree of annexation, and the
appropriateness of the chattel to the property’s use.
2. Transfers, Severance, and Change of Ownership
a. When both the chattel and the land are owned by the same person, a
deed to the real property will transfer all of the fixtures on the property.
However, the buyer and seller may agree that the seller may remove
certain fixtures from the property before title is transferred to the buyer.
EXAMPLE: A deed describing a lot of land also transfers the house on
the lot and the plumbing fixtures in the house.
b. An owner of a chattel may affix it to land also owned by that person
which later becomes encumbered by a mortgage. In that case, the
mortgage extends to both the land and the fixture. If the chattel is
attached to land already encumbered by a mortgage, the prior mort-
gage also encompasses the chattel, even though the mortgagee
thereby receives a security interest that is greater than the one
bargained for by the mortgagee.
c. If a fixture and the land to which it is attached are subject to common
ownership, the owner is free to sever the chattel physically and perma-
nently from the premises. In that case, the fixture regains its status
as personal property. A constructive severance of a fixture may occur
when the owner of the land expressly reserves the fixture in a contract
of sale for the land or deed of conveyance of the land.
d. Under Article 2 of the Uniform Commercial Code, a contract for the sale
of a structure to be removed from realty is a contract for the sale of
goods if the structure is to be severed from the land by the seller. Article
2 also applies to contracts for the sale of other things attached to realty
and capable of severance without material harm to the land, regardless
of whether the item is to be severed by the buyer or by the seller.
3. Trade Fixture
a. A trade fixture is a chattel that is annexed to the land by a tenant
(i.e., a life tenant, tenant for a term, periodic tenant, or tenant at will) to
advance his business or trade during his tenancy.
b. During a tenancy, a tenant is free to remove a trade fixture. If a tenant
removes a trade fixture, he is responsible for repairing damages
caused by its removal. In some states, the trade fixture exception has
been broadened to include domestic and/or ornamental fixtures.
c. At common law, if the tenancy is for a definite term and ends on a
certain day, trade fixtures must be removed before the end of the term
or they become part of the realty.

412
REAL PROPERTY

(1) Most states refuse to follow the common law rule that a tenant forfeits
his trade fixtures by failing to remove them before expiration of the lease
term. To encourage trade and industry, a tenant is permitted to remove
trade fixtures within a reasonable time after the expiration of a lease.
EXAMPLE: L leases Blackacre to T as a store for three years,
with a set termination date. During the term, T firmly affixes
shelves to the floor of the store building. These shelves are still
standing the day after the lease terminates. Under the common
law rule, the shelves remain fixtures on the property and belong
to L because T did not remove them before the end of the term.
However, under the modern view, T has not yet forfeited his
interest in the shelves. Instead, T will be given a reasonable time
after the termination date to remove the shelves.

D. Zoning
1. Authority and Scope
a. Authority
(1) The power to zone is granted by statute from a state to a city, county,
township, or other appropriate political subdivision. It gives the polit-
ical subdivision the ability to divide its geographical areas into zones
where some uses are permitted and other uses are prohibited.
(2) Ordinances that do not conform to state enabling acts are consid-
ered ultra vires, or unauthorized and beyond the scope of power
allowed or granted by law.
b. Classifications
(1) Common zoning classifications include residential, commercial,
agricultural, industrial, spatial, and mixed-use, as well as subsets
of these classifications.
EXAMPLE: A developer wishes to build a high-rise apartment
building in a desirable area of the city, but the area is zoned for
single-family residential uses only. The developer needs a district
that is zoned high-density residential.
EXAMPLE: A small airport is planned for a district that is currently
zoned agricultural. The district must be rezoned to add spatial as
an additional classification.
c. Purpose
(1) Zoning ordinances must promote the public health, safety, pros-
perity, morals, and welfare because the ordinances arise from the
state’s police power [Berman v. Parker, 348 U.S. 26 (1954)].
d. Uniformity
(1) Zoning ordinances must be uniform for each class or kind of
buildings and uses throughout each district, but the regulations in
one district may differ from those in other districts.

413
OUTLINE

e. Scope
(1) The regulations in each district may regulate, restrict, permit,
prohibit, and/or determine:
(a) the use of land, buildings, and structures;
(b) the size, height, area, location, construction, repair, and
removal of structures;
(c) the areas and dimensions of land, water, and air-space to be
occupied and open spaces to be left unoccupied; or
(d) the excavation or mining of soil or other natural resources.
EXAMPLE: A homeowner planned to build a garage that was
flush with the edge of his street, for ease of entering the garage.
However, he learned, upon applying for a building permit, that a
15-foot set-back from the road was required for any structure.
EXAMPLE: The owner of a shopping mall intended to expand
in order to build additional stores. Before proceeding too far
in the planning process, he reviewed the local zoning code to
see how many new parking spaces would be required.
f. Enforcement
(1) Notice of Zoning Violation
(a) When a property is found to be out of compliance with the
zoning code, the political subdivision will issue to the prop-
erty owner a Notice of Zoning Violation.
(2) Cease and Desist
(a) A Notice of Zoning Violation may contain instructions to
cease and desist from a certain use, or instructions on how
to modify the property to bring it into compliance, or other
information that informs the property owner about the viola-
tion and how to cure it.
EXAMPLE: The buyer of a house is told by the seller that
the house is a “legal triple,” by which he means that it is
permitted as a three-family home. The buyer sees the three
separate entrances and the three separate utility meters.
The buyer purchases the house intending to live in one part
and rent out the other two parts. As soon as the buyer rents
out the other parts of the house, the buyer receives a Notice
of Zoning Violation, stating that the area is zoned for single-
family houses only, that the house has no Certificate of
Occupancy for three families, and that the buyer must cease
and desist renting out the other parts of the house.
(b) If the property owner does not protest the notice, and does
not bring the property into compliance, the political subdivi-
sion may choose to obtain a court-ordered injunction or take
other steps to enforce the notice.

414
REAL PROPERTY

(3) Civil Fines


(a) A Notice of Zoning Violation usually recites that civil fines will
be owed if compliance is not achieved by the deadline set
in the notice. It is a common practice for fines to accumulate
daily until compliance is achieved.
(4) Criminal Misdemeanor
(a) In some jurisdictions, zoning violations can also be pros-
ecuted as criminal misdemeanors, if left uncorrected.
(5) Recording
(a) Some state laws provide that zoning violations that are not
corrected by the deadline contained in the notice will be
recorded, so that potential buyers of the property or lenders
are aware of the zoning violation.
(6) Challenges
(a) A Notice of Zoning Violation must include information on
how to protest the notice by appealing to the local board of
zoning appeals. If relief is not granted, an appeal to a court
must be provided.
(7) Tenants and Occupiers
(a) While the property owner is always liable for civil zoning
violations, whether committed by the owner or a tenant or
other occupier, in most jurisdictions a tenant or other occu-
pier who is committing the violation is also liable.
(b) An owner, however, cannot be liable for a criminal misde-
meanor committed by the tenant or occupier because the
element of intent is lacking.
2. Constitutional Limitations
a. In General
(1) The Due Process Clause of the Fourteenth Amendment provides
for both procedural and substantive due process.
(a) Procedural Due Process
1) Procedural due process requires notice and a fair
hearing, meaning an opportunity to be heard by a
neutral decision-maker.
2) When a property owner receives a Notice of Zoning
Violation and appeals it to the local board of zoning
appeals or other administrative board, there is a public
hearing. If the owner does not receive relief, an appeal
to the courts must be provided.
3) When a property owner requests a variance from the
zoning code, the owner is entitled to a public hearing
that has been noticed to all who might take an interest in
his request, including nearby property owners. If relief is
not granted, an appeal to the courts must be provided.

415
OUTLINE

4) Where appeal to the courts is taken, the court reviews the


administrative decision to see if it was illegal, arbitrary or capri-
cious, or unsupported by a preponderance of the evidence.
(b) Substantive Due Process
1) A zoning ordinance violates substantive due process if it
is arbitrary and capricious, meaning that it is not reason-
ably related to public health, welfare, or safety.
(2) Equal Protection
(a) A zoning ordinance may give rise to an equal protection chal-
lenge if similarly situated people are treated differently.
(b) However, where there is no fundamental right impacted, and
where there are no suspect classifications, then the rational basis
test will be applied. The zoning ordinance, as economic and
social legislation, will be upheld if it has a rational relationship to a
legitimate government interest, meaning the public health, safety,
and welfare [Village of Belle Terre v. Boraas, 416 U.S. 1 (1974)].
(3) First Amendment
(a) A zoning ordinance may also be subject to a First
Amendment challenge if it regulates billboards or aesthetics.
EXAMPLE: A city ordinance prohibiting the erection of bill-
boards in residential districts was found to be constitutional
as not unreasonable, arbitrary, or discriminatory, and as a
proper exercise of the police power [Thomas Cusack Co. v.
City of Chicago, 242 U.S. 526 (1917)].
b. Protection of Preexisting Property Rights—Takings
(1) The Fifth Amendment provides that private property shall not be taken
for public use without just compensation. This prohibition applies to the
states through the Due Process Clause of the Fourteenth Amendment.
(2) A land-use regulation is a “taking” if it denies an owner all reason-
able, economically beneficial uses of his land [Lucas v. South
Carolina Coastal Council, 505 U.S. 1003 (1992)].
(3) To analyze regulations that merely decrease economic value, the
court uses a balancing test to determine if there is a taking, which
happens rarely, considering the following factors [Penn Central
Transportation Co. v. New York, 438 U.S. 104 (1978)]:
(a) the economic impact of the regulation on the claimant;
(b) the extent to which the regulation has interfered with distinct
investment-backed expectations; and
(c) the character of the governmental action.
3. Rezoning and Other Zoning Changes
a. Rezoning
(1) If rezoning by the political subdivision is inconsistent with a
comprehensive plan for that area, then it must be based on a

416
REAL PROPERTY

change of conditions in the land, neighborhood, environment, or


public opinion.
(2) Rezoning of a particular piece of land is quasi-judicial and requires
procedural due process. A broader rezoning is legislative.
(3) If a property owner wants to use their land in a way that is not
allowed under the zoning classification, the owner may petition
for a rezoning to change the classification. While this is an option,
a use variance or special use permit is much easier to obtain.
b. Nonconforming Use
(1) A nonconforming use is a use permitted by zoning statutes or
ordinances to continue, notwithstanding the fact that similar uses
are not generally permitted in the area.
(2) A nonconforming use may not be expanded or rebuilt after
substantial destruction.
(3) Local ordinances often prohibit the enlargement, alteration, or
extension of a nonconforming use.
(4) Some local ordinances require certain nonconforming uses to be
amortized (reduced) over a specified period, at the end of which
they must be terminated.
c. Variances
(1) A variance is the permission by the local zoning authorities to use
property in a manner forbidden by the zoning ordinances in order
to alleviate conditions peculiar to a particular parcel of property.
(2) If a variance is sought from an area restriction, the petitioner must
show that there are practical difficulties in meeting the require-
ments of the zoning code or that the requirements are unreason-
able or create an undue hardship.
EXAMPLE: An accountant buys land in a commercial district to set up
a business office, but the zoning code requires 50 square feet of land-
scaping near the front door. There is no physical space for 50 square
feet of landscaping, so the accountant seeks an area variance.
(3) If the variance is sought from a use restriction, the petitioner must
show undue hardship, meaning that, without a use variance,
there is no viable use of the property.
EXAMPLE: An heir inherits a sawmill that has not been used in
decades, and is in a district that has been rezoned to retail. The
sawmill is not viable under the current zoning designation, nor is
it appropriate for a retail store. The heir seeks a use variance to
allow it to be remodeled into a warehouse.
d. Special Use Permits
(1) A special use permit is required for uses in an area not zoned for
those uses, but which would be beneficial to the public welfare
and compatible with the area.

417
OUTLINE

(2) An applicant for a special use permit is entitled to a public hearing


that has been noticed to all who might have an interest in the
application, including nearby property owners.
EXAMPLE: A hospital is planned to be built in a residential dis-
trict. A special use permit is needed because a hospital is not a
residential use.
EXAMPLE: A temple is planned to be built in a residential district.
A special use permit is needed because a temple is not a resi-
dential use.
e. Conditional Use Permits
(1) A conditional use permit is required for uses in an area not zoned
for those uses, but which would be beneficial to the public welfare,
and compatible with the area if certain conditions are met.
(2) The use is granted only if the applicant agrees to meet the addi-
tional conditions that the political subdivision imposes in order to
reduce the potentially objectionable impact of the use.
(3) An applicant for a conditional use permit is entitled to a public
hearing that has been noticed to all who might have an interest in
the application, including nearby property owners.
EXAMPLE: A group home for the developmentally disabled de-
sires to operate in a residential area. A conditional use permit is
granted on the conditions that the group home be fully staffed at
all hours and that the staff not park their cars on the street.
EXAMPLE: A lawyer wants to run his practice out of his home in
a residential area. A conditional use permit is granted on the con-
ditions that he not have more than two clients with cars visiting at
the same time, and that he post no signage.
f. Spot Zoning
(1) A parcel or small area may be zoned for a use or structure that
is inconsistent with the rationale of the overall plan or ordinance.
This is called spot zoning.
(2) Spot zoning is illegal when the zoning ordinance is designed
solely to serve the private interests of one or more landowners. It
is permissible if the purpose is to further the welfare of the entire
political subdivision.
g. Exactions
(1) An exaction is an approval of use in exchange for money or a
dedication of land. An exaction is permissible only if the local
government can demonstrate that the increased public need
is causally related (i.e., has an essential nexus) to the owner’s
use, and the amount of the exaction is approximately equal to
the additional public cost imposed by the use (i.e., has a rough
proportionality to the use).

418
REAL PROPERTY

EXAMPLE: The owner of a plumbing and electrical supply store


applied for a permit to expand her store and pave the parking lot.
The city was not allowed to insist that the owner also dedicate
land to a greenway and develop a pedestrian and bicycle path-
way by the store [Dolan v. City of Tigard, 512 U.S. 374 (1994)].

E. Support Rights
1. An owner of real property has the exclusive right to use and possess the
surface, airspace, and soil of the property.
2. Lateral Support
a. An owner of land has no liability if a subsiding of neighboring land is caused
by natural conditions on the owner’s land. However, a landowner may
be strictly liable if his excavation causes adjacent land to subside (sink).
If the adjacent land is improved, strict liability applies only if the adjacent
land would have collapsed in its natural state. Even if the adjacent land
would not have collapsed in its natural state, the landowner is only liable
for damages resulting from negligence. The courts are split on whether
money damages may be recovered for injury to the improvements.
3. Subjacent Support
a. The right of support extends to land in its natural state and buildings
existing on the date when the subjacent estate is severed from the
surface. However, the underground landowner is liable for damages
to subsequently erected buildings only if he is negligent.
b. An underground occupant (e.g. mining company) is liable for negli-
gently damaging springs and wells, but an adjoining landowner is not
liable for interfering with underground percolating water.

F. Water Rights
1. Riparian rights are rights in water enjoyed by an owner of land that abuts
a navigable natural river, stream, or lake. Several different sets of rules
have been developed concerning the use of riparian water by adjoining
landowners.
a. Traditionally, a riparian owner was entitled to make only such use of
riparian water as would not interrupt the natural flow of the body of water.
Under the natural flow doctrine, a landowner may make unlimited use of
riparian water for domestic or natural uses such as drinking and bathing.
However, a landowner may use riparian water for artificial uses, such as
irrigation, only if the use does not substantially diminish the flow of the
body of water. Today, very few jurisdictions follow the natural flow doctrine.
b. Under the reasonable use doctrine, a riparian owner may make reason-
able use of riparian water so long as his use does not interfere unreasonably
with the rights of other riparian owners. Such reasonable use is restricted
to riparian land, which is defined as land lying within the watershed. Most
American jurisdictions currently follow the reasonable use doctrine.

419
OUTLINE

c. The prior appropriation doctrine awards the right to use water to the first
person to take the water for beneficial purposes. The prior appropriation
doctrine was developed and is still followed in western states where water
resources are scarce. Prior appropriation rules are usually set by state statute.
2. Diffuse surface water is water from rain or melted snow that runs over the
surface of land outside of a recognizable body of water. Several different sets
of rules have been developed concerning the use of diffuse surface water.
a. Under the common enemy rule, an owner may use any method
available to keep diffuse surface water from coming onto his land.
Traditionally, the common enemy rule was followed in crowded urban
areas, primarily in northeastern states.
b. Under the civil law rule, an owner was not permitted to interfere with
the flow of diffuse surface water. Traditionally, the civil law rule was
followed in western states with vast open spaces.
c. Under the reasonable use rule, an owner may use reasonable means
to alter the flow of diffuse surface water, even if surrounding land-
owners are harmed.
3. Underground water is water that runs beneath the surface of the land.
a. The use of underground streams—where the water flows in a well-
defined, known, and permanent channel—is governed by the rules
applied to riparian waters.
b. Percolating underground water is water that percolates through
the subsurface of the land. The rules governing the use of percolating
underground water vary from state to state.
(1) Under the English absolute ownership doctrine, a possessor of
land may take as much of the water percolating under his land as
he desires. This is known as the rule of capture.
(2) Some states apply the reasonable use test to percolating under-
ground water.
(3) A few states use a correlative rights test, which provides that all
owners of land situated over a pool of underground water have
equal rights to use the water.
(4) In some states, prior appropriation statutes have been enacted to
govern the use of percolating underground water.

420
REAL PROPERTY

III. CONTRACTS

A. Creation and Construction


1. Statute of Frauds
a. The Statute of Frauds requires a writing signed by the party to be
charged for a transfer of an interest in real property.
b. Exceptions to the Statute of Frauds
(1) The doctrine of part performance may be used to enforce an
otherwise invalid oral contract of sale, provided the acts of part
performance unequivocally prove the existence of the contract.
To satisfy this doctrine, a showing of at least two of the following
three facts must be made:
(a) payment of all or part of the purchase price;
(b) taking of possession; and
(c) making substantial improvements.
(2) Equitable and, under the modern trend, promissory estoppel
may also be used to prove an oral contract for the sale of land.
(a) Equitable estoppel is based on an act or a representation.
(b) Promissory estoppel is based on a promise.
2. Essential Terms
a. The signed writing must include the following essential terms:
(1) description of the property;
(2) description of the parties;
(3) price; and
(4) any conditions of price or payment if agreed on.
3. Time for Performance
a. As with other contracts, if no time is stated, then performance is to
occur within a reasonable time.
4. Remedies for Breach
a. Seller’s Remedies for Buyer’s Breach
(1) Remedies at Law
(a) Traditionally, if the buyer breaches a contract for the sale of
real property, the seller is entitled to expectation damages,
measured by the difference between the contract price and
the market price at the time of the breach. To compensate
the seller in a falling market, some courts now measure
expectation damages based on the difference between the
contract price and the resale price.
(b) The seller may also recover foreseeable consequential
damages, such as mortgage interest payments that the
seller is required to make after the buyer’s breach.

421
OUTLINE

(c) The seller may recover reasonable reliance damages,


such as repairs and the cost of inspections.
(d) Traditionally, if the buyer breaches, the seller may elect to
retain the entire amount of the down payment, even in the
absence of a liquidated damages provision.
1) Most courts now restrict the traditional rule to
a down payment that equals 10% or less of the
purchase price.
2) The modern approach requires the seller to return to the
buyer the amount by which the down payment exceeds
the damages caused by the buyer’s breach.
(e) A contract for the sale of real property may include a
liquidated damages clause, which states that the seller
may retain the entire amount of the down payment if the
buyer breaches. If the contract does contain a liquidated
damages clause, most courts permit the seller to retain the
down payment if the clause is found to be reasonable. A
liquidated damage clause is reasonable, and therefore is
enforceable, if:
1) the injury caused by the breach is one that is difficult or
incapable of accurate estimation (traditionally judged at
the time the contract was made); and
2) the liquidated damages are a reasonable forecast of
the harm caused by the breach.

NOTE Traditionally, the reasonableness of the forecast was judged at the


time of the contract. Under the Second Restatement of Contracts, the
reasonableness of the forecast may be judged at the time the contract was
made (anticipated harm) or by the loss actually caused (actual harm).

(f) The seller may recover punitive damages if the buyer’s


breach was willful.
(2) Remedies in Equity
(a) If the buyer breaches, the seller may elect to seek
rescission of the contract.
(b) Traditionally, mutuality of remedy was required. Under this
rule, because the buyer is entitled to specific performance,
the seller is also entitled to specific performance. However,
as mutuality of remedy has fallen out of favor, some courts
have begun to question the general availability of specific
performance for sellers.
1) Some courts have now declined to classify a condo-
minium unit as unique realty. Those courts refuse to
grant a seller’s request for specific performance.

422
REAL PROPERTY

b. Buyer’s Remedies for Seller’s Breach of Contract


(1) Remedies at Law
(a) Traditionally, if the seller breaches a contract for the sale
of real property, the buyer may recover expectation
damages, measured by the difference between the market
price at the time of the breach and the contract price. To
compensate the buyer in a rising market, some courts now
measure expectation damages based on the difference
between the resale price and the contract price. The buyer
may also recover foreseeable consequential damages,
such as lost profits.
(b) The buyer may recover reasonable reliance damages,
such as the cost of inspections.
(c) If the seller breaches a contract for the sale of real property,
the buyer may seek restitution of the down payment.
1) Under the English rule, if the seller fails to deliver
marketable title, the buyer may seek restitution of his
down payment, plus interest and reasonable expenses
incurred in investigating title. However, under the
English rule, the buyer may not receive expectation
damages that exceed the down payment amount
unless the seller acted in bad faith or assumed the risk
of failing to obtain marketable title. A slight minority of
jurisdictions follow the English rule.
2) Under the American rule, if the seller fails to deliver
marketable title, the buyer is not restricted to restitution
of his down payment. Rather, the buyer may recover
expectation damages plus reasonably foreseeable
consequential damages. A majority of jurisdictions
follow the American rule.
(d) The buyer may recover punitive damages if the seller’s
breach is willful.
(2) Remedies in Equity
(a) If the seller breaches, the buyer may elect to seek rescis-
sion of the contract for sale, accompanied by restitution of
the down payment.
(b) Traditionally, each piece of land is considered unique.
Therefore, if the seller breaches the contract by failing
to sell the property to the buyer, the buyer may seek
specific performance of the contract for sale. If the
seller fails to deliver marketable title, the buyer may elect
to sue for specific performance, with an abatement in the
purchase price that reflects the decrease in value caused
by the title defect.

423
OUTLINE

B. Marketability of Title
1. All contracts for the sale of real property include an implied promise to
convey marketable title. Marketable title is title that is reasonably free
from doubt in both fact and law. Title is not reasonably free from doubt if it
contains any of the following defects:
a. defects in the chain of title, such as:
(1) adverse possession;
(a) Traditionally, title acquired by adverse possession
is unmarketable.
(b) However, recent cases (though still the minority view)
suggest that title acquired by adverse possession may be
marketable if:
1) the possession has been for a very long time;
2) the risk that the record owner will sue is remote; and
3) the probability of the record owner’s success is minimal.
(2) the defective execution of a deed; or
(3) significant variation of the description of land from one deed to
the next;
b. encumbrances;
(1) For the purposes of marketable title, an encumbrance is a
right or interest that another person has in real property that
diminishes the value of the property but is consistent with the
conveyance of a fee interest in the property.
(2) In this context, encumbrances include:
(a) mortgages;
(b) liens;
(c) easements; and
1) An easement that reduces the value of the property
(e.g., the burden of a right-of-way) renders title unmar-
ketable. However, an easement that benefits the
burdened estate, such as the installation of utilities,
and which is visible or known to the buyer, does not
render title unmarketable.
(d) covenants and servitudes.
(3) An encumbrance excepted in the contract may not serve as the
basis for a finding that title is unmarketable.
(a) A seller may satisfy a mortgage or lien at closing with the
proceeds of the sale. If the purchase price is sufficient and
satisfaction occurs simultaneously with the transfer of the
land, the buyer may not complain because closing will
result in marketable title.
c. encroachments; or
(1) A significant encroachment renders title unmarketable.

424
REAL PROPERTY

(2) In contrast, a slight encumbrance, such as a boundary overlap


of several inches only, will not render title unmarketable.
(3) An encroachment does not render title unmarketable if the owner
says he will not rely on the encroachment as a basis for suit.
d. zoning restrictions.
(1) The existence of a zoning restriction does not make title unmarketable.
(2) However, a zoning violation may render title unmarketable.
2. Physical defects in the property, such as termites, do not render title
unmarketable.

NOTE If title is unmarketable, the purchaser may get: (1) rescission; (2) money
damages for breach of contract; or (3) specific performance with an
abatement of the purchase price.

C. Equitable Conversion and Risk of Loss


1. A purchaser becomes an equitable owner of title at the time of the
execution of a binding contract.
2. Under the common law, the risk of loss is on the buyer on execution of a binding
contract for the sale of real property. The common law rule is the majority rule.
EXAMPLE: A house burns down after the contract is signed but before the
deed is signed; the purchaser must pay the purchase price.
3. Under the Uniform Vendor and Purchaser Risk Act, the risk of loss is placed on
the seller unless the legal title or possession of the property has passed to the
buyer. The rule of the Uniform Vendor and Purchaser Risk Act is the minority rule.

D. Options and Rights of First Refusal


1. Options and rights of first refusal are primarily contracts concepts, and
governed as such. They will be upheld if their terms are deemed reasonable.

E. Fitness and Suitability


1. Duty to Disclose Defects
a. A seller of a residential home has a duty to disclose to the buyer
material latent defects known to the seller but not readily observable
and not known to the buyer.
(1) Generally, the duty applies only to commercial builders and
developers of new residential homes.
(a) Some states extend the duty to all sellers.
(b) Real estate agents and brokers may also be included in
some jurisdictions.
(2) The duty to disclose defects applies to new homes. However,
some states extend the duty to used homes.
(a) In no state is the duty extended to commercial property.

425
OUTLINE

b. Material is often limited to defects that affect the health and safety
of the occupants. However, some states define material to include
defects that affect value as well as health and safety.
(1) In some states, materiality is judged by an objective standard,
which asks whether a reasonable person would attach impor-
tance to the defect in determining whether to purchase the home.
(2) In other states, materiality is judged by a subjective standard,
which asks whether the defect actually affected the value or
desirability of the property to the buyer.
c. At a minimum, defect means a physical defect on the premises.
(1) A few jurisdictions extend the duty to physical defects both on
and off the premises (e.g., nearby environmental hazards).
(2) Some states also extend the duty to physical and nonphysical
defects (e.g., noise from neighbors).
2. Implied Warranty of Quality
a. Most jurisdictions recognize an implied warranty of quality,
sometimes called an implied warranty of workmanlike quality, an
implied warranty of habitability, an implied warranty of fitness, or an
implied warranty of suitability.
b. The implied warranty of quality generally applies to the sale of new
or remodeled homes.
(1) The implied warranty of quality does not extend to
commercial structures.
(2) The implied warranty is imposed on contractors, developers,
and other commercial vendors of real property. The warranty
covers significant latent defects caused by the defendant’s
poor workmanship. The defects must be discovered within a
“reasonable time” of construction or remodeling.
(a) The jurisdictions are split on whether the implied warranty
of quality extends to subsequent purchasers, i.e.,
purchasers who are not in privity of contract with the
builder or remodeler. The jurisdictions are also split on
whether economic loss may be recovered for breach of
the implied warranty of quality.
c. Most jurisdictions permit the enforcement of an unambiguous disclaimer
of the implied warranty of quality. However, some jurisdictions do not
give effect to a general disclaimer (e.g., “as is”) for residential premises.
d. In some jurisdictions, the statute of limitations begins to run when
construction is completed. In other jurisdictions, the statute of
limitations begins to run when the buyer to whom the warranty was
first made takes possession of the premises, even if the buyer does
not know of the breach. In other jurisdictions, the statute of limitations
does not begin to run until the purchaser discovers or should have
discovered the breach.

426
REAL PROPERTY

TYPE OF PARTY
THEORY DEFINITION TYPE OF DEFECT
STRUCTURE BOUND

Title must be Residential and Any seller. Defects in title,


reasonably free commercial including chain-of-title
Marketable
and clear from property. defects, encumbrances,
Title
doubt at closing. encroachments, and
zoning violations.

Seller of New homes. Commercial Physical defects affecting


residential home builders and health and safety.
New and used
must disclose to developers. Physical defects affecting
Duty to homes.
buyer material All sellers. health, safety, and value.
Disclose
latent defects
known to seller All brokers. Physical and
but not to buyer. nonphysical defects.

Homes must be New and Contractors, Significant latent


Implied
built and remodeled remodeled developers, and defects caused by
Warranty
with workmanlike homes. other commercial the defendant’s poor
of Quality
quality. vendors. workmanship.

•Denotes jurisdictional variations

F. Merger
1. Traditionally, covenants in a contract of sale merge into the deed at
closing. However, under the modern trend, merger does not apply to
matters that are collateral to or not mentioned in the deed.
2. This means that, if the buyer discovers a problem with the title acquired
from the seller after closing, the buyer cannot sue the seller for breach
of the implied covenant of marketable title, because that covenant was
implied into the contract, which has now merged with the deed.
3. Instead, the buyer must now sue on the basis of a breach of any covenant
of title contained in the deed.
a. If the deed does not contain any covenants of title (i.e., a quitclaim
deed), then the buyer has no course of action.

427
OUTLINE

IV. MORTGAGES/SECURITY DEVICES

A. Types of Security Devices


1. Mortgages
a. In General
(1) A mortgage is a conveyance of an interest in real property
made to secure performance of an obligation. The obligation
often arises out of a loan of money made to facilitate the
purchase or development of real property. A mortgage is
typically evidenced by two documents:
(a) a mortgage deed; and
(b) a promissory note (mortgage note).
(2) A mortgage deed is a document that conveys an interest
in real property designed to secure performance of a debt.
Because a mortgage involves a transfer of an interest in realty,
a mortgage must be evidenced by a writing (e.g., a mortgage
deed) that is properly executed and delivered to the mort-
gagee. The mortgage deed must include, at a minimum, the
following elements:
(a) the identity of the parties;
1) The mortgagor is the owner of the real property, who
borrowed money and secured the debt with a mortgage
on the property.
2) The mortgagee is the lender of the money borrowed,
to whom the mortgage is made.
(b) a description of the property; and
1) The property should be described with sufficient detail
to put a subsequent bona fide purchaser on notice of
the mortgage (which may require more detail than is
required to satisfy the Statute of Frauds).
(c) the intent to create a security interest.
1) The mortgage deed should contain evidence of an
intent to create a security interest in the mortgagee.
2) The mortgage deed is a recordable document, which
is generally subject to the terms of the applicable
recording statute.
(3) The mortgage note represents the mortgage obligation. The
note is an “IOU” that creates personal liability in the mortgagor.
The note typically includes, inter alia, the following provisions:
(a) the loan amount;
(b) the interest rate, which may be fixed (set for the term of the
loan) or adjustable (it may vary over the term of the loan);
(c) the loan term (e.g., 15 or 30 years);

428
REAL PROPERTY

(d) a clause permitting prepayment but exacting a penalty for


the privilege of prepayment;
(e) a clause that permits the mortgagee to declare the entire
amount of the mortgage obligation due and payable if the
mortgagor defaults (an acceleration clause); and
(f) a “due on sale” clause, which requires the entire balance
due on the note to be paid before the property may be
transferred by the mortgagor/seller to a buyer.

NOTE When the mortgagor satisfies (pays) the mortgage note, the mortgagee executes a
document that releases the mortgage. The release document should be recorded.

b. Purchase-Money Mortgages
(1) A purchase-money mortgage (PMM) is a mortgage given to
secure a loan that enables the mortgagor to acquire title to the
property at issue, or to make improvements on the property.
(2) A purchase-money mortgage, whether recorded or unrecorded,
is entitled to priority over other liens on the property arising
through the actions of the buyer-mortgagor, even those recorded
earlier than the purchase-money mortgage, but only if they were
executed prior to the acquisition of title.
(a) The rationale is that the purchase-money mortgage enabled
the mortgagor to acquire title to the property (remember that
the purchase-money mortgage must be given as part of the
same transaction in which title is acquired). Therefore, the
purchase-money mortgage should have the superior right.

NOTE A mortgage given the day after the buyer acquired the property (through the
use of a PMM), if recorded before the PMM is recorded, would have priority
over the PMM based on recording statute priority.

(3) Where the instruments are silent, a purchase-money mortgage


given to the vendor of the property will have priority over one
given to a third-party lender.
EXAMPLE: Buyer acquires Blackacre by giving a purchase-
money mortgage to Seller and one to Bank, and defaults on both;
Seller has priority over Bank.
c. Future-Advance Mortgages
(1) Future-advance mortgages include line-of-credit or home-equity loans.
(2) The most common issue involving future-advance mortgages is
the fact that the arrangement is being executed at the present
time, but the funds are not being accessed until a date in the
future. The question then involves determining at what point the
mortgage attached to the property.
(a) If proper notice is given to future creditors, the mortgage
interest attaches on the date that the future-advance

429
OUTLINE

mortgage arrangement is made, not on the date that the


funds are actually accessed.
d. Mortgage Alternatives
(1) In a deed of trust, the debtor/note-maker is the settlor, who gives a
deed of trust to a trustee who is closely connected to the lender. In
the event of a default, the trustee is directed to proceed with a fore-
closure sale. Deeds of trust are generally treated like mortgages.
2. Installment Land-Sale Contracts
a. In an installment land-sale contract, the buyer takes possession under
a contract of sale and makes payments to the seller. The seller delivers
a deed and legal title only when the payments have been completed.
b. If the buyer defaults, an installment land-sale contract usually provides
for forfeiture of all installments paid, allowing the lender to retake the
property. However, some states require a foreclosure proceeding for an
installment land-sale contract.
(1) In some states, if a foreclosure occurs, the seller is required to
refund to the buyer all installments already paid, as long as these
payments are more than the damages suffered by the seller.
c. In many states, a buyer who defaults under an installment land-sale
contract is granted a grace period to pay off the loan. The buyer may
keep the land while he is paying under a new payment schedule.
(1) If a seller under an installment land-sale contract has accepted
late payments from the buyer, the seller may be deemed to have
waived his right to demand timely payment.
(2) If this is the case, when the seller wishes the buyer to begin
paying in a timely manner, he must send the buyer written
notice and allow the buyer a reasonable amount of time to make
back payments owed.
3. Absolute Deeds as Security
a. This situation arises when a debtor borrows money and issues a deed
to the creditor that appears to be absolute on its face. However, extrinsic
evidence shows an agreement between the debtor and creditor that, when
the debt is repaid, the creditor will reconvey the property to the debtor.
b. If that extrinsic agreement can be proven to the satisfaction of the
court, the arrangement will not be treated as an absolute deed of the
property, but rather, as a mortgage interest.
(1) The deed then becomes what is known as a mortgage deed.

B. Some Security Relationships


1. Theories
a. The jurisdictions are split on the nature of the interests held by
mortgagor and mortgagee after the execution of the mortgage
deed. The following three mortgage theories are in effect in various
jurisdictions of the United States.

430
REAL PROPERTY

b. Title Theory
(1) The title theory is the classic common law model for
determining the nature of the interests held by the mortgagor
and the mortgagee.
(2) Under the traditional title theory, the mortgagee receives legal
title to the mortgaged real property and has a right to take
possession of and to collect rents and profits from the property.
(a) The mortgagee’s title to the property is subject to a
condition subsequent that divests title from the mortgagee
if the mortgagor repays the loan by the due date.
(b) Until he repays the loan in a timely fashion, the mortgagor
retains only an equitable interest in the property.
(3) The traditional title theory is now the minority view.
(a) In addition, a title theory state is likely to recognize that the
mortgagee holds title for security purposes only. Under this
approach, the mortgagor is viewed as the owner of the land.
(b) Some title theory states have also eliminated or reduced the
incidents of legal title by, inter alia, giving the mortgagor the
right to possession until default.
c. Lien Theory
(1) In a lien theory jurisdiction, the mortgagee receives a lien,
and the mortgagor retains legal and equitable title and
possession to the mortgaged real property, unless and until
foreclosure occurs.
(2) Most states now adhere to the lien theory.
d. Intermediate Theory
(1) In an intermediate theory jurisdiction, the mortgagor retains
legal title until default occurs.
(2) After default, legal title and possession pass to the mortgagee,
who may then begin to collect rents and profits.
(3) Only a few states adhere to the intermediate theory. According
to the Restatement of Property, the intermediate theory differs
very little from the title theory, because title theory mortgagees
rarely assert a right to possession prior to default.
2. Rights and Duties prior to Foreclosure
a. Mortgage-Related Waste
(1) A person holding a remainder in mortgaged property has a duty
to pay the principal. A life tenant has a duty to pay the interest
on a mortgage. However, the life tenant’s duty is capped at:
(a) rents and profits derived from a third person in possession
of the property; or
(b) the reasonable rental value of the premises, if the life
tenant remains in possession of the property.

431
OUTLINE

(2) The mortgagor or a life tenant of mortgaged property also


commits waste if, without the mortgagee’s consent, the mort-
gagor or life tenant:
(a) fails to make timely payments of property taxes or
governmental assessments secured by a lien that have
priority over the mortgage;
(b) makes physical changes to the real property, negligently or
intentionally, that reduce the value of the property;
(c) fails to maintain and repair the property in a reasonable
manner, except for repair of casualty damage or acts of
third parties not the fault of the mortgagor;
(d) fails to comply materially with mortgage covenants
respecting the physical care, maintenance, construction,
demolition, or insurance against casualty of the property or
improvements on it; or
(e) retains rents to which the mortgagee has a right of
possession.

MORTGAGE-RELATED WASTE

My Rubbish Makes The Castle Ruined

Mortgage, Reduce Value, Maintain, Taxes, Covenants, Rents

(3) The mortgagee has the following remedies if the mortgagor or


life tenant commits waste:
(a) foreclosure or the exercise of other remedies available under
the mortgage for default on the secured obligation, if the
waste has impaired the mortgagee’s security;
(b) an injunction prohibiting future waste or requiring correc-
tion of waste already committed, but only to the extent that
the waste has impaired or threatens to impair the mortgag-
ee’s security; and
(c) recovery of damages, limited by the amount of the waste, to the
extent that the waste has impaired the mortgagee’s security.
(4) Absent a leasehold provision to the contrary, a tenant for
years or a periodic tenant has no common law duty to make
mortgage payments.
3. Right to Redeem and Clogging the Equity of Redemption
a. The mortgagor’s interest in the mortgaged property is called the
“equity,” which is short for “equity of redemption.”
b. At any time after default but before foreclosure, the mortgagor has the
right to redeem the property by paying the debt due.
c. The right to redeem may not be waived (“clogged”) at the time the
mortgage is created.

432
REAL PROPERTY

C. Transfers by Mortgagor
1. Assumption and Transfer Subject to the Mortgage
a. If the mortgagor transfers the property subject to the mortgage, and
mortgage payments are not made, the mortgagee may foreclose and
force the property to be sold. However, the transferee of the property
does not have personal liability for the debt.
b. If the transferee of mortgaged real property assumes the mortgage,
and mortgage payments are not made, the mortgagee may foreclose
and force the property to be sold.
(1) In addition, the transferee of the mortgage has personal
liability and can be held liable for any deficiency.
c. In a novation, the transferee of real property and the mortgagee
agree that the transferee will assume the mortgage and the
mortgagor will be released from liability.
2. Rights and Obligations of Transferor
a. If, after there has been an assumption, the debt falls into default, the
grantor can get an exoneration, which is a court order compelling the
grantee to pay the debt.
b. If, following the assumption, the grantor has made any payments on
the mortgage, the grantor can sue the grantee for reimbursement.
3. Application of Subrogation and Suretyship Principles
a. The grantor can pay off the debt and can then be subrogated to the mortgage
and note, allowing the grantor to sue the grantee in personam or in rem.
4. Restrictions on Transfer (including Due-on-Sale Clauses)
a. Due-on-sale clauses and due-on-encumbrance clauses are now
routinely upheld. Federal law preempts state laws restricting the
enforcement of such clauses.

D. Transfers by Mortgagee
1. The mortgagee may also transfer his interest in the mortgaged property.
2. The mortgage generally follows the transfer of the mortgage note. If the
note is negotiable, the transferee may qualify as a holder in due course,
who takes free and clear of certain “personal” defenses, such as lack of
consideration, duress by nonphysical threat, and fraud in the inducement.
a. However, the transferee will take subject to “real” defenses, such as
infancy, duress by physical threat, and fraud in the factum.

E. Discharge
1. Payment (including Prepayment)
a. There is no absolute right to prepay a mortgage debt early. Such an
option must be spelled out in the mortgage.
b. If the mortgage does allow for early payment, it will generally
always include a prepayment fee. Such a fee is valid, because it
compensates the creditor for loss of interest income.

433
OUTLINE

2. Deed in lieu of Foreclosure


a. A mortgagee can accept a deed to the property in lieu of foreclosure.
However, the mortgagee takes the deed with all mortgages still
attached to the land.
b. In other words, the mortgagee steps into the shoes of the mortgagor
in this situation.

F. Foreclosure
1. Types
a. In most jurisdictions, when there is a default on an obligation
secured by a mortgage, the mortgagee may:
(1) obtain a judgment against any person who is personally liable on
the obligation and, to the extent that the judgment is not satisfied,
foreclose the mortgage on the real estate for the balance; or
(2) foreclose the mortgage and, to the extent that the proceeds
of the foreclosure sale do not satisfy the obligation, obtain
a judgment for the deficiency against any person who is
personally liable on the obligation.
b. Today, a mortgagee may foreclose by forcing the sale of property secured
by a mortgage after the mortgagor has defaulted on the promissory note.
c. A foreclosure sale may be accomplished either through a power of
sale or by court order.
(1) A power-of-sale foreclosure occurs without judicial action, pursuant
to a power-of-sale clause included in the mortgage documents.
(2) A judicial foreclosure sale must:
(a) be public;
(b) be properly noticed;
(c) be conducted in a reasonable manner (usually regulated
by statute); and
(d) result in a “fair” sale price.
1) A fair price is not necessarily the fair market price.
Rather, it is arrived at as a result of the mortgagee’s
due diligence in conducting the foreclosure sale.
2) A foreclosure sale will not be set aside for inadequacy
of the sale price unless the inadequacy is so gross as
to shock the conscience.
d. A foreclosure sale terminates the mortgagor’s interest in the
mortgaged real estate.
2. Acceleration
a. An acceleration clause operates to make the entire debt become
due on the happening of some specified event, such as a default,
encumbrance, or sale.
b. Such clauses are generally upheld.

434
REAL PROPERTY

3. Parties to the Proceeding


a. Junior interests (i.e., second or later mortgages) in the property are
destroyed by a foreclosure sale.
(1) However, a junior mortgage is not extinguished if the junior mortgagee
is not made a defendant in the judicial proceeding culminating in a
foreclosure sale and does not receive notice of the foreclosure sale.
(2) Senior interests not participating in the foreclosure are not
affected by a foreclosure sale.
b. Modification
(1) If a senior mortgage is modified, a junior mortgage prevails over
the modification to the extent that the modification materially
prejudices the holder of the junior mortgage, such as by:
(a) increasing the amount of principal; or
(b) increasing the interest rate (if the rate under the original
mortgage was fixed).
EXAMPLE: First Bank has a senior mortgage on Black-
acre for $100,000. Second Bank has a junior mortgage for
$20,000. First Bank and the homeowner negotiate an in-
crease in the interest rate of the mortgage with First Bank,
which will result in First Bank receiving an extra $5,000.
This is a modification that materially prejudices Second
Bank. If the homeowner defaults and First Bank forecloses
on the property, joining Second Bank, First Bank will receive
the first $100,000 of the foreclosure sale price. Second Bank
will receive the next $20,000. Then, after Second Bank is
paid, First Bank would receive the additional $5,000.
(2) Modifications that normally do not materially prejudice the
holder of the junior mortgage include:
(a) extension of the mortgage maturity date; and
(b) rescheduling installment payments.
4. Deficiency and Surplus
a. The proceeds of a mortgage foreclosure sale will be distributed in
the following order:
(1) to the costs of the sale;
(2) to the security interest foreclosed;
(3) to junior lienholders terminated by the sale; and then
(4) to the mortgagor, if any proceeds remain.
b. If the proceeds of a foreclosure sale are not sufficient to cover the lien(s)
foreclosed, the mortgagee/creditor(s) may obtain a deficiency judgment
against the mortgagor (or a party that has assumed the mortgage) person-
ally, based on the note. Some states limit deficiency judgments by:
(1) prohibiting deficiency judgments when foreclosure is accom-
plished privately through a power-of-sale clause;

435
OUTLINE

(2) requiring deficiency judgments to be sought at the same time


as foreclosure;
(3) scrutinizing the fairness of the foreclosure sale, particularly if
the foreclosure is accomplished by a private sale; or
(4) requiring the mortgagee to set a minimum foreclosure sale
price (“upset price”).
5. Redemption after Foreclosure
a. About one-half of the states have a fixed time period (6–12 months)
after a foreclosure sale has occurred for the mortgagor to redeem
(by matching the foreclosure price).
b. Because this is a right created by statute, the ability of the parties to
waive (clog) this right would depend on the wording of the statute in
that jurisdiction.
6. Marshaling Assets
a. Marshaling is an equitable doctrine intended to prevent a senior creditor,
who has access to more than one source of funds from the debtor, from
prejudicing a junior creditor, who can seek recourse from only one source
of funds from the debtor, by foreclosing on the common source of funds
first. It compels a creditor with more collateral options at its disposal to
exhaust those other assets first, so as not to defeat another creditor.
b. The doctrine of marshaling is often raised by a junior lien creditor as an affir-
mative defense to a foreclosure action undertaken by a senior lienholder.
c. There are two common techniques to marshal assets: the two funds
doctrine and the inverse order of alienation doctrine.
d. Two Funds Doctrine
(1) Marshaling may be applied for the benefit of a junior lien creditor
when [In re Southeastern Materials, Inc., 452 B.R. 170 (Bankr.
M.D. N.C. 2011)]:
(a) the junior lien creditor and the senior lien creditor are dealing
with a common debtor who owns two or more properties;
(b) the senior lien creditor has a lien on two or more of the
debtor’s properties; and
(c) the junior lien creditor has a junior lien on fewer of the prop-
erties than the senior lien creditor.
EXAMPLE: Homeowner takes out a purchase-money mort-
gage with Bank 1 in order to finance his acquisition of Blackacre
and Whiteacre, two separate parcels of land located in a city.
Later, wanting to renovate Blackacre into a bed and breakfast,
Homeowner takes out a mortgage with Bank 2, using Blackacre
as collateral. Several years later, tourism to the city plummets,
and Homeowner defaults on his loan to Bank 1. Bank 1 initi-
ates a foreclosure action against Blackacre, joining Bank 2 as
a junior lienholder. The value of Blackacre at this point is well
below the outstanding amount of Bank 1’s mortgage. Bank 2

436
REAL PROPERTY

can raise the doctrine of marshaling as an affirmative defense


to this action and ask the court to compel Bank 1 to foreclose
first on Whiteacre so as to avoid prejudicing Bank 2, whose
only source of recovering from Homeowner is Blackacre.
e. Inverse Order of Alienation
(1) The inverse order of alienation doctrine provides that, where land
subject to a lien has been divided into separate parcels and sold
to different grantees successively and without a release from the
mortgagee, a mortgagee must first satisfy its lien out of the land
remaining in the grantor or original owner’s possession if possible
[Savings Bank v. Creswell, 100 U.S. 630 (1879)].
(2) If that land is insufficient to satisfy the debt (or the grantor did not
retain any property), the mortgagee must resort to the separate
parcels in the inverse order of their alienation (in other words, the
most recently conveyed parcel first, then working backwards to
the first parcel conveyed). The equity resides in an earlier (or the
earliest) grantee of a parcel of property.
EXAMPLE: Landowner mortgages Blackacre to Bank for
$500,000, which was duly recorded. Several years later, Landown-
er breaks Blackacre up into 10 equal lots of land. He keeps Lots 1
and 2 for himself, and then conveys Lots 3 through 10 to various
friends and buyers, in numerical order, all of whom take subject
to the mortgage. A couple of years after that, Landowner defaults
on the mortgage to Bank. If the court applies the inverse order of
alienation doctrine, Bank would need to foreclose first on Lots 1
and 2, and then on Lot 10, then Lot 9, and so forth, in descending
order of alienation, until the mortgage obligation is satisfied.
(a) The doctrine does not apply where it appears from the
conveyance or junior encumbrance that the subsequently
conveyed parcel was intended to be primarily liable for the
payment of the debt.
(3) Where a grantee assumes the mortgage, an opposite, “direct
order” of alienation applies. In this situation, as between the mort-
gagor and the grantee, the mortgagor is the holder of the equity,
and the grantee should pay the mortgage and his land be sold
first to satisfy the debt, before the mortgagor.

437
OUTLINE

V. TITLES

A. Adverse Possession
1. Adverse possession requires proof of possession that is all of the following:
a. open, visible, and notorious;
b. actual;
(1) An adverse possessor is required to possess the premises for
the requisite time period. Actual possession gives notice to the
world at large and the property owner in particular that someone
is possessing the premises.
(2) A person actually possesses property when he is on the
premises physically.
(3) A person may also possess the premises constructively.
Constructive adverse possession requires color of title
(usually a defective deed) and actual possession of at least a
significant part of the premises.
(4) If those elements are satisfied, the possessor will constructively
possess the whole property as described in deed. However,
there are certain exceptions to the constructive possession rule:
(a) if land is divided into two or more distinctive lots,
constructive possession will extend only to the lot that the
possessor actually occupies; and
(b) if a third person is in possession of part of the premises,
constructive possession will not extend to the portion the
third party possesses.

NOTE Many courts require some reasonable relationship between the area actually
possessed and the additional area alleged to be constructively possessed.
c. exclusive;
d. hostile and under a claim of title or right; and
(1) To acquire title by adverse possession, a person must possess
the premises in a “hostile” manner. In this context, a person
possesses hostilely if he holds the premises in a manner that is
inconsistent with the rights of the owner.
(2) Under the majority rule, hostility is judged objectively, which
means that the possessor’s state of mind is irrelevant. The
element is satisfied if the possessor intends to be on the prem-
ises, regardless of whether the possessor knows that the prem-
ises are owned by someone else.
(3) In a minority of jurisdictions, the possessor must act in good faith,
which means that the possessor must believe the land is his.
(4) A few courts require bad faith, which means that the possessor
must show that he knows that the land is not his but intends to
claim it nonetheless.

438
REAL PROPERTY

NOTE Adverse possession questions often involve mistaken boundaries. A typical


example involves a building on one parcel of land that encroaches slightly
on the adjoining parcel, often without the knowledge of the owner of the
adjoining parcel. Two standards have been developed to judge those cases.
Under the objective majority (“Maine”) view for mistaken boundaries,
possession is hostile so long as the possessor intends to claim the land
as his own, even if he is unsure as to the location of the boundary. Under
the subjective minority (“Connecticut”) view for mistaken boundaries, the
possessor must actually know that he has crossed over the boundary.

NOTE Boundary disputes may also be determined by methods other than adverse
possession. An oral agreement to settle a boundary dispute is enforceable
if the parties subsequently accept the line for a long period of time. Long
acquiescence (perhaps for a period shorter than the statute of limitations)
may also be used as evidence of an agreement between the parties to fix the
boundary line. In addition, if one party makes a representation through words
or acts to the other concerning the location of a common boundary, and the
other party changes position in reliance on that representation, the first party
may be estopped from denying the validity of his representation.

e. continuous for the statutory period.


(1) To acquire title by adverse possession, the possessor must show that
he has possessed the property continuously for the statutory period.
In some cases, a possessor may satisfy this element even if he has
not been on the premises every day during the statutory period.
(a) Seasonal use may suffice if that use constitutes the best use
of the property.
EXAMPLE: The annual summer use of an unheated cabin in
the north woods.
(2) The requisite time period for adverse possession is established
by statute. In many states, the common law period of 20 years
has been supplanted by a shorter statutory period, which is
often five, seven, or 10 years.
(3) If an adverse possessor has not been in possession for the full
statutory period, he may still meet the time period by tacking his
possession onto possession by a previous adverse possessor.
(a) Tacking is allowed if the adverse possessor and his
predecessor are in privity of estate.
(b) Privity of estate requires an intentional transfer of
possession from one person to the next.
(c) An adverse possessor may not tack his possession
onto that of a previous person if the adverse possessor
dispossesses the previous possessor.
(d) If the ownership of the property changes hands during the
period of adverse possession, the adverse possession

439
OUTLINE

continues to run uninterrupted against the subsequent owner


if the two owners are in privity of estate.
EXAMPLE: In 2000, A adversely enters onto Blackacre, which is
owned by O. In 2001, O conveys Blackacre to B for life, remain-
der to C. In 2008, B dies without ever having entered Blackacre.
The jurisdiction has a five-year statute of limitations for adverse
possession claims. C now seeks to quiet title against A. A en-
tered against O before O devised to B for life, remainder to
C. Therefore, the statute of limitations continued to run against
both B and C. Under the five-year statute of limitations, the
time period ran out in 2005; as a result, A will prevail over C.
(4) The running of a statute of limitations may be tolled in certain
circumstances.
(a) Tolling means that the statute will not run for a period of time.
(b) For tolling to occur, a disability must be in existence at the
time the adverse possessor enters.
1) Disability typically includes nonage, legal incompe-
tence, and imprisonment. Most states extend the time to
bring an action to a certain period after the disability has
been removed (e.g., 10 years).
EXAMPLE: In 1999, O, the owner of Blackacre, is adjudi-
cated legally incompetent. In 2000, A enters adversely on
land owned by O. In 2003, O dies without regaining legal
competence; however, O leaves a valid will that devises her
entire estate to B, who is under no disability at the time. The
jurisdiction has a statute of limitations for adverse posses-
sion, which provides that, in the case of disability, the statu-
tory period expires seven years after the adverse possessor
enters or 10 years after the owner’s disability is removed,
whichever date is later. B sues to quiet title to Blackacre in
2008. The statute is tolled in this case because O was under
a disability at the time that A entered the premises. O’s dis-
ability was removed in 2003, when O died. The statute gives
B, as the successor owner (who is in privity of estate with O
through O’s will), 10 years after the removal of O’s disability,
which means the statute would run out against B in 2013.
Therefore, B will prevail over A in B’s 2008 quiet title suit.

B. Transfer by Deed
1. Requirements for Deed
a. Conveyance of real property by deed requires:
(1) donative intent;
(a) The grantor must intend to transfer an interest immediately to
the grantee. If the grantor intends the deed to take effect only
on the death of the grantor, will formalities must be observed.

440
REAL PROPERTY

(2) delivery; and


(a) Delivery is usually accomplished through the physical
act of handing the deed over to someone. However, it is
possible to accomplish delivery by mere words (e.g., by
a declaration of intent and relinquishment of control). A
presumption of delivery arises if:
1) the deed is later found in the grantee’s possession;
2) the deed is properly executed and recorded; or
3) the deed contains an attestation clause that attests
to delivery.
(b) Handing the deed to the grantor’s agent does not constitute
valid delivery. Delivery does not occur until the grantor’s agent
delivers the deed to the grantee or grantee’s agent, which
then triggers the relation-back doctrine. However, handing the
deed to the grantee’s agent does constitute valid delivery.
(c) Doctrine of Relation Back
1) Handing the deed to a third party (i.e., an escrow agent)
constitutes valid delivery if the grantor relinquishes all
control over the deed.
2) If a delivery is accomplished through an escrow agent,
delivery will relate back to the date the grantor handed
the deed to the escrow agent.
EXAMPLE: If the grantor dies before the escrow agent
delivers the deed to the grantee, the delivery will relate
back to the date the grantor handed the deed to the agent.
Therefore, the deed was delivered during the lifetime of the
grantor, and will formality and probate issues are avoided.
(d) When the grantor hands the deed to the third party and
places conditions on the delivery, it gives rise to concerns
regarding a situation where the grantor transfers the property
to someone else before the condition occurs.
EXAMPLE: Grantor gives the deed to X, a third party, and says,
“X, give this deed to Grantee if Grantee graduates from law
school.” A month later, Grantor conveys the property to C. At
first glance, it would seem that C would win out. However, once
the condition is met, the conveyance will relate back to the date
Grantor gave X the deed, which was before the transfer to C.
1) The more conditions the grantor places on the convey-
ance, the more likely the conditioned conveyance will fail.
2) When a grantor gives the deed to a third party with
instructions to deliver the deed at the grantor’s death (a
death escrow), the grantor is making a present transfer
of a life estate with a remainder to the grantee.

441
OUTLINE

EXAMPLE: Grantor gives the deed to X, the third party,


and says, “X, give this deed to Grantee when I die.”
(e) If the grantor expressly retains the right to reclaim the
deed from the third party, transfer of title through the
escrow agent will fail.
(3) acceptance.
(a) Acceptance is presumed if the conveyance is beneficial to
the grantee.
b. To satisfy the Statute of Frauds, a deed must be in writing and:
(1) sufficiently identify the parties;
(2) contain words indicating an intent to make a present transfer of
the property;
(3) sufficiently describe the property; and
(4) be signed by the grantor.
c. Parol evidence is always admissible to show that a deed absolute on
its face is intended as a mortgage. Parol evidence may also be used to
show that the grantor did not intend the deed to have present effect.
(1) Most courts permit proof of oral conditions on deeds if delivery
is to a third party.
(2) Under the majority rule, if delivery has been made to the
grantee with an oral condition, the conveyance is valid, but the
oral condition may not be proved. Under the minority rule, if
delivery has been made to the grantee with an oral condition,
the conveyance is void (particularly if the condition is delivery
on the grantor’s death). In a few jurisdictions, if delivery
has been made to the grantee with an oral condition, the
conveyance is valid, and the oral condition may be proved.
d. Natural monuments prevail over all other descriptions. Artificial
monuments prevail over all descriptions except natural monuments.
Courses and angles prevail over distances. All of the descriptions
listed above prevail over a general description (e.g., “Blackacre”).
Natural monuments, artificial monuments, courses, and distances all
refer to a “metes and bounds” description. A description may also be
by government survey, name of parcel, or street address.
2. Types of Deeds
a. The following three types of deeds are generally used in
conveyances of real property.
(1) In a general warranty deed, the seller warrants that no title
defects have occurred during his ownership of the property.
The seller also warrants that there are no defects in the chain
of title from which he derived title.
(2) In a special warranty deed, the seller warrants that no title
defects have occurred during his ownership of the property.

442
REAL PROPERTY

However, the seller does not warrant that there are no defects
in the chain of title from which he derived title.
(3) In a quitclaim deed, the seller does not make any warranties.
Instead, the grantor simply conveys whatever interest he may have.
b. Covenants of Title
(1) A general or special warranty deed contains a series of promises
or covenants, which are divided into present and future covenants.
(2) Present covenants are broken, if at all, at the time of convey-
ance. Under the American rule, present covenants are consid-
ered to be personal and do not run with the land. They include
the following:
(a) in the covenant of seisin, the covenantor promises that he
owns and possesses the estate granted;
1) The existence of an encumbrance does not breach the
covenant of seisin.
(b) in the covenant of right to convey, the grantor promises
that he has the right to convey the property; and
1) The covenant of the right to convey is often
co-extensive with the covenant of seisin. However, there
are several differences between the two covenants:
a) a trustee may be seised of the fee but may not
have the right to convey if the trust deed prohibits
the trustee from conveying; and
b) an owner of a life estate may be seised but may
not have the right to convey if the life estate is
subject to a valid restraint on alienation.
(c) in the covenant against encumbrances, the grantor
promises that there are no encumbrances on the property.
1) For the purposes of the covenant against encum-
brances, an encumbrance is any right in a third person
that diminishes the value or limits the use of the land
granted. In this context, encumbrances include:
a) mortgage and judgment liens;
b) taxes;
c) leases;
d) water rights;
e) easements; and
f) restrictions on use.

NOTE An encumbrance expressly noted in the deed cannot serve as the basis for a
breach of the covenant against encumbrances. In most cases, if a defect renders
title unmarketable (e.g., a mortgage or an easement), the defect also breaches
the covenant against encumbrances. However, in a majority of jurisdictions, a
violation of a zoning ordinance renders title unmarketable but does not breach

443
OUTLINE

the covenant against encumbrances. The jurisdictions are split on whether a vis-
ible or known encumbrance breaches the covenant against encumbrances.
(3) Future covenants may be broken after the time of the conveyance,
and they run with the land. Future covenants include the following:
(a) in the covenant of quiet enjoyment, the grantor covenants
that the grantee will not be disturbed by a superior claim;
(b) in the covenant of warranty, the grantor guarantees that
he will assist in defending title against lawful claims and
will compensate the grantee for losses sustained by an
assertion of superior title; and
1) The covenant of warranty is virtually identical to the
covenant of quiet enjoyment.
(c) in the covenant of further assurances, the grantor
promises to take whatever steps may be required to
perfect defects of title.
1) Under this covenant the grantor may, for example, be
called on to defend a lawsuit or to execute a curative deed.

NOTE A future covenant is not breached until the grantee or his successor is
evicted from the property, buys up the paramount claim to avoid suit, or is
otherwise damaged.
c. Remedies for Breach of Covenant by the Grantor
(1) Monetary recovery is generally capped at the amount the
grantor/defendant received for the property (plus interest).
(a) For breach of the covenants of seisin, right to convey, general
warranty, and quiet enjoyment, the grantee may recover
the full purchase price (if the conveyance is voided) or a
percentage thereof (if a portion of the conveyance is voided).
(b) For breach of the covenant against encumbrances,
damages are measured by the cost of removing the defect,
if removal is possible, or the diminution in the value of the
property if the defect cannot be removed.

NOTE In some jurisdictions, attorney’s fees may also be recovered for breach of
future covenants if the grantee loses the property to a lawful superior claim.

(2) Some states permit a grantee to recover damages from a remote


grantor to the extent of the consideration received by the remote
grantor from his immediate grantee, even if the remote grantee
paid his immediate grantor less for the property. However, in some
states, if the remote grantee paid less than the remote grantor
received from his grantee, the remote grantee’s recovery is limited
to the amount the remote grantee paid to his immediate grantor.
(3) Specific performance is available for the covenant of further
assurances.

444
REAL PROPERTY

d. Statute of Limitations
(1) For present covenants, the statute of limitations begins to run
when the deed is delivered.
(2) For future covenants, the statute of limitations does not begin to
run until a third party asserts a superior title.
3. Drafting, Review, and Negotiation of Closing Documents
a. The contract between the parties to a real estate transaction sets out
the important points of agreement of the sale of the land, including
time of closing, time of possession, transfer of keys, responsibility for
payment of utilities owing and property taxes owing, what items remain
with the property, and in what condition the seller will leave the prop-
erty. These are in addition to the parties, the property description, the
price, and the payment terms, if any, which are essential terms.
EXAMPLE: A buyer contracted to purchase a house, but inserted a
clause that the contract would be rescinded if the buyer could not ob-
tain financing within 30 days.
b. After the contract has been executed by both parties, and the down
payment, if any, has been deposited into escrow, the contract is deliv-
ered to the entity that will handle the closing.
(1) In some jurisdictions, this entity is an attorney; in other jurisdic-
tions, the entity is a title company.
(2) The attorney or title company begins the process of performing the title
search and writing an Abstract of Title, in order to assure that market-
able title will be able to be transferred on the day of the closing. The title
company also prepares the title insurance that the buyer will purchase.
(3) The attorney or title company also reviews which liens have been filed
against the property, and obtains an accurate pay-off sum from each
lender, so that the lender on each lien can be paid in full on the closing
date. Each lender must agree to file a Release of Lien after being paid.
(4) The attorney or title company drafts the deed that will transfer title
from the grantor to the grantee. In many cases, the buyer has
stated in the contract how he would like to take title, whether in
his own name, as joint tenants with someone else, as a tenancy
by the entirety, as the trustee for a trust, or in some other manner.
(a) It is essential that the buyer know the language of the deed
before closing on the property in order to avoid errors in
titling the property. The buyer should clearly inform the
drafter of the deed of the manner in which he intends to take
title. The deed must also be read closely by the buyer on the
closing date, before the deed is recorded.
(b) It is essential that the property address and the address to
which any property tax bills will be mailed are both correct.
The buyer should request to see a draft of the deed before
closing, for these reasons.

445
OUTLINE

EXAMPLE: A buyer contracted to purchase a rental house


that was situated at 123 Berry Street. The buyer’s home ad-
dress was 789 Elm Street. The buyer did not intend to live
at the house, but to rent it out to tenants. Unfortunately, the
deed listed the property tax mailing address as 123 Berry
Street, when the buyer had specifically asked that his home
address be used for this purpose.
(5) In commercial transactions, many additional documents require
drafting, review, and negotiation. It is common for issues to arise after
contract execution but before closing that need a negotiated resolution.
EXAMPLE: A buyer contracts to purchase a shopping center,
contingent on obtaining a building permit to expand it. After the
contract is executed, the buyer finds that there is a leaking un-
derground storage tank on the property, left after a previous gas
station on the property closed. The buyer insists that the seller
perform the expensive environmental removal of the tank and
restoration or paving of the surface afterward, while the seller in-
sists that it is the buyer’s responsibility. Regardless of their nego-
tiated resolution, the closing cannot occur until the environmental
violation is cured. The closing documents are likely to include
waivers of liability granted by one party to the other.
4. Persons Authorized to Execute Documents
a. Ordinarily at a real estate closing involving individuals, the buyer and
the buyer’s spouse, if any, execute all documents requiring the buyer’s
signature. As a practical matter, photo identification is usually requested.
b. In the same way, at a real estate closing, the seller and the seller’s
spouse, if any, execute all documents requiring the seller’s signature.
Again, photo identification is usually requested.
c. In corporate transactions, the person executing the documents must
have corporate authority to engage in the transaction, with documenta-
tion to reflect this authority.
d. If, for some reason, a party cannot attend a real estate closing in person,
a Power of Attorney may be granted to a third party to act in their place.
e. A special type of power of attorney that is used frequently is the “durable”
power of attorney. A durable power of attorney differs from a traditional
power of attorney in that it continues the agency relationship beyond the
incapacity of the principal. Most often, durable powers of attorney are created
to deal with decisions involving either property management or health care.
EXAMPLE: A couple owned a house in State A, which they intended
to sell as soon as they could. The couple had already moved to State
B. Their realtor in State A informed them of an offer on the house and
emailed them the contract. The couple printed out the contract, signed
it, scanned it, and emailed it back, finalizing the deal. However, the
couple did not intend to return to State A for the closing, so they ex-

446
REAL PROPERTY

ecuted a Power of Attorney to the realtor so that the realtor could sign
all the closing documents on their behalf.”

C. Transfer by Operation of Law and Will


1. In General
a. If the seller dies after the execution of the contract for sale but before
closing, legal title passes to his heirs or devisees. However, they must honor
the sales agreement, and the purchase money passes as personalty.
b. If the buyer dies after execution of the contract but before closing, the buyer’s
estate will be bound to complete the closing and pay the purchase price.
2. Ademption
a. A testamentary gift is adeemed by extinction—that is, it fails—
when property specifically bequeathed or devised is not in the
testator’s estate at his death.
b. An ademption by satisfaction occurs when a testator makes an inter
vivos gift of property to a beneficiary of a general or residuary disposi-
tion with the intent that the provision of the will be thereby satisfied.
c. Property a testator gave in his lifetime to a person is treated as a
satisfaction of a devise only if [UPC § 2-609(a)]:
(1) the will provides for deduction of the gift;
(2) the testator declared in a contemporaneous writing that the gift
is in satisfaction of the devise or that its value is to be deducted
from the value of the devise; or
(3) the devisee acknowledged in writing that the gift is in
satisfaction of the devise or that its value is to be deducted
from the value of the devise.
3. Exoneration
a. If the buyer dies after the execution of the contract for sale but before
closing, the party that takes the decedent’s realty may demand a closing
and exoneration of liens from the personal estate of the decedent.
b. The majority rule is that the beneficiary of a devise or bequest under
a will takes the property subject to any lien or mortgage outstanding
at the testator’s death.
(1) In other words, the majority rule is that a beneficiary is not
entitled to an exoneration of liens.
(2) A testator can provide, expressly or by necessary implication, that the
lien on the property should be paid off. However, a general directive In
the will for the payment of debts does not qualify as such a provision.
4. Lapse
a. Under the common law, a lapse occurs when a disposition fails
because the beneficiary predeceases the testator.
b. The common law lapse doctrine applies, except where prevented by
an anti-lapse statute [UPC § 2-603, cmt.].

447
OUTLINE

c. Under such a statute, if no alternative disposition of the property in


question is made in the will, lapse will nonetheless be prevented in
certain circumstances if a devisee fails to survive the testator and is a
grandparent, a descendant of a grandparent, or a stepchild of either
the testator or the donor of a power of appointment exercised by the
testator’s will [UPC § 2-603(b)].

D. Title Assurance Systems


1. Recording Acts
a. In General
(1) Recordation of the deed of conveyance is not required to
validate the transfer of title. However, recording becomes
important when two or more parties claim that the owner has
conveyed (or mortgaged) the property to them.
(2) Recording statutes generally apply to conveyances of freehold
interests, easements, profits, covenants, servitudes, mortgages,
assignments, and liens.
(3) Title based on adverse possession or prescription is not recordable.
(a) Title based on adverse possession provides good title to the
property, superior to that of the original landowner.
(b) However, in order to be marketable and recordable, the adverse
possessor must first obtain a quiet title judgment in his favor.
(4) Recordation will not cure defects in a deed caused by lack of
delivery, forgery, or fraud.
b. Types of Recording Statutes
(1) Race Statute
(a) Under a race statute, the person who records first prevails.
(b) Typical race statutes read: “No (i) conveyance of land, or (ii)
contract to convey, or (iii) option to convey, or (iv) lease of land
for more than three years shall be valid to pass any property
interest against lien creditors or purchasers for a valuable
consideration from the donor, bargainor, or landlord but from the
time of registration thereof in the county where the land lies….”
EXAMPLE: O, the owner of Blackacre, conveys Blackacre to
A, who does not record her deed. O then conveys Blackacre
to B, who pays value, immediately records his deed, and has
actual notice of A’s deed. Although B had actual notice of A’s
deed, B will prevail over A because B recorded before A.
(2) Notice Statute
(a) Under a notice statute, an unrecorded conveyance or other
instrument is invalid as against a subsequent bona fide
purchase for value and without notice.
(b) To prevail under a notice statute, a claimant of real property
must prove the following three elements:

448
REAL PROPERTY

1) the claimant took subsequent in time to another person


claiming ownership of the real property in question;
2) the claimant was a bona fide purchaser for value; and
3) the claimant took the property without actual,
constructive, or inquiry notice.
(c) A subsequent purchaser must pay value to prevail under a
recording statute.
1) Value exists if the purchaser has paid all of the
purchase price.
(d) A purchaser under a contract of sale who has not yet paid
any money should not be protected under the recording
statute. However, if the purchaser has paid some but not
all of the purchase price before receiving notice, the courts
are split as to whether the purchaser is protected under the
recording statute.
1) Some courts state that partial payment is not sufficient
to protect the subsequent purchaser.
2) The majority holds that the buyer is protected pro tanto (so
much as he has paid), which may be accomplished by:
a) giving the land to the holder of the outstanding
interest and giving the buyer restitution (the most
common method);
b) awarding the buyer a fractional interest in the land
proportional to the amount paid prior to notice; or
c) allowing the buyer to complete the purchase but to
pay the remaining installments to the holder of the
outstanding interest.
(e) Typical notice statutes read: “A conveyance of an interest in
land shall not be valid against any subsequent purchaser for
value, without notice, unless the conveyance is recorded.”
EXAMPLE: O, the owner of Blackacre, conveys Blackacre to
A, who does not record her deed. O then conveys Blackacre
to B, who pays value, does not have actual or inquiry notice
of A’s deed, and does not record his deed. Although B did
not record his deed, B prevails over A because B paid value
and did not have notice of A’s deed; B did not have actual or
inquiry notice of A’s deed and B could not have constructive
notice of A’s deed because the deed was not recorded.
EXAMPLE: O, the owner of Blackacre, conveys Blackacre
to A, who promptly records her deed. O then conveys Black-
acre to B, who pays value, does not have actual or inquiry
notice of A’s deed, and promptly records his deed. A will pre-
vail over B because B had constructive notice of A’s deed.

449
OUTLINE

(3) Race-Notice Statute


(a) Under a race-notice statute, an unrecorded conveyance or
other instrument is invalid against a subsequent bona fide
purchaser for value, without notice, who records first.
(b) To prevail under a race-notice statute, a claimant of real
property must prove the following four elements:
1) the claimant took subsequent in time to another person
claiming ownership of the real property in question;
2) the claimant was a bona fide purchaser for value;
3) the claimant took the property without actual,
constructive, or inquiry notice; and
4) the claimant recorded first.
(c) Typical race-notice statutes read: “Any conveyance of an interest
in land shall not be valid against any subsequent purchaser
for value, without notice, whose conveyance is first recorded.”
EXAMPLE: O, the owner of Blackacre, conveys Blackacre
to A, who does not record her deed. O then conveys
Blackacre to B, who pays value, does not have actual or
inquiry notice of A’s deed, and promptly records his deed.
A then records her deed. B prevails over A because B paid
value, did not have notice of A’s deed, and recorded first.
EXAMPLE: O, the owner of Blackacre, conveys Blackacre to
A, who does not record her deed. O then conveys Blackacre
to B, who pays value, does not have actual or inquiry notice
of A’s deed, and does not record his deed. A prevails over
B because B did not record his deed, and therefore, A pre-
vails under the common law rule of first-in-time, first-in-right.
c. Indexes
(1) In tract index jurisdictions, the searcher looks at the legal
description of the tract of land followed by a chronological listing
of all conveyances involving that piece of land.
(a) This type of search always provides constructive notice of
a claim to property.
(b) A minority of jurisdictions require tract index searches.
(2) In grantor and grantee index jurisdictions, the searcher
establishes a chain of title by searching two index books.
(a) First, the searcher looks back in time within the grantee-
grantor index.
(b) Then, the searcher looks forward in time within the grantor-
grantee index to see if any grantor conveyed an interest to
someone outside of the backwards chain.
(c) A majority of jurisdictions require grantor and grantee
index searches.

450
REAL PROPERTY

d. Chain of Title
(1) The shelter rule provides protection for a subsequent taker
who does not satisfy the applicable recording statute. Under the
shelter rule, a person who is a successor in interest to a person
protected by the recording statute is also protected.
EXAMPLE: O, the owner of Blackacre, conveys Blackacre to
A, who does not record. O then conveys Blackacre to B, who
pays value, records immediately, and takes without notice of A’s
deed. B then conveys Blackacre to C, who takes by gift. Under
either a notice or a race-notice statute, C cannot prevail over A
because C did not pay value. However, when B took Blackacre,
B would have prevailed over A under the relevant recording stat-
ute. C may take shelter under B’s protected status. Therefore, C
will prevail over A under either a notice or a race-notice statute.
(2) Exceptions to Shelter Rule
(a) A subsequent person may not take advantage of the shelter
rule if that person:
1) attempts to “wash” his deed by conveying to a third
person and then immediately taking a reconveyance of
the property; or
2) commits fraud in respect to the deed.
e. Protected Parties
(1) Recording statutes only apply to protect subsequent
purchasers for value.
(a) For purposes of recording statutes, this includes
mortgagees and lien creditors.
(2) A grantee who acquires the property by gift, devise, or adverse
possession will not qualify as a subsequent purchaser for
recording statute purposes.
(a) However, such a grantee may still record their interest to
protect it against subsequent purchasers of the property.
EXAMPLE: O, the owner of Blackacre, conveys Blackacre
to A as a gift in 2000. A does not record. In 2002, O sells
Blackacre to B, who has no notice of the previous gift to A.
B immediately records. A then records. B will have a superior
claim to the property over A in any recording jurisdiction.
EXAMPLE: O, the owner of Blackacre, conveys Blackacre to
A for value. A does not record. Five years later, O dies, and
bequeaths Blackacre to his son, B. B immediately records.
A then records. Under any recording statute, A will prevail.
This is because B took title to Blackacre by devise, and as
such, is not a subsequent purchaser for value. Therefore, B
is unable to obtain priority through a recording statute.

451
OUTLINE

(3) Judgment and Tax Liens


(a) A judgment lien is a lien filed in the county where a defen-
dant resides after the defendant has lost a lawsuit.
(b) The lien, once recorded, covers all real property that the
defendant owns, or may own in the future, in that county.
1) In some states, a judgment lien covers personal
property as well.
(c) A tax lien is recorded in a similar manner to judgment liens,
and it covers all real property and personal property that the
taxpayer owns or may own in the future in that county.
(d) Either type of lien can be enforced through a specific levy or
foreclosure action on a piece of property, or by waiting until
the debtor sells a parcel of real estate, at which time the
liens are paid off as part of the closing.
(e) In most jurisdictions, a judgment creditor is deemed not to
have paid value for the property, and so will not be entitled to
protection under the recording statute.
f. Priorities
(1) Under the common law, if two persons claimed title under
deeds to one parcel of real property, the property was awarded
to the person who took the property first in time.
(a) The common law rule still applies if the person who took later
in time fails to qualify under the relevant recording statute.
(2) If two parties claim under a relevant recording statute, the first to
satisfy the requirements of the recording statute will have priority.
g. Notice
(1) Actual notice exists if the claimant actually sees the deed under which
the other party is claiming, or otherwise knows of the conveyance.
(2) Constructive or record notice exists if the other party’s deed
is recorded in the proper place in the record books.
(a) Wild deeds are deeds recorded outside of the chain of
title, and which do not impart constructive notice on a
subsequent purchaser.
(b) The courts are divided on whether an easement or
restriction that appears in a prior deed to one lot made by
a common grantor is constructive notice to the grantee of
another lot from the common grantor.
(c) The courts are also split on whether a subsequent bona
fide purchaser has constructive notice of a prior deed from
the grantor that is recorded after a subsequent deed from
the grantor is recorded.
(3) Inquiry notice exists if the appearance of the property is such
that the claimant should have asked more questions about the
title to the property.

452
REAL PROPERTY

(4) Lis pendens notice is notice of a pending lawsuit that will


affect a particular piece of property; it is not a lien itself, but
provides notice that there is or may be a lien against the prop-
erty as a result of a judgment. When the lis pendens notice is
properly recorded, it will serve as constructive notice to other lien
holders, but does not serve to record the lien itself.
2. Title Insurance
a. Title insurance acts like a contract of indemnity. It is an alternative to
doing a lengthy and expensive title search.
(1) A buyer can purchase an insurance policy on the title received,
such that if there is a flaw in the title, the insurance policy will pay
for the loss suffered.
(2) The specifics will be detailed in the insurance policy itself.
b. Marketable Title Act
(1) A marketable title act provides a “cut-off” point, which limits the
time period during which a subsequent purchaser is required to
search the records. The cut-off point is fixed by identifying a deed
that has been of record for the required period, which is usually
30 or 40 years. Any interest that is not recorded or refiled within
the required period is extinguished.
EXAMPLE: In 1960, O, the record owner of Blackacre, grants
a right-of-way easement across Blackacre to A, which was
recorded. In 1967, O conveys Blackacre to B in a deed that
does not mention A’s easement. In 2004, B conveys Blackacre
to C. In 2005, A attempts to use the easement, but C prevents
A from crossing over Blackacre. The jurisdiction has a 30-year
marketable title act. The 1967 deed becomes the “root of title”
in 1997, when it had been recorded for 30 years. Under the
marketable title act, all competing interests recorded prior to
the root of title are extinguished. Therefore, A’s easement was
extinguished in 1997.

E. Special Problems
1. After-Acquired Title and Estoppel by Deed
a. Under the majority rule, estoppel by deed is an equitable remedy
that is applied on a case-by-case basis. Pursuant to this rule, when
a grantor grants title to property that she does not own, and then
subsequently acquires title to that property, she will be estopped from
asserting anything in derogation of the deed.
(1) The grantee has to go to court to assert title against the grantor,
or to otherwise enforce the deed against the grantor.
b. Under the minority rule, after-acquired title is a legal theory that
provides for automatic flow-through of the property.

453
OUTLINE

(1) With the after-acquired title doctrine, title to the property auto-
matically inures to the benefit of the grantee when the grantor
subsequently acquires title.
2. Forged Instruments and Undelivered Deeds
a. A forged deed is deemed to be a void deed, and does not operate to
transfer title.
(1) This is most likely to arise in the situation with a potential bona
fide purchaser. One who would otherwise be a bona fide purchaser
will lose out to the original grantor where the property has been
taken away from the original grantor by way of a forged deed.
b. An undelivered deed does not operate to pass title.
3. Judgment and Tax Liens
a. Judgment and tax creditors are not generally considered to have paid value
for the property. As such, they will not be considered bona fide purchasers
for value and will not qualify for protection under recording statutes.

454
Torts
TABLE OF CONTENTS

I. INTENTIONAL TORTS

Elements of Intentional Torts........................................................................................................458


Battery..........................................................................................................................................459
Assault.........................................................................................................................................459
False Imprisonment......................................................................................................................460
Intentional Infliction of Emotional Distress...................................................................................460
Trespass to Land..........................................................................................................................461
Trespass to Chattels....................................................................................................................463
Conversion...................................................................................................................................464
Defenses and Privileges to Intentional Torts................................................................................466

II. NEGLIGENCE

Duty..............................................................................................................................................475
Standard of Care..........................................................................................................................490
Breach of Duty.............................................................................................................................495
Cause-in-Fact (Actual Cause)......................................................................................................497
Proximate (Legal) Cause.............................................................................................................499
Damages......................................................................................................................................502
Defenses to Negligence...............................................................................................................503

III. STRICT LIABILITY

In General....................................................................................................................................508
Categories....................................................................................................................................508
Defenses......................................................................................................................................509

IV. PRODUCTS LIABILITY

Strict Products Liability in Tort......................................................................................................511


Products Liability on a Negligence Theory...................................................................................518
Products Liability on a Warranty Theory......................................................................................519

V. NUISANCE

Types of Nuisance........................................................................................................................522
Defenses......................................................................................................................................524
Remedies.....................................................................................................................................524

VI. DEFAMATION

In General....................................................................................................................................528
Defamatory Message...................................................................................................................528
Pleading Problems.......................................................................................................................529
Publication....................................................................................................................................530

456
Type of Defamation......................................................................................................................531
Damages......................................................................................................................................532
Common Law Defenses...............................................................................................................532
Constitutional Issues....................................................................................................................534

VII. INVASION OF PRIVACY

Intrusion into Seclusion................................................................................................................537


Appropriation of Identity or Likeness............................................................................................537
Public Disclosure of Private Facts................................................................................................537
Portrayal in a False Light.............................................................................................................538
Defenses and Privileges..............................................................................................................538
Constitutional Principles...............................................................................................................539

VIII. WRONGFUL INSTITUTION OF LEGAL PROCEEDINGS

Malicious Prosecution..................................................................................................................540
Wrongful Institution of Civil Proceedings......................................................................................541
Abuse of Process.........................................................................................................................541

IX. ECONOMIC TORTS

Intentional Misrepresentation (Fraud)..........................................................................................543


Negligent Misrepresentation........................................................................................................545
Interference with Contractual Relations ......................................................................................545
Interference with Prospective Advantage.....................................................................................546
Injurious Falsehood (Trade Libel).................................................................................................547

X. MISCELLANEOUS TORT CONCEPTS

Vicarious Liability..........................................................................................................................549
Joint and Several Liability............................................................................................................552
Survival of Action and Wrongful Death.........................................................................................553
Satisfaction and Release.............................................................................................................554

457
OUTLINE

I. INTENTIONAL TORTS

A. Elements of Intentional Torts


1. An intentional tort consists of the following elements:
a. a voluntary act;
(1) A defendant is not liable in tort for acts that are not voluntary.
Acts are not voluntary if they are a product of pure reflex or if the
defendant is unconscious when the act is performed.
EXAMPLE: Dex, during a sudden epileptic seizure, hits Pon.
While Dex did not intend the harm, there is also no liability be-
cause there was no voluntary act by Dex.
b. intent;
(1) All intentional torts require the defendant to have a certain
mental state when he performs the wrongful act. This mental
state is called intent. For most intentional torts, intent is estab-
lished if the defendant either:
(a) desires that his act will cause the harmful result described by
the tort; or
(b) knows that it is substantially certain that such a result will occur.
(2) Incompetency
(a) The fact that a defendant is mentally incompetent or a minor
does not preclude a finding that he possessed the intent
to commit an intentional tort, but incompetency may affect
whether such intent actually existed.
(3) Transferred Intent
(a) If a defendant acts with the necessary intent to inflict
certain intentional torts, but for some reason causes injury
to a different victim than intended, the defendant’s intent
is “transferred” to the actual victim. This transfer of intent
applies only to assault, battery, false imprisonment, and
trespass to land or to chattels.
EXAMPLE: Darryl swung a baseball bat, intending to strike
Astrid. However, the person Darryl hit was Plato. Darryl’s
intent to cause a harmful touching of Astrid will be “trans-
ferred” to Plato so that Darryl will be liable for committing a
battery against Plato.
c. causation;
(1) As to intentional torts, the defendant’s act or a force set in motion
by that act must cause the plaintiff’s injury.
d. harm; and
e. lack of a privilege or defense.

458
TORTS

B. Battery
1. A battery is an intentional act that causes a harmful or offensive contact
with the plaintiff or with something closely connected thereto.
2. The defendant must either:
a. desire to cause an immediate harmful or offensive contact; or
b. know such contact is substantially certain to occur.
EXAMPLE: Dagwood intentionally swings his fist into Pickles’ face,
intending to hit Pickles. Dagwood has committed a battery.
3. The harmful or offensive contact element is satisfied if the contact would
inflict pain or impairment of any body function, or if a reasonable person
would regard it as offensive.
a. It is sufficient for a battery if the defendant causes a contact with
something close to the plaintiff, as where the defendant snatches a
hat from the plaintiff’s hand.
b. Unlike assault, plaintiff need not be aware of the contact.
EXAMPLE: Ryan intentionally spits on Prudence while she is
asleep. Several weeks later, Prudence learns of Ryan’s act. Ryan is
liable for battery.

C. Assault
1. An assault is an intentional act that causes the plaintiff to experience a
reasonable apprehension of an immediate harmful or offensive contact.
2. The defendant must act with the desire to cause an immediate harmful or
offensive contact or the immediate apprehension of such a contact, or know
that such a result is substantially certain to occur.
3. Liability for assault will not be found unless a reasonable person in the
same position as a plaintiff would have experienced the same apprehen-
sion. However, if the plaintiff’s apprehension is reasonable, the fact that
the defendant lacked the actual ability to cause the harmful or offensive
contact does not defeat liability.
EXAMPLE: An assault is committed if Delson points an unloaded gun at
Paulson, as long as Paulson reasonably thought the gun was loaded.
4. This element is satisfied if the contact threatened would inflict pain or
impairment of any body function or if a reasonable person would regard
it as offensive.
EXAMPLE: Donahue holds a knife to Petunia’s throat and threatens to hurt
her. He is liable for assault and battery.
EXAMPLE: Delilah, a bank robber, points a gun into a crowded bank and
says, “Everyone lie down and keep quiet or else I’ll shoot.” This conditional
threat is an assault.

459
OUTLINE

D. False Imprisonment
1. False imprisonment is an intentional act that causes a plaintiff to be
confined or restrained to a bounded area against the plaintiff’s will, and the
plaintiff knows of the confinement or is injured thereby.
a. The defendant has the requisite intent for false imprisonment if he:
(1) desires to confine or restrain the plaintiff to a bounded area; or
(2) knows that such confinement is virtually certain to occur.
b. The plaintiff may be confined by the use of physical barriers (e.g.,
locking the plaintiff in a room), by failing to release the plaintiff where
the defendant has a legal duty to do so, or by the invalid assertion of
legal authority. No duration of confinement is required—a very brief
confinement will suffice, though the duration of the confinement may
affect the amount of damages.
(1) The plaintiff is under no duty to resist if the defendant uses or makes a
credible threat to use physical force. A plaintiff is not “confined” if there
is a reasonable means of escape of which he is actually aware.
EXAMPLE: Stuart takes all of Susan’s clothes and leaves her in
the middle of the woods. Because Susan does not have a means
of escape, Stuart has falsely imprisoned her.
(2) In general, the plaintiff must be aware of the confinement or must
suffer actual harm as a result of the confinement. Some cases have
held that infants or incompetents who are incapable of being aware of
their confinement can, nevertheless, recover for false imprisonment.

E. Intentional Infliction of Emotional Distress


1. Intentional infliction of emotional distress is an intentional or reckless
act amounting to extreme and outrageous conduct that causes the plaintiff
severe mental distress.
a. The defendant must act with intent to cause severe mental distress or
be reckless in creating the risk of emotional distress.
(1) Recovery for infliction of emotional distress is allowed where
a defendant acts recklessly, meaning that the defendant acts
in deliberate disregard of a high degree of probability that the
emotional distress will follow. This is contrary to the general notion
that reckless conduct is somehow “less wrongful” than intentional
conduct, and should be considered an exception limited to this tort.
b. The element of extreme and outrageous conduct is satisfied if the
defendant’s conduct is beyond the bounds of decency—conduct that a
civilized society will not tolerate.
(1) Offensive or insulting language is generally not considered outra-
geous, except in cases involving defendants who are common
carriers or innkeepers, or plaintiffs with known sensitivity, such as
the elderly, children, or pregnant women.

460
TORTS

EXAMPLE: Damien, who does not like children, dresses up like


a monster and runs into a kindergarten classroom shouting, “I’m
going to eat all of you little children!” Damien’s conduct may be
extreme and outrageous.
(2) There is a trend toward permitting recovery for offensive insults
made by one in a position of authority (e.g., a workplace supervisor).
c. The plaintiff must prove that the distress suffered was severe—
more than the level of mental distress a reasonable person could be
expected to endure. The emotional distress must be substantial, not
trivial or transitory. The more outrageous the defendant’s conduct, the
easier it will be for plaintiff to establish the requisite mental injury.
(1) Most states no longer require the plaintiff to show that actual
physical injury accompanied the severe emotional distress.
2. Where the defendant’s conduct is directed at a third party, the defendant
is subject to liability to a plaintiff, assuming the other elements of the tort
are satisfied, if the defendant intentionally or recklessly causes severe
emotional distress:
a. to a plaintiff who is an immediate family member or close relative of the
third party, where the plaintiff is present at the time and the defendant
is aware of the plaintiff’s presence; or
(1) Recovery is available whether or not such distress results in
bodily harm.
b. to any other plaintiff (regardless of relationship) who is present at the
time, if such distress results in bodily harm and the defendant is aware
of the plaintiff’s presence.

F. Trespass to Land
1. Trespass to land is an intentional act that causes a physical invasion of
the plaintiff’s land.
a. A defendant need only act with intent to cause a physical invasion of a
particular piece of land, not the specific intent to invade the plaintiff’s land.
Intentional entry onto land is a trespass even though the defendant does not
realize he has crossed a boundary line, or has a good faith belief that his
entry is lawful. In other words, mistake is not a defense as to a trespass action.
EXAMPLE: Don is out for a walk with his dog, Rover. Believing that he
is the rightful owner of a grove of pecan trees near his property line,
Don allows Rover to wander into the trees to “do his business.” The
trees are actually on Pam’s property. Although Don is mistaken about
his ownership of the trees, he may still be liable for trespass.
b. In order to bring an action for trespass, the plaintiff must be in actual
possession or have the right to immediate possession of that land. It
is important to distinguish “possession” from “ownership”—an adverse
possessor or a lessee may maintain a trespass action against a defen-
dant entering wrongfully onto land possessed, but not owned, by them.

461
OUTLINE

However, if the person who holds legal title to the land is not in posses-
sion, that person may not maintain a trespass action as to that land.
c. The element of physical invasion is satisfied if the defendant enters or
causes a third person or object to enter onto the plaintiff’s land, enters
onto the plaintiff’s land lawfully but then remains when under a legal
duty to leave, or fails to remove an object from the plaintiff’s land when
under a legal duty to do so.
EXAMPLE: Don is hitting baseballs in his backyard. For kicks, he
decides to see if he can hit a baseball into his neighbor Paxton’s yard,
100 feet away. If he succeeds in hitting a baseball over the fence into
Paxton’s yard, absent Paxton’s consent, Don is liable for trespass.
EXAMPLE: Paxton calls Don and asks him to remove a baseball that
Don accidentally hit onto his land. If Don fails to remove the baseball,
he may be liable for trespass.
2. Some events that might logically be considered intrusions onto land, such
as airborne pesticides that float onto a plaintiff’s land from the defendant’s
crop dusting of adjacent property, were traditionally addressed by tort law
under nuisance or strict liability principles. Some jurisdictions have begun
treating such invasions as trespasses to land if actual harm was caused.

EXAM TIP Consider nuisance and strict liability, along with trespass to land, whenever there
are facts involving something entering the plaintiff’s land and causing harm.

3. Plaintiff’s land includes the area both above and beneath the surface.
Traditionally, plaintiff’s land was thought to include the airspace and the
subsurface to a level that the plaintiff did or could make beneficial use of.
EXAMPLE: Debbie loves to climb trees. She regularly climbs a tall tree in
her backyard. One day, she climbs high into the tree and out onto a limb
that overhangs onto Peggy’s property. Debbie may be liable for trespass.
EXAMPLE: Dirk digs a tunnel to escape from his basement bedroom,
where his mother thinks he is sleeping. As he digs, he digs under his own
house and under the neighbor’s yard. Dirk may be liable for trespass.
a. Aircraft flying at or above normal flight altitude do not “trespass” on
the land above which they are flying. An intrusion into a plaintiff’s
“airspace” may be trespassory if it both enters into the “immediate
reaches” of that airspace and interferes substantially with the use and
enjoyment of the plaintiff’s land.
4. Traditionally, nominal damages are recoverable where the defendant tres-
passes but causes no real injury, as a way to vindicate the land possessor’s
rights. If the trespasser causes injury during the trespass, however, he is
liable for that harm as well. Further, where the defendant acts willfully or
maliciously, he may be liable for punitive damages.
EXAMPLE: Dex drives onto Paula’s land, believing wrongly that he has per-
mission to do so. While on the land, driving with all possible care, Dex hits a

462
TORTS

concealed pothole, loses control of the car, and destroys a rare and expen-
sive bush on Paula’s property. Dex is liable for the harm to the bush.
5. Ejectment
a. Ejectment is an action at law to recover possession of real property.
b. The following elements are required for ejectment:
(1) proof of legal title;
(2) proof of the plaintiff’s right to possession; and
(3) wrongful possession by the defendant.
c. A successful plaintiff is entitled to judgment for recovery of the property
and for mesne damages. Mesne damages compensate for the loss of
use of the land and are measured by the rental value of the property or
the benefit gained by the wrongful possessor, whichever is greater.
d. At common law, where the defendant mistakenly trespasses on or takes
possession of the plaintiff’s property and makes improvements thereon,
the plaintiff is entitled to recover the property and need not compensate the
defendant for these improvements. This is true even though the defendant
acted in good faith, believing that he had rightful possession of the property.

G. Trespass to Chattels
1. Trespass to chattels is an intentional act by the defendant that interferes
with the plaintiff’s chattel, causing harm.
a. Intent is satisfied when the defendant intentionally performs the phys-
ical act that interferes with the plaintiff’s chattel. The defendant is liable
even though he did not intend or recognize the legal significance of his
act. Mistake is not a defense to trespass to chattels.
EXAMPLE: Dorothy needs a black evening bag to carry to her sorority
formal. Without asking Paula’s permission, Dorothy goes into Paula’s
room in the sorority house, takes Paula’s black bag, carries it to the for-
mal, then returns it to Paula’s closet the next morning. Because Doro-
thy has Paula’s bag, Paula does not have a bag to carry to the formal.
Dorothy may be liable for trespass to chattels.
b. Chattel means tangible personal property or intangible property that
has a physical representation, such as a promissory note, or docu-
ments in which title to a chattel are merged, such as warehouse
receipts or bills of lading.
c. Interference with plaintiff’s chattel is actionable if it constitutes dispos-
session or intermeddling. More serious interferences with the plaintiff’s
chattel may amount to a conversion, discussed below.
(1) Dispossession is a direct interference with the plaintiff’s posses-
sion, such as where a defendant temporarily takes the plaintiff’s
chattel or wrongfully refuses to return it.
(2) Intermeddling is an interference with a chattel that does not
directly affect the plaintiff’s possession.

463
OUTLINE

EXAMPLE: Smearing mud on the plaintiff’s truck or kicking the


plaintiff’s dog.
2. In order to bring a trespass to chattel action, the plaintiff must have been in
actual possession or have had the right to immediate possession of the chattel.
3. Unlike other intentional torts, proof of actual damages is an element of the
cause of action for trespass to chattels. Actual damages would include the
value of loss of use (e.g., rental value) of the chattel during a dispossession
or the cost to remedy an intermeddling.

H. Conversion
1. Conversion is an intentional act by a defendant that causes the destruction
of or a serious and substantial interference with the plaintiff’s chattel.
a. As with trespass to chattels, mistake is not a defense to conversion. A
defendant is liable even though he did not intend or recognize the legal
significance of his act.
EXAMPLE: Drew sees Peter’s 1978 Volkswagen Bug parked by the curb.
Drew has always wanted to drive a vintage Bug, so she decides to hotwire
it and take it for a spin. Seven hours later, she returns the Bug to the same
parking spot where she found it, full of gas and in perfect condition. Drew may
be liable for conversion. If, while Drew is driving the car, the Bug is hit and
totaled, Drew is liable for conversion, even if the accident is not Drew’s fault.
b. “Destruction” or “serious and substantial interference” with a plaintiff’s
chattel is alternatively described as “the exercise by defendant of dominion
and control” over the chattel. This is an interference with the plaintiff’s
property interest that is more serious than in a trespass to chattels. In
distinguishing between the two torts, the longer the period of interference
and the greater the use of the chattel by the defendant, the more likely
it will be considered a conversion rather than a trespass to chattels. The
following types of acts are likely to be classified as conversions:
(1) wrongful acquisition (e.g., theft, embezzlement, and receiving
stolen property);
(2) wrongful transfer (e.g., selling, misdelivering, or pledging);
(3) wrongful detention (withholding from owner);
(4) loss, destruction, or severe damage;
(5) material alteration; or
(6) significant misuse.
c. In determining the seriousness of the interference and the justice of requiring
the defendant to pay the full value, the following factors are important:
(1) the extent and duration of the defendant’s exercise of dominion
or control;
(2) the defendant’s intent to assert a right inconsistent with the
other’s right of control;
(3) the defendant’s good faith;

464
TORTS

(4)the extent and duration of the resulting interference with the


plaintiff’s right of control;
(5) the harm done to the chattel; and
(6) the inconvenience and expense caused to the plaintiff.
2. The plaintiff is generally permitted to elect either recovery of damages, usually
fair market value at the time of conversion plus consequential losses, or replevin/
detinue/claim and delivery, compelling the defendant to return a converted
chattel, with recovery of damages attributable to its wrongful detention.
3. If the defendant offers to return the plaintiff’s chattel, this does not alleviate
the conversion, and the plaintiff need not accept the return. Such an offer
might be considered in mitigation of damages by a defendant who “inno-
cently” converted the plaintiff’s chattel (e.g., the defendant unknowingly
received property stolen from the plaintiff by a third person).

TRESPASS TO CHATTELS CONVERSION

Intentional tort. Intentional tort.

Committed by intentionally dispossessing or Committed by intentionally exercising dominion


intermeddling with a chattel in the possession or control over a chattel and seriously interfering
of another. with the rights of the owner.

Defendant is liable for damage or diminished Defendant is liable for the full value of the chattel
value of chattel. at the time of the conversion.

EXAMPLE: On leaving a restaurant, A mistakenly takes B’s hat from the rack,
believing it to be his own. When he reaches the sidewalk, A puts on the hat, dis-
covers his mistake, and immediately reenters the restaurant and returns the hat
to the rack. This is not a conversion. However, if A keeps the hat for six months
before discovering his mistake and returning it, this is a conversion. If A reaches
the sidewalk, puts on the hat, and a sudden gust of wind blows it off his head
and into an open manhole, this is also a conversion. If A takes B’s hat from
the rack intending to steal it, and he approaches the door, sees a policeman
outside, and immediately returns the hat to the rack, this too is a conversion.
4. Replevin
a. Replevin is an action at law for the recovery of specific chattels that
have been wrongfully taken or detained. It is also called “claim and
delivery” in some states.
b. Replevin is a possessory action that permits the plaintiff to recover
immediate possession of the property (at the beginning of the action).
(1) When the plaintiff seeks to recover the chattel at the beginning
of the action, he must post a bond as security against the possi-
bility that judgment will be found for the defendant. Note that the
defendant may post bond if he wishes to retain the chattel until
the action has concluded.

465
OUTLINE

(2) No seizure of the chattel is allowed until a hearing has taken


place to determine the plaintiff’s entitlement to the chattel
[Fuentes v. Shevin, 407 U.S. 67 (1972)].
c. Any damages suffered from the deprivation may also be recovered.
(1) The measure of damages is either the market value of the chattel
at the time of the deprivation minus the market value at the time
the action is commenced (if the chattel is held for sale) or the
value of lost use (rental value or lost profits).
(2) Where judgment is for plaintiff but the chattel is not returned, the plain-
tiff recovers the present value of the chattel as established at trial.
d. If the defendant has hidden the chattel or removed it from the jurisdiction,
and it cannot be seized, then an injunction for equitable replevin may issue.
e. Replevin may be brought only to recover tangible personal property; it is
not available for recovery of real property or intangible personal property.
(1) Under the Uniform Commercial Code, the buyer has a right of
replevin for goods identified to the contract if [UCC § 2-716(3)]:
(a) after reasonable effort he is unable to effect cover for such goods;
(b) the circumstances reasonably indicate that such effort will be
unavailing; or
(c) if the goods have been shipped under reservation and
satisfaction of the security interest in them has been
made or tendered.
(2) In the case of goods bought for personal, family, or household
purposes, the buyer’s right of replevin under Article 2 vests upon
acquisition of a special property interest, even if the seller had not
then repudiated or failed to deliver.
f. Article 2 of the UCC also provides for restitution of amounts paid in
sales of goods. Where the seller justifiably withholds delivery of goods
because of the buyer’s breach, the buyer is entitled to restitution of any
amount by which the sum of his payments exceeds [UCC § 2-718(2)]:
(1) the amount to which the seller is entitled by virtue of terms liqui-
dating the seller’s damages; or
(2) in the absence of such terms, 20% of the value of the total perfor-
mance for which the buyer is obligated under the contract or
$500, whichever is smaller.

I. Defenses and Privileges to Intentional Torts

DEFENSES

POPCANS
Privilege, Defense of Others, Defense of Property, Consent, Authority, Necessity, Self-Defense

466
TORTS

1. Privilege
a. Under certain circumstances, a defendant may not be liable for
conduct that would ordinarily subject him to liability.
b. A privilege may exist where:
(1) the person affected by the defendant’s conduct consents;
(2) some important personal or public interest will be protected by the
defendant’s ordinarily prohibited conduct, and this interest justifies
the harm caused or threatened by the defendant’s conduct; or
(3) the defendant must act freely in order to perform an essential function.
EXAMPLE: Delbert looks out his window and see his neighbor’s
house on fire. He grabs his hose and goes over to the house, putting
out the fire. Delbert’s entry onto his neighbor’s land will be privileged.
c. The defendant has the burden of proving the existence of a privilege and
that the privilege was exercised reasonably under the circumstances.
2. Consent
a. Even though a defendant has otherwise committed an intentional tort,
he is not liable if the plaintiff consented to the act which constituted the
tort. In order to invoke this defense, the consent must be effective, and
the defendant must not exceed the scope of the consent.
b. A plaintiff can manifest consent expressly, by implication, or as a
matter of law.
(1) Express consent exists where the plaintiff affirmatively commu-
nicates permission for a defendant to act.
(2) Consent is implied under circumstances where a reasonable person
would interpret the plaintiff’s conduct as evidencing permission to act.
EXAMPLE: David, a football player, tackles Pat, a player on the
other team. Because the boys are engaged in a football game, it
is apparent from Pat’s conduct that he consents to the tackle.
(3) Consent may be found to exist as a matter of law where the
plaintiff is unable to consent, and:
(a) emergency action is necessary to prevent his death or
serious injury;
(b) a reasonable person would be expected to consent under
the circumstances; and
(c) no reason exists to believe that the plaintiff would not consent.
EXAMPLE: Della falls and hits her head, splitting it open.
She is unconscious. A surgeon may operate to repair the
damage under the premise that Della would have consented
had she been awake.
c. Defenses to Consent
(1) Even where consent is expressly or impliedly given by the plain-
tiff, the circumstances may be such that this consent is ineffective

467
OUTLINE

and will not operate as a defense for the defendant. The most
frequently tested situations involve mistake, fraud, duress, inca-
pacity, and violation of criminal statutes.
(2) Mistake
(a) Consent is not effective if:
1) it is the product of a mistake of fact or law as to the
nature or consequences of the defendant’s act; and
2) the defendant is aware of the mistake.
(3) Fraud
(a) Consent is not effective if it is induced by the defendant’s
intentional deceit as to the essential nature or consequence
of his act. If the fraud relates to a collateral matter, the
consent may still be effective, but the fraud itself may be
independently tortious as to a plaintiff.
(4) Duress
(a) Consent is not effective if it is induced by a threat of immi-
nent harm to the plaintiff or by a false assertion of lawful
authority over the plaintiff. The same principle operates if the
threat or false assertion of authority is made as to a member
of the plaintiff’s immediate family.
(5) Incapacity
(a) As a matter of law, young children and people whose mental
capacities are impaired by mental disease, mental defect, or
intoxication are incapable of consenting to tortious conduct.
Without particular knowledge, the defendant may interpret a
plaintiff’s actions as manifesting consent.
(6) Violation of Criminal Statute
(a) Most jurisdictions treat consent as ineffective where the defen-
dant’s tortious conduct also constitutes a crime. A minority of
jurisdictions and the Restatement regard consent to a criminal
act as effective for purposes of civil liability for that conduct.
1) Minority rule jurisdictions consider consent to be effec-
tive, for tort law purposes, to an act that is also a violation
of a criminal statute, so long as the defendant’s act does
not constitute a breach of the peace. The majority rule
is that a person cannot consent to a crime. The minority
rule (Restatement rule) is that a person can consent to
a crime, provided it is not a crime that is also a breach
of the peace (in other words, a person cannot consent
for everyone). In addition, a person cannot consent to a
violation of a criminal statute that was meant to protect
him (i.e., consent is not a defense to statutory rape).
2) All jurisdictions regard consent as ineffective if the plain-
tiff is a member of the class of persons protected by the

468
TORTS

violated criminal statute. This is thus an “exception” to


the minority/Restatement approach as well as the “no
breach of peace” principle in a majority jurisdiction.
d. While a physician’s treatment of a patient without the patient’s informed
consent is typically treated as a form of negligence liability, there
remain situations where there is such a gross deviation on the part of
the defendant-doctor that a battery action will lie.
EXAMPLE: Parth gives Dr. Darth permission to remove his tonsils.
Parth awakes from surgery to find that Dr. Darth cut off one of his toes
instead. This would likely be a battery, as Dr. Darth far exceeded the
consent given to her.
e. If the defendant’s conduct substantially exceeds, in degree or nature,
the scope of the plaintiff’s otherwise effective consent, the defendant
may still be held liable for his tortious actions.
3. Self-Defense
a. A defendant charged with an intentional tort may defend on grounds
that he used reasonable force to prevent the plaintiff from engaging in
an imminent and unprivileged attack.
(1) A defendant otherwise acting in self-defense may only use the
degree of force reasonably necessary to avoid the harm threat-
ened by the plaintiff. A defendant could not successfully assert self-
defense if he used deadly force against a plaintiff whose conduct
did not threaten death or serious bodily harm to that defendant.
EXAMPLE: Dylan sees Percy approaching him in a bar with a
baseball bat poised to hit him. Percy shouts, “I’m going to get you,
Dylan!” If necessary, Dylan may tackle Percy to the ground or grab
his arms to prevent Percy from hitting him. However, if Dylan sees
Percy approaching him with a flyswatter, Dylan may not shoot
Percy, as he would be meeting non-deadly force with deadly force.
(2) A defendant cannot successfully assert self-defense when the
purported threat represented by the plaintiff’s conduct is not
about to happen, has been averted, or has ended.
EXAMPLE: Percy says to Dylan, “I’m going to come back here
tomorrow and kill you.”
(3) Where the plaintiff’s conduct, which purportedly threatens an immi-
nent attack, is privileged, a defendant may not invoke self-defense
and will be liable for any tortious acts committed toward the plaintiff.
EXAMPLE: police officer makes a lawful arrest of defendant.
(4) A defendant cannot successfully assert self-defense if he used
force to defend himself when he knew an impending attack was
based on a mistake as to his identity, and he would have had
time to correct the mistake and prevent the attack.

469
OUTLINE

b. Even where there is actually no harm threatened against the defen-


dant, he may successfully assert self-defense if a reasonable person in
the same circumstance would have believed that he was under attack.
Thus, so long as the defendant subjectively (i.e., honestly and in good
faith) believes that a sufficient threat exists to justify defensive force,
and there is an objective basis for that belief (i.e., a reasonable person
would believe so under the circumstances), self-defense is available.
c. In a majority of jurisdictions, a defendant acting in self-defense has no
duty to retreat; even if a safe retreat is possible, the defendant may
choose to use reasonable force against the attacking plaintiff. In a
minority of states, a defendant outside his home must retreat before
using deadly force if that is safely possible; if inside his own home or
where safe retreat is not possible, use of defensive force is permitted.
d. Where a defendant otherwise properly acts in self-defense, he is not
liable for an intentional tort if he thereby inadvertently inflicts injuries
on innocent third persons. However, that defendant will be liable if he
deliberately injures a third party, and may be liable for negligence if he
unreasonably inflicts such injuries in the course of defending himself
against an attack by the plaintiff.
4. Defense of Others
a. A defendant is entitled to defend another person from an attack by
the plaintiff to the same extent that the third person would be lawfully
entitled to defend himself from that plaintiff.
EXAMPLE: Dora and Terry are walking down the street. Dora sees
Terry’s ex-girlfriend, Phoebe, coming toward Terry with a knife. Realiz-
ing that Terry does not see Phoebe, Dora jumps out and grabs Phoe-
be’s wrist, wrestling the knife from her. Because Terry could have been
injured, Dora is privileged to protect him.
b. At common law, a defendant who made a mistake about whether
defense of a third person was justified, or as to the degree of force that
was reasonable, could not assert the defense and would be liable to
the plaintiff for an intentional tort.
c. The modern majority rule applies the reasonable mistake doctrine,
which states that a defendant is relieved of liability where the third
person would not be permitted to assert self-defense against the
plaintiff if a reasonable person in the defendant’s position would have
believed that defense of the third person was justified, and that the
defendant’s action was necessary to prevent harm to the third person.
EXAMPLE: Dakota sees Xenobia run out of a doorway pursued by
Patton. Patton tackles Xenobia and they struggle. Xenobia cries out,
“Help, he’s hurting me!” Dakota seizes Patton and forces him to re-
lease Xenobia. It is subsequently established that Xenobia had robbed
Patton of his wallet and Patton was lawfully seeking to restrain Xeno-
bia. At common law, Dakota was liable to Patton for battery. At modern

470
TORTS

law, Dakota will not be liable to Patton for battery if a reasonable per-
son in Dakota’s position would have believed that Xenobia was entitled
to exert force in defense against Patton.
5. Defense of Property
a. A defendant is permitted to use reasonable force to prevent a plaintiff
from committing a tort against the defendant’s property.
(1) The defendant must first demand that the plaintiff desist the conduct
that threatens injury to his property before he can use force in defense,
unless it would be futile or dangerous to make such a demand.
(2) The amount of force used by the defendant must be no greater than
necessary to prevent the threatened harm. In addition, it is never
permissible to use deadly force to protect one’s property from injury.
(a) A defendant may not indirectly use a greater degree of force
than would be justified if he were acting personally against a
plaintiff. Thus, use of dogs or mechanical devices to protect
property will generally result in liability, even if the plaintiff’s
conduct is otherwise tortious, because such force inflicted on
the plaintiff is almost always considered unreasonable.
b. A defendant may use reasonable force to promptly recover his
personal property if tortiously dispossessed of that property by the
plaintiff. The defendant may also use reasonable force to recover such
property from a guilty third party (one who took possession knowing of
the tortious dispossession).
(1) The defendant may apply only such force against the plaintiff as
is reasonably necessary to recover the property. A defendant may
never use deadly force to recover property.
(2) A defendant must act with reasonable diligence to discover the
dispossession and to recover his property. This has often been
described by the courts as a requirement that the defendant be “in hot
pursuit” of the tortiously dispossessing plaintiff or the guilty third party.
(3) Before being otherwise entitled to use force to recover personal prop-
erty, the defendant must demand that it be returned by the wrongfully
dispossessing plaintiff or guilty third party in wrongful possession.
EXAMPLE: Pam takes Darla’s gold bracelet. Pam knows that the
bracelet belongs to Darla, but she really likes it, so she takes it.
Darla asks Pam to give it back, but Pam refuses. Darla may go
up and grab the bracelet away from Pam.
(4) Even if otherwise proper, a defendant may not use force to
recover property as to which the plaintiff came into possession
under a claim of right.
EXAMPLE: Defendant may not use force to recover property
from a withholding bailee or a purchaser on credit who has de-
faulted on the obligation to pay.

471
OUTLINE

(5) A defendant cannot successfully assert the defense of


recovery of property if he is mistaken about the fact that he
was tortiously dispossessed of it by the plaintiff. This is so
even if the mistake is reasonable.
(6) If otherwise proper for the defendant to use reasonable force to
recover property, that defendant may enter upon the land of the
plaintiff or guilty third party in order to effectuate the recovery. Such
entry must be at a reasonable time and must be accomplished in a
reasonable manner. The same rule applies if the property is on the
land of an innocent possessor. However, the defendant will be held
liable for any actual damage such entry causes.
EXCEPTION: Defendant may not enter upon the land of an in-
nocent party and recover tortiously dispossessed property if that
property is so situated as a result of the defendant’s fault.
c. Recovery of Wrongfully Dispossessed Land
(1) In the majority of jurisdictions, the modern rule is that a defendant
may never use force to recover land of which he has been wrong-
fully dispossessed. Such jurisdictions provide civil statutes that
offer prompt judicial remedies and thus avoid any necessity for
violent confrontations over possession of land.
(2) A minority of states still apparently follow the traditional rule that
permits a defendant to use reasonable, non-deadly force to
recover tortiously dispossessed land, so long as he acts promptly
after discovering the dispossession.
6. Necessity
a. A defendant is permitted to injure a plaintiff’s property if this is reason-
ably necessary to avoid a substantially greater harm to the public, to
himself, or to his property.
(1) A defendant may successfully assert this defense if a reasonable
person in the same circumstance would believe it necessary to
injure the plaintiff’s property. This is an objective standard.
(2) If the defendant reasonably but mistakenly believes that his
actions are justified under the objective standard set forth, he is
privileged to act, even if it subsequently is established that there
was no actual necessity.
(3) If the defendant is acting to protect private, individual interests,
he is justified in doing so if the threatened harm he is acting to
avoid is substantially greater than the harm that will result from
the action he actually takes.
(4) If the defendant is acting to protect the public interest, he is justi-
fied in doing so only if the threatened harm is severe—essentially,
a disaster (e.g., the 1906 San Francisco earthquake and fire).
(5) In most jurisdictions, when a defendant acts out of private neces-
sity, the defendant is still liable for any actual damage to the

472
TORTS

plaintiff’s property, but is not liable for any technical tort (such as
trespass). In some jurisdictions, necessity is a complete defense.
(6) If a defendant acts out of public necessity, he incurs no liability
whatsoever for damage to the plaintiff’s property.
7. Authority
a. Arrest
(1) Where the defendant is a police officer acting pursuant to a duly
issued warrant, valid on its face, he is not liable in tort for the
fact of arrest. This rule applies to both felony and misdemeanor
arrests, and applies even if the warrant is subsequently held to
be invalid by a court.
(a) This defense is available even though the arrested plaintiff
is not the person against whom the warrant was issued, so
long as the defendant-police officer’s mistake as to the iden-
tity of the person to be arrested was reasonable.
(2) Where a police officer or private citizen acts to prevent a felony that is
being committed or appears about to be committed in his presence,
he is not liable for an intentional tort based upon such an arrest.
(a) The defendant may assert the defense even though he was
mistaken in his belief that a felony was being or about to be
committed, so long as the mistake is reasonable.
(3) Both police officers and private citizens may assert the defense
for arrest of a person who has, in fact, committed a felony.
Treatment varies where a mistake is made as to either the fact
that a felony has been committed or the identity of the felon,
depending upon whether the defendant is a police officer or not.
(a) A police officer is not liable in tort for a warrantless felony
arrest, even if he makes a mistake about whether a felony
was committed or about the identity of the person who
committed the felony, so long as the mistake is reasonable.
(b) Where a private citizen makes a felony arrest but makes a
mistake about whether a felony was committed the defense
of authority is not available, even if the mistake is reason-
able. If a felony was committed but the private citizen makes
a mistake about the identity of the person who committed
the felony, the defense of authority is available if defendant’s
mistake as to identity is reasonable.
(4) A police officer or private citizen is not liable for an arrest made
without a warrant as to a breach of the peace (e.g., a misde-
meanor involving violence) that is committed or appears about to
be committed in his presence.
(a) A defendant may assert this defense even though mistaken
in his belief that a breach of the peace was being or about to
be committed, so long as the mistake is reasonable.

473
OUTLINE

(5) In the majority of jurisdictions, a defendant may not assert the


defense of authority if he makes a warrantless arrest for commis-
sion of a misdemeanor not involving a breach of the peace.
(a) Statutes in a few jurisdictions give police officers the
authority to make warrantless misdemeanor arrests not
involving a breach of the peace if the misdemeanor is
committed in their presence.
(6) If a defendant is otherwise entitled to make an arrest, the defen-
dant may also enter upon the plaintiff-arrestee’s land to effec-
tuate the arrest.
b. Shopkeeper’s Privilege
(1) A defendant-shopkeeper is not liable for false imprisonment or a
related tort if he has a reasonable suspicion that the plaintiff has
stolen goods, uses reasonable force to detain the person, and
detains the plaintiff for a reasonable period and in a reasonable
manner, either on the premises or in the immediate vicinity.
(a) The defendant-shopkeeper may assert the defense, even
if the detained plaintiff has not, in fact, stolen any property,
so long as the defendant’s mistake is reasonable. It is often
stated that the shopkeeper must have a reasonable suspi-
cion that goods were stolen and that the detained plaintiff
was the person who stole them.
c. Discipline
(1) If a defendant is charged with maintaining discipline (e.g., a parent
or teacher), he may use reasonable force to perform this duty.
(a) Force that is reasonably necessary to maintain discipline
varies according to the circumstances. Factors considered
include the nature of the misconduct; the age, sex, and
physical condition of the disciplined plaintiff; and the motiva-
tion under which the defendant acted.

474
TORTS

II. NEGLIGENCE

A. Duty
1. The element of duty is usually described as an obligation, recognized by
law, requiring the defendant to conform to a certain standard of conduct for
the protection of others against unreasonable risk. Where the defendant
engages in conduct that is claimed to have injured the plaintiff, the issue
can be framed as: did the defendant have a duty to the plaintiff to conform
to a certain standard of conduct?
2. In some situations the general duty rule will not apply.
3. Foreseeable Plaintiffs
a. In the famous Palsgraf case, Justice Cardozo articulated the rule that a
defendant owes a duty only to foreseeable plaintiffs [Palsgraf v. Long
Island R.R. Co., 162 N.E. 99 (N.Y. 1928)]. (Justice Andrews’ dissent in that
case argued that “[e]veryone owes to the world at large the duty of refraining
from those acts which unreasonably threaten the safety of others.”)
(1) Justice Cardozo’s view is the prevailing view.
(2) If taken literally, the Cardozo view that a duty is owed only to
foreseeable plaintiffs could prevent some worthy persons from
recovering for negligence. A key example of a worthy plaintiff is
a rescuer who is injured due to a person’s negligence. Cardozo
claimed, and virtually all jurisdictions have agreed, that rescuers
are per se foreseeable plaintiffs, and thus are owed a duty.
b. The traditional rule is that there is no affirmative duty to take action
to aid or protect a plaintiff who is at risk of injury unless such action
is taken. This is based on the distinction between nonfeasance and
misfeasance. The law does recognize certain situations, however, in
which a duty to take action does arise.
EXAMPLE: Darren sees Polly injured by the side of the road. Late for
an appointment, Darren does not stop to help Polly. Polly bleeds to
death. Polly’s estate does not have a cause of action against Darren.
(1) If the defendant’s conduct is responsible for placing the plaintiff in
a position where he requires aid, the defendant has a duty to take
action to aid the plaintiff.
(a) A negligent omission occurs when the defendant fails to do
something that a reasonable person would have done, such
as stopping at a stop sign. Negligent omissions are treated
as misfeasance for which a duty is typically owed.
(b) The traditional application of this exception recognized a duty
to act only when a defendant’s conduct that caused a plaintiff
to be in peril was itself negligent. The modern approach, which
is still a minority view, is to recognize a duty to take action even
when a defendant’s conduct creating the peril was not tortious.

475
OUTLINE

EXAMPLE: Danielle is on her way to the bar exam. She is wor-


ried about being late, so she is speeding. As she turns a corner,
she fails to notice Pedro on his bike. Her car strikes Pedro’s bike
and causes Pedro to fall into the road and become trapped under
his bicycle. Pedro sustains a broken ankle. Danielle must stop
and help Pedro out of the road so that another car will not hit him.
(2) Although a defendant has no general duty to take affirmative
action to aid a plaintiff, in many jurisdictions, once a defendant
actually takes such action, he has a duty to exercise due care as
to his subsequent conduct.
(a) Some jurisdictions state that if a person undertakes to act,
he is liable if he leaves the plaintiff-victim in a worse position.
(b) As a general rule, a rescuer must act reasonably in effecting
the rescue. Many jurisdictions, however, have Good Samaritan
statutes which limit the liability of rescuers who provide emer-
gency aid. For example, some only impose liability for reckless
or intentional wrongdoing on the part of the rescuer.
(c) The majority rule is that a defendant who gratuitously prom-
ises to take action to aid a plaintiff has no duty to actually
take the promised action. This is so even if the plaintiff relied
on the promise to his detriment. This is often referred to as
nonfeasance, a complete failure to render the promised aid.
(d) If a defendant gratuitously promises to aid a plaintiff, once
the defendant attempts to give the promised aid, the defen-
dant has a duty to exercise reasonable care in doing so. Due
to the perceived harshness of the nonfeasance rule, courts
readily find that a defendant who makes a gratuitous promise
to aid a plaintiff has subsequently taken action that consti-
tutes an attempted performance of the promise.
(e) In a minority of jurisdictions, a defendant is liable for a complete
failure to perform a gratuitous promise of aid if the plaintiff relied
on the promise to his detriment (e.g., by foregoing other aid).
(3) The defendant has a duty to take affirmative action in aid of a
plaintiff where a special relationship exists between the defen-
dant and plaintiff.
(a) If the defendant derives or occupies a position of power
over the plaintiff (e.g., the plaintiff is particularly vulnerable
and dependent upon the defendant), there is a duty to take
action. The following relationships have been generally
recognized as triggering the duty to care for the plaintiff:
1) employer-employee during and in the scope of employment;
2) common carrier- and innkeeper-customer;
3) school-pupil;
4) parent-child;

476
TORTS

5) business-patron; and
6) jailer-prisoner.
(4) Duty to Control Third Parties
(a) There is no duty to control the conduct of a third person as to
prevent him from causing physical harm to another, unless:
1) a special relationship exists between the defendant and
the third party that imposes a duty upon the defendant
to control the third party’s conduct; or
EXAMPLE: Dean is in a movie theater sitting next to
Perry. Dean and Perry do not know one another; they
just happen to be sitting in adjacent seats. Perry gets
very angry with the usher because the usher tells him to
take his feet off the seat in front of him. If Perry decides
to slug the usher, Dean has no duty to control him.
2) a special relationship exists between the defendant and
the third party that gives the third party a right of protection.
EXAMPLE: Dean is in the movie theater sitting next
to his son when Eliza enters. Eliza takes one look at
Dean’s son, realizes that he is the kid who bit her child
on the playground, and lunges at him. Dean has a duty
to protect his son from Eliza and to control her conduct.
(b) A parent is under a duty to exercise reasonable care to
control his minor child so as to prevent the child from inten-
tionally harming others or creating an unreasonable risk of
bodily harm to them, if the parent:
1) knows or has reason to know that he has the ability to
control his child; and
2) knows or should know of the necessity and opportunity
for exercising such control.
EXAMPLE: Milly and Erica, two 13-year-olds, are build-
ing bombs in Milly’s garage. Their parents do not su-
pervise their free time and giggle when they find bomb-
making manuals in the house, saying, “Kids will be kids!”
If Milly and Erica bomb their school and kill many of their
classmates and teachers, their parents may be liable.
(c) A master is under a duty to exercise reasonable care to control
his servant while acting outside the scope of his employment in
order to prevent the servant from intentionally harming others or
creating unreasonable risk of bodily harm to them, if:
1) the servant:
a) is upon the master’s premises or premises upon which
the servant is privileged to enter only as his servant; or
b) is using a chattel of the master; and

477
OUTLINE

2) the master:
a) knows or has reason to know that he has the ability
to control his servant; and
b) knows or should know of the necessity and oppor-
tunity for exercising such control.
EXAMPLE: Darnell owns Silver Skates, an ice-skat-
ing rink in Coldville. Ellie is a skating teacher at the
rink. Darnell is in the office overlooking the rink one
day when he sees Ellie, who is supposed be taking
the day off, doing skating lifts with skaters that Darnell
knows to be beginners. Darnell knows that beginning
skaters are not ready to do lifts and can really hurt
themselves if they do. Darnell has a duty to intervene
to protect the students if he is able to do so.
(d) The defendant-employer has a duty to a plaintiff to exercise
reasonable care in hiring employees, such that a defendant
may be liable to a plaintiff if an employee subsequently injures
the plaintiff. This is distinguished from the vicarious liability of
an employer for his employee’s torts—respondeat superior,
where the negligent conduct is that of the employee and liability
is attributed by law to the employer. In negligent hiring, the
employer is liable for his own negligence in hiring the employee,
not vicariously liable for the wrongful conduct of the employee.
(e) Where a defendant permits a third person to use his
personal property, the defendant has a duty to control such
use and to exercise due care in permitting the third person to
use the property. The context in which this issue is frequently
tested is an auto owner-defendant who permits a member of
his family to drive his car.
(f) In some jurisdictions, a defendant who has the requisite
special relationship with a third person and who becomes
aware that the third person intends to do specific harm to an
identified plaintiff has a duty to warn the plaintiff of the harm.
The special relationships that have been recognized as trig-
gering this duty include:
1) psychotherapist-patient; and
2) custodian-prisoner.
EXAMPLE: Zazu tells his therapist, Dr. Drew, that he
intends to kill his ex-wife, Pru. Dr. Drew takes no action
to warn Pru of the threat, which Dr. Drew believes to be
credible. Pru is killed. In a negligence action against Dr.
Drew, most jurisdictions would find that he had a duty
to take reasonable steps to warn Pru of the impending
danger based on his relationship with Zazu, his patient.

478
TORTS

(5) Based on the distinction between nonfeasance and misfeasance,


one generally does not have a duty to protect a person from
third-party criminal conduct. In certain circumstances, however, a
special relationship (such as a landlord-tenant or business-invitee)
will trigger a duty. Jurisdictions vary on the amount of foreseeability
that must exist before a court will find a duty to protect.
(a) Some jurisdictions will only find a duty to protect where
there were prior similar incidents, making the third-party
criminal conduct particularly foreseeable. Others use a
more flexible totality of the circumstances test, while
some others balance the degree of foreseeability against
the burden that would be placed on the defendant to
protect the plaintiff from harm.
c. When the defendant is a governmental entity, the question of whether
the defendant owes a duty to the plaintiff will depend on the function
the government is fulfilling that gives rise to the cause of action.
(1) If the governmental entity is acting in a proprietary function–
that is, acting in an area traditionally occupied by private entities–
the government will be treated as any other defendant for the
purpose of determining duty.
(2) When the governmental entity is engaged in a discretionary
activity–that is, where the governmental entity is using judgment
and allocating resources–the courts will not find a duty.
(3) When the governmental entity is acting in a ministerial function,
courts will find a duty; once the governmental entity has under-
taken to act, it must do so non-negligently.
(a) Under the public duty doctrine, when a government agency
(e.g., the police or fire department) is sued for failing to
provide an adequate response, courts will find no duty unless:
1) there has been reliance on the response of the agency;
2) there is a special relationship between the plaintiff and
the agency; or
3) the agency has increased the danger beyond what
would otherwise exist.
EXAMPLE: Paul’s third-story apartment catches fire, and
he calls 911. Paul asks the dispatcher if he should jump
out the window, and she tells him that the fire department
is only a minute away, and he should wait for the firemen
to arrive so that he can climb down the ladder. In reality,
the firemen have stopped off to grab a snack before re-
sponding, and by the time they arrive the fire has spread,
blocking Paul’s access to the window to jump.
(b) If the defendant is a utility, courts have refused to impose
duty beyond those who are in privity of contract to the utility.

479
OUTLINE

4. Negligent Infliction of Emotional Distress


a. If the plaintiff’s injury is not personal injury or property damage, duty
issues arise.
b. Courts have traditionally been reluctant to allow liability for emotional
distress, and apply special rules for claims for pure emotional distress.

NOTE Pain and suffering, though emotional damages, are not subject to the limita-
tions placed on claims for pure emotional distress.

c. Direct Claims
(1) In most jurisdictions, to recover for emotional distress the plaintiff must:
(a) have been in the zone of danger—that is, the area in which
he was at risk of being physically injured; and
EXAMPLE: Paula and Peter are crossing the street when
Dexter comes driving down the road at twice the speed limit.
Paula manages to get out of the way, but Peter is hit by Dex-
ter. Paula was in the zone of danger and may have a cause
of action for emotional distress.
(b) have suffered some accompanying physical manifestation
of the emotional distress.
(2) Two exceptions exist to the zone of danger and physical mani-
festation requirements:
(a) if the defendant negligently transmits a telegram announcing
the death of a loved one; and
(b) if the defendant negligently mishandles a corpse.
(3) In a minority of jurisdictions, if the defendant has a preexisting
duty to the plaintiff, the plaintiff may recover for negligent infliction
of emotional distress.
(4) Some jurisdictions have eliminated the requirement of physical
manifestation of emotional distress and allow plaintiffs to prevail
based on a showing of severe emotional distress without accom-
panying physical symptoms.
d. Bystander Actions
(1) In a bystander action, the physical harm occurs to a loved one,
and the plaintiff sues for his emotional distress as a result of the
injury to another. It is premised upon the defendant’s violation
of the duty not to negligently cause emotional distress to people
who observe the conduct which causes harm to another.
(2) The majority rule is that a plaintiff may recover for negligent inflic-
tion of emotional distress under a bystander theory if he:
(a) was located near the scene of an accident;
(b) suffered a severe emotional distress (“shock”) resulting from the
sensory and contemporaneous observance of the accident; and
(c) had a close relationship with the victim.

480
TORTS

(3) Before the recognition of bystander liability, American courts


permitted a person to recover for negligently caused emotional
disturbance only if the person suffered physical impact due to the
defendant’s negligent conduct or, later, was personally in the zone of
danger created by the defendant’s negligent conduct. Most American
courts have now adopted some version of the bystander liability rule.
(4) Bystander emotional distress is a derivative claim in most jurisdic-
tions. As a result, the bystander’s recovery may be reduced propor-
tionally if the injured party is found to be comparatively negligent.

EXAM TIP The requirements for negligent infliction of emotional distress (“NIED”) are
different than the requirements for intentional infliction of emotional distress
(“IIED”). IIED is covered in depth earlier in this outline. See the chart below
for a recap of the different elements required for both NIED and IIED.

INTENTIONAL INFLICTION OF EMOTIONAL DISTRESS


Elements Required for Plaintiff Recovery When Conduct Directed at Third-Party

Theory 1 Theory 2

1. Plaintiff must be present when conduct 1. Plaintiff (no special relation to the third
occurs to third party/victim; party/victim required) must be present
2. Plaintiff must be a close relative of the when conduct occurs;
third party/victim; OR 2. Plaintiff suffers actual bodily harm (a
3. Defendant is aware of plaintiff’s physical manifestation of the emotional
presence; and distress); and
4. Plaintiff suffers severe emotional distress 3. Defendant is aware of plaintiff’s presence.
(whether or not it results in bodily harm).

NEGLIGENT INFLICTION OF EMOTIONAL DISTRESS


Elements Required for Recovery

Theory 1: Direct Claim Theory 2: Bystander Action

1. Plaintiff is within the “zone of danger”; and 1. Plaintiff is present at the scene and
2. Plaintiff suffers emotional distress OR witnesses the event;
and some accompanying physical 2. Plaintiff is a close relative of the third
manifestation of the emotional distress. party/victim; and
3. Plaintiff suffers severe emotional distress.

5. Wrongful Conception, Wrongful Birth, Wrongful Life


a. Wrongful conception applies where the injury is the birth of a healthy child.
(1) Generally, wrongful conception actions arise where the plaintiff
has had a negligently performed vasectomy or other negligently
administered form of birth control.

481
OUTLINE

(2) Damages typically involve the cost of the birth and the cost to
rectify the ineffective contraceptive measure.
(3) Courts are very reluctant to award the costs of raising a child
through the age of majority.
b. Wrongful birth is the claim of the parents for the birth of an unhealthy child.
(1) Wrongful birth claims generally stem from a physician’s failure to
diagnose a disability in the fetus, which the plaintiff claims would
have led her to not give birth to the child.
(2) Many courts will not recognize a claim for wrongful birth.
(3) Some courts will award the extraordinary costs of having a child
with special needs, but the jury may offset this award by the
benefit obtained from having the child.
c. Wrongful life is the child’s action for having been born unhealthy.
(1) Most courts will not award damages for wrongful life.
(2) A small number of courts have awarded damages for the costs of
the child’s special needs after the age of majority.
6. Land Possessor Liability
a. The standard of care applied to owners and occupiers of land varies
according to which of three categories of danger or activity were
involved in the injury to the plaintiff. These categories are:
(1) activities—the injury to the plaintiff derived from the conduct of
persons on the land;
(2) artificial conditions—the injury to the plaintiff derived from circum-
stances created by persons on the land, such as buildings, exca-
vations, cultivation, etc.; and
(3) natural conditions—the injury to the plaintiff derived from circum-
stances not created by persons but existing on the land, such as
natural bodies of water, trees occurring naturally, falling boulders, etc.
b. In addition to considering the category of danger or activity that injured
the plaintiff, the analysis of the standard of care varies according to the
categories of the plaintiffs who claim injury.
c. Plaintiffs on the land
(1) Invitees
(a) An invitee is a person who enters onto the defendant’s land at
the defendant’s express or implied invitation, and who enters for a
purpose relating to the defendant’s interests or activities. Invitees
are classified as either business invitees or public invitees.
1) A business invitee is an invitee who enters onto the
defendant’s land for a purpose related to the defen-
dant’s business activities or interests.
EXAMPLE: Customers and persons accompanying them,
delivery persons, salespersons (if reasonable for them to
expect that someone on the non-private, non-residential

482
TORTS

premises may be interested in purchasing), and job ap-


plicants (if reasonable for them to expect that employment
may be available) are examples of business invitees.
2) A public invitee is a member of the public who enters
onto the defendant’s land for a purpose as to which the
land is held open to the public.
EXAMPLE: Visitors to airports and visitors to churches.
a) In a minority of jurisdictions, the classification of public
invitee is not recognized. However, courts in such jurisdic-
tions readily find a business purpose in visits by persons
who would otherwise be considered public invitees.
(b) A defendant has a duty to exercise reasonable care to prevent
injuries to invitees caused by activities conducted on his land.
(c) The defendant also has a duty to exercise reasonable care to
discover dangerous artificial conditions that invitees would not
reasonably be aware of, and to warn invitees of the existence
of such conditions or to make the conditions safe. If it would be
insufficient to make the dangerous condition reasonably safe by
providing only a warning, the defendant’s duty includes a duty
of reasonable care to provide other precautions.
(d) The defendant’s duty to invitees with regard to dangerous natural
conditions is the same as that applicable to artificial conditions.
(e) An invitee may be regarded as a licensee or even as a tres-
passer if the invitee enters areas of the defendant’s property to
which his invitation does not extend, or if the invitee stays in a
permitted area longer than was contemplated by the invitation.
(2) Licensees
(a) A licensee is a person who enters onto the defendant’s land
with the defendant’s express or implied permission, and who
does not enter for a purpose benefiting the defendant or the
defendant’s activities.
EXAMPLE: Visiting relatives, social guests, and door-to-
door salespersons.

NOTE Invitees who exceed the scope of defendant’s invitation are treated as licensees.

(b) As to licensees, a defendant has a duty to exercise reason-


able care to protect them from injury arising from activities
conducted by the defendant or on the defendant’s behalf. It is
usually sufficient for the defendant to warn the plaintiff-licensee,
but the defendant’s duty includes the exercise of reasonable
care to discover licensees of whom he is not aware.
(c) A defendant has a duty to exercise reasonable care to warn
of any artificial conditions of which he is aware, which present

483
OUTLINE

an unreasonable danger, and of which the plaintiff-licensee is


unaware and unlikely to discover. The defendant has no duty
to inspect his land for such dangerous artificial conditions.
1) The standard of care applicable to licensees requires
defendant to protect them from dangerous natural condi-
tions to the same extent required for artificial conditions.
(3) Trespassers
(a) If a plaintiff enters onto a defendant’s land without the defen-
dant’s permission or without a privilege to so enter, the plain-
tiff is classified as a trespasser.
EXAMPLE: Demi wants to see the great view of the Missis-
sippi from the top of Blueberry Hill. Blueberry Hill is entirely
on Philip’s property. If Demi hikes up Blueberry Hill without
Philip’s permission, she is a trespasser.
(b) The trespasser category is further broken down into four
subcategories, as to which the standard of care may vary.
1) Unknown
a) A defendant has no duty of care as to a trespasser
whose presence is unknown to him. The defendant
also has no duty to inspect his land to attempt to
discover unknown trespassers.
2) Known
a) If the defendant becomes aware that a particular plain-
tiff has trespassed on his property or becomes aware
of facts from which he should reasonably conclude
that a plaintiff has trespassed, the plaintiff is regarded
as a known trespasser. The applicable standard of
care varies according to the category of danger.
b) Most jurisdictions require the defendant to exercise
reasonable care to protect a known trespasser from
injuries deriving from activities conducted on his land.
c) A defendant will be liable to a known trespasser for
failing to exercise reasonable care in warning them
of an artificial condition maintained on the premises
by the defendant if [Restatement (2d) of Torts § 337]:
i) the possessor knows or has reason to know of
their presence in dangerous proximity to the
condition; and
ii) the condition is of such a nature that the defen-
dant has reason to believe that the trespasser
will not discover it or realize the risk involved.
d) The defendant has no duty to protect a known
trespasser from injuries deriving from natural
conditions on his land.

484
TORTS

3) Frequent
a) If the defendant knows or reasonably should know
that trespassers frequently enter upon a portion
of his land, the standard may be higher than that
normally applicable to unknown trespassers. For
example, if the defendant observed that a beaten
path cuts across his property, he would be alerted
to the presence of frequent trespassers. Frequent
trespassers are owed the same duty of care owed
to known trespassers, even if the defendant is not
aware that a particular plaintiff is present on his land
or has ever previously trespassed upon his land.
4) Children
a) Where activities and natural conditions are involved,
the standard of care as to children who trespass is
the same as that for the applicable category of adult
trespassers. A heightened standard of care may apply
as to artificial conditions on the defendant’s land.
i) Horses, livestock, pets, and other owned
animals are “artificial conditions” for these
purposes insomuch as the owner placed them
where they are, if they are not naturally occur-
ring in the landscape. However, note that they
may not pose a foreseeable, unreasonable risk
of danger to children trespassers unless the
owner knows or reasonably should know of the
danger posted, and so the liability of the owner
will depend on the facts of the case.
b) If the heightened standard of care as to children tres-
passers is invoked, a defendant has a duty to exer-
cise reasonable care to prevent injury to the children,
applicable to dangerous artificial conditions. The
heightened standard of care set forth above arises if
four prerequisites are shown to exist:
i) the artificial condition is a foreseeable risk of
unreasonable danger to trespassing children;
a. An artificial condition on a defendant’s land
poses a foreseeable, unreasonable risk of
danger to children trespassers if a defendant
knows or reasonably should know of the exis-
tence and nature of the artificial condition. A
defendant has no independent duty to inspect
the land to discover such artificial conditions.
b. It is relevant to whether a condition poses a
foreseeably unreasonable danger to children

485
OUTLINE

trespassers that the danger is one which a


child might ordinarily be expected to recog-
nize. Thus, a child old enough to play without
immediate parental supervision can ordinarily
be expected to recognize the danger repre-
sented by water, fire, and falling.
ii) it is foreseeable that children are likely to tres-
pass where the artificial condition is located;
c. If a defendant has no reason to anticipate
that children are likely to trespass where
an artificial condition is located, the height-
ened standard of care is not triggered.
Foreseeability of the likelihood of children
trespassers may arise from a defendant’s
knowledge of past trespasses, proximity to
places where children are likely to be, acces-
sibility to the artificial condition, and other
relevant factors. Thus, if the artificial condition
is located near a park, playground, or street,
or is easily climbed, a defendant should
reasonably know that children are likely to
trespass there. It was from this aspect of
foreseeability that the traditional label attrac-
tive nuisance doctrine was derived.
iii) the child trespasser is unaware of the risk; and
d. This prerequisite is satisfied if the child
trespasser, because of his age or imma-
turity, did not discover the condition or
appreciate the danger it represented. If
the child trespasser is aware of the condi-
tion, understands the risk of danger it
poses, and is able to avoid that risk, the
defendant owes no heightened duty to
prevent injury to that child (i.e., the child is
treated as if he were an adult trespasser).
iv) the risk of danger of the artificial condition
outweighs its utility.
e. It is said that the utility of maintaining the
dangerous artificial condition must be
“slight” compared to the risk to trespassing
children in order for this prerequisite to be
satisfied. This is similar to the negligence
calculus (cost of precautions balanced
against probability and gravity of harm
considering social utility).

486
TORTS

EXAMPLE: Paloma is a six-year-old girl who frequently


trespasses on Donatella’s land. Donatella knows that
Paloma sometimes trespasses on her land. If there is
a dangerous dumpsite on Donatella’s land, Donatella
must make it safe. If there is an abandoned car that
is rusty and dangerous on Donatella’s land, Donatella
should remove it or cover it to prevent Paloma from
injuring herself. However, Donatella does not have to
build a fence around her pond, as natural bodies of
water are generally not included within the rule.
(4) Privileged Entrants
(a) Where there has been no express or implied permission
or invitation extended by a defendant, certain persons are
nevertheless privileged by law to enter onto the defendant’s
land. These include police officers or firefighters responding
to an emergency, census takers, or private persons exer-
cising a privilege (e.g., where unauthorized entry is neces-
sary to avoid a greater harm). Privileged entrants are
classified as either licensees or invitees, usually depending
on the purpose for which they entered.
(b) If the public purpose for which the privileged entrant
was acting has a connection with the defendant’s activi-
ties conducted on the premises, the privileged entrant is
regarded as an invitee for standard of care purposes. Courts
readily find that such a connection exists.
(c) Police and firefighters entering onto the premises have
traditionally been regarded by the courts as licensees. As set
forth above, a defendant has no duty to inspect and discover
dangerous conditions as to licensees.
(d) A private person entering onto a defendant’s land under
some legal privilege is regarded as a licensee.

STATUS DUTY OWED


Trespasser— No duty.
Undiscovered

Trespasser— Ordinary care—duty to warn of dangerous conditions


Known or that are known to possessor (exception: no duty to warn
Anticipated of obvious natural conditions of the land (e.g., lake)).

Ordinary care—duty to warn of dangerous conditions


Licensee
which are known to possessor.

Ordinary care—duty to (1) inspect premises and/or land;


Invitee
and (2) make safe for protection of invitees who enter.

487
OUTLINE

d. Plaintiffs Not on the Land (But Adjacent to It)


(1) A defendant must exercise reasonable care to prevent a plaintiff
not on his land from an injury deriving from the defendant’s activi-
ties or the activities of others conducted on his land.
(2) A defendant must exercise reasonable care to prevent a plain-
tiff not on his land from an injury deriving from unreasonably
dangerous artificial conditions that abut or protrude onto adjacent
land. A defendant must also exercise reasonable care to protect
passersby on a public street from injury deriving from dangerous
artificial conditions on his land.
(3) A majority of jurisdictions impose no duty on a defendant to
protect a plaintiff not on the defendant’s land from dangers
deriving from natural conditions on his land. The minority rule
is that the defendant must exercise reasonable care to prevent
injury to a plaintiff not on his land from dangers deriving from
natural conditions on his land.
(4) Many jurisdictions require that in urban areas, a defendant must exer-
cise reasonable care to protect a plaintiff passing by his land on adja-
cent public streets from injury deriving from native trees on his land.
(5) The defendant to whom the standard of care is applied is the person
in possession of the land, which includes the owner, a tenant, a
purchaser, or an adverse possessor. The standard of care may also
vary if the possessor of land is a landlord or seller of land.
(6) A substantial number of jurisdictions have eliminated the distinc-
tions between the various classes of persons entering the land,
and simply hold the landowner to a “reasonable under the
circumstances” test. In applying the test, the nature of the entry is
simply one factor in determining foreseeability and reasonability.
e. Landlords and Tenants
(1) The standard of care applicable to owners and occupiers of land
is invoked in connection with possession of land. Thus, the appro-
priate standard of care (determined as discussed above) applies
when a defendant is a tenant in possession of leased premises.
Areas retained in the landlord’s possession, such as common
areas in multiple housing units, remain the landlord’s responsibility.
Under certain circumstances, however, a landlord may be liable
to the tenant or to third persons for injuries they suffer in areas to
which the landlord has surrendered possession to the tenant.
(a) The nature and extent of the landlord’s duty to protect a tenant
in possession from harm arising from a dangerous condition on
the premises varies according to whether the defect was patent
or latent, or whether it arose after the transfer of possession.
1) Dangerous natural or artificial conditions that are or should
be reasonably apparent to the tenant upon transfer of
possession are called patent defects. The landlord is

488
TORTS

under no duty to warn of or repair such obvious conditions.


2) Dangerous natural or artificial conditions of which the
tenant is unaware and which are not reasonably apparent
to him, and of which the landlord is aware, are classified
as latent defects. The landlord has a duty to warn the
tenant of such dangers or repair them. The landlord has
no duty to inspect the premises for latent dangers.
3) The landlord has no duty to exercise reasonable care as
to dangerous conditions that arise after possession has
been transferred to the tenant, unless the landlord actu-
ally undertakes to repair such conditions or covenants
to repair them. This latter view is a modern minority
position, imposing liability where the tenant is injured
because the landlord failed to perform a contractual duty
to repair a dangerous condition of which the landlord
knew or should have known and as to which the land-
lord had a reasonable opportunity to repair.
(b) Under the traditional view, a landlord had no duty to protect
third persons from injuries arising from dangerous conditions
or activities on the leased premises. Today, many jurisdic-
tions treat third persons who would otherwise be classified as
licensees or invitees of the tenant as though they were tenants
themselves in assessing the tort liability of the landlord.
1) Even under the traditional view, if the landlord knew that
the tenant intended to open the leased premises to the
public, the landlord had a duty to prevent injury to members
of the public who came onto the premises. This standard
of care required the landlord to exercise reasonable care
to discover and repair any dangerous natural or artificial
conditions existing at the time of transfer of possession.
f. Sellers of Land
(1) Where a defendant transfers both possession and ownership
of land to another, there is generally no further duty to protect
anyone from injuries arising from the conditions of the land
or activities conducted thereon. Under limited circumstances,
however, the vendor-seller of land has a continuing duty even
after the transfer of possession and title.
(a) A defendant-seller has a duty to disclose to the purchaser of land
any hidden dangerous natural or artificial conditions of which the
seller knows or reasonably should know and which the seller
reasonably could anticipate the purchaser will not discover. The
duty to disclose hidden dangers continues until the purchaser has
had a reasonable chance to discover and remedy the dangers.
(b) The seller’s duty to disclose hidden dangerous conditions
applies to the purchaser and the purchaser’s invitees and

489
OUTLINE

licensees as to injuries occurring on the land, and to persons


not on the land if the dangerous conditions create an unrea-
sonable risk to such persons.
(c) Where the defendant-seller actively conceals the existence
of a dangerous condition, the duty to disclose continues
until the purchaser actually discovers the dangerous condi-
tion and remedies it (i.e., the seller may not argue that the
purchaser reasonably should have discovered the condition).
(d) In many jurisdictions, the builder of a residence has a tort
law duty to exercise reasonable care in construction so that
such builder remains liable for defects in its construction.

B. Standard of Care
1. Reasonably Prudent Person under the Same or Similar Circumstances
a. In general, a defendant breaches the duty to a plaintiff if he fails to conduct
himself as a reasonable person would in the same circumstances.
(1) Exceptions
(a) The reasonable person standard is applied as though the
reasonable person possessed the same physical character-
istics as the defendant. Thus, the trier of fact assesses what
conduct a reasonable person of the same height, weight,
ability to see or hear, and disabilities as the defendant would
have engaged in under the circumstances.
EXAMPLE: Del, blind since birth, is walking down the street
using a cane. He knocks into Pol, injuring him. In Pol’s negli-
gence action against Del, Del will be held to the standard of
care of a reasonably prudent blind person.

NOTE The reasonable person standard is not altered to account for a defendant’s physi-
cal disability if the disability is the result of the defendant’s voluntary intoxication.

(b) For purposes of analyzing a negligence action, the conduct


of defendants whose cognitive abilities are diminished due to
mental illness, mental disability, or intoxication is assessed
without such diminishment of abilities.
1) However, to the extent that a defendant possessed
greater knowledge or expertise than the average
person, the “reasonable” person to whom the defen-
dant is compared is thought of as also possessing that
greater knowledge or expertise.
EXAMPLE: Dennis has an IQ of 75. Dennis places a large
number of rocks in the back of his pickup truck but does not
tie them down. When he drives his truck on the highway, a
lot of rocks fall out of the back of the truck, causing a pile-up

490
TORTS

accident behind him. In a negligence analysis, Dennis’s ac-


tions will be compared with persons of average intelligence,
not those of other people with mental challenges.
(c) Where a defendant’s conduct occurs in an emergency situ-
ation, it is recognized that a reasonable person might accept
greater risk or have less opportunity for reflection in determining
the reasonable course of action. The defendant must act as a
reasonable person would behave in the emergency situation.
1) If a defendant’s own negligent conduct is responsible
for creating the emergency situation, the above principle
is not applied in assessing the reasonableness of the
defendant’s conduct during the emergency.
(2) In determining whether a defendant’s conduct constituted a breach
of duty under the “reasonable person” standard, Judge Learned
Hand offered the following analysis: if the burden (or cost) to the
defendant of taking precautions against the threatened risk was
outweighed by the likelihood (or probability) that the plaintiff would
be injured by the risk-producing activity, considering also the gravity
(or severity) of the injury to the plaintiff if the risk manifests itself, and
considering the social utility of the activity in which the defendant is
engaged, then the defendant failed to act as a reasonable person
would. In short, if the burden of taking precautions is less than the
probability of injury “times” the severity of that injury, plus the social
utility of the defendant’s activity, the defendant breached his duty of
care by not taking such precautions. Although legal problems cannot
be reduced to simple mathematical formulae, this is a useful way to
remember and to articulate the relevant considerations.
(a) The greater the benefit society derives, viewed in the aggre-
gate, from the activity engaged in by a defendant, the more
likely the defendant will not be found to have breached a
duty to a plaintiff merely by engaging in such an activity or by
not undertaking certain precautions.
EXAMPLE: Driving a car presents a fairly high probability of
colliding with others and a relatively severe danger of death
or great injury, which might be thought to justify significant
precautions, such as driving very slowly or avoiding driving
at all when there is great congestion or pedestrian activity.
However, the social utility of driving is also very great, so
that such precautions are considered to be “outweighed” in
the Learned Hand calculus, and a reasonable person would
not undertake them. Less burdensome precautions, such as
remaining alert and operating the vehicle competently, are
regarded as sufficient to fulfill the duty of care.
(b) The determination of whether a person has acted unreason-
ably is very fact-intensive. Use Hand’s formula as a guide in

491
OUTLINE

asking whether the probability of harm multiplied by the likely


magnitude of that harm outweighs the burden of avoidance.
(c) The reasonable person standard of care is an objective one.
In other words, the standard is applied as though the reason-
able person possessed the experience, knowledge, and
mental capabilities of an average member of the community,
even if the defendant did not himself actually possess it.
(d) In determining how a reasonable person would behave
under the circumstances, it is relevant how a person
engaging in the defendant’s activity would customarily
behave. However, evidence of custom is not conclusive;
the trier of fact may find either that the customary manner
of behavior was not reasonable under the circumstances or
that a reasonable person would have engaged in behavior
other than what is customary.
1) A plaintiff may submit evidence of a defendant’s devia-
tion from custom as evidence of the defendant’s unrea-
sonable conduct. Conversely, a defendant may put
on evidence of compliance with custom to show the
reasonableness of his conduct.
EXAMPLE: In trying to show that her landlord, Darv, was
negligent for failing to install unbreakable glass in her
shower, Penny may put on evidence that most landlords
use unbreakable glass. This evidence is persuasive, as it
suggests that the burden of using that glass is not too great.
2. Children
a. The reasonable person standard specifically takes account of age
when a defendant is a minor.
(1) In the majority of jurisdictions, a minor defendant’s conduct is
assessed according to what a reasonable child of the same age,
education, intelligence, and experience would have done.
(2) A minority of states follow the traditional rule, which divided
minors into three age levels:
(a) age six and below, as to which the defendant was conclu-
sively presumed incapable of being negligent;
(b) ages seven to 13, as to which it was rebuttably presumed
that the defendant was not negligent; and
(c) age 14 and up, as to which it was rebuttably presumed that
the defendant was capable of being negligent.
EXAMPLE: Darlene is seven years old and is very intelli-
gent. One day, she leaves her roller skates on the front walk.
Patrick, who is walking down the walk, does not see the
skates and slips and falls on them, breaking his leg. If Pat-
rick sues Darlene, her act of leaving the skates on the walk

492
TORTS

will be compared with that of a seven-year-old with similar


experience and intelligence.
b. Children engaging in adult activities (e.g., operating an automobile, boat,
or airplane), however, are required to conform to an adult standard of care.
EXAMPLE: Nine-year-old Dilbert takes his dad’s motorboat out for a spin.
While driving the boat across the bay, he hits Patricia, a swimmer, and
knocks her unconscious. Patricia drowns. If Patricia’s estate sues Dilbert,
he will not be compared with other children’s standard of care. Instead, he
will be held to the objective, adult reasonable person’s standard of care.
3. Statutory
a. A statute that provides for civil liability supersedes the common law of
torts. A defendant’s civil liability will be determined by the specific statu-
tory provisions. Such civil statutes are rare.
EXAMPLE: A legislature could enact a statute stating that anyone
injured in a car accident who was not wearing a seatbelt is barred from
pursuing a negligence action.
b. Where a defendant’s conduct also violates a statute that does not
provide for civil liability (usually a criminal statute), the statute may
establish the standard of conduct for breach of duty purposes. In a
majority of jurisdictions, this means that an unexcused violation conclu-
sively establishes that the defendant breached his duty to the plaintiff
(often referred to as negligence per se).
(1) Other (minority) jurisdictions regard a qualifying violation of
statute by a defendant as either raising a rebuttable presump-
tion (so that the plaintiff wins unless the defendant introduces
enough evidence to overcome the presumption) or as prima facie
evidence (so that the plaintiff wins unless the defendant intro-
duces any evidence, but the plaintiff retains the burden of proof
if the defendant offers evidence) that the defendant’s conduct
breached the duty of care owed to the plaintiff.
c. In a jurisdiction that has adopted a negligence per se rule, a defendant’s
violation of a criminal statute has the effect of establishing the standard
of care only when all of the following three conditions are present:
(1) the injury caused by the defendant’s conduct is the type that the
statute was intended to prevent;
(2) the plaintiff is a member of the class intended to be protected by
the statute; and
(3) the defendant’s violation of the statute is not excused.
d. A defendant’s violation of an applicable statute is excused if compli-
ance with the statute:
(1) would have resulted in a harm greater than the harm produced by
the violation; or
(2) would have been impossible.

493
OUTLINE

e. A few statutes are regarded as so important that their violation cannot


be excused as a matter of law (e.g., the statutory obligation to maintain
a vehicle’s brakes in proper working order).
EXAMPLE: To prevent children from getting poisoned, the legislature pass-
es a law making it a crime to sell toxic substances without a child-proof cap.
DunCo fails to put a child-proof cap on its rat poison. Four-year-old Pablum
finds the rat poison, drinks it, and suffers harm. Even though the statute
is criminal in nature, in most states it would become the standard of care
because it was designed to protect children from ingesting poison. Breach
would be the violation of that statute—i.e., not putting a child-proof cap on
the poison. The plaintiff must still prove the other elements of negligence.
f. Compliance with a statute is regarded as mere evidence on the issue
of whether a defendant breached his duty of care, and does not raise
any presumption in the defendant’s favor.
4. Professionals
a. Historically, defendants who engaged in certain activities were held to
a higher standard of care to the public or to their customers.
(1) Common carriers (e.g., trains, bus lines, etc.) were traditionally said
to have a duty to avoid harm to their passengers through exercise of
“the highest degree of vigilance, care and precaution,” or “the utmost
caution characteristic of very careful, prudent persons.”
(2) Some jurisdictions utilize the same standard of care for assessing
the conduct of innkeepers toward their customers as that applied
to a common carrier regarding its passengers.
(3) The dangerous products or services dispensed by public utilities (e.g.,
electricity, and gas) caused many jurisdictions to impose a duty of care
which required that such utilities “take every reasonable precaution
suggested by experience or prudence” to avoid injury to the public.
b. Professionals are treated differently from other defendants in negligence.
For most defendants, custom is only evidence related to providing
breach of duty. For professionals, such as doctors, lawyers, and accoun-
tants, custom is everything. The customary practice of professionals
in good standing sets the standard of care. If the defendant deviates
from that custom, he has breached his duty; if he has complied with that
custom, he cannot be found to have breached his duty.
EXAMPLE: Dr. Dell uses a certain procedure to remove Penn’s tonsils, and
in so doing causes Penn harm. In Penn’s malpractice action against Dr. Dell,
if it is established that Dr. Dell performed as other doctors in good standing
would have performed, she is free from liability. Conversely, if the evidence
is that Dr. Dell deviated from the customary way tonsils are removed, even
if she did so in good faith, she will have committed malpractice.
(1) The effect of this rule is that a defendant will be found to have
breached the standard of care if he did not conduct himself as
would a competent member of the profession with minimally

494
TORTS

adequate knowledge and expertise. If the defendant is a


specialist within a profession, the standard of care is applied
according to the standards appropriate to that specialty.
(a) Where there are several accepted ways of acting, the defen-
dant’s compliance with any of these accepted schools of
thought protects him from liability.
(2) Traditionally, the conduct of a defendant with specialized skill
or expertise was assessed for reasonable person/breach of
duty purposes with regard to the community in which the defen-
dant practiced his profession. Some jurisdictions apply a more
modern, national standard, recognizing that technology permits
communication and exchange of knowledge that transcends
traditional geographic limitations. Specialists are particularly likely
to be held to a national standard.
(3) While expert testimony is almost always needed to establish the
requisite standard of care for that professional and to establish
breach of that standard, no expert testimony is required when
the subject matter is within the common knowledge of untrained
persons. This is known as the “common knowledge exception.”
(a) This occurs when the failure was so egregious, so obvious,
and so flagrant that no one could miss it. The judge deter-
mines when no expert testimony would be required.
c. Traditionally, a physician was liable for the intentional tort of battery if
he failed to properly inform a plaintiff-patient about the risks and alter-
natives of a proposed medical procedure or treatment; the plaintiff’s
consent was said to be negated by the lack of disclosure. This is still
the case where there is a gross deviation from consent, such as where
a patient agrees to a tonsillectomy, but her leg is amputated instead.
Most jurisdictions now treat nondisclosure as a form of malpractice.
(1) In some jurisdictions, malpractice based on the failure to disclose
is treated like any other kind of medical malpractice. The physi-
cian-defendant must disclose risks that doctors in good standing
customarily divulge. The failure to do so is malpractice.
(2) The trend, based on recognition of patient autonomy, has been to
require physicians to divulge all material risks—that is, risks that
a reasonable patient would want to know in deciding whether to
undergo a specific procedure. The failure to divulge a material
risk is malpractice provided the patient can show that he would
have refused the procedure had the risk been divulged.

C. Breach of Duty
1. Res Ipsa Loquitur
a. A plaintiff generally meets his burden of proving breach of duty by estab-
lishing that the defendant’s conduct fails to conform to the applicable
standard of care. This is rendered difficult or impossible where a plaintiff

495
OUTLINE

does not know and cannot effectively determine specifically what a


defendant’s injurious conduct was. The doctrine of res ipsa loquitur (“the
thing speaks for itself”) helps the plaintiff in such situations.
(1) In the majority of jurisdictions, if a plaintiff makes a qualifying
showing of res ipsa loquitur, he has produced evidence suffi-
cient, if believed by the jury, to support a finding that a defen-
dant breached his duty. Res ipsa loquitur is thus merely a
means of adducing evidence of breach of duty to satisfy the
burdens of producing evidence and persuasion. In most jurisdic-
tions, res ipsa loquitur permits the jury to infer the defendant’s
breach of duty.
(2) A small minority of jurisdictions regard a qualifying showing of
res ipsa loquitur as shifting the burden of producing evidence to
the defendant as to the issue of breach of duty. The defendant
must then produce sufficient evidence to support a verdict in
his favor, or the court will instruct the jury that breach of duty is
established. If the defendant produces qualifying evidence, the
burden of persuasion remains on the plaintiff, and the jury may
find for the defendant if the plaintiff has not established breach
of duty by a preponderance of the evidence.
(3) Some jurisdictions—another small minority—give a qualifying
showing of res ipsa loquitur the effect of shifting the burden of
persuasion to the defendant. If the defendant does not then
persuade the jury, by a preponderance of the evidence, that he did
meet the applicable standard of care, breach of duty is established.
b. A plaintiff must establish that three requisites are present in order to
invoke the doctrine of res ipsa loquitur:
(1) the event that caused the plaintiff’s injury was one which would
not ordinarily occur in the absence of negligence;
(a) The trier of fact may determine whether an event is one that
would not ordinarily occur in the absence of negligence with
reference to common experience, or if that is not sufficient,
by resort to evidence provided by the plaintiff. Expert testi-
mony may be required to establish that negligence is the
likely cause of the injury-causing event. The plaintiff is not
required to show that negligence is the only possible cause
of the injury-causing event; it is enough if the trier of fact
finds that negligence was more likely than not the cause.
(2) it is more likely than not that it was the defendant’s negligence
that was responsible for the injury-causing event; and
(a) Traditionally, a plaintiff was required to show that the defen-
dant was in exclusive control of the instrumentality that
inflicted injury upon the plaintiff. Today, this requirement has
become less rigid in many states. In some states, for instance,
it is sufficient if a plaintiff shows that it is more likely than not

496
TORTS

that the defendant is responsible for the negligent event.


Control, either exclusive or not, is merely a circumstance
relevant to determination of the defendant’s responsibility.
(3) the plaintiff was not responsible for the event that caused injury.
(a) A plaintiff must demonstrate that he did not set in motion the
forces that resulted in his injury. However, it is immaterial that
the plaintiff placed himself in a situation of peril or that he did
not take precautions to avoid injury. In light of the current move
to comparative fault, this element has become less important.

D. Cause-in-Fact (Actual Cause)


1. The element of cause-in-fact ties the defendant’s breach of duty to the
plaintiff’s injury. Without proof of this element by a preponderance of the
evidence, the plaintiff loses his negligence claim.
EXAMPLE: Dorkas fails to use his headlights after dusk, as required by
statute, and hits Petunia when she comes running out between two parked
cars. Even though Dorkas has breached the duty he owes to Petunia, she
would lose her negligence claim if the jury determines that he still would
have hit Petunia even if his lights had been on.
2. “But-For” Test
a. A defendant’s conduct was the cause-in-fact of an event if that event
would not have occurred but for the existence of the conduct.
EXAMPLE: City negligently mixes its drinking-water line with its
sewage-water line. Pyn, who drinks City-provided water, contracts
typhoid. Pyn may recover if he can show that it is more likely than
not that “but for” the negligent mixing of the water lines, he would not
have contracted typhoid.
3. “Substantial Factor” Test
a. A defendant’s conduct is also the cause-in-fact of a plaintiff’s injury if
that conduct was a substantial factor in bringing about the injury. This
accounts for the situation where the conduct of two or more defendants
results in injury to the plaintiff, and each individual defendant’s conduct,
taken alone, would have been sufficient to directly cause the injury. A
mechanical application of the “but-for” test would permit each defen-
dant to claim that the plaintiff’s injury would have occurred whether he
acted or not and thus “but for” causation did not exist.
EXAMPLE: Kramer negligently sets a fire that would have burned
down Jerry’s house on its own. Separately, Elaine negligently set a fire
that also would have destroyed Jerry’s house. The fires combine and
burn down Jerry’s house. Each fire is a substantial factor of the harm
and both Kramer and Elaine are the cause-in-fact of the harm. Note
that the “but-for” test doesn’t work where there are multiple causes,
either of which would have brought about the harm.

497
OUTLINE

b. Where there are multiple negligent parties, each of whom contributes


to the plaintiff’s indivisible harm, they are jointly and severally liable.
That means that the plaintiff may recover fully against any of the defen-
dants, and those defendants can sue each other for contribution.
EXAMPLE: If Jerry’s torched house is worth $1 million, he could sue just
Elaine to recover that amount. Elaine could then sue Kramer, a joint tortfea-
sor, for contribution (partial repayment of that judgment). Note that Jerry
cannot recover more than the total amount of his damages—$1 million.
4. Alternative Liability Theory
a. Where a plaintiff’s injury arises from the negligent conduct of two or
more independently acting defendants, only one of whom can actually
be responsible, and the plaintiff is unable to establish which defendant
is in fact responsible, each and every defendant’s conduct is regarded
as a cause-in-fact of the injury unless a defendant can prove that he
did not cause the plaintiff’s injury. In effect, the burden of proof as to
cause-in-fact is shifted from the plaintiff to the defendant [Summers v.
Tice, 199 P.2d 1 (Cal. 1948)].
EXAMPLE: Wyn negligently shoots in the direction of a rustling bush
at the same moment that Fahn does. One bullet hits Pantaloon, who
cannot identify which gun the bullet came from. Because both Wyn and
Fahn breached a duty owed to Pantaloon, and because both are be-
fore the court, the court will shift the burden of proof on the element of
cause-in-fact. If Wyn and Fahn cannot prove they were not the respon-
sible party, they will be jointly and severally liable.
5. Where a plaintiff demonstrates that the injury resulted from the conduct of
several defendants, each acting independently, but cannot identify which
particular defendant’s conduct actually injured him, each defendant’s
conduct may be regarded as a cause-in-fact of the plaintiff’s injury. Liability
is apportioned among the defendants based upon the economic benefit,
as measured by percentage or market share, each defendant derived from
the risk-producing conduct. Market share liability applies only to generic
products such as DES.
6. As a general matter, like the other prima facie elements of negligence, the
plaintiff must prove cause-in-fact by a preponderance of the evidence (i.e.,
by more than 50%). This has led to the result that where a doctor commits
malpractice on a patient who, due to illness, had a likelihood of death, the
doctor escapes liability for his malpractice. A growing number of jurisdictions
allow the plaintiff to proceed by recharacterizing the injury as “loss of chance.”
EXAMPLE: Dr. Dyl commits malpractice and fails to diagnose cancer in his
patient, Prax. Had Dr. Dyl done so, Prax would have had a 40% chance of
survival. By the time the cancer is found, the cancer is incurable. Under the
traditional cause-in-fact rule, Prax loses his malpractice case because he
cannot show that it is more likely than not that, but for Dr. Dyl’s malpractice,
he would have survived. however, some jurisdictions would allow the case

498
TORTS

to proceed, finding that it is more likely than not that, but for Dr. Dyl’s mal-
practice, Prax would not have lost his 40% chance of survival.
a. Statutes in many jurisdictions reduce the duty of care owed by the
driver of an automobile to a guest (a non-paying passenger) in that
auto. Such statutes frequently provide that the driver of an auto is not
liable for injuries suffered by the guest unless the driver’s wrongful
conduct amounted to gross negligence or recklessness.
b. Slip-and-Fall Cases
(1) In cases where a person is injured because of a fall on the defen-
dant’s premises, the plaintiff must show evidence from which a
jury may reasonably infer unreasonable conduct on the part of the
defendant. Typically, the plaintiff must put on evidence about the
condition of the item on which he fell so that a jury may infer that
the object was there long enough that the defendant was unrea-
sonable in not discovering and remedying the dangerous condition.
EXAMPLE: Pashanda is injured when she slips on a grape on
the floor in the produce section of Dexto’s market. If this is all
the evidence she presents, Pashanda’s negligence claim will be
thrown out. If she can show, however, that the grape on which
she slipped was blackened and gritty, she will proceed to a jury
because there is circumstantial evidence that would permit the
jury to find that the grape had been on the floor long enough that
Dexto’s should have discovered it.

E. Proximate (Legal) Cause


1. In addition to being a cause-in-fact of a plaintiff’s harm, the defendant’s
conduct must also be a proximate, or legal, cause of the injury.
a. A defendant owes a duty of reasonable care only to foreseeable plain-
tiffs (i.e., those individuals who are within the risk of harm created by
the defendant’s unreasonable conduct). The majority view holds that
a defendant only owes a duty of care to foreseeable plaintiffs who are
within the zone of danger (i.e., under the circumstances, a reason-
able defendant would have foreseen a risk of harm to the plaintiff). The
broader minority view allows recovery to any person thereby harmed
due to a breach of the defendant’s duty of care.
2. Not all injuries “actually” caused by the defendant will be deemed to have been
proximately caused by his acts. As such, the doctrine of proximate or legal cause
deals with a limitation of liability with respect to persons and consequences
that bear some reasonable relationship to a defendant’s tortious conduct.
a. Unforeseeable Extent of Harm
(1) If the defendant’s conduct is a substantial factor in bringing about
the harm to another, the mere fact that the defendant neither
foresaw nor should have foreseen the extent of the harm does
not prevent him from being liable.

499
OUTLINE

(2) Under the so-called thin-skulled or eggshell plaintiff rule,


a defendant is liable for the full consequences of a plaintiff’s
injury, even though, due to the plaintiff’s peculiar susceptibility
to harm (of which the defendant was unaware), those conse-
quences were more severe than they would have been in a
normal person.
EXAMPLE: Peyton is a hemophiliac. Derek leaves a bunch of
paper and boxes lying around in the hall just outside his cubicle.
When Peyton walks by, he falls and hits his head on the side of
Derek’s desk. He begins to bleed heavily. Eventually, he loses so
much blood that he must be hospitalized. Derek will be liable to
Peyton for the full extent of Peyton’s injuries, even if he did not
know about Peyton’s inability to clot blood.
b. Unforeseeable Type or Manner of Harm
(1) Superseding Cause
(a) A superseding cause is an unforeseeable, intervening
cause that breaks the chain of causation between the initial
wrongful act and the ultimate injury, and thus relieves the
original tortfeasor of any further liability.
(b) Commonly occurring examples of superseding causes:
1) naturally occurring phenomena;
2) criminal acts of third persons;
3) intentional torts of third persons; or
4) extraordinary forms of negligent conduct.
(c) An intervening force is one that actively operates in
producing harm to another after the actor has already
committed his negligent act or omission.
1) As a general rule, a defendant will be held liable for
harm caused by foreseeable intervening forces.
2) Because rescuers are foreseeable, the original
tortfeasors will be held liable for the ordinary
negligence of the rescuer.
EXAMPLE: Don is throwing balls around at a play-
ground. Don hits Poppy in the head, causing her to
lose consciousness. Casey arrives and tries to re-
suscitate Poppy, breaking Poppy’s ribs while doing
compressions. Because Casey was merely negligent,
Don will be liable for both Poppy’s head injury and
broken ribs.
3) The original tortfeasor is usually held liable for the ordinary
negligence of the plaintiff’s treating physician or nurse.
EXAMPLE: When Poppy is hit in the head, Casey does
not come to her aid. Instead, an ambulance takes her to

500
TORTS

the hospital. The triage nurse does not examine Poppy


carefully and concludes that she does not need medical
attention. The nurse keeps Poppy lying on a gurney
in the hallway for eight hours. During that time, Poppy
has a cerebral hemorrhage and sustains irreversible
brain damage. Don will be liable for Poppy’s brain
hemorrhage and the strike on the head.
4) The original tortfeasor is usually held liable for
diseases contracted or subsequent injuries sustained
because of the impairment of the plaintiff’s health
resulting from the original injury caused by the defen-
dant’s tortious conduct.
EXAMPLE: When Don hits Poppy in the head, her sinus-
es are affected. As a result, Poppy becomes much more
susceptible to sinus infections. She must go to the doctor
frequently and take expensive medications. Don will be
liable for both the head injury and the sinus infections.
5) A defendant will be held liable for negligent efforts on
the part of persons to protect life or property interests
endangered by his negligence.
6) In situations where the plaintiff suffers a subsequent
injury after the original injury, and the original injury was
a substantial factor in causing the second accident, the
original tortfeasor is held liable for damages.
EXAMPLE: Doug negligently fractures Prudence’s
left leg. while walking on crutches, Prudence trips and
falls, breaking her right leg. Doug will be liable for
both leg injuries.
(d) Considerations of importance in determining whether an inter-
vening force is a superseding cause of harm to another include:
1) the fact that its intervention brings about a harm
different in kind from that which would otherwise have
resulted from the defendant’s negligence;
2) the fact that its operation or the consequences thereof
appear to be extraordinary and unforeseeable after
the event; and
3) the fact that the intervening force is operating independently
of any situation created by the defendant’s negligence.
3. Another highly tested area of proximate causation deals with rescuers. A negligent
defendant owes an independent duty of care to a rescuer. Even where the rescue
efforts are done negligently (but provided they are not wanton), the negligent
defendant will be liable for both personal injury and property damage, whether
the rescuer succeeds in injuring himself, the person rescued, or a stranger.

501
OUTLINE

UNFORESEEABLE
FORESEEABLE
(I.E., SUPERSEDING)

Chain of proximate causation unbroken— Chain of proximate causation broken—


Effect original defendant remains liable. original defendant’s liability cut off for
consequences of antecedent conduct.

• Subsequent medical malpractice, • Criminal acts and intentional torts


including aggravation of plaintiff’s of third parties, but only where they
condition. are unforeseeable under the facts
• Subsequent disease or accident, or circumstances.
Typical including all illnesses and injuries • Highly extraordinary harm arising
Examples resulting from plaintiff’s weakened from defendant’s conduct, as viewed
condition, but not deadly, rare diseases. by the court, including grossly
• Negligent rescue efforts. negligent conduct of third parties.
• Unforeseeable, naturally occurring
phenomena.

F. Damages
1. A plaintiff must affirmatively prove actual damages. Nominal damages are
not available, and punitive damages generally are not allowed.
2. Personal injury and property damages are recoverable. Included are
general and special damages, past and future pain and suffering, medical
expenses, lost wages, and loss of consortium, but not attorney’s fees. The
plaintiff’s duty to mitigate damages applies.
3. Payments made to or benefits conferred on the injured party-plaintiff from
other (i.e., collateral) sources are not credited against the tortfeasor’s
liability, even where they cover all or a part of the harm for which the tort-
feasor is liable. This is known as the collateral source rule.
a. The rule that collateral benefits are not subtracted from the plaintiff’s
recovery applies to the following types of benefits:
(1) insurance policies, whether maintained by the plaintiff or a third party;
(2) employment benefits;
(3) gratuities, including cash gratuities and the rendering of
services; and
(4) social legislative benefits.
EXAMPLE: Della negligently injures Pasha when she sets off
firecrackers incorrectly. Pasha has second-degree burns over
most of her body. If Pasha collects disability or health insurance
benefits, these will not be subtracted from the amount of dam-
ages that Della owes Pasha.
EXCEPTION: Payments made by a tortfeasor or by a person
acting for him (e.g., the defendant’s insurance company) to the
injured plaintiff are credited against the defendant’s tort liability.

502
TORTS

4. Punitive Damages
a. Punitive damages are an amount over and above the compensation
needed to make the plaintiff whole. They are intended to punish the defen-
dant for egregious conduct and to act as a deterrent against future conduct
by the defendant or others, and are thus also called “exemplary” damages.
b. Punitive damages are available in tort cases when there has been
willful, wanton, or malicious conduct. They are generally not available
in negligence actions or in contract actions.
c. A plaintiff is never entitled to an award of punitive damages, but a jury
may award them at its discretion. Jury rewards may be reversed or
overturned if excessive. Due process requires that punitive damage
awards not be grossly excessive on three measures [State Farm Mut.
Auto. Ins. Co. v. Campbell, 538 U.S. 408 (2003); BMW of No. America
v. Gore, 517 U.S. 559 (1996)]:
(1) the degree of reprehensibility of the defendant’s conduct, consid-
ering such factors as whether:
(a) the defendant acted intentionally, maliciously, or with reck-
less disregard for harm;
(b) such conduct was repeated or isolated; and
(c) the harm caused to the plaintiff was economic or noneco-
nomic (e.g., lost profits or personal injury);
(2) the ratio between the plaintiff’s compensatory damages and the
amount of the punitive damages (presumptively, punitive damages
should not exceed 10 times the compensatory damage award); and
(3) the difference between the punitive damage award and the civil or
criminal sanctions that could be imposed for comparable misconduct.
d. There may also be specific limitations on entitlement to punitive
damages under state law.

G. Defenses to Negligence
1. Contributory Negligence
a. Tort law requires a plaintiff to exercise due care to protect himself from
injury by the defendant. Thus, the plaintiff’s own negligence—called
contributory negligence at common law—may bar the plaintiff’s
recovery. The analysis is similar to that for the defendant’s negligence—
whether the plaintiff acted as a reasonable person would under the
same circumstances. Contributory negligence is conduct on the part of
the plaintiff that falls below the standard to which he should conform for
his own protection, and which is a legally contributing cause cooperating
with the defendant’s negligence in bringing about the plaintiff’s harm.
b. To show contributory negligence, the defendant must prove by a
preponderance of the evidence that the plaintiff fell below the relevant
standard of care and that this failure was the cause-in-fact and proxi-
mate cause of the plaintiff’s damages.

503
OUTLINE

(1) Traditionally, if a plaintiff is found to have engaged in contributory


negligence, he is barred from recovery for a defendant’s other-
wise negligent conduct, even if the plaintiff is only 1% at fault.
EXAMPLE: Delbert, who is speeding, hits Pol’s car one night. Pol
will be barred from recovery because he is contributorily negligent
per se if he was hit because, in violation of the statute, Pol did not
have his headlights on.
(2) Contributory negligence does not bar recovery if the plaintiff’s
theory is intentional tort, recklessness, or strict liability.
c. Risks or dangerous acts undertaken by a plaintiff that might otherwise
be regarded as contributory negligence are often viewed as reason-
able in an emergency (i.e., the plaintiff is threatened with immediate
danger of bodily harm or death because of the defendant’s negligence)
or if the plaintiff is attempting to rescue someone.
d. The analysis of whether a child plaintiff is contributorily negligent is
similar to the negligence analysis for children generally.
(1) A majority of jurisdictions apply the “reasonable child” standard—
whether a child of the same age, education, intelligence, and experi-
ence, in the same circumstances, would have acted as the plaintiff did.
(2) A minority of states utilize the traditional age categories:
(a) below age six, contributory negligence is precluded as a
matter of law;
(b) ages seven to 13, the child-plaintiff is rebuttably presumed
incapable of contributory negligence; and
(c) 14 and older, the child-plaintiff is rebuttably presumed
capable of contributory negligence.
e. The concept of negligence per se is applicable to the plaintiff in a
contributory negligence situation.
(1) Contributory negligence is not available, even though the plaintiff’s
conduct violates a statute, where the defendant’s negligent act
violates the same statute, the plaintiff is a member of the class of
persons the statute was intended to protect, and the harm suffered
by the plaintiff is the type the statute was intended to avoid.
EXAMPLE: 15-year-old Pasha, who was hired by DunCo in viola-
tion of a statute prohibiting the hiring of children, will not be found
to be contributorily negligent.
f. In order to invoke contributory negligence, a defendant must prove that
the plaintiff’s conduct is the actual and proximate cause of the injuries
suffered by the plaintiff.
g. In two limited situations, the contributory negligence of a third person may
be imputed to the plaintiff so as to bar recovery against the defendant:
(1) the plaintiff and the contributorily negligent third person are suffi-
ciently related such that the plaintiff would be vicariously liable

504
TORTS

for the third person’s negligence (e.g., partners and joint enter-
prisers, employers and employees); and
(2) the plaintiff’s claim against the defendant is completely deriva-
tive of the third person who was contributorily negligent (e.g.,
wrongful death, loss of consortium).
(a) A minority of states do not impute one spouse’s contributory
negligence to another in an action against a defendant for
loss of consortium.
h. The last clear chance doctrine provides a basis for recovery even
where a plaintiff is otherwise contributorily negligent. The focus is on
the time period after the plaintiff has engaged in contributory negli-
gence. If injury to the plaintiff could still have been avoided through a
subsequent exercise of due care by the defendant, then the defendant
is said to have had the last clear chance to avoid harm, and the plain-
tiff’s contributory fault will not bar recovery.
(1) If the defendant is aware that his breach of duty has placed the plain-
tiff in danger, the last clear chance doctrine is always available to the
plaintiff. The majority of jurisdictions deny a plaintiff the benefit of the last
clear chance doctrine if the defendant is not aware that the plaintiff is in
danger, and the plaintiff is in “helpless peril” rather than “inattentive peril.”
(a) A plaintiff is in helpless peril when his negligence has placed
him in a position of danger from which he cannot extricate
himself. A plaintiff is in inattentive peril when his contributory
negligence has placed him in a position of danger from which he
could escape if observant enough to recognize his peril.
(b) In a majority of jurisdictions, if a defendant is under a duty
to discover danger to the plaintiff (e.g., an owner/occupier
of land has duty to exercise due care to discover dangerous
conditions and protect invitees from them), no distinction is
made between helpless and inattentive danger.
2. Comparative Negligence
a. Under a modern comparative negligence regime, where a plaintiff’s
negligence has contributed to his own injuries, the total damages
caused by the defendant may be apportioned based upon a determina-
tion of the relative fault of each party. Virtually every state has adopted
a comparative negligence system, either by statute or judicial decision.
The analysis of comparative fault is identical to the analysis of contribu-
tory negligence, though the effect is different.
(1) Some states have adopted a pure comparative negligence
scheme. In such states, apportionment of damages tracks appor-
tionment of fault perfectly—if a defendant is 25% responsible and
a plaintiff is 75% responsible, plaintiff recovers from the defen-
dant 25% of the total damages he suffered.
(2) Other states have adopted a partial comparative negligence system.
Here, damages are apportioned only if the defendant’s responsibility

505
OUTLINE

exceeds the plaintiff’s responsibility. The plaintiff is denied any


recovery if he is responsible for 50% or more of his own damages.
(3) If there are two or more defendants who are not jointly liable, two
different systems of comparative negligence have been devel-
oped—the “aggregate” system and the “individual equality” system.
(a) In an aggregate system, the plaintiff does not recover anything
if his responsibility exceeds the total percentage responsibility
of all the defendants combined. Thus, if two defendants each
contributed 24% of a plaintiff’s damages, totaling 48%, and the
plaintiff contributed 52%, he would not recover.
(b) Under an individual equality system, a plaintiff does not
recover anything if his responsibility exceeds that of any
single defendant. Thus, if two defendants each contributed
33% of a plaintiff’s damages, and the plaintiff contributed
34%, the plaintiff would be barred from recovery.

EXAM TIP On the MBE, assume that a pure comparative negligence system applies un-
less the question states otherwise.

3. Assumption of Risk
a. A plaintiff “assumes the risk” of injury from a defendant’s negligence if
the plaintiff expressly or impliedly consents to undergo the risk created
by the defendant’s conduct.
(1) A plaintiff is barred from negligence recovery when he has, by
written or oral words, expressly relieved the defendant of his
obligation to act non-negligently toward the plaintiff. As long as
the waiver is not void as against public policy and the language is
clear, the waiver will be enforced by most courts.
(2) A plaintiff may also impliedly assume the risk of a defendant’s conduct.
(a) A plaintiff is barred from recovery, or recovery will be
reduced, under the assumption of the risk doctrine if the
defendant establishes that:
1) the plaintiff had knowledge of and appreciated the
nature of the danger involved;
a) This is a subjective standard. Youth, lack of infor-
mation, or lack of experience may justify a finding
that a plaintiff actually failed to comprehend the
risk involved. If so, there can be no assumption of
the risk, even if a “reasonable plaintiff” would have
recognized the danger under similar circumstances.
2) the plaintiff appreciated the specific danger that injured
him; and
EXAMPLE: A plaintiff who knowingly agrees to ride in a
car where the driver is speeding does not assume the
risk that the driver might also be intoxicated.

506
TORTS

a) Some risks are regarded as so obvious that any


competent adult is expected to be aware of them.
EXAMPLE: The danger of falling through an un-
guarded opening, slipping on ice, or being struck by
a baseball at a baseball game.
3) the plaintiff voluntarily chose to subject himself to that danger.
a) Voluntariness can be established by express
consent (e.g., an exculpatory clause in a contract
providing that one party will hold the other party
harmless for injuries caused by the first party), or
consent may be implied from the fact that the plain-
tiff continued in the face of a known danger. With
regard to implied assumption of the risk, courts
closely examine whether behavior is voluntary
when there is a lack of alternatives.
(b) In some circumstances, a court will determine that a defen-
dant has no obligation to be non-negligent toward the plaintiff
because of the nature of the activity that they are engaged
in. The most common context is sports.
(c) Assumption of the risk may not be asserted where a plaintiff
is a member of a class intended to be protected by a statute,
and the defendant’s conduct both violates the statute and
threatens the risk the statute was designed to prevent. In
some states, any risk-creating violation by a defendant of a
public safety statute has the above effect.
EXAMPLE: Petunia and Drucilla are playing basketball. Pe-
tunia’s eye is severely injured when Drucilla pokes Petunia in
the face trying to get a rebound. Even if Drucilla was negli-
gent, many courts would conclude that the harm that befell
Petunia was an inherent risk of the game, and thus Drucilla
had no duty to avoid such harm. However, if a court found
Drucilla’s conduct to be more than negligent, Petunia would
be permitted to proceed with her tort claim.

507
OUTLINE

III. STRICT LIABILITY

A. In General
1. In strict liability, a defendant is liable for injuring a plaintiff whether or not the
defendant exercised due care. As to certain activities, the policy of the law is
to impose liability regardless of how carefully a defendant conducted himself.

B. Categories
1. A defendant may be held strictly liable as to two categories of activities:
a. possession of animals; and
(1) The analysis of strict liability in connection with the possession of
animals varies according to the nature of the plaintiff’s injury and
whether the animal is wild or domestic.
(2) A defendant can be held strictly liable for personal injuries
inflicted by his animal if it has “known dangerous propensities.”
(a) This is, in part, a “scienter-like” element, in that the defen-
dant is subject to strict liability only if he knew or had reason
to know of the dangerous quality of the animal.
1) Wild animals generally have “known dangerous
propensities” for this purpose. A wild animal is one not
customarily devoted to the service of humankind at the
time and in the place where it is kept.
2) Domestic animals have “known dangerous propensities”
only if a reasonable owner would realize that the animal
presented a danger of death or injury. The classic
example of a domestic animal with known dangerous
propensities is a dog that has previously bitten a human
being (the “one bite” rule—after one bite, the owner is
presumed to know that the dog is dangerous).
a) Strict liability does not apply to possession of domestic
animals as to which dangerous propensities are
normal, such as bulls, stallions, mules, rams, and bees.
EXAMPLE: Parker is seriously injured when bitten
by Damien’s pet tiger. Damien is strictly liable for this
injury even if Damien’s tiger has been as gentle as a
kitten until that moment. If Parker had been bitten by
Damien’s cat, Damien is only strictly liable once Damien
should have known of his cat’s dangerous propensity.
(b) If the plaintiff is an unknown trespasser, most jurisdictions do
not impose any liability for injuries inflicted by a defendant’s
animals while the plaintiff is on the defendant’s land, even as
to animals with known dangerous propensities. If the plaintiff
is any other type of trespasser (known, frequent, or child),

508
TORTS

a defendant is liable only for negligence. The plaintiff must


establish that the defendant failed to exercise due care to
warn of or protect the trespasser from his animal.
(3) A defendant is held strictly liable for any trespass to the land or chat-
tels of a plaintiff by wild animals if of a kind likely to escape, trespass,
and do damage, or by “livestock” possessed by the defendant.
Domestic pets (e.g., dogs and cats) are not considered livestock.
b. abnormally dangerous activities.
(1) A judge determines whether an activity is abnormally dangerous
by considering if:
(a) the activity creates a risk of serious injury as to the plaintiff,
his land, or his chattels;
(b) this risk cannot be eliminated by the exercise of due care; and
(c) the activity is not usually conducted in that area. Such things
as dynamiting, crop-dusting, and exterminating have been
found to be abnormally dangerous.
(2) Principles of causation generally applicable to negligence are
also applied in a strict liability analysis.
(3) In order for strict liability to apply, the harm to the plaintiff must
have resulted from the type of danger that justified classifying the
animal or activity as dangerous.
(4) An unforeseeable, intervening force could relieve a strictly liable
defendant of liability as well.

C. Defenses
1. Contributory Negligence
a. Where strict liability is applicable, a defendant generally may not raise
contributory negligence as a defense.
EXCEPTION: Where a plaintiff knew of the danger that justified imposition of
strict liability, and his contributory negligence caused exactly that danger to be
manifested, such contributory negligence will bar the plaintiff’s recovery, as-
suming the jurisdiction applies the traditional contributory negligence doctrine.
EXAMPLE: If Pat O’Pheline stands next to a circus tiger’s cage knowing
that the tiger is dangerous and can reach between the bars of the cage,
and despite seeing and understanding warning signs and ropes posted
to keep circus patrons at a safe distance, he may be barred from recov-
ery by the doctrine of contributory negligence if the tiger claws him.

NOTE Contributory negligence is not a complete defense to strict liability; it can only
reduce recovery.
2. Comparative Negligence
a. Some states that have adopted comparative fault systems reduce the
recovery of plaintiffs whose negligence contributes to their own injuries
involving strict liability situations.

509
OUTLINE

3. Assumption of Risk
a. A plaintiff may be found to have assumed the risk of injury and be
completely barred from recovery in a strict liability situation if the plain-
tiff knows of and appreciates the danger justifying imposition of strict
liability and voluntarily exposes himself to such danger.

510
TORTS

IV. PRODUCTS LIABILITY

A. Strict Products Liability in Tort


1. Strict products liability is invoked when a defective product, for which an appro-
priate defendant is responsible, injures an appropriate plaintiff. To analyze an
action for strict products liability, it is necessary to consider certain specific factors.
2. Proper Plaintiff
a. In general, if strict liability is otherwise applicable, any plaintiff injured
while using a defective product may recover damages from an appro-
priate defendant. This includes purchasers and consumers, as well as
families, friends, guests, and employees.
b. There is a modern trend to permit one not using a defective product to
invoke strict liability against an appropriate defendant if it was reason-
ably foreseeable that such a plaintiff might be injured by the defective
product. Examples include bystanders (e.g., the plaintiff was a pedes-
trian injured when an automobile manufactured by the defendant with
defective steering went out of control and struck him) and rescuers
(e.g., the plaintiff attempted to rescue a third person who was endan-
gered by a defective product manufactured by the defendant).
3. Proper Defendant
a. Commercial suppliers at all levels of the distribution chain (i.e., the
manufacturer, distributor, or retailer) as well as commercial lessors,
new home developers, and sellers of used goods are all potential
defendants. Occasional sellers and those supplying services (i.e., an
optometrist) cannot be strictly liable, but may be sued for negligence.
(1) A person who assembles component parts into a finished product is
strictly liable for defects in the components used. The manufacturer
of the component part is also strictly liable for defects in the compo-
nent. If the finished product is defective not because of a defect in
a component part but because the assembler put the component
to a use for which it was not suited, the component manufacturer
is generally not liable unless the component manufacturer knew
or should have known that his component was being misused. A
person who rebuilds or reconditions used goods is often held to be
the equivalent of a manufacturer, and may be subject to strict liability
for injuries caused by defects in the rebuilt or reconditioned goods.
EXAMPLE: Dunlop Tire Co. manufactured a tire that proves to
be defective. Ford Motor Company purchases Dunlop tires to put
onto its new car model, the Stallion. Ford sells a Stallion to Jen
Smyth Ford, a car retailer. While driving a Stallion purchased by
Paxtona at Jen Smyth Ford, Penelope is physically injured be-
cause of the defect in the tire. Dunlop, Ford, and Jen Smyth are
all proper defendants for a strict products liability claim because
they are all in the business of dealing with this product and part of

511
OUTLINE

the marketing chain. Note that these defendants are liable even
though there is likely no fault on their part.
b. Retailers and commercial lessors are subject to strict liability for
defects in new goods that they sell or lease. Commercial lessors are
subject to strict liability for defective used goods leased, but there is a
split in authority as to whether a retailer of used goods is strictly liable
for defects in those goods.
c. Most jurisdictions hold sellers of mass-marketed new residences
strictly liable for defects in those homes. There is a split in authority
as to whether other new home sellers, such as custom builders who
construct a few houses at a time or building contractors who construct
residences under specific contracts, may be held strictly liable for
defects in those constructions. No court has imposed strict liability on a
defendant who sold a home he did not construct.
d. Occasional or one-time sellers are not proper defendants for purposes
of strict products liability because they are not in the position to further
the goals of the tort—safer products and cost-spreading.
EXAMPLE: Pumpkin is injured as a result of a defect in a new widget
she bought at Della’s garage sale. Della is not a proper defendant for
strict products liability. To recover, Pumpkin will have to prove negli-
gence or a breach of warranty.
e. A few states have imposed strict liability on a franchisor, at least where
it was found that the franchisor exercised substantial control over the
operations of the franchisee.
4. Proper Context for Strict Products Liability
a. Generally, providers of services are not held strictly liable for inju-
ries received by their customers. If defective goods are supplied
along with services, strict liability is still not applicable so long as
the goods supplied were merely “incidental” to rendition of the
services. In this regard, restaurants are frequently regarded as
sellers of goods (food) subject to strict liability. Doctors, dentists,
and blood banks are usually regarded as primarily providing
services, so defective products (medicines, blood, etc.) provided are
incidental and not subject to strict liability.
5. Defect
a. Almost all jurisdictions impose strict liability where a product is “in a
defective condition unreasonably dangerous.”
b. Formulations of liability occur under three categories of defects.
(1) Manufacturing Defects
(a) A product manufactured in a form other than the manufac-
turer intended contains a manufacturing defect. All jurisdic-
tions impose strict liability on the manufacturer and everyone
else in the chain of distribution for personal injury or property
damage to a plaintiff caused by a manufacturing defect.

512
TORTS

EXAMPLE: Pachina cuts her lip on a piece of metal in the


tuna fish she was eating. The tuna fish was manufactured
by Tuna-of-the-Ocean and purchased from Delmon Grocery.
Because Tuna-of-the-Ocean did not intend for there to be a
piece of metal in the tuna fish, this is a manufacturing defect.
They, along with Delmon as part of the marketing chain, will
be strictly liable to Pachina for her personal injury.
(2) Design Defects
(a) A product manufactured as the manufacturer intended, but that
still presents a danger of personal injury or property damage to
a plaintiff, suffers from a design defect. The analysis of whether
such a defect is sufficient to invoke strict liability is complex.
1) Under the consumer expectation test standard, a
product is in an unreasonably dangerous defective condi-
tion when it is more dangerous than would be contem-
plated by the ordinary consumer who purchases it, with
the ordinary knowledge common to the community as to
its characteristics. Thus, a product is dangerously defec-
tive if a reasonably foreseeable purchaser would not have
expected it to present the danger that resulted in his injury.
a) One interpretation of the consumer expectations
test would hold that obvious danger could not be
a dangerous defect because it would not be unex-
pected to a reasonable consumer.
b) Many jurisdictions that apply the consumer expecta-
tion test expand “consumer” to include a nonpur-
chasing user. A product is thus considered defective
and unreasonably dangerous if more dangerous
than a reasonable user would have expected.
2) Under the danger-utility test approach, a product is
defective if a jury determines that the danger it threatens
(the cost in human injury and property damage)
outweighs its utility to society. The danger-utility test
balances the likelihood, nature, and potential severity of
injuries caused by a product against the usefulness of
the product, considering the availability and cost of safer
alternative designs. A product’s design is usually defec-
tive under this test if an alternative design could have
reduced the danger at about the same cost.
a) Almost all jurisdictions that utilize the danger-utility
test apply it so that the inquiry as to the availability
of alternative designs focuses on the time the
product was put on the market rather than consid-
ering alternatives that were discovered in the
period up until trial.

513
OUTLINE

3) Under the hindsight-negligence test, a product is


defective if a reasonable person, knowing of the danger
it presented, would not have placed it in the stream of
commerce. This test imposes constructive prior knowl-
edge of the defect on the defendant, in effect presuming
that the defendant knew of the risk, whether or not he
actually did know or reasonably could have known of it.
EXAMPLE: Dun Motors manufactures a new lightweight,
fuel-efficient car, the Stallion, which because of the place-
ment of the gas tank proves to be vulnerable to explosion
upon a low-speed rear-end collision. Pafto is injured when
the Stallion he is driving is rear-ended and explodes due
to the placement of the gas tank. This is a design defect
because the product is in the condition intended by the
manufacturer. It may be defective if it is more dangerous
than an ordinary consumer would expect when using the
car in its intended or foreseeable manner. However, a
consumer may have no particular expectations about the
placement of the gas tank, so many jurisdictions would
permit Pafto to show that the car was defective because
the risk of the car with the gas tank placed where it was
outweighs the benefit of such a design.
(3) Absence of Warnings
(a) A product may be considered dangerously defective when it
is accompanied by an inadequate warning—for example, its
message to the user (e.g., a child who cannot read) fails to
sufficiently describe the danger, fails to mention all dangers, or
is inconsistent with the instructions for use of the product.
(b) A defendant’s failure to warn a plaintiff that a product pres-
ents a threat of personal injury or property damage may be
considered a defect. Where a plaintiff establishes that the
manufacturer of a product knew or reasonably should have
known of a danger presented by the product and failed to
take the precautions a reasonable person would have taken
to warn adequately of that danger, the absence of such
warning is sufficient to impose strict liability.
1) Certain products are regarded as so obviously
dangerous that a warning is considered unnecessary,
(e.g., a sharp knife).
2) Whether absence of a warning of allergic or other
adverse reactions is a dangerous defect depends upon
the severity of the potential reaction and the number
of people expected to be affected. A small number
of reactions justifies a warning if the adverse reac-
tion threatens death or serious illness. A mild reaction

514
TORTS

might require a warning if a substantial number of


people likely will experience it.
6. Cause-in-Fact
a. That the injury is attributable to the defendant is usually proven by
showing that the defect that injured plaintiff was in existence at the
time it left defendant’s control. It may be necessary in this regard to
establish that intermediate handlers (distributors and suppliers) did not
mistreat or alter the product.
b. Plaintiff shows that the injury was caused by the defect by proving that
the defect was a substantial factor in bringing about his injury.
c. Warning defect claims often pose difficult issues of causation because
the plaintiff must prove that she would not have been injured by the
product had there been a warning on the product or had the warning
been adequate. Some jurisdictions employ a “heeding presump-
tion,” which puts the burden of proof on the defendant to prove that a
warning or adequate warning would not have made a difference.
7. Proximate Cause
a. Principles of legal or proximate cause relevant to negligence are
applied in like fashion to strict products liability, except for where the
intervening conduct of third persons occurs.
b. The negligent handling of a product by a third party after it leaves the
defendant’s control is regarded as foreseeable and is not a super-
seding intervening force. This includes negligent handling not directly
affecting the defectiveness of product, e.g., where a third party negli-
gently collides with plaintiff’s car, which is dangerously defective in a
manner attributable to defendant.
c. A defendant will be relieved of strict products liability where an inter-
vening force is the sole proximate cause of the harm. In other words,
the intervening force will sever the defendant’s liability where it is such
that the injury would not have been suffered but for that superseding
or intervening force, independently of the act or omission which consti-
tuted the defendant’s negligence.
d. A third party’s criminal or intentionally tortious conduct is regarded
as an unforeseeable intervening force that supersedes a defendant’s
wrongdoing, unless the defect somehow increased the risk that a third
party would engage in such conduct.
8. Damages
a. General principles of tort damages apply to strict liability, except for
the following:
(1) In a majority of jurisdictions, pure economic losses, such as
loss of profits due to the defective product not performing as
expected, additional expenses incurred in obtaining replacement
equipment or materials, etc., may not be recovered by a plaintiff
in strict liability. However, most jurisdictions permit recovery of

515
OUTLINE

economic losses as derivative elements of otherwise appropriate


personal injury or property damage claims.
(2) Some jurisdictions look to the nature of the harm threatened
by the defect to determine the availability of damages in strict
liability, rather than distinguishing between property damage
and economic losses. Under this approach, any loss suffered is
recoverable if caused by a defect that threatened personal injury,
even if the actual loss did not include personal injury. If the defect
was not such that it would threaten personal injury, no property
damage or economic losses are recoverable in strict liability.
9. Defenses
a. Misuse
(1) If a plaintiff uses a product in a manner that is neither intended nor
foreseeable, he has misused the product and it cannot be defective.
EXAMPLE: Parker stands on a chair manufactured by DrekCo
to reach a pot in his kitchen. The chair collapses under him.
While sitting is the intended use of a chair, it is foreseeable that
a person would stand on a chair, and thus, there is no misuse. If
Parker was using the chair as part of an act in which he has an
elephant balance on the chair, this would be a misuse because it
would be an unforeseeable use of the chair.
b. Alteration
(1) A manufacturer or seller may have their liability reduced (or in
a contributory negligence jurisdiction, relieved entirely) for a
product where the product is altered after leaving its hands.
(a) Alteration or modification includes changes in design,
formula, function, or use of a product from that originally
designed, tested, or intended by the product seller [Potter v.
Chicago Pneumatic Tool Co., 694 A.2d 1319 (Conn. 1997)].
(b) An alteration must be substantial in order to relieve the
manufacturer or seller of liability. A substantial change
consists of any change which [E.Z. Gas, Inc. v. Hydrocarbon
Transp., Inc., 471 N.E.2d 316 (Ind. Ct. App. 1984)]:
1) increases the likelihood of a malfunction;
2) is the proximate cause of the harm; and
3) is independent of the expected and intended use to
which the product is put.
(2) Alteration or modification of a product constitutes a defense to
claims in strict liability, negligence, and warranty.
(a) In product liability actions based on negligence or breach of
warranty, the defense of alteration is raised as an intervening
or superseding cause.
(3) In order to preclude liability, an alteration or modification must
occur between the time the product leaves the manufacturer’s

516
TORTS

control and the time of the plaintiff’s injury [Banner Welders, Inc.
v. Knighton, 425 So. 2d 441 (Ala. 1982)].
(4) For the defense to prevail, the modification or alteration must be
independent of the expected and intended use of the product.
(5) The mere fact that the product has been altered or modified does
not necessarily relieve the manufacturer or seller of liability. The
defendant will remain liable if the alteration or modification:
(a) did not in fact cause the injury; or
(b) was reasonably foreseeable to the manufacturer or seller.
(6) Material alteration is not a defense in a products liability action
based on a design defect theory; it is only a defense when the
alteration makes it impossible to conclude that a defect at the time
of manufacturer was a cause of the injury giving rise to the suit.
c. Contributory Negligence
(1) As initially conceived, a plaintiff’s unreasonable conduct was not a
defense to a strict products liability action unless the plaintiff knew
of the defect, comprehended the risks posed by the defect, and
voluntarily elected to expose himself to those risks. That is, only an
assumption of the risk was a defense to strict products liability.
EXAMPLE: Because of a defect, Parker’s television set, manu-
factured by DuMont, starts to spark. A reasonable person would
have noticed this, but Parker, because he is totally engrossed in
his favorite sitcom, does not. DuMont has no defense.
(2) At modern law, contributory negligence only applies as a defense
to strict products liability if the plaintiff’s conduct rises to the level of
misuse, abnormal use, or independent negligence (i.e., not where
the plaintiff’s wrongful conduct is a failure to discover the defect).
(3) A plaintiff’s continued use of a product which the plaintiff knows to
be defective is not voluntary, and thus not an assumption of the
risk, if there are no practicable alternatives to such use.
EXAMPLE: If while in the desert hundreds of miles from a city,
Plaintiff discovers that his car has a dangerous steering defect, it
would not be an assumption of the risk that Plaintiff continued to
use the car to drive back to civilization.
(4) Where a defendant can show that his product was subse-
quently altered in an unforeseeable manner by someone in the
chain of distribution or a third party, courts usually relieve that
defendant of liability.
d. Comparative Negligence
(1) Some jurisdictions that have adopted a comparative negligence
system as to negligence also apply that system to strict liability.
A plaintiff’s wrongful conduct that contributes, along with the
defective product, to his own injury reduces his recovery in some

517
OUTLINE

amount. This is usually limited to misuse, abnormal use, or


independent negligence situations (i.e., not where the plaintiff’s
wrongful conduct is a failure to discover the defect).
(2) In some jurisdictions that permit comparative fault, the plaintiff’s
unreasonable conduct in failing to discover and guard against the
defect is not a defense.

B. Products Liability on a Negligence Theory


1. Proper Defendants
a. If negligence, rather than strict liability, is the theory of recovery
in a products liability action, the plaintiff must establish a greater
personal degree of fault. Negligent conduct should result in liability
for the following:
(1) a defendant selling used goods (i.e., a commercial seller of used
goods unreasonably passes on a defective product or negligently
fails to discover a dangerous defect);
(2) a defendant repairing used goods (i.e., where the negligent repair
of used goods renders or leaves them dangerously defective);
(3) a defendant leasing real property;
(4) a defendant providing services; and
(5) franchisors.
2. Elements
a. The elements of a products liability claim based in negligence are the
same elements that are part of the negligence cause of action in general.
b. Duty
(1) A duty is owed to any foreseeable plaintiff. No contractual rela-
tionship is required.
EXAMPLE: If Tom buys a car negligently manufactured by Ford
for his wife, Katie, she may bring a negligence claim against
Ford, as she is a foreseeable plaintiff.
c. Breach of Duty
(1) The defendant will typically be judged by the reasonable person
standard of care, and breach of duty will be unreasonable
conduct on the part of the defendant. On a negligence theory,
proof that a product is defective does not automatically establish
that each defendant in the chain of distribution breached a duty.
The plaintiff must therefore establish that each defendant failed to
exercise due care.
(2) If a reasonable person would have realized that a product was
dangerous in normal use, a defendant’s failure to inspect and
discover the defect is a breach of duty under negligence principles.
It is relevant to consider the nature of the product, the source of the
product, and the extent of information available to the defendant.

518
TORTS

(3) In general, a failure to inspect packaged goods for defects is not a


breach of duty if they come from a reputable manufacturer or distrib-
utor. If the goods are manufactured or otherwise supplied by a previ-
ously unknown or questionable source, a defendant’s unreasonable
failure to inspect is a breach of duty. It likely would be a breach of
duty not to inspect a particular product as to which a defendant had
received or otherwise became aware of complaints from customers.
(4) If the defendant inspects or becomes aware of a defect, his
subsequent failure to find the defect or to take reasonable
precautions is judged by the reasonable person standard.
d. Cause-in-Fact and Proximate Cause
(1) The principles of actual (cause-in-fact) and legal (proximate) causa-
tion which govern a negligence analysis generally apply equally to
products liability claims where negligence is the theory of recovery.
e. Damages
(1) Damages in negligence products liability are assessed according
to the same principles applicable to strict liability.
f. Defenses
(1) Those defenses generally applicable to negligence are also avail-
able in products liability sounding in negligence.

C. Products Liability on a Warranty Theory


1. Today, there is little left of the historical requirement for products liability on
a warranty theory that the parties be in privity. Only where a plaintiff seeks
recovery for pure economic losses and the theory of recovery is implied
warranty is privity a relevant consideration. Privity in this context refers to
the relationship between the injured plaintiff and the last commercial seller
in the chain of distribution of the defective product.
2. UCC Section 2-318 offers three alternatives regarding privity (a legislature
could enact any one of the three). Each provides that if the purchaser of
goods was the beneficiary of a warranty, privity extends to:
a. members of the purchaser’s family or household, plus guests of the
purchaser, if they suffer personal injury;
b. any natural persons who may reasonably be expected to use, consume,
or be affected by the goods, if they suffer personal injury; and
c. any persons who may reasonably be expected to use, consume, or be
affected by the goods, if they suffer injury.
3. On a warranty products liability theory, liability arises from the fact that a product
is not as represented (i.e., it breaches a warranty made either expressly or
impliedly by the defendant). The plaintiff must establish that a warranty existed
as to the product and that the product does not conform to the warranty.
a. An express warranty exists where the defendant made a representation
as to the nature or quality of the product. This can occur via advertising,
during negotiations for purchase, or as a provision of the contract of sale.

519
OUTLINE

b. The UCC provides that an express warranty may be created by a


defendant’s promise, by affirmation of fact, by description, or by use of
a sample or model [UCC § 2-313].
c. An implied warranty may arise under a variety of circumstances.
(1) The UCC is limited by its own terms to the sale of goods. There are two
types of warranties that may be implied under the proper circumstances:
(a) where a merchant deals in goods of a particular kind, the sale
of such goods constitutes an implied warranty that those goods
will be merchantable—that is, they are of average quality for
goods of that kind and generally fit for the purpose for which
such goods are normally used [UCC § 2-314(1)]; and
(b) where a defendant knows or has reason to know that
[UCC § 2-315]:
1) the plaintiff is purchasing the goods for a particular
purpose; and
2) the plaintiff is relying upon the defendant’s skill or judg-
ment to furnish appropriate goods—sale of the goods
will constitute an implied warranty that those goods will
actually be fit for the plaintiff’s purpose.
(2) Many states recognize judicially created implied warranties of merchant-
ability and fitness for a particular purpose that apply to leased goods.
(3) The decisional law of a small minority of states extends the implied
warranties applicable to sales of goods to real property as well.
4. Recovery of damages for breach of warranty is governed by the same prin-
ciples applicable to strict liability in tort, except that pure economic losses
are recoverable (e.g., loss of profits, cost of obtaining replacement equip-
ment or materials, etc.). The plaintiff must also be in privity with the defen-
dant, as discussed above.
5. Defenses
a. Disclaimer
(1) The UCC enables a defendant to disclaim or limit the applicability
of all warranties by a sufficiently conspicuous writing.
(2) The UCC provides that it is prima facie unconscionable for a
seller to attempt to limit the remedies available for breach of
warranty so as to exclude recovery for personal injuries (e.g., by
limiting responsibility for breach to repair or replacement) where
consumer goods are involved.
(3) Where goods are sold “as is,” it is often implied that the seller is
disclaiming any warranties, implied or express.
(4) Federal law provides that a seller who gives a written warranty to
a consumer may not thereby disclaim any implied warranties.
b. Failure to Notify Seller
(1) The UCC provides that a plaintiff-buyer must notify a defendant-seller
of the breach of warranty within a reasonable time after the buyer

520
TORTS

discovers or should have discovered it. This provision is often not


applied where the plaintiff is a bystander who suffers personal injury.
c. Contributory and Comparative Negligence
(1) With respect to personal injury or injury to property, contributory
negligence and comparative fault defenses are available where
warranty is the theory on a similar basis to strict liability in tort.
d. Assumption of Risk
(1) As to products liability in warranty, this defense is similar in appli-
cation to strict products liability in tort. If the plaintiff voluntarily
and unreasonably continues to use a product after he discovers
that it is not fit for the purpose purchased or not of merchantable
quality, the defendant is exonerated from liability for personal inju-
ries or property damage suffered by the plaintiff.

521
OUTLINE

V. NUISANCE

A. Types of Nuisance
1. Public Nuisance
a. A public nuisance is an unreasonable interference with a right
common to the general public.
b. Circumstances that may sustain a holding that an interference with a
public right is unreasonable include the following:
(1) whether the conduct involves a significant interference with the
public health, safety, peace, comfort, or convenience;
(2) whether the conduct is proscribed by a statute, ordinance, or
administrative regulation; and
(3) whether the conduct is of a continuing nature or has produced a
permanent or long-lasting effect, and as the actor knows or has
reason to know, has a significant effect upon the public right.
c. To recover damages in an individual action for a public nuisance, a
plaintiff must have suffered harm of a kind different from that suffered
by other members of the public.
d. To maintain a proceeding to enjoin or abate a public nuisance, a plaintiff must:
(1) have the right to recover damages;
(2) have authority as a public official or public agency to represent
the state or a political subdivision in the matter; or
(3) have standing to sue as a representative of the general public, as a
citizen in a citizen’s action, or as a member of a class in a class action.
2. Private Nuisance
a. A private nuisance is a thing or activity that substantially and unrea-
sonably interferes with the plaintiff’s use and enjoyment of his land.
b. The interference with the plaintiff’s use and enjoyment must be
substantial. This means that it must be offensive, inconvenient, or
annoying to an average person in the community. A plaintiff cannot, by
devoting his land to an unusually sensitive use, complain of a nuisance
based on conduct that would otherwise be relatively harmless.
c. The interference must be unreasonable, which means that either:
(1) the gravity of the plaintiff’s harm outweighs the utility of the defen-
dant’s conduct; or
(2) if intentional, the harm caused by the defendant’s conduct is substan-
tial and the financial burden of compensating for this and other
harms does not render unfeasible the continuation of the conduct.
d. A trespass is an invasion of a plaintiff’s interest in the exclusive
possession of land (e.g., an entry of something tangible onto the prop-
erty). On the other hand, a nuisance is an interference with a plaintiff’s
interest in the use and enjoyment of the land, which does not neces-
sarily require a physical intrusion.

522
TORTS

EXAMPLE: Amanda is a writer. Every day, she tries to get work


done in the morning. If her neighbor, Callie, calls her every morning,
this will not be a private nuisance. However, if Callie calls her fifteen
times a morning after Amanda has asked her not to, this may be a
private nuisance.
e. For a private nuisance, there is liability only to those who have property
rights and privileges in respect to the use and employment of the land
affected, including:
(1) possessors of the land;
(2) owners of easements and profits in the land; and
(3) owners of nonpossessory estates in the land that are detrimen-
tally affected by interferences with its use and enjoyment.
f. A defendant is subject to liability for a private nuisance if, but only if,
his conduct is a legal cause of an invasion of another’s interest in the
private use and enjoyment of land, and the invasion is either:
(1) intentional and unreasonable; or
(2) unintentional and otherwise actionable under the rules controlling
liability for negligent or reckless conduct.

EXAM TIP Often the key determination on an examination question is to evaluate the
reasonableness or unreasonableness of the defendant’s conduct. This
analysis involves weighing the gravity of the harm done to the plaintiff
against the utility of the defendant’s activity. Unlike in trespass, the court
will balance several factors: compliance with applicable zoning ordinanc-
es; priority of occupation; the frequency and extent of the interference,
applied objectively to normal persons; and the utility and social value of
the defendant’s activity.

TRESPASS ON LAND NUISANCE

A defendant who intentionally, Substantial and unreasonable


negligently, or recklessly enters the interference with the plaintiff’s use
land in the possession of another or and enjoyment of the land; basis
causes a thing or a third person to do of nuisance may be intentional,
so is subject to liability. negligent, or absolute.

Consists of intrusions upon, Consists of: (1) interference with the


beneath, and above the surface of physical condition of the land (such
the earth. as by vibrations or blasting which
damages a house, the destruction of
crops, the flooding or pollution of a
stream, etc.); or (2) a disturbance of the
comfort or convenience of the occupant
(such as by unpleasant odors, smoke,
dust, loud noise, excessive light, or
even repeated telephone calls).

523
OUTLINE

B. Defenses
1. Contributory Negligence
a. When a nuisance results from negligent conduct of the defendant,
the contributory negligence of the plaintiff is a defense to the same
extent as in other actions founded on negligence.
b. When the harm is intentional or the result of recklessness, contributory
negligence is not a defense.
c. When the nuisance results from an abnormally dangerous condition
or activity, contributory negligence is a defense only if the plaintiff has
voluntarily and unreasonably subjected himself to the risk of harm.
2. Assumption of Risk
a. In an action for a nuisance, the plaintiff’s assumption of risk is a
defense to the same extent as in other tort actions.
3. Coming to the Nuisance
a. The fact that the plaintiff has acquired or improved his land after a
nuisance will not by itself bar his action, but it is a factor to be consid-
ered in determining whether the nuisance is actionable.
4. Compliance with Statute
a. A relevant and persuasive, but not absolute, defense to nuisance arises upon
evidence that the defendant’s conduct was consistent with applicable admin-
istrative regulations (i.e., a zoning ordinance or pollution control regulation).

C. Remedies
1. For a private or public nuisance, the usual remedy is damages.
2. Where the legal remedy (i.e., money damages) is inadequate or unavail-
able, courts may grant injunctive relief. The legal remedy may be deemed
to be inadequate for a number of reasons (e.g., the nuisance is a continuing
wrong or the nuisance is of a kind which will cause irreparable harm). In
determining whether an injunction will be granted, the court will undertake
to balance the equities, namely taking into account:
a. the relative economic hardship to the parties for granting or denying
the injunction; and
b. the public interest in the defendant’s activity continuing.

EXAM TIP Keep in mind that a court may require the defendant to pay damages while
denying injunctive relief.

3. A plaintiff has the privilege to enter upon the defendant’s land and personally
abate the nuisance after notice to the defendant and his refusal to act. The privi-
lege extends to the use of all reasonable action that is necessary to terminate
the nuisance, even to the destruction of valuable property, provided the damage
done is not greatly disproportionate to the threatened harm, but does not extend
to unnecessary or unreasonable damage. There will be liability for any excess.

524
TORTS

EXAMPLE: It may not be justifiable, for instance, to destroy a house merely


because it is used for prostitution.
4. A plaintiff who has suffered some unique damage has a similar privilege
to abate a public nuisance by self-help. However, a public nuisance may
be abated by a private individual only when it causes or threatens special
damage to himself apart from that to the general public, and then only to
the extent necessary to protect his own interests.
5. Injunctions
a. Nature and Forms
(1) The injunction is the most common form of equitable remedy. An
injunction is a court order directed to a person or entity, usually
to refrain from doing a particular act (a prohibitory injunction).
(2) Less commonly, a court may issue an order to do a particular
act (a mandatory injunction). Requests for mandatory injunc-
tions are subject to heightened scrutiny, requiring a showing that
the facts and law clearly favor the moving party [Dahl v. HEM
Pharmaceuticals Corp., 7 F.3d 1399 (9th Cir. 1993)].
EXAMPLE: Plaintiff, a 30-year-old male, brings an action against
Health Club, an all-women’s health facility, to allow him to take
aerobics classes there. If the court requires the health club to
admit him, it may do so in the form of a mandatory injunction.
EXAMPLE: Plaintiff, a women’s health clinic, brings an action against
Defendant, an anti-abortion group, to enjoin it from harassing patients
and passersby on the sidewalk in front of the clinic. If the court en-
joins Defendant, it will do so in the form of a prohibitory injunction.
b. Availability
(1) In accordance with the general rules on the availability of equi-
table remedies, injunctive relief is generally available when
money damages would not afford adequate relief, such as where
it would be extremely difficult to ascertain the amount of compen-
sation that would afford adequate relief or where injunctive relief
is necessary to prevent a multiplicity of judicial proceedings.
EXAMPLE: Injunctive relief has been the primary remedy in civil
rights litigation because a deprivation of the individual’s constitu-
tional rights is not deemed remediable by money damages.
EXAMPLE: If the defendant repeatedly trespasses on the plain-
tiff’s land, equity may enjoin the trespass rather than force repeat-
ed actions at law to redress the injury.
(2) The plaintiff who seeks an injunction must specifically show that:
(a) he is about to suffer or will continue to suffer an irreparable injury
for which money damages would be inadequate compensation;

525
OUTLINE

(b) the balance of hardships between the plaintiff and defendant


supports an equitable remedy; and
(c) the public interest would not be disserved by the grant of
an injunction.
(3) Injunctive relief is generally not available to:
(a) stay court proceedings, unless necessary to prevent a
multiplicity of proceedings;
(b) enforce the criminal laws other than those that restrain a
public nuisance; or
(c) prevent the breach of a contract that is not specifically
enforceable, such as where the terms of the agreement are
not sufficiently certain to make the precise act which is to be
done clearly ascertainable.
c. Temporary Restraining Orders and Preliminary Injunctions
(1) Showing Required
(a) Courts apply a number of other factors to determine whether
preliminary injunctive relief is appropriate. The following
criteria are the most commonly applied:
1) the plaintiff is likely to succeed on the merits of the
underlying claim;
2) the complaint shows that the plaintiff is entitled to the
relief demanded (restraining the commission or contin-
uance of the act complained of) either for a limited
period or perpetually;
3) denial of injunctive relief would produce waste, or great
or irreparable injury, to the plaintiff, such as where it
appears, during litigation, that a party to the action is
doing or is about to do some act in violation of the rights
of another party to the action that would tend to render
the judgment ineffectual; and
4) the public interest as well as the balancing of hardships
between the parties favors the grant of an injunction.
(2) Notice Requirements
(a) A preliminary injunction may not be granted without notice
to the other party. During the time required for notice and
hearing on the preliminary injunction, however, the moving
party may seek immediate relief through a temporary
restraining order (“TRO”).
(3) Duration
(a) A TRO granted without notice remains in effect for only a
specified period of time, generally 10 to 20 days (a TRO
issued without notice in a federal court is effective for a
maximum of 14 days [Fed. R. Civ. P. 65(b)(2)]).

526
TORTS

d. Permanent Injunctions
(1) The plaintiff must have prevailed on the substantive claim and
shown a need for continuing protection to be eligible for perma-
nent injunctive relief.
(a) EXAMPLE: When a claimant who insisted the famous lawyer
Johnnie Cochran owed him money, and picketed Cochran’s
office with signs containing insults and obscenities, indicated
that he would continue to engage in the activity absent a
court order, the court permanently enjoined the claimant from
making defamatory statements about Cochran and his firm in
any public forum [Tory v. Cochran, 544 U.S. 734 (2005)].
(2) Unlike the interlocutory temporary injunction, a permanent injunc-
tion is a final judgment and continues in force until dissolved, but
the permanent injunction need not be perpetual—it may be set to
expire by its own terms.

527
OUTLINE

VI. DEFAMATION

A. In General
1. For a defamation action, a plaintiff must establish that the defendant
published defamatory material concerning the plaintiff that caused reputa-
tional damage. In analyzing an action for defamation, one must check for:
a. a defamatory message;
b. certain pleading problems;
c. publication of the message;
d. the type of defamation;
e. damages;
f. common law defenses; and
g. constitutional issues.

B. Defamatory Message
1. A message is defamatory if it lowers a plaintiff in the esteem of the commu-
nity or discourages third persons from associating with him.
a. A defamatory message has been characterized as one that holds
a plaintiff up to hatred, ridicule, contempt, or scorn. Whether or not a
defendant’s statement includes this notion of disgrace, it should be
actionable if it causes third persons to avoid contact with the plaintiff. A
message is also defamatory to an entity if it causes customers to stop
doing business with the entity, causes persons to stop making chari-
table contributions to it, or causes them to avoid membership.
2. In general, only statements of fact are actionable as defamatory. However,
expressions of opinion which imply that the speaker knows certain facts to
be true, or which imply that such facts exist, may be sufficient to classify the
messages as defamatory. If a reasonable person would interpret a state-
ment as one of fact, considering the context and the nature of the utter-
ance, a jury may find that the message is defamatory.
a. A statement is more likely to be regarded as a statement of fact rather
than opinion to the extent that it is more specific and detailed. It is not
sufficient to change what is otherwise a statement of fact to one of
opinion by adding qualifiers such as, “I think…“ or “It is my opinion….”
EXAMPLE: The Daily may be subject to defamation liability to Paxton for
stating falsely: “In our opinion, Paxton was involved in the planning of the as-
sassination of Robert F. Kennedy.” The Daily cannot be liable for publishing:
“Paxton was once a very stylish dresser, but this year his clothes seem drab.”
3. A message is defamatory if it has the required injurious effect on any substan-
tial minority of reasonable people. If a negligible number of people are affected,
or only those whose views are too antisocial, the message is not defamatory.
a. A messages is defamatory per se if it is apparent on the face of the
message that it will injure the plaintiff’s reputation. If the message at issue

528
TORTS

does not on its face seem injurious to the plaintiff’s reputation, or defama-
tion per quod, the plaintiff must plead additional, extrinsic facts that render
the message defamatory. The plaintiff must also explain how the message,
with the extrinsic facts, injures his reputation. Where a statement is defam-
atory only upon a showing of extrinsic facts, the plaintiff must:
(1) plead and prove inducement;
(2) establish a defamatory meaning by innuendo; and
(3) show that he himself was the intended plaintiff by colloquium.
EXAMPLE: The Daily prints, “Petunia married Donald yesterday.”
This does not appear reputation-harming on its face, but it may
become so through the inducement that Petunia is already mar-
ried to someone else.
4. The defamatory message must be understood by the person who receives it.
A judge decides whether a communication could be understood as defama-
tory, and the jury decides whether it was defamatory in the case before it.
EXAMPLE: The Daily falsely states in an article that Penelope used to be
best friends with a drug addict. A judge may determine that a false statement
that a person was once friends with a person who was addicted to drugs is
not capable of causing reputational harm. If the judge instead determines
that it could be harmful, then it would be up to the jury to determine if such a
statement harmed Penelope’s reputation in the situation before it.

C. Pleading Problems
1. To bring an action for defamation, the party suffering the defamation must
have been a living person or an existing organization.
a. A defamatory message published about a third person is action-
able only to the extent that it also defames the plaintiff. A defamatory
message concerning only a third person, even one closely related to
the plaintiff, cannot defame the plaintiff.
b. Where a defendant makes an otherwise defamatory statement about a
group of persons, his liability to individual plaintiffs varies according to
the size of the group and the nature of the defamatory message.
(1) A defamatory message made concerning all members of a large
group does not create a right of action in favor of any particular
member of that group. However, if the circumstances would indi-
cate to a reasonable person that a particular plaintiff is the actual
subject of the defamatory message, the fact that the message is
spoken of the group does not relieve the defendant of liability.
(2) If the group that is the subject of the defendant’s defamatory message
is sufficiently small, each member of the group is generally regarded as
sufficiently identified so that each could bring an action for defamation.
EXAMPLE: In a case involving a sportswriter who published an
article stating that all 20 members of the University football team

529
OUTLINE

had used illegal steroids, each member of that team was permit-
ted to bring an action against the sportswriter for defamation.
(3) Traditionally, no individual member of a small group was
permitted to bring an action for defamation where the defendant
defamed some but not all of them. Today, whether a particular
member was sufficiently identified is examined on a case-by-case
basis, considering all the circumstances.

D. Publication
1. A defamatory message is commonly a spoken or written statement. Any
form of communication may be defamatory, however, including television
and radio broadcasts, films, plays, novels, cartoons, sculpture, etc.
a. A plaintiff must prove that some reasonable third person who received
the defamatory message understood it to refer to the plaintiff. Thus,
if the plaintiff is not specifically named in the allegedly defamatory
communication, he must allege through colloquium that some people
will interpret the communication to be about him.
EXAMPLE: The Daily states that the longest serving Torts professor at
Acme School of Law does not know the difference between battery and
assault. Pinnafore may proceed in his action for defamation by alleging
and proving that he is the longest serving Torts professor at Acme.
2. It is not actionable to utter a defamatory message to the plaintiff alone. For
the defendant’s message concerning the plaintiff to constitute defamation, it
must be communicated to a third person, who receives and understands it.
a. In most situations, it is apparent that the defendant desired that third
persons receive the defamatory message. However, a message is
“published” if the defendant negligently permitted it to be communi-
cated to third persons. If it is reasonably foreseeable that an eaves-
dropper might overhear a message, for example, and one does so,
there is a sufficient publication.
3. In addition to the defendant who originates the defamatory message, other
persons who repeat it may be liable to the plaintiff, varying according to
their relationship to the original publisher. Such republication may also
increase the originator’s liability to the plaintiff.
a. All persons who participate in originating a defamatory message are
liable as primary publishers.
EXAMPLE: The author of a book defamatory to a plaintiff, the editor
who selected it for publication and provided editorial services, and the
company that employed the editor and printed the book would all be
liable as primary publishers.
b. Any person who repeats the defamatory message is liable as a
publisher. This is so even where the repetition is qualified by such
terms as “alleged” or is said not to be the opinion of the republisher.

530
TORTS

(1) Where the original publisher could reasonably foresee that the
defamatory message would be republished, he is liable to the
plaintiff for additional damages caused by the republications.
c. A person who distributes the original defamatory message as a
commodity (e.g., a bookseller, newspaper vendor, retailer, etc.) is liable
only if he knew or should have known that the material distributed
contained the defamatory message.

E. Type of Defamation
1. Historically, libel was a written form of defamation. Today, a defamatory
message embodied in any relatively permanent form is a libel.
EXAMPLE: A sound recording, video recording, picture, sculpture, etc.
a. Libel per quod is a libel as to which it is not apparent on the face of
the communication that it is defamatory. A plaintiff must plead and
establish extrinsic facts to establish that the libel was defamatory and
that it referred to the plaintiff. A large minority of jurisdictions require
proof of special damages for libel per quod.
2. Historically, slander was defamation in spoken, rather than written, form. Today,
a defamatory message not preserved in permanent form is classified as slander.
a. Slander per se is a type of slander historically regarded as so harmful
that it was presumed that the plaintiff suffered damage from the very
fact of its utterance. Four types of slander were so classified:
(1) a slander that imputed to the plaintiff the commission of a crime involving
moral turpitude or infamous punishment (imprisonment or death);
(2) allegations of the plaintiff having a loathsome disease are slander;
(a) Historically, a loathsome disease was one that was incur-
able and persisted over time, such as venereal disease or
leprosy. Allegations of insanity or tuberculosis have been
held not within the slander per se category.
(3) slander which imputes to the plaintiff behavior or characteristics
that are incompatible with the proper conduct of his business,
profession, or office; and
(4) it was slander per se to falsely impute unchastity to a woman.
(a) Scholars have concluded that the same conclusion would
follow if the target of the defamation were a man, but no
case has so held. The Restatement defines this form of
slander per se in terms of false imputation of “serious sexual
misconduct” to any person. A small minority of jurisdictions
hold it slander per se to falsely impute impotency to a man.
In addition, commentators have suggested that false imputa-
tion of deviant sexual behavior might fall within this category.
3. Where it is not clear whether a defamatory message is libel or slander,
factors to distinguish the two include:
a. the permanence of the form;

531
OUTLINE

b. the area of dissemination; and


c. the extent to which the message was planned rather than
spontaneous.
(1) The more permanent the form, widely disseminated, and planned a
defamatory message is, the more likely it is to be considered libel.

F. Damages
1. Three different types of damages are potentially recoverable for defama-
tion. In some situations, the need for actual proof of damages is affected by
the type of defamation involved.
a. Pecuniary damages are quantifiable monetary losses suffered by
the plaintiff due to the injury to his reputation. Examples include loss
of customers, loss of a job, or other diminishment of economic advan-
tage. The plaintiff must present evidence of specific actual monetary
losses in order to recover pecuniary damages.
(1) Proof of pecuniary damages is necessary to establish a prima
facie case if the form of defamation is slander and, in some
states, if it is libel per quod. Once pecuniary damages are estab-
lished, presumed damages are also available.
b. In certain circumstances, the jury is permitted to presume that plaintiff
suffered general damages as a result of the defendant’s defamatory
statement. These include nonpecuniary aspects of the injury to reputa-
tion, such as humiliation, loss of friends, etc. The jury is instructed to
estimate the amount of presumed damages based upon the extent of
injury to the plaintiff’s reputation.
(1) It is “presumed” that the plaintiff suffered general damages, and thus
no proof of actual damage need be offered, when the form of defa-
mation is slander per se or “ordinary” libel (i.e., not libel per quod).
c. Damages that are assessed against the defendant to punish and deter
future wrongful conduct are called punitive damages. The plaintiff
must make some additional evidentiary showing of vexatiousness or
evil intent (i.e., common law malice) to recover punitive damages.

G. Common Law Defenses


1. Truth
a. Historically, falsity was presumed once the plaintiff established the
publication of a defamatory communication. In other words, truth was a
substantial defense. The defendant had to prove that the communica-
tion was “substantially true.”
b. Now, in all defamation cases except possibly those where the plaintiff
is a private plaintiff and the matter is of private concern, the plaintiff
must prove falsity as part of his prima facie case.
2. Absolute Privilege
a. Where an absolute privilege applies, the defendant may not be held
liable for an otherwise defamatory message as a matter of law.

532
TORTS

(1) A legislator is not liable for a defamatory message uttered while


on the floor of the legislature or during hearings or committee
proceedings. The nature or content of the defamatory message
or its relationship to any matter before the legislature is immate-
rial to availability of this privilege.
(2) A participant in judicial proceedings (e.g., a judge, attorney,
witness, or juror) is not liable for any defamatory message that is
reasonably related to the proceedings.
(3) The privilege for statements made during judicial proceedings
arises upon filing of the complaint and continues until final termi-
nation of the action. It includes pleadings, pretrial proceedings,
and discovery, and may include settlement negotiations occur-
ring prior to the filing of the litigation. Quasi-judicial administrative
hearings are considered judicial proceedings for this privilege.
(4) An absolute privilege from liability for defamation applies to policy-
making officials of the executive branches of state and federal
governments, so long as the defamatory utterance was made in
the course of their duties and was relevant to those duties.
(5) A defamatory message communicated by one spouse to another
is absolutely privileged from defamation liability.
(6) The U.S. Supreme Court has held that a broadcast media defendant
compelled by the fairness doctrine to permit a third person to utilize
its facilities is absolutely privileged as to defamatory statements made
by the third person. Commentators suggest the same rule might be
applied to a newspaper compelled by law to print public notices.
b. The defendant has the burden of establishing that an absolute privilege
applies to his defamatory message.
3. Qualified Privilege
a. If a qualified privilege is applicable, a defendant is not held liable for
otherwise defamatory messages he utters unless he loses the protec-
tion of the privilege.
b. A defendant is qualifiedly immune from liability for defamatory
messages made in a communication that appears reasonably neces-
sary to protect or advance the defendant’s own legitimate interests.
c. A defendant who communicates on a matter of interest to the recipient
of the communication or a third person is qualifiedly immune from
liability for defamatory messages in the communication.
EXAMPLE: Duncan was employed for seven years with ABC Compa-
ny. After being terminated, he sought employment with XYZ Company.
XYZ contacted ABC and asked for a recommendation. The president
of ABC stated, “If I were you, I wouldn’t hire Duncan—he’s a thief.” As
long as the speaker reasonably believed the information to be true,
there is a qualified privilege to act in the interest of others. Thus, ABC
would not be liable for defamation.

533
OUTLINE

d. A defendant who communicates concerning a matter of public interest


to one empowered to protect that interest is qualifiedly privileged as to
a defamatory message contained in the communication.
e. A defendant has a qualified privilege as to defamatory messages
contained in a criticism of a matter of public interest.
f. A defendant is qualifiedly immune from liability for defamatory
messages that are republished in a report of public hearings or meet-
ings, so long as the defendant’s report is fair and accurate.
g. The defendant bears the burden of establishing that a qualified privi-
lege is available.
h. The defendant loses an otherwise available qualified privilege if:
(1) he acts out of malice;
(a) Malice is present if the defendant’s primary motive in
publishing the defamatory message was something other
than furthering the interest that justified the privilege.
(2) he exceeds the scope of the privilege; or
(a) A qualified privilege may be lost if the defamatory message
includes matters not relevant to the interests protected
by the privilege or is published in a manner or to persons
outside the legitimate scope of the privilege.
(3) he does not believe the truth of the defamatory communication.
(a) A qualified privilege is lost if the defendant does not possess an
honest belief in the truth of the defamatory message. Some states
also require that the defendant have a reasonable, honest belief.
i. While a defendant bears the burden of establishing that a qualified
privilege is applicable, a plaintiff bears the burden of establishing that
conditions exist under which the privilege is lost.
4. Consent
a. Generally applicable principles relating to the defense of consent
apply to defamation.

H. Constitutional Issues
1. Decisions interpreting the Free Speech Clause of the First Amendment
have altered the common law of defamation with regard to:
a. the degree of fault required of a defendant;
(1) At common law, defamation was a strict liability offense, in the sense
that the plaintiff need only establish that the defamatory statement
was made in order to recover (all other elements being present), not
that defendant had any particular mental state or degree of fault.
b. the nature of the plaintiff;
(1) Where a plaintiff is a public official or a public figure, the plaintiff must
establish that the defendant acted with “malice” before any recovery
may be had [New York Times v. Sullivan, 376 U.S. 254 (1964)].

534
TORTS

(a) A public official is a government official who has or appears


to have substantial responsibility over governmental opera-
tions. While this does not include every public employee,
courts have, for example, generally included police officers
among public officials.
(b) A public figure is a person who has either:
1) achieved such pervasive fame or notoriety such that he
becomes a public figure for all purposes (e.g., a celebrity); or
2) voluntarily injected himself or allowed himself to be
drawn into a particular public controversy such that he
becomes a public figure as to the limited issues present
in that controversy.
(c) Malice, for this purpose, is defined as knowing falsity or
recklessness as to truth or falsity. In order for malice to exist,
the defendant must have actually known the defamatory
message was false or actually entertained serious doubts
about its truth (i.e., had reckless disregard) [Id.].
(d) The plaintiff must establish malice with “convincing clarity.”
Commentators have analogized this language from New York
Times v. Sullivan to the “clear and convincing” evidence stan-
dard, which requires something more than proof by a prepon-
derance of the evidence, and something less than the “beyond
a reasonable doubt” standard used in criminal matters.
c. the subject matter of the defamatory statement;
(1) Where the plaintiff is neither a public official nor public figure, consti-
tutional protection of free speech still precludes application of the
common law strict liability scheme if the subject matter of the defa-
mation is a matter of public concern. In such a case, the defendant
must be shown to have exhibited some degree of fault higher than
strict liability, which presumably means negligence. However, actual
malice must be proved for presumed or punitive damages.
(2) Where a private plaintiff sues in defamation and the subject
matter is also private (i.e., not a matter of public concern), the
U.S. Constitution does not require that the plaintiff prove actual
malice to recover presumed or punitive damages.
(3) This can be a blurry line, but the test is to look at the “form,
content, and context” of the communication. As a practical matter,
if the defendant is a member of the media, or if the communi-
cation is widely disseminated, it is probably of public concern.
Conversely, if the case involves slander that is shared with only
very few people, it might be a matter of private concern.
d. the availability of presumed and punitive damages; and
(1) General damages may be presumed if otherwise available under
defamation law, and punitive damages may be awarded upon
proper proof in two situations:

535
OUTLINE

(a) in any case where the plaintiff establishes that the defendant
acted with malice; and
(b) where the common law still applies—that is, where a private
plaintiff sues for defamation as to a matter not of public concern.
(2) Where negligence is the applicable degree of fault required by
constitutional principles and the plaintiff does not show malice,
the plaintiff may only recover actual damages.
(3) Actual damages are broader and more inclusive than the
common law pecuniary damages, and include all injuries to the
plaintiff’s reputation. They need not be supported by evidence of
quantifiable monetary losses, but there must be introduction of
some evidence as proof.
(4) The U.S. Supreme Court has held that speech such as parodies,
cartoons, and other satirical utterances cannot be actionable as
an intentional infliction of emotional distress unless they contain
false statements of fact made with malice.
e. the media status of the defendant.

536
TORTS

VII. INVASION OF PRIVACY

A. Intrusion into Seclusion


1. This form of invasion of privacy is present when a defendant unreasonably
intrudes into the plaintiff’s seclusion.
a. The wrongful conduct is an interference with the plaintiff’s seclusion,
including physical intrusions (placing a webcam in the plaintiff’s bath-
room) and non-physical intrusions (i.e., photographing the plaintiff in
his backyard from off the property).
b. To be actionable, the defendant’s intrusion must be one that would be
highly objectionable to a reasonable person.
c. Seclusion refers to a plaintiff’s right to physical solitude or to the
privacy of personal affairs or concerns. In circumstances where the
plaintiff has no reasonable expectation of solitude or privacy, conduct
by the defendant that might otherwise be intrusive is not an invasion of
seclusion (e.g., eavesdropping on a conversation between the plaintiff
and another person as they walk down a public sidewalk).
2. Damages recoverable for invasion of seclusion include compensatory
damages (e.g., mental distress unaccompanied by physical injury) and,
under appropriate circumstances, punitive damages.

B. Appropriation of Identity or Likeness


1. Appropriation is an unauthorized use of the plaintiff’s identity or likeness
for the defendant’s commercial advantage.
a. The plaintiff bears the burden of proving that he did not consent to the
defendant’s use of his identity or likeness.
b. Use of “identity or likeness” is present if the defendant uses any object
or characteristic sufficient to identify the plaintiff. Use of an object or
characteristic that does not identify the plaintiff is not actionable.
c. The wrongful use of the plaintiff’s identity or likeness must be in
connection with the promotion of a product or service. The fact that the
defendant derived economic benefit from the use of the plaintiff’s iden-
tity is not enough alone to constitute a violation (e.g., a biographer paid
to write an unauthorized book about a famous plaintiff).
2. Compensatory damages are recoverable, measured by the reasonable
value of the use of the plaintiff’s identity or likeness. Punitive damages
should be recoverable on a proper showing.
EXAMPLE: Jonathan wishes to advertise a weight-loss product. Without
asking her permission, Jonathan puts Calista Flockhart’s photo in an ad for
the product. Flockhart will have a cause of action against Jonathan.

C. Public Disclosure of Private Facts


1. This form of invasion of privacy is present when a defendant unreasonably
discloses private facts about a plaintiff to the public.

537
OUTLINE

a. Disclosure is actionable if it would be highly offensive to a reasonable


person and not of legitimate public concern.
b. The private facts must be disseminated to the public. Communicating
them to a third person is not sufficient to constitute this tort (as distin-
guished from defamation, where communication to a single third
person is a sufficient publication).
c. The information or material disclosed by the defendant must be an
aspect or component of the plaintiff’s life not open to public view or
inspection and not a matter of public record.
2. The plaintiff can recover compensatory damages, including mental distress
unaccompanied by physical injury.
3. Public disclosure of private facts is not actionable where the publication is
newsworthy. This broad defense can apply to pictures published in newspa-
pers as well as magazine articles on former celebrities and public figures.
Private matters contained in public records are absolutely privileged.

D. Portrayal in a False Light


1. This tort is present when a defendant publishes matters that portray a plain-
tiff in a false light.
a. The defendant must communicate the material to a substantial
number of people.
b. Portraying in a false light means attributing to the plaintiff views he
does not hold or attributing actions to him that he did not take. In addi-
tion, the false light in which the plaintiff is placed must be such that a
reasonable person would find it highly offensive.
(1) This is a “lesser” form of offensive falseness than required
for defamation.
2. The plaintiff can recover compensatory and, in a proper case, punitive damages.

E. Defenses and Privileges


1. Truth
a. Truth is not a defense to appropriation or intrusion, since truth or
falsity is not a relevant issue. However, truth is a complete defense to
portrayal in a false light.
2. Consent
a. Consent is a defense to disclosure, intrusion, and false light to the
same extent it is to any other intentional tort.
b. The plaintiff is required to prove lack of his consent as an element of
his prima facie case for appropriation.
3. Privilege
a. The absolute and conditional privileges applicable to defama-
tion should be available as to public disclosure of private facts and
portrayal in a false light.

538
TORTS

F. Constitutional Principles
1. Where a plaintiff is portrayed in a false light as to a matter of public interest,
the plaintiff must prove that the defendant had acted with malice—knowl-
edge of falsity or reckless disregard for truth or falsity—in order to recover.

539
OUTLINE

VIII. WRONGFUL INSTITUTION OF LEGAL PROCEEDINGS

A. Malicious Prosecution
1. Malicious prosecution is the institution of criminal proceedings by a
defendant, done for an improper purpose and without probable cause, that
terminate favorably for the plaintiff and cause the plaintiff damages.
a. The wrongful conduct is the taking of action that results in the
commencement of the criminal prosecution of the plaintiff. Such
actions include persuading a prosecutor to bring charges against a
plaintiff, signing an affidavit for a warrant, or giving false information to
the authorities with knowledge of its falsity. It does not constitute mali-
cious prosecution to give information to authorities while relying upon
their discretion as to whether to prosecute.
b. The defendant must act for a primary purpose other than to bring a guilty
person to justice. That there was no probable cause supporting the initia-
tion of proceedings is evidence that the defendant’s purpose was improper.
c. The defendant lacks probable cause for initiation of criminal proceed-
ings when either:
(1) a reasonable person possessing the same facts as the defen-
dant would not have believed that the plaintiff was guilty of the
charged offense; or
(2) defendant did not actually believe that the plaintiff was guilty.
EXAMPLE: Dahlia is still angry with her former boyfriend, Preston.
Dahlia contacts the police and has Preston arrested on suspicion
of being a terrorist. Preston is not a terrorist, and Dahlia knows this
fact. A judge eventually dismisses the case. Preston will have a
cause of action against Dahlia for malicious prosecution.
d. A grand jury indictment returned against a plaintiff after the defendant’s
action or a magistrate holding the plaintiff to answer after a prelimi-
nary hearing are prima facie evidence that probable cause existed
to believe the plaintiff was guilty. A plaintiff’s conviction on criminal
charges is conclusive evidence of probable cause, even if the convic-
tion is subsequently reversed on appeal.
e. An attorney’s advice to the defendant to institute criminal proceed-
ings conclusively establishes probable cause in most jurisdictions, at
least where the defendant made a full and fair disclosure of all relevant
facts, and the attorney is competent and duly admitted to practice in
the jurisdiction or otherwise qualified to render an opinion.
f. The criminal prosecution must have terminated in a fashion indicating
that the plaintiff was innocent of the charges. Terminations on the merits
(i.e., acquittal after trial or court dismissal for lack of sufficient evidence)
are sufficient in this regard; terminations based on procedural or tech-
nical defects, prosecutorial discretion, or similar grounds are not.

540
TORTS

2. When a plaintiff establishes the essential elements of a cause of action for


malicious prosecution, he is entitled to recover damages for:
a. the harm to his reputation resulting from the accusation brought
against him; and
b. the emotional distress resulting from the bringing of the proceedings.
3. Defenses
a. Privilege
(1) Judges and prosecutors are absolutely privileged as to
malicious prosecution.
(2) Law enforcement officers have a more limited immunity. A
law enforcement officer will not be protected from malicious
prosecution if the particular conduct complained of consisted of
acts outside the scope of the officer’s official duties or authority.
If they act within the scope of their authority or with probable
cause, they will enjoy immunity from malicious prosecution.
b. A defendant may prevail in a malicious prosecution action by a plaintiff if the
defendant can establish by a preponderance of the evidence that the plain-
tiff was actually guilty of the crime for which the plaintiff was prosecuted.
(1) A termination of the criminal proceeding in favor of the plaintiff does
not preclude this result, having no res judicata effect due to the higher
standard of proof in criminal proceedings (because the prosecution
must prove the defendant’s guilt beyond a reasonable doubt).

B. Wrongful Institution of Civil Proceedings


1. A person who takes an active part in the initiation, continuation, or procure-
ment of civil proceedings against another is subject to liability to the other
for wrongful civil proceedings, if:
a. he acts without basis and primarily for a purpose other than that of securing
the proper adjudication of the claim on which the proceedings are based; and
b. except when they are ex parte, the proceedings have terminated in
favor of the person against whom they are brought.
EXAMPLE: Deborah dislikes her new neighbor, Parker. To try to get
him to move out of the neighborhood, Deborah sues Parker for pri-
vate nuisance, alleging that his music is too loud. Deborah knows that
Parker rarely plays his music so that it can be heard outside of his
home and he always turns it down if asked. When the court hears the
evidence against Parker, it dismisses the case. Parker has a cause of
action against Deborah for the wrongful institution of civil proceedings.

C. Abuse of Process
1. Abuse of process exists where a defendant intentionally misuses a judicial
process (whether civil or criminal) for a purpose other than that for which
the process is intended. This tort also parallels malicious prosecution.

541
OUTLINE

2. The plaintiff need not show the defendant’s lack of probable cause, as proof
of the defendant’s improper purpose serves the same function.

542
TORTS

IX. ECONOMIC TORTS

A. Intentional Misrepresentation (Fraud)


1. An intentional misrepresentation by a defendant, made with scienter,
which is material and justifiably relied upon by a plaintiff and which causes
damages to the plaintiff, is actionable.
2. An actionable misrepresentation is an assertion of a false past or present fact.
The generally cited principle that a misrepresentation of opinion is not actionable
is subject to exceptions, which are discussed in connection with reliance.
3. A misrepresentation can consist of:
a. a false, affirmative assertion;
b. active concealment; or
(1) In a majority of jurisdictions, a defendant cannot be liable for active
concealment if his transaction with the plaintiff is stated to be “as is,”
or the plaintiff is otherwise put on notice as to the concealed facts.
c. an omission of fact (i.e., a failure to disclose).
(1) Traditionally, a defendant was not liable for misrepresentation if
his only wrongdoing was the omission of facts. Many exceptions
have been developed over the years, including:
(a) where the defendant is a fiduciary for the plaintiff;
(b) where the defendant makes an assertion believing it to be true,
subsequently discovers that it was false or that circumstances have
changed, and fails to disclose the truth or changed circumstances;
(c) where the defendant makes an incomplete or ambiguous asser-
tion, omitting additional facts that render his assertion misleading;
(d) where the defendant makes a false assertion not intending
that anyone rely upon it, subsequently discovers that the
plaintiff intends to act in reliance upon the false assertion,
and fails to disclose that the assertion was false; and
(e) where the plaintiff reasonably expects disclosure.
1) A minority of modern jurisdictions impose liability on a
defendant for the omission of facts where, under the
circumstances, the plaintiff could reasonably expect
disclosure. Factors to consider include:
a) the relation of the parties;
b) the nature of the undisclosed facts; and
c) the nature of the transaction.
4. The defendant must intend that the plaintiff or a class of persons of which the
plaintiff is a member will act or fail to act in reliance on his misrepresentation.
EXCEPTION: Any plaintiff may recover for misrepresentation regardless of
defendant’s intent if the misrepresentation is characterized as ongoing (e.g.,
a mislabeled product).

543
OUTLINE

a. Some cases have found liability for misrepresentation in any situation


where the defendant could reasonably foresee that someone would
rely upon his misrepresentation, regardless of whether the defendant
intended that the particular plaintiff rely on it.
5. Scienter is present when the defendant makes a misrepresentation
knowing it to be false or recklessly possessing insufficient information as
to its truth or falsity.
6. A representation or omission is material if it would influence a reasonable
person in determining his course of action in the particular transaction at issue. A
fact is also material, even if a reasonable person would not regard it as impor-
tant, if the defendant knows that the plaintiff actually regards it as important.
7. The plaintiff must rely on the defendant’s misrepresentation, and that reli-
ance must be justified. In general, reliance on an assertion is justified if a
reasonable person would have relied upon it. Factors considered in this
analysis include the nature of the misrepresentation, the parties, and the
relationship of the parties.
a. Reliance on a misrepresentation of fact is generally regarded as justi-
fied except where the representation is patently false. A plaintiff has no
duty to investigate, even where not burdensome to do so. If the plaintiff
actually investigates, he may not rely on representations inconsistent
with what he actually or might reasonably have discovered.
b. In general, reliance on a misrepresentation of opinion is not justified.
There are numerous exceptions to this principle, however.
(1) A plaintiff’s reliance on a defendant’s assertion of opinion is justi-
fied where the defendant owes the plaintiff a fiduciary duty.
(2) In circumstances not amounting to a fiduciary relation, but where
a defendant has the confidence of a plaintiff (e.g., the defendant
is the plaintiff’s uncle), reliance on the defendant’s misrepresenta-
tions may be justified.
(3) Reliance on a defendant’s opinion is more likely to be justified if
the defendant possesses much greater expertise than the plaintiff
as to the subject of the transaction.
(4) If the defendant has an interest that he fails to disclose to the
plaintiff, it is justifiable for the plaintiff to rely on the defendant’s
expression of opinion on the subject of that interest.
c. Traditionally, representations of law were regarded as expressions of
opinion, and thus a plaintiff’s reliance was not justified. Today, courts
apply the principles discussed above. In addition, courts are quite
willing to find implied statements of fact in legal opinions.
8. The element of causation is met if a defendant’s misrepresentation played a
substantial part in inducing the plaintiff to act as he did.
9. A plaintiff may recover compensatory damages for the value of what he would
have received if not for the misrepresentation. A plaintiff may also recover puni-
tive damages upon a showing that the defendant acted with common law malice.

544
TORTS

B. Negligent Misrepresentation
1. The traditional rule, followed by a majority of jurisdictions, is that negligent
misrepresentations are not actionable. Many of these jurisdictions neverthe-
less allow recovery where there is arguably no intentional misrepresentation
by resorting to legal fictions that permit their courts to find intent because a
defendant’s honestly held belief in the truth of his assertion is unreasonable.
a. Many jurisdictions impose liability for negligent misrepresentation only in
certain situations where a special relationship exists between the defendant
and the plaintiff, and the nature of the defendant’s activity justifies holding
the defendant liable for a failure to exercise due care. The most widely
recognized circumstance involves a defendant in the business of supplying
information to be used by others in making economically significant deci-
sions (e.g., accountants, title abstractors, or lawyers). Such defendants
are liable if they fail to exercise due care in determining the truth or falsity
of the representations they make. The elements of this cause of action are
identical to those for fraud, with the following differences:
(1) The mental state a defendant must have for liability under negli-
gent misrepresentation is the same for negligence analysis
generally. Thus, a defendant’s representations, made in good
faith, are actionable if they are inaccurate because the defendant
failed to exercise due care.
(2) A defendant is liable for a negligent misrepresentation only to:
(a) the person to whom the misrepresentation was made; and
(b) to any other specific persons or identifiable group of persons
that the defendant knew would rely upon the misrepresentation.
1) Note the absence of foreseeability, a common negli-
gence concept; courts expanding the reach of fraud into
negligent misrepresentation sought to limit the class of
potential plaintiffs.

C. Interference with Contractual Relations


1. The main type of interference with economic relations that has been
marked out by the courts and regarded as a separate tort is referred to as
inducing breach of contract or interference with contract.
2. Virtually any type of contract may be the basis for this type of tort action.
The contract must:
a. be in force and effect;
b. be legal; and
c. not be opposed to public policy.
3. For reasons that have not been clearly stated, contracts to marry have
received special treatment, and almost without exception the courts have
refused to hold that it is a tort to induce the parties to break them.
4. To be held liable for interference with a contract, the defendant must be
shown to have caused the interference. It is not enough that he merely has

545
OUTLINE

reaped the advantages of the broken contract after the contracting party
has withdrawn from it.
EXAMPLE: Dani calls Jeffrey, whom she knows to be involved in a contract
with PubCo. Dani promises Jeffrey that she will go on a date with him if
he breaks the contract with PubCo. Unbeknownst to Jeffrey, Dani plans to
have her company take over Jeffrey’s business with PubCo once Jeffrey
breaks the contract. If Jeffrey does breach as a result of Dani’s bribe, Dani
will be liable for interference with a contract.
5. There have also been many decisions in which the action has been allowed
where the defendant has merely prevented the performance of a contract or
has made the performance more difficult and onerous.
EXAMPLE: Donald Defendant prevents Peter Promisor from supplying Paul
Plaintiff with goods by calling an illegal strike among his workmen. Donald
will be liable for interference with a contract.
6. Interference with a contract is almost entirely an intentional tort. Liability
has not been extended to the various forms of negligence by which perfor-
mance of a contract may be prevented or rendered more burdensome.
7. Where the damages suffered can be compensated with money, then an
action at law is appropriate.
a. If substantial loss has occurred, one line of cases tends to adopt the
contract measure of damages, limiting recovery to those damages that were
within the contemplation of the parties when the original contract was made.
b. Another line of cases, however, applies a tort measure, but limits
the damages to those which are sufficiently “proximate,” with some
analogy to the rules of negligence.

D. Interference with Prospective Advantage


1. This tort protects the probable “expectancy” interests of the future contrac-
tual relations of a party, such as the prospect of obtaining employment or
the opportunity to obtain customers.
2. Modern decisions have expanded this tort action to protect such noncommer-
cial expectancies as interference with an expected gift or legacy under a will.
a. In such cases, courts of equity have granted relief by imposing a
constructive trust.
b. It should be noted, however, that all such cases (e.g., suppression of a
will or fraudulently inducing testator to make a will or prospective gift),
whether in a tort action or under a constructive trust, have involved
tortious conduct such as fraud, duress, or defamation.
3. Although earlier decisions required so-called “malice,” modern decisions
hold a defendant liable where his conduct is unlawful in itself (e.g., where
it involves violence, intimidation, defamation, injurious falsehood, fraud,
etc.) or is malevolent, such as evincing a desire to do harm to the plaintiff
for its own sake.

546
TORTS

a. Proof of the intentional interference and resulting damages estab-


lishes what the courts have called a “prima facie tort,” and cast upon
the defendant the burden of avoiding liability by showing that his
conduct was privileged.
b. The most common defense centers around the privilege of competition.
In sum, it is not a tort to beat a business rival to prospective customers.
Thus, in the absence of prohibition by statute, illegitimate means,
or other unlawful conduct, a defendant seeking to increase his own
business may cut rates or prices, allow discounts, or enter into secret
negotiations behind the plaintiff’s back; refuse to deal with plaintiff; or
threaten to discharge employees who do.

E. Injurious Falsehood (Trade Libel)


1. An injurious falsehood is a false statement made to another by the defen-
dant that causes economic injury to the plaintiff.
a. The plaintiff bears the burden of proving the falsity of the
challenged statement.
(1) The false statement need not be defamatory, personally relate to
the plaintiff, or cause others to shun the plaintiff’s company.
b. The false statement must be made to a third person. An otherwise
actionable statement made to the plaintiff will not result in liability.
c. The defendant must intend to cause others not to do business with the
plaintiff or to otherwise interfere with the plaintiff’s relations with others
to the plaintiff’s economic disadvantage. In many jurisdictions, such
motives will be found upon proof that the defendant knowingly made
the false statement or was reckless with regard to its truth or falsity.
2. Recovery is only for those pecuniary losses that the plaintiff proves have
been realized or liquidated (e.g., specific lost sales). Proof is generally
sufficient if equivalent to that which would establish lost profits in a breach
of contract case.
EXAMPLE: On The Day Break Show, Brian Gumdrop insults Milan Hyatt’s
new line of handbags, calling them “made of pigskin.” Sales of the bags no-
ticeably diminish, and Milan is forced to declare bankruptcy. Milan may have
a cause of action against Brian for trade libel if they are not made of pigskin.
3. Defenses
a. Consent
(1) A defendant will not be liable for an injurious falsehood if
speaking with the consent of the plaintiff.
b. Privilege
(1) The same absolute and qualified privileges applicable to defama-
tion are available to a defendant in an injurious falsehood action.
If the plaintiff establishes that the defendant acted with common
law malice, conditional privilege will be lost.

547
OUTLINE

NOTE If the defendant’s speech involves a matter of public concern or affects a


plaintiff who is a public official or figure, the constitutional principles applicable
to defamation law may also be applicable to injurious falsehood. The com-
mercial nature of this tort suggests that most cases would affect only matters
of private concern, and thus not invoke constitutional limitations.

548
TORTS

X. MISCELLANEOUS TORT CONCEPTS

A. Vicarious Liability
1. Vicarious liability describes liability imposed on a defendant because of his
relationship with the actual wrongdoer that directly caused injury to the plaintiff.
2. Employer-Employee (Respondeat Superior)
a. An employer is liable for injuries caused by the negligence or strict
liability of an employee if the tortious act occurred within the scope of
the employment.
(1) To determine whether the tortious acts occurred within the scope
of employment, a distinction is made based on whether the
tortious conduct was committed while the employee was on a
frolic (major deviation; outside of scope) or on a detour (small
deviation from an employer’s directions; within the scope).

EXAM TIP The more minor the deviation is in time and geographic area, the more likely it
will be only a detour, and therefore torts committed during that time will be con-
sidered within the scope of employment, making the employer vicariously liable.

b. Acts are within the scope of employment if they are so closely


connected with what the employee was hired to do and so fairly and
reasonably incidental to it that they may be regarded as methods, even
though improper, of carrying out the objectives of the employment.
c. Intentional torts committed by an employee are generally not given
respondeat superior effect, even if committed during working hours.
However, if an employee uses force, even misguidedly, wholly or partly
to further the employer’s purpose, such use of force may fall within the
scope of employment, resulting in vicarious liability for the employer.
3. Independent Contractor
a. A defendant generally is not liable for torts committed by someone he
has engaged as an independent contractor, because the defendant
has no right to control the activity of the contractor.
b. In two situations, contrary to the general rule set forth above, a defen-
dant may be held vicariously liable for the torts of an independent
contractor. These are applicable where:
(1) the contractor undertakes a duty the law does not permit to be
delegated to another (a “nondelegable” duty); and
EXAMPLE: Keeping streets in good repair, maintaining a fence
around an excavation, or an owner-occupier’s duty to have safe
premises for business invitees.
(2) a contractor engages in inherently dangerous activities, defined
as any activity as to which there is a high degree of risk in rela-
tion to the particular surroundings, recognizable in advance as
requiring special precautions.

549
OUTLINE

EXAMPLE: Fumigating with poisonous gases or using explosives


in an urban setting.
4. Joint Enterprise (Partners and Joint Venturers)
a. Partners and joint venturers are vicariously liable for each others’ torts
if those torts were committed in the course and scope of the partner-
ship or joint venture.
b. A partnership is a legal relationship arising from an agreement between
two or more persons to operate a business for profit. A joint venture is like a
partnership, except it is of more limited scope and duration. A joint venture
is present when two or more people engage in concerted activity for a
common business purpose and each person has a mutual right to control
the activity—for example, two people on a shared-expense auto trip.
5. Negligent Entrustment
a. Courts generally recognize liability for negligent entrustment, whereby
a defendant will be held liable for negligently permitting a third party to
use a thing or engage in an activity.
b. To be liable for negligent entrustment, the plaintiff must generally prove that:
(1) the entrustee was incompetent, unfit, inexperienced, or reckless;
(2) the entrustor knew, should have known, or had reason to know of
the entrustee’s condition or proclivities;
(a) Some jurisdictions require actual knowledge.
(3) there was an entrustment of the chattel;
(a) The entrustment does not need to be express; the entrust-
ment or permission can be implied. The test is whether the
owner knew, or had reasonable cause to know, that he was
entrusting his property.
(b) While negligent entrustment is usually applied to motor vehicles
or firearms, it has been applied to other chattels which, if placed
in the hands of an incompetent or inexperienced person, present
a likelihood of unreasonable risk of harm to third persons.
(c) Some courts refer to entrustment of a “dangerous instrumen-
tality.” Other courts, as well as the Restatement (2d) of Torts,
simply refer to supplying chattel that is likely to be used in a
manner involving risk of physical harm to others.
(4) the entrustment created an appreciable risk of harm to others; and
(5) the harm to the injury victim was proximately caused by the negli-
gence of the entrustor and the entrustee.
6. Owner of Auto or Driver
a. At common law, the owner of a motor vehicle was not liable for its negli-
gent operation by another using it with or without his permission, unless:
(1) the operator was acting as his or her agent;
(2) the owner was present in the car and maintained some control
over its operation;

550
TORTS

(3) the owner negligently entrusted the vehicle; or


(4) the owner and driver were engaged in a joint enterprise or
partnership activity.
b. At modern law, the owner of a motor vehicle is liable for its negligent
operation by any person granted permission to use or operate it, for
whatever reason.
(1) The owner’s liability is generally limited to acts of the driver which the
owner would have been primarily liable for if operating the vehicle.
EXAMPLE: If a third party is driving the vehicle and a gratu-
itous guest is injured, the owner is generally accorded the benefit
of a guest statute limiting his liability to cases of gross negli-
gence, or where there has been willful or wanton misconduct.
c. Under the family-purpose doctrine, adopted in some jurisdictions, the driver
of a family car, in pursuit of recreation or pleasure, is considered to be
engaged in the owner’s business and is viewed as either an agent or servant
of the owner. The driver must be a family member for the doctrine to apply.
d. To sustain a claim against a vehicle’s owner under the family-purpose
doctrine, the plaintiff must show that [Hicks v. Newman, 641 S.E.2d 589
(Ga. Ct. App. 2007)]:
(1) the defendant owned or had an interest in or control over the vehicle;
(2) the defendant made the vehicle available for family use;
(3) the driver was a member of the defendant’s immediate household
at the time of the collision; and
(4) the driver drove the vehicle with the defendant’s permission.
e. While many jurisdictions have expressly rejected the family-purpose
doctrine, the practical results of most jurisdictions, including many which
rejected the doctrine, reach similar liability results through statutes holding
the owner responsible for all injuries negligently inflicted while his motor
vehicle is being used by another with the owner’s express or implied
consent, including members of the owner’s family.
7. Parent-Child
a. A parent is normally not vicariously liable for a tort committed by his child.
A few courts that have imposed vicarious liability on a parent for the
tort of a child characterized the relationship as “principal-agent,” akin to
employer-employee, where the child was running an errand for the parent.
8. Bailor-Bailee
a. A bailor is not vicariously liable for the torts of a bailee.
9. Tavernkeepers
a. Historically, a tavernkeeper was not vicariously liable for injuries
inflicted by an intoxicated patron, whether the person injured was a
third person or was the intoxicated patron himself.
b. Some states statutorily impose vicarious liability on tavernkeepers as
to injuries to third persons caused by intoxicated patrons.

551
OUTLINE

B. Joint and Several Liability


1. Where two or more defendants acting in concert injure the plaintiff or where
two or more defendants acting independently injure the plaintiff, and the
resulting damages cannot be allocated to particular defendants, all of the
defendants are liable for the entirety of the plaintiff’s injury. The plaintiff can
execute against each defendant for the total damages suffered, although
the plaintiff may only recover from any or all defendants an amount equal to
the total damages awarded.
EXAMPLE: Matt was walking down the street one day when he was hit in
the head by a flower pot that fell off the roof of a building. It was discov-
ered that the flower pot was knocked off the roof during a drunken brawl
by two men, Randall and Perry. Matt sued Randall and Perry in negligence
for damages, and a jury found them liable. Matt may recover up to the full
amount of damages from either Randall or Perry (though he will only be
able to recover a total amount equal to the total damages awarded).
2. In jurisdictions that adopt comparative negligence systems, joint and
several liability is still available to a plaintiff. Any defendant compelled to
pay damages to a plaintiff greater than the percentage amount for which
he was found responsible may usually obtain contribution for the pro rata
shares of the other defendants.
3. Contribution
a. Most jurisdictions have by statute permitted one of several defendants
responsible for negligently injuring a plaintiff to compel the others to
contribute an equal share toward any judgment satisfied by the plaintiff
against one of them.
(1) Contribution is not applicable where the defendants have inten-
tionally injured the plaintiff.
(2) A minority of states retain the traditional common law rule that did
not permit contribution among tortfeasors.
b. Most jurisdictions apportion contribution equally. Thus, the judg-
ment against all defendants is divided into as many equal portions as
there are defendants, and a defendant against whom the plaintiff has
executed for the full amount of damages may obtain the appropriate
amount paid from each of the other defendants.
c. Other jurisdictions allocate the amount payable by each defendant by
reference to the share of the plaintiff’s injuries for which that defendant
is responsible, as would be determined under a comparative negli-
gence system for example.
EXAMPLE: Same facts as above, only the jurisdiction is one that has
adopted comparative negligence. The jury determined that Randall
was 60% liable and Perry was 40% liable for causing Matt’s injury, and
awarded Matt $100,000 in damages. If Matt recovers the full $100,000
from Randall, Randall can seek contribution from Perry for his share of
the damages, or $40,000 (40% of the total $100,000 damage award).

552
TORTS

4. Indemnity
a. One of two or more defendants responsible for a plaintiff’s injuries may
in some situations cause one or more of the other defendants to satisfy
the entire amount of the plaintiff’s damages.
(1) Where one defendant is only vicariously liable for the tort of another
directly liable defendant, the first defendant may recover the entire
amount of any damages paid to the plaintiff from the second defen-
dant, who was actually responsible for the plaintiff’s injury.
EXAMPLE: A fiddler was injured by a truck driver that negligently
went through a red light while making a delivery. The fiddler sued the
truck driver as well as his employer, through respondeat superior.
The court found in favor of the fiddler and awarded him $100,000 in
damages. The fiddler recovered this amount from the truck driver’s
employer. The employer may then sue the truck driver for indemnity.
(2) In some jurisdictions, where a plaintiff has obtained and satisfied a
judgment against one defendant who is jointly liable with others, that
defendant may recover from the other defendants if their conduct
was “more wrongful” than the first defendant’s. In general, intentional
tortfeasors are viewed as “more wrongful” than negligent tortfeasors.
(a) Courts also make distinctions between “active” wrongdoing
(e.g., a manufacturer who makes a defective product) and
“passive” wrongdoing (e.g., the retailer who fails to discover
the defect), attributing “more wrongfulness” to the former.
(3) A defendant who injures a plaintiff may be liable when another
defendant subsequently injures the plaintiff and aggravates the
injuries the plaintiff suffered from the first defendant. Courts
generally permit the first defendant to obtain indemnity from the
second for the portion of additional damages imposed on the first
defendant attributable to the aggravating injury.

C. Survival of Action and Wrongful Death


1. Traditionally, the death of either the victim or the tortfeasor abated a tort
action between them. Nearly all jurisdictions provide via statute (called
survival statutes) that the death of the victim or tortfeasor no longer
abates the tort action. Statutory treatment varies according to the type of
injury suffered and the damages recoverable.
a. Most jurisdictions provide for survival of actions where a plaintiff
suffered personal injuries. A minority permit survival of the action only
where the injury suffered is property damage.
b. Only a few jurisdictions allow survival of the action where the injury
suffered is reputational (defamation) or involves privacy interests.
c. Some jurisdictions do not permit recovery in a survival action for the
deceased victim’s pain and suffering if the victim died before his case
went to judgment.

553
OUTLINE

d. Most jurisdictions do not permit any recovery of punitive damages


against the estate of a tortfeasor who dies before judgment is rendered.
2. All jurisdictions statutorily provide for an action by which either the heirs of
a deceased victim or the personal representative of the victim’s estate may
bring an action against the tortfeasor responsible for the victim’s death.
Recovery may generally be had for pecuniary losses resulting from the
death, such as wages the victim might have earned over his lifetime, and
for damages, such as victim’s medical expenses or loss of wages.

D. Satisfaction and Release


1. A satisfaction occurs when more than one defendant is liable for a plain-
tiff’s injuries, but the plaintiff recovers fully from one defendant. The effect
of a satisfaction is that the plaintiff may not seek further recovery from any
other defendant. The defendant against whom satisfaction was had may
seek contribution or indemnity from the other defendants.
2. At common law, if a plaintiff agreed with one defendant to give up his rights in
exchange for a settlement, the plaintiff lost his rights against all other defendants
who might be otherwise jointly responsible for the plaintiff’s injuries. Most states
have modified this doctrine by permitting the plaintiff to release one defendant
while expressly retaining his rights against others, or by permitting a plaintiff
to make a covenant not to sue with one defendant while proceeding against
the others. Other jurisdictions have entirely abolished the doctrine of release.

554

You might also like